Sie sind auf Seite 1von 206

www.crackjee.

xyz

www.crackjee.xyz
Head Office : B-32, Shivalik Main Road, Malviya Nagar, New Delhi-110017

• Sales Office : B-48, Shivalik Main Road, Malviya Nagar, New Delhi-110017
Tel. : 011-26691021 / 26691713

Page Layout : Prakash Chandra Sahoo

Typeset by Disha DTP Team

DISHA PUBLICATION
ALL RIGHTS RESERVED

© Copyright Author

No part of this publication may be reproduced in any form without prior permission of the publisher. The author and the
publisher do not take any legal responsibility for any errors or misrepresentations that might have crept in. We have
tried and made our best efforts to provide accurate up-to-date information in this book.

For further information about the books from DISHA,


Log on to www.dishapublication.com or email to info@dishapublication.com
www.crackjee.xyz
STUDY PACKAGE IN PHYSICS FOR JEE MAIN & ADVANCED
Booklet No. Title Chapter Nos. Page Nos.
Ch 0. Mathematics Used in Physics
Ch 1. Units and Measurements
Units, Measurements &
1 Ch 2. Vectors 1-202
Motion
Ch 3. Motion in a Straight Line
Ch 4. Motion in a Plane
Laws of Motion and Ch 5. Laws of Motion and Equilibrium
2 203-318
Circular Motion Ch 6. Circular Motion
Ch 7. Work, Energy and Power
Work Energy, Power &
3 Ch 8. Collisions and Centre of Mass 319-480
Gravitation
Ch 9. Gravitation
4 Rotational Motion Ch 1. Rotational Mechanics 1-120
Ch 2. Properties of Matter
Properties of Matter &
5 Ch 3. Fluid Mechanics 121-364
SHM
Ch 4. Simple Harmonic Motion
Ch 5. Thermometry, Expansion &
Calorimetry
6 Heat & Thermodynamics Ch 6. Kinetic Theory of Gases 365-570
Ch 7. Laws of Thermodynamics
Ch 8. Heat Transfer
Ch 9. Wave – I
7 Waves 571-698
Ch 10. Wave –II
Ch 0. Mathematics Used in Physics
8 Electrostatics Ch 1. Electrostatics 1-216
Ch 2. Capacitance & Capacitors
Ch 3. DC and DC circuits
9 Current Electricity Ch 4. Thermal and Chemical effects of 217-338
Current"
Ch 5. Magnetic Force on Moving
Charges & Conductor
Ch 6. Magnetic Effects of Current
10 Magnetism, EMI & AC Ch 7. Permanent Magnet & Magnetic 339-618
Properties of Substance
Ch 8. Electromagnetic Induction
Ch 9. AC and EM Waves
Ch 1. Reflection of Light
Ch 2. Refraction and Dispersion
11 Ray & Wave Optics Ch 3. Refraction at Spherical Surface, 1-244
Lenses and Photometry
Ch 4. Wave optics
Ch 5. Electron, Photon, Atoms,
Photoelectric Effect and X-rays
12 Modern Physics 245-384
Ch 6. Nuclear Physics
Ch 7. Electronics & Communication
www.crackjee.xyz
Contents
Contents
Study Package Booklet 1 - Units, Measurements & Motion
0. Mathematics Used in Physics 1-16 3. Motion in a Straight Line 93-146

3.1 Concept of a point object 94


1. Units and Measurements 17-60
3.2 Rest and motion are relative terms 94
Definitions Explanations and Derivations 18 3.3 Motion 94
1.1 Fundamental quantities 18
3.4 Motion parameters 94
1.2 Derived quantities 18
1.3 The SI system of units 18 3.5 Equations of motion 97
1.4 Definitions of SI units 19 3.6 Study of motion by graphs 105
1.5 Advantages of SI system 20 3.7 Relative velocity 112
1.6 Dimensions of a physical quantity 20
3.8 Motion with variable acceleration 118
1.7 Order of magnitude 22
3.9 Problems based on maxima and minima 118
1.8 Rules of significant figures 22
1.9 Errors in measurement 26 Exercise 3.1 Level 1 (Single correct option)
1.10 Indirect methods of measuring Exercise 3.1 Level 2 (Single correct option)
large distances 31
Exercise 3.2 (more than one correct options)
1.11 Indirect method of measuring
small distances 33 Exercise 3.3 (Assertion and Reasoning type questions)

1.12 Vernier callipers and ccrew gauge 34 Exercise 3.4 (Passage & Matrix)
Exercise 1.1 Level 1 (Single correct option) Exercise 3.5 (Past years JEE-(Main and Advance)
Exercise 1.1 Level 2 (Single correct option) Hints and Solutions (Solution of all exercises)
Exercise 1.2 (more than one correct options)
Exercise 1.3 (Assertion and Reasoning type questions) 4. Motion in a Plane 147-202
Exercise 1.4 (Passage & Matrix)
4.1 Introduction 148
Exercise 1.5 (Past years JEE-(Main and Advance)
Hints and Solutions (Solution of all exercises) 4.2 Position vector and displacement 148
4.3 Average velocity 148
2. Vectors 61-92 4.4 Average acceleration 149
Definitions Explanations and Derivations 62 4.5 Motion in a plane with constant
2.1 Scalar quantity or scalar 62 acceleration 150
2.2 Vector quantity or vector 62 4.6 Relative velocity in two dimensions 151
2.3 Vectors operations 65 4.7 Projectile motion 157
2.4 Addition or subtraction of two vectors 65
4.8 Projection up on an inclined plane 169
2.5 Addition or subtraction of more than
4.9 Projection down the inclined plane 170
two vectors 68
2.6 Product of two vectors 73 4.10 Motion along a curved path 171

2.7 Geometrical interpretation of scalar 4.11 Constraint relations 179


triple product 77 Exercise 4.1 Level 1 (Single correct option)
Exercise 2.1 Level 1 (Single correct option)
Exercise 4.1 Level 2 (Single correct option)
Exercise 2.1 Level 2 (Single correct option)
Exercise 4.2 (more than one correct options)
Exercise 2.2 (more than one correct options)
Exercise 4.3 (Assertion and Reasoning type questions)
Exercise 2.3 (Assertion and Reasoning type questions)
Exercise 2.4 (Passage & Matrix) Exercise 4.4 (Passage & Matrix)

Exercise 2.5 (Past years JEE-(Main and Advance) Exercise 4.5 (Past years JEE-(Main and Advance)
Hints and Solutions (Solution of all exercises) Hints and Solutions (Solution of all exercises)
www.crackjee.xyz
Mathematics Used in Physics 1
www.crackjee.xyz
2 MECHANICS
ALGEBRA
Common Identities
(i) (a + b)2 = a2 + b2 + 2ab = (a – b)2 + 4ab
(ii) ( a – b)2 = a2 + b2 – 2ab = (a + b)2 – 4ab
(iii) a2 – b2 = (a + b) (a – b)
(iv) (a + b)3 = a3 + b3 + 3ab(a + b)
= a3 + b3 + 3a2b + 3ab2
(v) (a – b) = a3 – b3 – 3ab (a – b)
3

= a3 – b3 – 3a2b + 3ab2
(vi) a3 + b3 = (a + b) (a2 – ab + b2)
= (a + b)3 – 3ab(a + b)
(vii) a – b = (a – b) (a2 + ab + b2)
3 3

= (a – b)3 + 3ab (a – b)
(viii) (a + b)2 + (a – b)2 = 2(a2 + b2)
(ix) (a + b)2 – (a – b)2 = 4ab.
QUADRATIC EQUATION
An algebraic equation of second order (highest power of variable is 2) is called a quadratic equation
e.g.
ax 2 + bx + c = 0, a¹0
It has solution for two values of x which are given by

-b ± b2 - 4ac
x =
2a
2
The quantity b – 4ac, is called discriminant of the equation.
BINOMIAL THEOREM
(i) The binomial theorem for any positive value of n

( x + a)n = x n + n C1ax n -1 + n C2 a 2 x n - 2 + .......... + n Cr a r x n -r + ......... + a n

n!
where a is constant and n =
Cr r !(n - r )!

Here n! = n(n - 1)(n - 2)..............3 ´ 2 ´ 1


So 5! = 5 ´ 4 ´ 3 ´ 2 ´1 =120

n(n - 1) 2 n( n - 1)( n - 2) 3
(ii) (1 + x) n = 1 + nx + x + x + .....
2! 3!

For x << 1 , we can neglect the higher power of x.

So (1 + x)n ; 1 + nx

Similarly, (1 - x)n ; 1 - nx

(1 + x) - n ; 1 - nx

(1 - x) - n ; 1 + nx
Here n may have any value.
www.crackjee.xyz
Mathematics Used in Physics 3
Ex. 1 Evaluate (1.01)

Sol. (1.01)1/ 2 = (1 + 0.01)1/ 2


1
; 1+ ´ 0.01
2
= 1.005

ARITHMETIC PROGRESSION (A.P.)


A sequence like a, a + d, a + 2d, ........... is called arithmetic progression. Here d is the common
difference.
(i) The nth term of an A.P. is given by
an = a + (n - 1)d
(ii) The sum of first n term of an A.P. is given by
n n
Sn =
2
[ I term + last term ] = 2 (a1 + an )
Here a1 = a and an = a + (n – 1)d
n
\ Sn = [2a + (n - 1)d ]
2

GEOMETRIC PROGRESSION (G.P.)


The progression like, a, ar, ar2, .......... is called geometric progression, here r is called geometric ratio or
common ratio.
(i) The nth term of G.P. is given by
an ar n -1
=
(ii) The sum of the first n terms of G.P. is given by

a(r n - 1)
Sn = for (r > 0)
(r - 1)

(1 - r n )
and Sn = a for (r < 0)
(1 - r )
(iii) The sum of infinite term of G.P. for r < 1, is given by

Ist term
S =
1 - Geometric ratio

a
or S =
1- r

1 1 1
Ex. 2 Find sum of the progression; 1, , , , .........¥.
2 4 8
a
Sol. We have S =
1- r
1
Here, a = 1, r =
2
1
\ S = =2
1 - 1/ 2
www.crackjee.xyz
4 MECHANICS

EXPONENTIAL SERIES
n
æ 1ö 1 1 1
The value of e ; e = lim ç 1 + ÷ = 1 + + + + .........¥
n®¥ è nø 1! 2! 3!

1 1 1
= 1 + 1+ + + + .......¥ = 2.718
2 6 24

x x 2 x3
\ ex = 1 + + + + ..........¥
1! 2! 3!

x x 2 x3
and e- x = 1 - + - + ..........¥
1! 2! 3!
LOGARITHMIC SERIES

x 2 x3 x 4
loge (1 + x ) = x - + - + ..............¥
2 3 4
1 1 1
loge (2) = log e (1 + 1) = 1 - + - + ..............¥
2 3 4

x2 x3 x4
log(1 - x ) = - x - - - - ..............¥
2 3 4
TRIGONOMETRIC SERIES

x3 x 5
sin x = x- + - ...........
3! 5!

x 2 x4
cos x = 1- + - ...........
2! 4!
LOGARITHMS
For a positive real number a and a rational number m, we have, am = b. The another way of expressing
the same fact in that of logarithms of b to the base a is m
i.e., log a b = m
There are two bases of logarithms that are used these days. One is base e and the other base 10. The
logarithms to base e are called natural logarithms. The logarithms to base 10 are called the common
logarithms.
Thus we can write
(i) 1000 on the base of 10 as 103, and in logarithms it is; log101000 = 3.
(ii) Similarly ex = y can be written as
loge y = lny = x
Here log e ® ln
log a 1 = 0 ; log1010 = 1; log10 2 = 0.693; log e10 = 2.303

LAWS OF LOGARITHMS
Ist Law log a (mn) = log a m + log a n

æ mö
IInd Law loga ç ÷ = log a m - log a n
è nø

IIIrd Law log a (m)n = n log a m


www.crackjee.xyz
Mathematics Used in Physics 5
ANGLES
(i) Degree measure
One sixtieth of a degree is called a minute, and written 1¢,
and one sixtieth of a minute is called second, written as 1¢¢.
Thus 1° = 60¢
and 1¢ = 60¢¢ Figure. 0.1

(ii) Radian measure


1 radian : An angle with its vertex at the centre of a circle which intercepts an arc equal in length
to the radius of the circle is said to have a measure of 1 radian.
The circumference, s, of a circle of radius r is 2pr.
Thus one complete revolution subtends an angle
2 pr
q = = 2p rad
r
Thus if a circle of radius r, an arc of length l subtends an angle q radian at the centre, we have
l
q =
r
(iii) Relationship between degree and radian
2p radian = 360°
or p radian = 180° Figure. 0.2
180°
or 1radian = ; 57°16 '
p

degree 30° 45° 60° 90° 180° 270° 360°


p p p p 3p
radian p 2p
6 4 3 2 2

TRIGONOMETRIC FUNCTION
In a right angled triangle ABC, we can define that
y
sin q =
r
x
cos q =
r
y
tan q =
x
x
cot q = y

r
cosecq =
y
r
secq =
x Figure. 0.3
From above ratios, we have
1
(i) cosecq =
sin q
1
sec q =
cosq
1
and tan q =
cot q
www.crackjee.xyz
6 MECHANICS

(ii) For small angle (q ® 0) , r ® x and y ® 0


\ sin q = tan q
and cos q ® 1
(iii) sin 2 q + cos 2 q = 1
1 + tan 2 q = sec2 q
1 + cot 2 q = cosec 2 q
TRIGONOMETRIC RATIO

Angle 0° 30° 45° 60° 90° 120° 135° 150° 180°


1 1 3 3 1 1
sin 0 1 0
2 2 2 2 2 2
3 1 1 1 1 3
cos 1 0 - - - –1
2 2 2 2 2 2
1 1
tan 0 1 3 ¥ - 3 -1 - 0
3 3

THE VALUE OF TRIGONOMETRIC RATIO IN DIFFERENT QUADRANTS

Angle -q 90° - q 90° + q 180° - q 180° + q 270° - q 270° + q 360° - q 360° + q

sin - sin q cos q cos q sin q - sin q - cos q - cos q - sin q sin q

cos cos q sin q - sin q - cos q - cos q - sin q sin q cos q cos q

tan - tan q cot q - cot q - tan q tan q cot q - cot q - tan q tan q

RATIO OF DIFFERENT TRIGONOMETRIC ANGLE


(i) Consider an arc BC length l which subtends an angle q radian at A. Draw a perpendicular on AC,
we have
»
BC
q =
AB
BD
and sin q =
AB

sin q BD
\ = »
Figure. 0.4 q BC
sin q
» = BD
When q ® 0, BC lim = 1
q® 0 q

(ii) In a right triangle of sides 3, 4, 5, we have


4
sin 53° = = 0.8 , cos 37° = 0.8
5
3
and cos 53° = = 0.6 , sin 37° = 0.6.
Figure. 0.5 5
www.crackjee.xyz
Mathematics Used in Physics 7
IMPORTANT TRIGONOMETRIC FORMULAE
(i) sin (A + B) = sin A cos B + cos A sin B
(ii) sin (A – B) = sin A cos B – cos A sin B
(iii) cos (A + B) = cos A cos B – sin A sin B
(iv) cos (A – B) = cos A cos B + sin A sin B

tan A + tan B
(v) tan (A + B) =
1 - tan A tan B

tan A - tan B
(vi) tan (A – B) =
1 + tan A tan B
For A = B
(vii) sin 2A = 2sin A cos A
(viii) cos 2A = cos2 A – sin2A

2 tan A
(ix) tan 2A =
1 - tan 2 A
SUM AND DIFFERENCE FORMULAE

A+ B A-B
(i) sin A + sin B = 2sin .cos
2 2

A+ B A- B
(ii) sin A – sin B = 2 cos .sin
2 2

A+ B A-B
(iii) cos A + cos B = 2cos .cos
2 2

A+ B B- A
(iv) cos A – cos B = 2sin .sin
2 2
PRODUCT FORMULAE
(i) 2 sin A cos B = sin (A + B) + sin (A – B)
(ii) 2 cos A sin B = sin (A + B) – sin (A – B)
(iii) 2 cos A cos B = cos (A + B) + cos (A – B)
(iv) 2 sin A sin B = cos (A – B) – cos (A + B)
PROPERTIES OF TRIANGLE

(i) Laws of sines


The sides of a triangle are proportional to the sines of the opposite angle,

a b c
i.e., = = .
sin A sin B sin C
(ii) Laws of cosines
In any triangle, the square of any side is equal to the sum of the squares of the other two sides Figure 0.6
minus twice the product of these two sides into the cosine of their included angle,
i.e., a 2 = b2 + c2 – 2bc cos A
b 2 = a2 + c2 – 2ac cos B
and c 2 = a2 + b2 – 2ab cosC
www.crackjee.xyz
8 MECHANICS
1 foot = 12 inch
1 yard = 3 feet = 91.44 cm
1 mile = 1609 m
1 ton = 1000 kg
1 hectare = 10000 m2
1 m3 = 1000 litre

AREA AND VOLUME


(i) Area of triangle of height h and base b;
1
A = bh
2

Figure. 0.7

(ii) Area of trapezium


1
A = (a + b)h
2

Figure. 0.8

(iii) Area of circle

A = pR 2

Figure. 0.9

(iv) Surface area of cone, A = pR l


pR 2 h
Volume of cone, V =
3

Figure. 0.10

(v) Surface area of sphere, A = 4pR 2


4 3
Volume of sphere, V= pR
3
DIFFERENTIATION
If y is the function of x, then we can write
Figure. 0.11
y = f (x)
Here x is the independent variable and y is the dependent variable. If x varies from x to x + Dx, then
www.crackjee.xyz
Mathematics Used in Physics 9
y + Dy = f ( x + Dx)
\ Dy = ( y + Dy ) – y
= f ( x + Dx ) – f ( x )
Dy f ( x + Dx ) - f ( x)
And the ratio =
Dx Dx
Taking limits on both sides of above expression, we have
æ Dy ö f ( x + Dx) - f ( x )
lim ç lim
Dx ÷ø
=
Dx ®0 è Dx ® 0 Dx

æ Dy ö æ dy ö
The quantity lim ç ÷ is called differentiation of y w.r.t. x and we can written as ç dx ÷ .
Dx ®0 è Dx ø è ø
Thus we write,
dy f ( x + Dx) - f ( x )
= lim
dx Dx ® 0 Dx

Ex. 3 Given y = 3x2 + 5, differentiate y w.r.t. x. (iv) Differentiation of quotient of two functions :
u
Sol. Step - I Substitute (x + Dx) in place of x in the given function , Suppose y =
so we have v

y + Dy = 3( x + Dx )2 + 5 æuö é du dv ù
dy
dç ÷
è vø êëv dx - u dx úû
Step -II Dy = ( y + Dy ) - y = =
dx dx v2
= [3( x + Dx )2 + 5] - [3 x 2 + 5] (v) Differentiation of a function of a function :

or Dy = 2 2
[3( x + Dx + 2 xDx ) + 5] - [3 x + 5] 2 dy æ dy ö æ du ö
We can write = çè ÷ø ´ çè ÷ø
dx du dx
= 3( Dx 2 + 2 x Dx)
dy
2
Ex. 4 Given, y = (ax + b )2 , evaluate .
Dy 3( Dx + 2 xDx) dx
Step -III =
Dx Dx Sol. Method -I
= 3( Dx + 2 x ) Substituting (ax + b) = u
du d ( ax + b )
æ Dy ö Then = =a
Step - IV lim çè ÷ø = 3(0 + 2 x ) dx dx
D x ® 0 Dx

= 3 × 2x dy d (ax + b) 2 d (u )2
and = = = 2u
= 6x du du du

dy d [3 x 2 + 5] dy dy du
That is = = 3 ´ 2 x 2 -1 + 0 = 6 x \ = ´ = 2u ´ a = 2(ax + b) a
dx dx dx du dx
Similarly, we can get = 2a (ax + b)

d Method - II y = ( ax + b )2
(i) ( ax n + b) = anx n -1
dx = a 2 x 2 + b 2 + 2abx
Where n may have any value.
dy d é 2 2
(ii) Differentiation of sum or difference of two or more function: Then = a x + b 2 + 2abx ù
dx dx ë û
Suppose y = u ± v, u and v are function of x.
d 2 2 d d
dy d du dv = ( a x ) + (b 2 ) + (2abx)
Then = (u ± v ) = ± dx dx dx
dx dx dx dx
(iii) Differentiation of the product of two functions : d ( x )2 dx
Suppose y = uv = a2 + 0 + 2ab
dx dx
dy d (uv ) dv du
Then = = u +v = a2 × 2x + 2ab × 1
dx dx dx dx
= 2a (ax + b)
www.crackjee.xyz
10 MECHANICS
DIFFERENTIATION OF TRIGONOMETRIC FUNCTION

dy
Let y = sin x, then find .
dx
Step -I Substitute x + Dx in place of x in the function,
we have y + Dy = sin (x + Dx)
Step - II Dy = ( y + Dy ) - y
= sin( x + Dx ) - sin x

æ x + Dx + x ö æ x + Dx - x ö
= 2 cos ç ÷ sin ç ÷
è 2 ø è 2 ø

æ 2 x + Dx ö æ Dx ö
= 2cos ç ÷ sin ç ÷
è 2 ø è 2 ø

æ 2 x + Dx ö æ Dx ö
Dy 2cos ç ÷ sin ç ÷
Step - III = è 2 ø è 2 ø
Dx Dx

æ Dx ö
sin ç ÷
æ 2 x + Dx ö è 2 ø
= cos ç ÷
è 2 ø Dx
2

ì æ Dx ö ü
ï sin ç ÷ï
Dy ì æ 2 x + Dx ö ü ï è 2 øï
Step - IV lim = í lim cos ç ÷ ý ´ í lim ý
Dx ®0 Dx î Dx®0 è 2 ø þ ï Dx®0 æ Dx ö ï
ïî ç 2 ÷ ï
è ø þ

æ 2x + 0 ö
= cos ç ÷ ´1
è 2 ø
= cos x
dy d d cos x
(i) Thus = (sin x ) = cos x (ii) = - sin x
dx dx dx
Similarly, we can get
d d
(iii) (tan x ) = sec 2 x (iv) (cot x ) = -cosec 2 x
dx dx

d d
(v) (cosec x ) = - cot x cosec x (vi) (sec x ) = tan x sec x
dx dx

d –1 1 d –1 1
(vii) dx (sin x) = (viii) dx (cos x ) = -
1 - x2 1 - x2

DIFFERENTIATION OF LOGARITHMIC AND EXPONENTIAL FUNCTIONS

d (e x )
(i) Let y = ln x (ii) = ex
dx

dy d (ln x) 1
Then = =
dx dx x
www.crackjee.xyz
Mathematics Used in Physics 11
OR
dy
Ex. 5 Given y = sin 2x, then find . dy d
dx = (sin 2 x )
dx dx
dy d
Sol. dx
=
dx
(sin 2 x ) d
sin(2 x ) ´
d (2 x)
=
d (2 x ) dx
d (sin u ) du = cos 2 x ´ 2 = 2cos 2 x
= ´
du dx
dy
Here u = 2x Ex. 6 Given y = ln (ax + b), then find .
dx
du d (2 x ) d (sin u ) dy d
\ = = 2 and = cos u
dx dx du Sol. dx
=
dx
ln( ax + b)

dy d ln(ax + b) d (ax + b)
\ = cos u ´ 2 = ´
dx d (ax + b) dx
= 2 cos 2x a
=
ax + b

SUCCESSIVE DIFFERENTIATION

dy
is called differentiation of y w.r.t. x or first derivative of y.
dx

d2y d æ dy ö
= ç ÷ is called second derivative of y and so on.
2 dx è dx ø
dx

dy d 2 y d3 y d2y d
Ex. 7 y = x 3 – 4 x 2 + 5 , find , and . and = (3x 2 - 8 x)
dx dx 2 dx 3 dx 2 dx
d d
dy d 3 = (3 x 2 ) - (8 x)
Sol. = ( x - 4 x 2 + 5) dx dx
dx dx
= 3 × 2x – 8
= 6x – 8
d ( x3 ) d d
= - (4 x 2 ) + (5)
dx dx dx d3y d
Also = (6 x - 8)
= dx3 dx
3x 2 - 4 ´ 2 x + 0
d d
= 3x2 - 8 x = (6 x ) - (8)
dx dx
= 6
GEOMETRICAL MEANING OF DIFFERENTIATION
Figure represents the graph of y versus x. Choose two points P(x, y) and
Q(x + Dx, y + Dy) on the curve. The slope of line PQ is given by
Dy
= tan q
Dx
If point Q approaches P, the slope tan q of the line PQ approaches the
slope of the tangent at P. Thus we have

æ Dy ö dy
lim ç
Dx ÷ø
=
Dx ®0 è dx
= tan q
æ dy ö
i.e., ç ÷ at any point of the curve gives slope of the tangent at that point.
è dx ø Figure 0.12
www.crackjee.xyz
12 MECHANICS

INTEGRATION
The integration is the inverse operation of differentiation. Thus if the differentiation of a function f (x)
w.r.t. x is f ¢(x), then the integration of f '(x) w.r.t. x will be f (x). That is
d
f ( x) = f '( x)
dx

Then ò f '( x)dx = f ( x)


Constant of integration : The differentiation of a constant is zero, therefore in integration there may
involve constant of integration, let C. Thus we can write ò f '( x ) dx = f ( x ) + c

(i) ò dx = x + c ò
(vii) sin x dx = - cos x + c

(ii) ò c dx = c ò dx = cx ò
(viii) cos x dx = sin x + c

x n+1
(iii) ò x n dx =
(n + 1)
+c (n ¹ -1) ò
2
(ix) sec x dx = tan x + c

òx ò
-1 2
(iv) dx = ln x + c (x) cosec x dx = - cot x + c

(v) ò (u + v) dx =ò (u) dx + ò (v) dx + c ò


(xi) cosec x cot x dx = - cosec x + c
where u and v are function of x.

òe ò
x
(vi) dx = e x + c (xii) sec x tan x dx = sec x + c

INTEGRATION BY SUBSTITUTION dx
Ex. 9 Find value of ò .
(ax + b)
Ex. 8 Find value of ò (ax + b)3/2 dx . Sol. Substitute (ax + b) = z in the given function.
Sol. Substitute (ax + b) = z in the given function. d
( ax + b)
dz
Also =
d dz dx dx
Also, ( ax + b) =
dx dx dz
or a =
dx
dz
or a = dz
dx \ dx =
a
dz
dx
\
ò (ax + b)
( dz / a )
ò
dx =
a and =
z
dz
and
ò (ax + b) 3/ 2
dx = òz 3/ 2
´
a =
1
a òz
dz

1 3/ 2
=
a ò
z dz =
1
a
ln z + c

Substitute back for z, we get


æ3 ö
çè +1÷ø dx
ò (ax + b)
2 1
= 1 z = ln( ax + b ) + c
+c a
a æ3 ö
çè + 1÷
2 ø Ex. 10 Find value of ò sin2 x dx.
1 z5 / 2 Sol. Substitute 2x = z in the given function.
= +c
a 5/2 d dz
Also (2 x ) =
Substitute back for z, we get dx dx
2 dz
ò (ax + b) 3/ 2
dx =
5a
(ax + b )5 / 2 + c or 2 =
dx
www.crackjee.xyz
Mathematics Used in Physics 13
dz 1
\ dx = = ( - cos z ) + c
2 2
Substitute back for z, we get
æ dz ö
and ò sin 2x dx = ò sin z ç ÷
è 2ø
ò sin 2x dx =
1
( - cos 2 x) + c
2
1
=
2òsin z dz

DEFINITE INTEGRAL
When a function is integrated between lower and upper limit, it is called definite integral.
If a and b are the lower and upper limits of variable x, then
b

ò f '( x)dx = [ f ( x) + c]ba


a

= { f (b) + c} - { f (a) + c}
= f(b) – f(a)
Here constant of integration c get cancelled so there is no need to place it in definite integration.
i i
di dz /( - R ) i
Ex. 11 Given, di
ε–L
dt
= iR , find the value of i at any time t in
and ò ( e - iR ) = ò z
æ 1 ö
= ç
è - R ÷ø òz
dz

terms of constant e, L and R. At t = 0, i = 0. 0 0


0

di æ 1ö i
Sol. We have, e-L = iR = çè - ÷ø ln z 0
dt R

di æ 1ö i
or ( e - iR ) = L = çè - ÷ø ln(e - iR) 0
dt R
æ 1ö
di dt = çè - ÷ø { ln(e - iR) - ln(e - 0)}
or = ....(i) R
( e - iR) L
Integrating both sides of the equation (i), we get æ 1 ö (e - iR )
= çè - ÷ø ln ....(ii)
di R e
ò
dt
( e - iR)
=
L ò t
dt
Here limit of time varies from 0 to t and corresponding limits of i varies
from 0 to i.
and RHS òL
0
æ 1ö t 1
= çè ÷ø t 0 = (t - 0) =
L L
t
L
i t From equations (i) and (ii), we have
di dt
\ ò ( e - iR ) = ò L
æ 1 ö æ e - iR ö
çè - ÷ø ln çè ÷ =
t
0 0 R e ø L
For integration of LHS, substitute e – iR = z.
æ e - iR ö R
ln ç - t
d
( e - iR ) =
dz or è e ÷ø = L
Also
di di
Rt
æ e - iR ö -
dz dz or çè ÷ = e L
or (0 – R ) = \ di = e ø
di (-R) tR
e -
or i = (1 - e L )
R
PARTIAL DIFFERENTIATION
In physics, we often come across quantities which depend on two or more variables. For example
electric potential V depends on x, y coordinates as : V = xy. For given pair of value of x and y, V has a
definite value. If we differentiate quantity V w.r.t. x keeping y constant, then it is known as partial
¶V
differentiation and represented by . Similarly differentiation of V w.r.t. y keeping x is constant is
¶x
¶V
represented by .
¶y
www.crackjee.xyz
14 MECHANICS

¶V ¶ ( xy )
Thus = =y
¶x ¶x

¶V ¶ ( xy )
and = =x
¶y ¶y
In general if f is a function of n variables x1, x2,......xn, then partial differential coefficient of f with respect
¶f
to x1, keeping all the variables except x1 as constant can be written as .
¶x1

Ex. 12 Given f = ( )
a x 2 + y 2 + bz 2 , where a and b are con- ¶f
¶y
=
¶ é
¶y ë
( )
a x 2 + y 2 + bz 2 ù
û
stants. Find partial differentiation of f w.r.t. x, y and z.
= a × 2y = 2ay,
Sol.
¶f
¶x
=
¶ é
¶x ë
( )
a x 2 + y 2 + bz 2 ù
û and
¶f
=
¶ é
( )
a x 2 + y 2 + bz 2 ù
¶z ¶z ë û
= a × 2 x = 2 ax,
= b × 2z = 2bz
SOME USEFUL PHYSICAL CONSTANTS
(i) Acceleration due to gravity, g = 9.8 m/s2
(ii) Speed of light, c = 3 × 108 m/s
N - m2
(iii) Universal gravitation constant, G = 6.67 ´10-11
kg 2
TERRESTRIAL CONSTANTS
(i) Mean radius of Earth, R = 6.37 × 106 m ; 6.4 ´106 m
(ii) Mass of the Earth, M = 6 × 1024 kg
(iii) Mass of the Sun, Ms = 1.99 × 1030 kg
(iv) Mass of the Moon, M m = 7.34 × 1022 kg
(v) Earth – Moon distance = 3.84 × 108 m
(vi) Earth – Sun distance = 1.49 × 1011 m.
Nature of curve
The nature of curve along which the particle move can be understood by making the relationship
between x, y coordinates of the curve. Some of the common curves are;

1. Straight line : y = mx + c

Figure 0.13

2. Circle : x2 + y2 = R2

Figure. 0.14
www.crackjee.xyz
Mathematics Used in Physics 15
3. Parabola : The following may be the equations of a parabola.
y

y = kx2

(i)
Figure 0.15

x2 y2
4. Ellipse : + =1
a2 b2

b2
Also eccentricity, e = 1-
a2

2b 2
Latus rectum, AB =
a

5. Rectangular hyperbola : xy = constant Figure 0.16

Figure 0.17

6. Sinusoidal curve :
(a) y = A sinx

Figure 0.18
www.crackjee.xyz
16 MECHANICS
(b) y = A cosx

Figure 0.19

Some Important Constants


Name Symbol Value
Speed of light in vacuum c 2.9979 ´ 10 ms– 1
8

Charge of electron e 1.602 ´ 10 –19 C


Gravitational constant G 6.673 ´ 10 –11 N m2 kg–2
Planck constant h 6.626 ´ 10 –34 J s
Boltzmann constant k 1.381 ´ 10 –23 J K– 1
Avogadro number NA 6..022 ´ 10 23 mol–1
Universal gas constant R 8.314 J mol –1 K– 1
Mass of electron me 9.110 ´ 10 –31 kg
Mass of neutron mn 1.675 ´ 10 –27 kg
Mass of proton mp 1.673 ´ 10 –27 kg
Electron-charge to mass ratio e/mr 1.759 ´ 10 11 C/kg
Faraday constant F 9.648 ´ 10 4 C/mol
Rydberg constant R 1.097 ´ 10 7 m–1
Bohr radius a0 5.292 ´ 10 –11 m
Stefan-Boltzmann constant s 5.670 ´ 10 –8 W m–2 K–4
Wien’s Constant b 2.898 ´ 10 –3 m K
e0 8.854 ´ 10 –12 C2 N –1 m–2
Permittivity of free space
1/4p e0 8.987 ´ 10 9 N m 2 C2
Permeability of free space 4p ´ 10 –7 T m A–1
m0
@ 1.257 ´ 10 –6 Wb A–1 m–1
Other useful Constants
Name Symbol Value
Mechanical equivalent of heat J 4.186 J cal –1

Standard atmospheric pressure 1 atm 1.013 ´ 10 5 Pa


Absolute zero 0K –273.15° C
Electron volt 1 eV 1.602 ´ 10 –19 J
Unified Atomic mass unit 1u 1.661 ´ 10 –27 kg
Electron rest energy mc2 0.511 MeV
Energy equivalent of 1 u 1 uc2 931.5 MeV
Volume of ideal gas(0° C and 1 atm) V 22.4 L mol–1
Acceleration due to gravity
g 9.78049 ms –2
(sea level, at equator)
www.crackjee.xyz
www.crackjee.xyz
18 MECHANICS
Definitions Explanations and Derivations
1.1 FUNDAMENTAL QUANTITIES
The physical quantities which are independent of other quantities are called fundamental quantities.
Example : mass, length, time etc.
1.2 DERIVED QUANTITIES
The physical quantities which are derived from fundamental quantities are known as derived quantities.
Example : area, velocity, force etc.
1.3 THE SI SYSTEM OF UNITS
In 1971, General Conference of Weights and Measures introduced a logical and rationalised system of
units known as International System of Units, abbreviated as SI in all language. In this system, there are
seven fundamental quantities and two supplementary quantities.
Fundamental quantities and their units

S .No. Phys ical quantity Unit S ymbol


1 Length metre m

2 M as s kilogram kg
3 Time s econd s

4 Temperature kelvin K
5 Electric current ampere A

6 Luminous intens ity candela cd


7 A mount of subs tance mole mol

Supplementary quantities and their units


S .No. Phys ical quantity Unit S ymbol
1 Plane angle radian rad
2 Solid angle s teradian sr

Rules of writing unit


1. In writing the unit of any quantity, small letters must be used for symbol of unit. Example : m,
m/s etc.
2. Symbol are not followed by a full stop.
3. If any unit is named after a scientist, its initial letter of a symbol is to be capital. Example N
(Newton), W (Watt), K (Kelvin) etc.
4. The full name of a unit always begins with a small letter even if it is named after a scientist.
Example : 5 N or 5 newton.
5. Symbols do not take plural form.
Some practical units
There are some practical units which are simultaneously used with SI units.
(i) 1 fermi = 10–15m (ii) 1 angstrom (Å) = 10–10m
(iii) 1 nanometer (nm) = 10 –9m (iv) 1 micron ( mm ) = 10 –6m
(v) 1 light year = 9.46 × 1015 m (vi) 1 astronomical unit (AU)
(vii) 1 parsec = 3.03 × 10 18m = 1.496 × 1011m
(viii) 1 amu = 1.66 × 10 –27kg (x) 1 tonne = 1000 kg
(xi) 1 lunar month = 27.3 days (xii) 1 solar day = 365.25 average solar days
(xiii) 1 shake = 10– 8s = 366.25 sidereal days
www.crackjee.xyz
Units and Measurements 19
1.4 DEFINITIONS OF SI UNITS
(i) Metre (m) : One metre defined as the length of the path travelled by light in vacuum in 1/(299,
792,458) of a second. (1983)
(ii) Kilogram (kg) : One kilogram is the mass of prototype [a certain platinum-iridium cylinder]
preserved at the International Bureau of Weights and Measures, at Severs, near Paris. (1889)
(iii) Second (s) : One second is the duration of 9,192,631,770 periods of the radiation corresponding
to the transition between the two hyperfine levels of the ground state of the cesium-133
atom. (1967)
(iv) Ampere (A) : One ampere is that constant current which , if maintained in two straight parallel
conductors of infinite length, of neglegible circular cross-section, and placed 1 metre apart in
vacuum would produce between these conductors a force equal to 2 × 10–7 newton per metre
of length. (1946)
(v) Kelvin (K) : One Kelvin is the fraction 1/(273.16) of the thermodynamic temperature of the
triple point of water. (1967)
(vi) Candela (Cd) : One candela is the luminous intensity, in a given direction, of a source that
emits monochromatic radiation of frequency 540 × 1012 hertz and that has a radiant intensity
of 1/683 watt per steradian in that direction.
(vii) Mole (mol) : One mole is that amount of substance which contains as many elementary
entities as there are atoms in 0.012 kg of carbon-12 isotope. The entities may be atoms,
molecules, ions etc.
The two supplementary SI units are defined as follows
(i) Radian (rad) : 1 radian is the angle subtended at the centre of a circle by an arc equal in length
to the radius of the circle.

Arc r
Thus q = = = 1rad
Radius r
(ii) Steradian (sr) : 1 steradian is the solid angle subtended at the centre of a sphere by a surface
of the sphere equal in area to that of a square, having each side equal to the radius of the
sphere. Figure. 1.1

Surface area
Thus w =
Radius2
2
r
= 2 = 1 sr
r
Definition of some practical units
(i) Light year : It is the distance travelled by light in vacuum in one year. Thus Figure. 1.2
1 light year = Speed of light in Vacuum × 1 year
= 3 × 108 × (365.25 × 24 × 60 × 60)
or 1 ly = 9.46 × 1015 m
(ii) Astronomical unit : It is the average distance of earth from the sum (centre to centre).
1 astronomical unit = 1 AU = 1.496 × 1011 m
(iii) Parsec (parallactic second) : It is defined as the distance at which an arch of length 1 AU
subtends an angle of 1 second of arc. If r is the distance, then
l l
q = or r=
r q
1AU
Thus 1 parsec =
1"
1.496 ´1011
= = 3.08 × 1016 m
æ p 1 1 ö
ç ´ ´ ÷
è 180 60 60 ø Figure. 1.3
Also 1 parsec = 3.26 ly.
www.crackjee.xyz
20 MECHANICS

1.5 ADVANTAGES OF SI SYSTEM


(i) SI is a coherent system of units. All derived units can be obtained by simple multiplication or
devision of fundamental units without introducing any numerical factor.
(ii) SI is a rational system of units. It uses only one unit for a given physical quantity. For example
all forms of energy are measured in joule, heat energy in calories and electrical energy in watt
hour.
(iii) SI is a metric system. The mulitples and subsultiples of SI units can be expressed as powers of
10.
(iv) SI is an absolute system of units. It does not used gravitational units. The use of ‘g’ is not
required.
(v) SI is an internationally accepted system of units.

1.6 DIMENSIONS OF A PHYSICAL QUANTITY


The dimensions of a physical quantity are the powers to which the unit of fundamental quantities are
raised to represent that quantity.
Dimensions of fundamental quantities
S .No. Phys ical quantity Dimens ion
1 Length [L]

2 M as s [M ]
3 Time [T]

4 Temperature [K]
5 Electric current [A]

6 Luminous intensity [Cd]


7 Amount of s ubs tance [M ol]

Note: Two supplementary fundamental quantities that is plane angle and solid angle have no
dimensions.

Dimensional equation : The equation obtained by equating a physical quantity with its dimensions
formula is called dimensional equation of the given physical quantity. Example : The dimensional
equation of momentum is
[Momentum] = [MLT–1]
Dimensions of some physical quantities

S.No. Physical Quantity Relation with Other Quantities Unit Dimensional Formulae

1. Force Mass × Acceleration N [MLT –2 ]


2. Work Force × Displacement J [ML2T–2]
Force
3. Pressure N/m2 [ML–1 T–2]
Area
Force
4. Force constant N/m [ML0T–2]
Distance
Force × distance 2
5. Gravitational constant G Nm2/kg 2 [M–1 L3T –2 ]
Mass2
6. Impulse of force Force × Time Ns [MLT –1 ]
Force
7. Stress N/m2 [ML–1 T–2]
Area
www.crackjee.xyz
Units and Measurements 21
Change in dimension
8. Strain –––– [M0L0 T0]
Original dimension
Stress
9. Modulus of elasticity N/m2 [ML–1 T–2]
Strain
Force
10. Surface tension N/m [ML0 T–2 ]
Length
Force×distance
11. Coefficient of viscosity N-s/m 2 [ML–1 T–1]
Area × velocity
Heat
12. Latent heat J/kg [M0L2 T–2]
Mass
13. Electric charge Current × time C [M0L0 TA]
Work
14. Electric potential J/C or V [ML2 T–3 A–1]
Charge
Potential
15. Resistance R ohm (W) [ML2 T–3 A–2]
Current
Charge
16. Capacitance C farad (F) [M–1 L–2 T4 A2]
Potential
Potential
17. Inductance L henry (H) [ML2 T–2 A–2]
Current/time
Force
18. Magnetic field B tesla (T) [ML0 T–2 A–1]
Charge × velocity
Energy
19. Plank’s constant h J-s [ML2T–1]
Frequency
q1q2
20. Permittivity, e e= A2C2N–1 m–2 [M–1L–3T4A2]
2
Fr
4 p.F
21. Permeability, µ m= N/A2-m [MLT –2A–2]
I1I 2l

Note:
1. Some of physical quantities have no dimensions (dimensionless). Example : plane angle,
solid angle, specific gravity, strain, refractive index.
2. Quantities having same dimensions
(a) Momentum and impulse
(b) Work, energy, torque
(c) Pressure, stress and modulus of elasticity.
3. Dimensionless physical parameters : Reynolds number, Mack number, refractive index.

L
4. CR, and LC have dimensions of time.
R
Principle of homogeneity of dimensions
According to this principle, the dimensions of all the terms occuring on both sides of the equation
must be same.

Uses of dimensions
1. Conversion of unit of one system to another : It is based on the fact that product of numerical
value contained in and the unit of physical quantity remains constant, that is, larger unit has
smaller magnitude or n [u] = constant.
www.crackjee.xyz
22 MECHANICS
If a physical quantity has dimensional formula [ MaLbTc] and units of that quantity in two
systems are[M1a L1b T1c] and [M2a L2b T2c] respectively, then
n1 [u1] = n2 [u2]
[u1 ]
\ n2 = n1
[u2 ]
a b c
éM ù éL ù éT ù
or n2 = n1 ê 1 ú ê 1 ú ê 1 ú
ë M 2 û ë L 2 û ë T2 û
where n1 and n2 are numerical values in first and second system of units.
2. To check the correctness of a physical relations : This is based on the principle of homogeneity
of dimensions.
3. Deriving the relation among the physical quantities : By using the principle of homogeneity of
dimensions, we can derive an expression of a physical quantity if we know the various factors on
which it depends.
Let physical quantity X depends on other quantities P, Q and R, then we can write
X = k Pa Qb R c
where k is a dimensionless constant, whose value can be determined by experiment or otherwise,
but not by dimensions. By equating dimensions of both sides of equation, we can get required
relation between the quantities.
4. Finding the dimensions of constants : It is based on homogeneity of dimensions.
Limitations of dimensional analysis
1. The method of dimensional analysis does not give any information about the constant k.
2. It fails to derive the relation if any quantity depends on more than fundamental quantities (in
2T cos q
mechanics three fundamental quantities). Example : Capillary rise h = , here h depends
r rg
on four quantities of mechanics. We have only three equations.
1 2
3. It fails to derive the relation like s = ut +
at .
2
4. The method fails to derive relationship which involves trigonometric, logarithmic or exponential
functions.
1.7 ORDER OF MAGNITUDE
The order of magnitude of a physical quantity is that power of 10 which is closest to its magnitude. It
gives an idea about how big magnitude. It gives an idea about how big and how small a given physical
quantity is?
A number N can be expressed as N = n × 10x.
If 0.5 < n £ 5 , then x will be the order of magnitude of N.
1.8 RULES OF SIGNIFICANT FIGURES
Significant figures
In any measurement, the reliable digits plus the first uncertain digit are known as significant figures.
e.g., The length of an object measured to be 475.2 cm. Here the digits 4, 7, 5 are reliable while the
digit 2 is uncertain. The significant figures in above measured values are four.
All the non-zero digits are significant. All zeros between non-zero digits are significant e.g., 2.005 has four
significant figures.
The zeros on the right of decimal point but left of the first non-zero digit are not significant. Trailing zeros
in a number with a decimal are significant e.g., 0.0002500 have four significant figures.
The trailing zeros in a number without a decimal point are not significant e.g., 2500 have two significant figures.
In addition or subtraction, the final result should have as many decimal places as are there in the number
with the least decimal places, e.g., in the sum of 2.50 cm and 4.275cm. Their arithmetic sum is 6.775 cm
but the least precise measurement is 2.50 cm. So, the final result should be 6.78 cm.
www.crackjee.xyz
Units and Measurements 23
In multiplication or division, the final result should have as many as significant figures as are the figures least
significant in any number taking part in the operation.
e.g. 1.25 × 2.0 = 2.50, should be 2.5.

Note: The number of significant figures do not change with the change in system of units e.g. The
observed length 5.208 cm has four significant figures.
In different units, it can be written as 52.08 mm, 0.05208 m or 52080 µm. All these numbers have the same
number of significant figures i.e., four.

FORMULAE USED
1. n1u1 = n2u2
2. n1[M1aL1bT1c] = n2[M2aL2b T2c]

a b c
é M1 ù é L1 ù é T1 ù
3. n2 = n1 ê ú ê ú ê ú
ë M 2 û ë L 2 û ë T2 û

EXAMPLES BASED ON UNITS DIMENSIONS AND SIGNIFICANT FIGURES


Example 1. Express 1 parsec in terms of metre. Write its order Example 4. A new unit of length is chosen such that the speed
of magnitude. of light in vacuum is unity. What is the distance between the Sun
Sol. 1 parsec =3.08 × 1016 m and the Earth in terms of the new unit, if light takes 8 min and
20 sec to cover this distance? [NCERT]
Here 0.5 < 3.08 < 5
Sol. Velocity of light = c
\ Order of magnitude = 16 Ans.
=1 new unit of length s–1
Example 2. Write the order of magntitude of the following Time taken by light of Sun to reach the Earth = t = 8
measurements : (i) 45,710,000 m (ii) 0.00000 532 kg min 20 s = 8 × 60 + 20 = 500 s
Sol. \ Distance between the Sun and Earth,
(i) 45,710,000 = 4.571 × 107 m x = c × t = 1 new unit of length s–1 × 500 s
Here 0.5 < 4.571 < 5, = 500 new units of length.
\ Order of magnitude is = 7 Ans. Example 5. The density of a material in CGS system is 8 g/
(ii) 0.00000532 = 0.532 × 10–5 kg cm3. In a system of units in which unit of length is 5 cm and unit of
mass is 20 g, what is the density of material ?
Here 0.5 < 0.532 < 5, Sol. The dimensions of density are [ML–3].
\ Order of magnitude is = –5 Ans. We know that n1 [u1] = n2 [u2]
Example 3. A calorie is a unit of heat or energy and it equals [u1 ]
about 4.2J, where 1J = 1 kgm2s–2. Suppose, we employ a system \ n2 = n1 [u ]
2
of units in which the unit of mass equals a kg, the unit of length
equals bm and the unit of time is gs. Show that a calorie has 1 3
é M1 ù é L2 ù
a magnitude of 4.2 a –1b –2g 2 in terms of new units. = n1 ê ú ê ú
[NCERT] ë M 2 û ë L1 û
Sol. 1 calorie = 4.2 J = 4.2 kg m 2 s–2 æ 1g ö æ 5cm ö
3
If a kg = new unit of mass = 8 ç ç ÷
è 20 g ÷ø è 1cm ø
1 = 50
Then, 1kg = new unit of mass
a i.e., the density of material in new system is 50 unit.
= a-1 new unit of mass Example 6. State the number of significant figures in the
Similarly, 1m = b -1 new unit of length following:
1s = g-1 new unit of time (i) 0.007 m2 (ii) 2.64 × 1024 kg
Now, 1 calorie = 4.2 (a-1 new unit of mass) (iii) 0.2370 g/cm3 (iv) 6.320 J
(b -1 new unit of length)2 (v) 6.032 N/m2 (vi) 0.0006032 m2
(g-1 new unit of time)–2 (vii) 2.000 m (viii) 5100 kg
= 4.2 a–1 b –2g2 unit of energy. (ix) 0.050 cm
www.crackjee.xyz
24 MECHANICS
Sol. 2 pt 2 pt ö
(i) One : 7 (ii) three : 2, 6, 4 (d) ( æ
y = a 2 ç sin
è
)T
+ cos
T ø
÷
(iii) Four : 2, 3, 7, 0 (iv) Four : 6, 3, 2, 0
(a = maximum displacement of the particle,
(v) Four : 6, 0, 3, 2 (vi) Four : 6, 0, 3, 2
v = speed of the particle, T = time-period of motion).
(vii) Four : 2, 0, 0, 0 (viii) Four : 5, 1, 0, 0
Rule out the wrong formulas on dimensional grounds.
(ix) Two : 5, 0
Sol. According to dimensional analysis an equation must be
dimensionally homogeneous.
Note: 5100 kg is the measured value, and so it has four significant
2 pt
figures. If it simply a numerical value 5100, then significant number in (a) y = a sin
T
it will be two. Here, [L.H.S.] = [y] = [L]
Example 7. Solve the following and express the result to an é 2p t ù
and [R.H.S.] = ê a sin
T úû
appropriate number of significant figure :
ë
(i) Add 6.2 g, 4.33 g and 17.456 g.
(ii) Subtract 63.54 kg from 187.2 kg é Tù
= ê Lsin ú = [ L]
(iii) 75.5 × 125.2 × 0.51 ë Tû
So, it is correct.
2.13 × 24.78 2.51× 10-4 × 1.81× 107
(iv) (v) (b) y = a sin vt
458.2 0.4463 Here, [y] = [L]
Sol. and [a sin vt] = [L sin (LT –1.T)]
= [L sin L]
(i) 6.2 g + 4.33 g + 17.456 g = 27.986 g
So, the equation is wrong.
The result should be rounded off to first decimal place.
\ = 28.0 g æaö t
(c) y = ç ÷ sin
(ii) 187.2 kg – 63.54 kg = 123.66 kg T
è ø a
The result should be rounded off to first decimal place. Here, [y] = [L]
\ = 123.7 kg
éæ a ö t ù éL Tù
(iii) 75.5 × 125.2 × 0.51 = 4820.826 and ê ç
T ÷ sin a ú = ê T sin L ú
The result should be rounded off to two significant figures, because ëè ø û ë û
of (0.51).
= é LT -1 sin TL-1 ù
\ = 4800 ë û
So, the equation is wrong.
2.13 ´ 24.78
( ) 2 πt ö
(iv) = 0.115193 æ 2πt
458.2 y = a 2 ç sin + cos
T ÷ø
(d)
è T
The result should be rounded off to three significant figure because
of (2.13). Here, [y] = [L], éë a 2 ùû = [ L ]
\ = 0.115
é 2πt 2πt ù é T Tù
2.51 ´ 10-4 ´ 1.81 ´ 107 and êsin + cos = êsin T + cos T ú
(v) = 10.1795 × 103 ë T T úû ë û
0.4463 Q [LHS] = [RHS]
The result should be rounded off to three significant figure because = dimensionless
of 1.81 So, the equation is correct.
\ = 10.2 × 103 Example 9. When the planet Jupiter is at a distance of 824.7
million km from Earth, its angular diameter is measured to be
Example 8. A book with many printing errors contains four
35.72" of arc. Calculate the diameter of Jupiter? [NCERT]
different formulas for the displacement y of a particle undergoing
Sol. r = 824.7 × 106 km
a certain periodic motion: [NCERT]
35.72 p
2 pt q = 35.72" = × radian
(a) y = a sin 60 ´ 60 180
T
(b) y = a sin vt 35.72 p
Q l = r q = 824.7 × 106 × × km
æaö t 60 ´ 60 180
(c) y = ç ÷ sin
èT ø a = 1.429 × 105 km.
www.crackjee.xyz
Units and Measurements 25
Example 10. A great physicist of this century (P.A.M. Dirac) Sol. Dimensions of b = dimensions of x2
loved playing with numerical values of Fundamental constants = L2
of nature. This led him to an interesting observation. Dirac found x2
that from the basic constants of atomic physics (c, e, mass of Dimensions of = dimensions of P
at
electron, mass of proton) and the gravitational constant G, he
could arrive at a number with the dimension of time. Further, é x2 ù
\ Dimensions of a = dimensions of ê ú
it was a very large number, its magnitude being close to the ëê Pt ûú
present estimate on the age of the universe ( ~15 /billion years).
From the table of fundamental constants in this book, try to see L2
= = M -1T 2
if you too can construct this number (or any other interesting ML2T -3T
number you can think of ). If its coincidence with the age of the Hence dimensions of a × b = L2 × M–1 T2
universe were significant, what would this imply for the constancy = M –1L2T 2 Ans.
of fundamental constants? [NCERT]
Example 12. Check by dimensions whether the equation
Sol. Using basic constants such as speed of light (c), charge on
rg
electron (e), mass of electron (me), mass of proton (mv) and gravitational tan q = is correct, where r is the radius of the path, g acceleration
constant (G), we can construct the quantity, v2
due to gravity and v speed of the vehicle q is the banking angle.
æ e2 ö
2 Sol.
1
t =ç ÷ ´ The dimensions of LHS = [M 0L0T 0]
ç 4πe 0 ÷ m m 2c 3G
è ø p e rg
The dimensions of RHS =
v2
é e2 ù é 1 e2 2 ù
Now ê ú=ê r ú = é Fr 2 ù é L . LT -2 ù
êë 4πe 0 úû êë 4πe 0 r úû ë û
2
= ê -1 2 ú
êë ( LT ) úû
= [MLT –2 . L 2 ] = [ML3T –2 ] = [M 0 L 0T 0 ]
Since both sides of equation has same dimensions, therefore given equation
2
é ML3T -2 ù is dimensionally correct.
ë û
\ [t] = 3
= [T]
[ M ][ M ]2 éë LT -1 ùû é M -1L3T -2 ù
ë û Note:
Clearly, the quantity t has the dimensions of time.
Put G = 6.67 ´ 10 –11 Nm 2 kg–2, rg
c = 3 ´ 10 8 m/s 1. The equation tan q = is not physically correct. The correct
v2
e = 1.6 ´ 10 –19 C, m e = 9.1 ´ 10–31 kg,
m = 1.67 ´ 10–27 kg v2
equation is tan q = .
rg
1
and = 9 ´109 Nm 2 C 2 2. The dimensionally correct equation need not be physically correct.
4pe 0
Example 13. The velocity (v) of water waves may depend upon
2 their wavelength l, the density of water r and the acceleration due
é
( ) ùúû
2
9 -19
ê9 ´10 ´ 1.6 ´10 to gravity g. Find the relation between these quantities by method
\t = ë
of dimensions.
( ) ´ (3 ´10 ) ´ 6.67 ´10
2 3
1.67 ´10-27 ´ 9.1´ 10-31 8 -11
Sol. Suppose, v = k l arb g c
= 2.13 ´ 10 16 s
Substituting dimensions of all quantities in a above equation, we get
2.13 ´ 1016 [M 0 LT –1 ] = [L]a [ML–3]b [LT –2]c
= years = 0.667 ´ 109 years. or [M 0 LT –1 ] = [MbLa – 3b + c T–2c]
7
3.156 ´ 10
Equating dimensions of both sides, we get
= 0.667 billion years.
This time is slightly less than the age of the universe (» 15 billion years). b = 0
It implies that the values of the basic constants of physics should a – 3b + c = 1
change with time because the age of the universe increases with time. and – 2c = – 1

Example 11. Find the dimensions of a × b in the relation 1 1


After solving we get, a= , b = 0 and c =
2 2
b – x2
P= ; where P is power, x is distance and t is time. The required relation is v = k l1/ 2 g1/ 2 Ans.
at
www.crackjee.xyz
26 MECHANICS

In Chapter Exercise 1.1

1. Why length, mass and time are choosen as base 7. An artificial satellite is revolving around a planet of mass
quantities in mechanics? [NCERT Exemplar] M and radius R, in a circular orbit of radius r. From kepler’s
2. Given an example of the following : [NCERT Exemplar] third law about the period of a satellite around a common
(a) A physical quantity which has a unit but no central body, square of the period of revolution T is
dimensions.
proportional tothe cube of the radius of the orbit r. Show
(b) A physical quantity which has neither unit nor
dimensions.
k r3
(c) A constant which has a unit. using dimensional analysis, that T = , where k is a
(d) A constant which has no unit. R g
3. If the unit of force is 100 N, unit of length is 10 m and dimensionless constant and g is acceleration due to gravity.
unit of time is 100 s, what is the unit of mass in this system [NCERT Exemplar]
of units? [NCERT Exemplar] 8. The number of particles crossing a unit area perpendicular
Ans. 10 5 kg
2 –5 –2 to x-axis in unit time is given by
4. In the expression P = El m G where E, m, l and G denote
energy, mass, angular momentum and gravitational n2 - n1
constant, respectively. Show that P is a dimensionless n=–D x -x
2 1
quantity. [NCERT Exemplar]
Ans. [M0L 0T 0] where n1 and n2 are number of particles per unit volume
5. If velocity of light c, Planck’s constant h and gravitational for the values of x meant to x1 and x2. Find the dimensions
constant G are taken as fundamental quantities, then of the diffusion constant D. Ans. [D] = [L2T–1]
express mass, length and time in terms of dimensions of 9. A body of mass m is moving in a circle of radius r with
three quantities. [NCERT Exemplar] angular velocity w . Find the expression for centripetal
ch hG hG force acting on it by the method of dimensions.
Ans. m = k , L=k 3 , T =k 5 Ans. F = Kmw2r.
G c c
6. A new system of units is proposed in which unit of mass a
is a kg, unit of length is b m and unit of time is g s. How 10. Find the dimensions of in the equation; F = a x + bt2,
b
much will 5 J measure in this new system?
where F is force, x is distance and t is time.
5g 2 Ans. L–1/2T 2
[NCERT Exemplar] Ans. 2
ab

1.9 ERRORS IN MEASUREMENT


Every measurement is limited by the reliability of the measuring instrument and skill of the person
making the measurement. If we repeat a particular measurement, we usually do not get the same result
every time. This imperfection in measurement can be expressed in two ways :
Accuracy and precision
Accuracy refers to the closeness of observed values to its true value of the quantity while precision
refers to closeness between the different observed values of the same quantity. High precision does
not mean high accuracy. The difference between accuracy and precision can be understand by the
following example : Suppose three students are asked to find the length of a rod whose length is known
to be 2.250 cm. The observations are given in the table.

Student Measurement-1 Measurement-2 Measurement-3 Average length


A. 2.25 cm 2.27 cm 2.26 cm 2.26 cm
B. 2.252 cm 2.250 cm 2.251 cm 2.251 cm
C. 2.250 cm 2.250 cm 2.251 cm 2.250 cm
It is clear from the above table, that the observations taken by student A are neither precise nor
accurate. The observations of student B are more precise. The observations of student C are precise as
well as accurate.
Error : Each instrument has its limitation of measurement. While taking the observation, some uncertainty
gets introduced in the observation. As a result, the observed value is somewhat different from true
value. Therefore,
Error = True value – Observed value
www.crackjee.xyz
Units and Measurements 27
Systematic errors : The errors which tend to occur of one sign, either positive or negative, are called
systematic errors. Systematic errors are due to some known cause which follow some specified rule. We
can eliminate such errors if we know their causes. Systematic errors may occur due to zero error of an
instrument, imperfection in experimental techniques, change in weather conditions like temperature, pressure
etc.
Random errors : The errors which occur randomly and irregularly in magnitude and sign are called
random errors. The cause of random errors are not known. If a person repeat the observations number of
times, he may get different readings every time. Random errors have almost equal chances for positive and
negative sign. Hence the arithmetic mean of large number of observations can be taken to minimize the
random error.
Mean value of a quantity : Since the probability of occurrence of positive and negative errors are same, so
the arithmetic mean of all observations can be taken as the true value of a observed quantity.
If a1, a2, ................an are the observed values of a quantity, then its true value a can be given by
a1 + a2 + ................ + an
a = amean =
n
n
1
= n å ai
i =1
The absolute errors in individual observations are:
Da1 = a - a1
Da2 = a - a2
...........................
Dan = a - an
The mean absolute error is defined as
| Da1 | + | Da2 | +.................+ | Dan |
Da =
n
n
1
= ` å
n i=
| Dai |
1
Thus the final result of the observed quantity can be expressed as a = a ± Da .
It is clear from above that any observed value can by (a - Da ) £ a £ (a + Da ) .

Relative or fractional error : The ratio of the mean absolute error to the true value of the quantity is called
relative error.
Da
Thus relative error =
a
Percentage error : If relative error is expressed in percentage is called percentage error.
Da
Thus percentage error = ´ 100
a

Note: Absolute error has the unit of quantity. But relative error has no unit.

Combination of errors
4 3
Let we want to get the volume of sphere, V = pr . There involves multiplication of radius three times.
3
The measurement of radius has some error, then what will be error in calculating the volume of sphere?
The error in final result depends on the individual measurement as well as the mathematical operations
involved in calculating the result. Following rules are used to evaluate maximum possible error in any
computed quantity.
www.crackjee.xyz
28 MECHANICS
1. Error in addition
Let Z = X + Y. Suppose ± Dx be absolute errors in X and ± Dy be the absolute error in Y, then
we have
Z + Dz = (X ± Dx) + (Y ± Dy)
= (X + Y) ± (Dx + Dy)
\ Dz = (Z + Dz) – Z
= ± (Dx + Dy)

Note: The maximum value of Dx or Dy can be least count of the instrument used.
Example : x = 2.20 cm, Dx will be 0.01 cm.

RULE : The maximum possible error in the addition of quantities is equal to the sum of their
absolute error.

Dz é Dx + Dy ù
% error in Z, × 100 = ±ê ú × 100
Z ë X +Y û

2. Error in subtraction
Let Z = X–Y
Z + Dz = (X ± Dx) – (Y ± Dy)
= (X – Y) ± (Dx m Dy)
\ Dz = (Z +Dz) – Z
= ± Dx m Dy
For maximum possible error Dx and Dy must be of same sign.
\ Dz = ± (Dx + Dy)

RULE : The maximum possible error in subtraction of quantities is equal to the sum
of their absolute errors.

Dz é Dx + Dy ù
% error in Z, × 100 = ± ê ú.
Z ë X -Y û

3. Error in product
Let Z = XY
Z + Dz = (X ± Dx) (Y ± Dy)
= XY ± DxY ± XDy ± DxDy
\ Dz = (Z + Dz) – Z
= ± (DxY + XDy) ± DxDy
If Dx and Dy are both small, their product be very small, therefore we can neglect it.
\ Dz = ± (DxY + XDy)
The maximum fractional error in Z,
Dz é Dx Dy ù
= ±ê +
Z ëX Y úû
and maximum percentage error in Z,
Dz é Dx Dy ù
× 100 = ±ê + × 100
Z ëX Y úû

RULE : The maximum fractional error in the product is equal to the sum of the fractional
errors in the individual quantities.

Note: The product Dx Dy can not be neglected if the errors in x and y are order of 10% or more.
The product can be neglected, if the error in x and y are 1% or little more than this (say 2 to 3%).
www.crackjee.xyz
Units and Measurements 29
4. Error in quotient or division
X
Let Z =
Y
X ± Dx
Then Z + Dz =
Y ± Dy

æ Dx ö
X ç1 ±
è X ÷ø
=
æ Dy ö
Y ç1 ±
è Y ÷ø

-1
X æ Dx öæ Dy ö
= ç1 ± ÷ç 1 ± ÷
Y è X øè Y ø

X æ Dx öæ Dy ö
=
Y ç1 ± X ÷ç1 m Y ÷
è øè ø

æ Dx öæ Dy ö
Z + Dz = Z ç1 ± ÷ç 1m
Y ÷ø
or
è X øè

Dz æ Dx ö æ Dy ö
= ç1 ± 1m
X ÷ø çè Y ÷ø
1+
Z è

Dx Dy Dx Dy
= 1± m ± .
X Y X Y
Dx Dy
As the term and are small, so their product can be neglected. The maximum fractional
X Y
error is given by

Dz æ Dx Dy ö
= ±ç +
Z è X Y ÷ø
And maximum possible percentage error in Z,

Dz æ Dx Dy ö
\ × 100 = ± ç +
Y ÷ø
× 100
Z è X
RULE : The maximum fractional error in the quotient is equal to the sum of their individual
fractional errors.
5. Error in the power of a quantity
Let Z = Xn
Z + Dz = (X ± Dx)n
n
æ Dx ö
= Xn ç 1 ± ÷
è X ø

æ Dx ö
; Z ç1 ± n X ÷
è ø

Z + Dz æ Dx ö
= ç1 ± n
X ÷ø
or
Z è

Dz Dx
or 1+ = 1±n
Z X
www.crackjee.xyz
30 MECHANICS

Dz Dx
\ = ±n
Z X
Maximum percentage error in Z

Dz æ Dx ö
× 100 = ± n ç ´ 100 ÷
Z è X ø

RULE : The fractional error in the quantity with n powers is n times the fractional
error in that quantity.

Note: Here n may have any value. It may be a whole number, fraction, positive or negative.

X aY b
General case : If Z = , the maximum possible fractional error in Z,
Wc

Dz é Dx Dy Dw ù
=± êa X + b Y + c W ú
Z ë û
The maximum possible percentage error
Dz é Dx Dy Dw ù
× 100 = ± ê a +b +c × 100
Z ë X Y W úû
The above used algebraic method in many operations become difficult to operate. In such situations
we can used differential method to find the error.

Differential method of calculation of errors


X aY b
1. Let Z = k
Wc
where k is a constant.
Taking logarithms of both sides of equation, we get
ln Z = ln k + a ln X + b ln Y – c ln W
Now differentiating partially the above expression, we have
dz dx dy dw
= a +b -c
Z X Y W
We can write above equation by writing D in place of d;
Dz Dx Dy Dw
= a +b -c
Z X Y W
Errors calculated by above equation, is known as mathematical error. But our interest is in
finding the maximum possible error.

Dz é Dx Dy Dw ù
\ × 100 = ± ê a +b +c ×100
Z ë X Y W úû

W
2. Let Z =
(X +Y )
Taking logarithms of both sides of above equation, we have
ln Z = ln W – ln (X + Y)
Differentiating partially, we get
dz dw d( x + y ) d w (d x + d y )
= - = -
Z W (X +Y ) W X +Y
www.crackjee.xyz
Units and Measurements 31
(a) The maximum possible error in Z

Dz é Dw Dx + Dy ù
= ±ê + ú
Z ë W (X +Y)û

W
(b) For Z =
X -Y

Dz é Dw Dx + Dy ù
= ±ê +
Z ëW X - Y úû

XY
3. Let Z =
U +V
Taking logarithms of both sides of above equation, we have
ln Z = ln X + ln Y – ln (U + V)
Differentiating partially, we get
dz d x d y d (u + v )
= + -
Z X Y U +V

dx dy (du + dv )
= + -
X Y U +V
The maximum possible error in Z

Dz é Dx Dy ( Du + Dv ) ù
(a) = ±ê + +
Z ëX Y U + V úû

XY Dz é Dx Dy Du + Dv ù
(b) For Z = , =±ê + +
U -V Z ëX Y U - V úû
4. Let Z = sinx
Differentiating partially, we get
dz = cos x dx
or Dz = cos x Dx

Dz cos x 1 - sin 2 x
and = Dx = Dx
Z sin x sin x

Dz 1 - z2
or = Dx
Z Z
1.10 INDIRECT METHODS OF MEASURING LARGE DISTANCES
Triangulation method
It is based on the relationship between sides and angles of a triangle.
(i) Height of an accessible object :
Let h be the height of the tree or tower to be measured. Place a sextant at a distance x from the
foot and measure the angle of elevation. If q is the angle of elevation of the top, then

h
tan q =
x
Figure. 1.4
or h = x tan q
knowing the distance x, the height h can be determined.
www.crackjee.xyz
32 MECHANICS
(ii) Height of an inaccessible object :
Let h is the height of the mountain to be measured. Measure angles of elevation of the top of the
mountain by using a sextant. If q1 and q2 are the angles taken from C and D respectively, then
in DABC,

x
cot q1 =
h

d+x
and in D ABD, cot q2 =
h

d
\ cot q2 - cot q1 =
h

é d ù
or h = ê ú
ëê cot q 2 - cot q1 ûú
Figure. 1.5
Parallax method
Parallax : It is the apparent shift in the position of an object with respect to another when we shift
our eye sidewise.
To understand it, hold a pencil P at a distance s from eyes. Look towards the pencil first by left eye
L (closing right eye) and then by the right eye R(closing left eye). The position of the pencil
appears to change with respect to the background. This shift in position of the object is called
parallax. The distance between the two points of observation is called basis. In the figure the
distance LR between the eyes is the basis, and angle q is called parallax angle or parallactic angle.
(i) Distance of moon or near by heavenly body
To measure the distance s of the moon, we observe it simultaneously from two different positions
Figure. 1.6
on the earth, separated by a large distance. We select a distant star (for reference) whose positon
can be taken approximately same during the observations. In figure q1 and q2 are the angular
positions (from reference star) of the moon taken simultaneously from A and B respectively.
The parallactic angle q = q1 + q2
arc b
= =
radius s

b
\ s =
q
(ii) Distance of a nearly star
If figure N is the near by star whose distance s is to be found. Taking a distance star F(fixed star)
whose position remains fix for all positions of the earth in its orbital motion. When the earth is at
Figure. 1.7 positon A, let q1 is the angle subtended by star from reference AF and q2 when earth is at the
position B.
The parallactic angle q = q1 + q2

Arc AB
= =
Radius s

AB
\ s= .
q
The distance AB is the diameter of the orbital plane of earth around sun. This method is
useful for the determination of distances which are less than 100 light years away from
Figure. 1.8
the earth.
www.crackjee.xyz
Units and Measurements 33

Note: For a star more than 100 light years away, the parallax angle is so small that it
cannot be measured accurately.

Reflection method
In this method waves are to be send towards the obstruction and time of reflected waves
is to be noted. If t is the time the waves taken and v is the speed, then
vt
s = .
2 Figure. 1.9
(i) LASER method : The word LASER stands for Light Amplification by Stimulated
Emission of Radiation. The Laser light can travel long distances without fading its
intensity. A laser beam is sent towards the object (moon etc) whose distance is to
be measured and its reflected pulse is received. If t is the time elapsed between the
instants the laser beam is sent and return back, then the distance of the moon from
the earth is given by

ct
s = , where c is the speed of light, which is 3 × 108 m/s. Figure. 1.10
2
(ii) RADAR method : The word RADAR stands for Radio Detection and Ranging. A
radar can be used to measure accurately the distance of an aeroplane etc.
Radiowaves are sent from a transmitter which after reflection from the aeroplane
are detected by the receiver. If t is the time between the instants the radiowaves
are sent and received, then the distance of the aeroplane is given by

ct
s= , where c = 3 × 108 m/s is the speed radio waves.
2
(iii) SONAR method : The word SONAR stands for Sound Navigation And Ranging.
Figure. 1.11
This method is used to detect the submarines or to find the depth of sea. Ultrasonic
waves (waves of frequency greater than 20000 Hz) are sent into the sea; they are
reflected by the bottom of sea and received by the receiver. Transmitter and receiver
are set into the ship. If t is the time taken by the ultrasonic waves from the instant
of transmission to receiving, then depth of sea is given by

vt
s =
2
where v is the speed of sound waves in water, which is nearly 1498 m/s.
1.11 INDIRECT METHOD OF MEASURING SMALL DISTANCES
Figure. 1.12
Atomic radius by Avogadro's hypothesis
When large number of atoms are packed together, some empty spaces are left between them. According
to Avogadro's hypothesis, the actual volume occupied by the atoms is two third of the volume of the
substance.
If M be the molecular mass of a substance, then number of atoms in it is N (Avogadro number).
Consider m gm of the substance.
m
The number of moles in the substance = ,
M
mN
and the number of atoms in it =
M
If r is the radius of the each atom, then volume of the atoms in the substance
www.crackjee.xyz
34 MECHANICS

æ mN ö 4 3
Vatoms = ç ÷´ pr … (i)
è M ø 3
If r is the density of the substance, then its volume

m
Vsubstance = …(ii)
r
According to Avogadro's hypothesis
2
Vatoms = Vsubstance
3
Figure. 1.13 æ mN ö 4 3 2m
or ç ÷´ pr =
è M ø 3 3r
1/ 3
é M ù
or r = ê 2 p Nr ú
ë û

1.12 VERNIER CALLIPERS AND CCREW GAUGE


Introduction : The metre scale which commonly used in practice is the simplest instrument for
measuring length. By metre scale we can measure upto 1 mm because the length of the smallest

æ 1 ö
division made on the scale is 1 mm. In order to measure still smaller lengths accurately upto ç ÷ th or
è 10 ø

æ 1 ö
ç 100 ÷ th of a millimeter, the instruments commonly used in laboratory are :
è ø
1. Vernier callipers
2. Screw gauge
Vernier callipers
It was invented by French Mathematician Pierre Vernier and hence the instrument is named Vernier. It

æ 1 ö
is used to measure accurately upto ç ÷ th of millimeter..
è 10 ø
0 1 2
Vernier callipers comprises of two scales, viz., main scale S and vernier scale V which is
0 10 called auxiliary scale. The main scale is fixed but the vernier scale is movable. The
Vernier scale Main scale
divisions of vernier scale are usually a little smaller in size than the smallest division on
Figure. 1.14 the main scale. It also has two jaws, one attached with the main scale and other with the vernier scale.
The purpose of jaws are to grip the object between them. Vernier has a strip, which slide along with
vernier scale, over the main scale. This strip is used to measure the depth of hollow object.
Construction : The main parts of Vernier callipers are :

Figure. 1.15
www.crackjee.xyz
Units and Measurements 35
Vernier constant (VC) : Suppose the size of one main scale division is S and that of one vernier scale
division is V units. Also suppose that length of n vernier division is equal to the length of (n – 1)
division of main scale. Thus we have
(n – 1) S = n V
or nS – S = n V
S
or S–V =
n
The quantity (S – V) is called vernier constant (VC).
Least count : The smallest value of a physical quantity which can be measured accurately with an
instrument is called the least count (LC) of the instrument. For vernier callipers, its least count is equal
to its vernier constant. Thus
S
Least count = S – V =
n
or LC = [length of one division of main scale – length of one division of vernier scale]
length of one division of main scale
= number of divisions on vernier scale

Reading a vernier : Suppose that while measuring the length of an object, the positions of the main
scale and vernier scale are shown in figure. First of all we read the position of the zero of the vernier on
the main scale. As it is clear that the zero position of the vernier lies between 4.2 cm and 4.3 cm. In fact
the objective of the instrument is to measure accurately small length ‘x’ which lies between zero mark
of the vernier scale and 4.2 cm mark on the main scale. We can see that x can not be read directly on the
main scale, as this scale is smaller than the smallest division on the main scale.
Next find out which division on the vernier scale exactly coincides with some division of the main
scale. In figure it is quite clear that 3rd division of vernier scale coincide with some division of the main
scale. Therefore, the value of length x will be given by the relation :
4.2 cm + x + 3 vernier scale division = 4.2 cm + 3 main scale division 4 5 6
Þ x = 3 main scale division – 3 vernier scale division
0 3 10 Main scale
= 3 (1 main scale division – 1 vernier scale division) Vernier scale
= 3 (0.10 – 0.09) Figure. 1.16
= 3 × 0.01
= 0.03
\ The required length is given by
L = 4.2 + 0.03
= 4.23 cm
Thus, Length of the object = Main scale reading + n (LC)
Where n, vernier division exactly coinciding with some main scale division.
Determination of zero error : When jaws of the vernier are made touch each other and the zero mark
of the vernier scale coincide with the zero mark of the main scale, there will no zero error in the
instrument. However, in practice it is never so. Due to wear and tear of the jaws and due to some
manufacturing defect, the zero mark of the main scale and vernier scale may not coincide, it give rise to
an error, is called zero error. It may be positive or negative zero error.
Positive and negative zero error : When the zero mark of the vernier scale lies towards
the right side of the zero of main scale when jaws are in contact, the measured length 0 1 2 cm
will be greater than the actual length. Because of this fact the zero error is called
positive zero error. On the other hand, when zero mark of the vernier scale lies towards 0 10 Vernier scale Main scale
the left side of the zero of the main scale when jaws in contact with each other, the Jaws in contact
length of the object measured by the instrument will be less than the actual length of No zero error of the vernier calliper
the object. Because of this reason is called negative zero error. Figure. 1.17
True reading = Observed reading – Zero error with proper sign
www.crackjee.xyz
36 MECHANICS
Correction for positive zero error : Let us see the vernier callipers, shown in figure. When its jaws are
in contact with each other, suppose 3rd vernier division coincides with the any of the division of main
scale. Thus we have
0 1 2 cm Zero error = + [0.00 cm + 3 (LC)]
= + [0.00 + 3 × 0.01 cm]
Main scale
= + 0.03 cm
Jaws in contact Third division coinciding Correct reading = observed reading – (0.03 cm)
Figure. 1.18 = observed reading – 0.03 cm
Correction for negative error : Let us see the vernier callipers shown in figure when jaws are in
contact with each other, suppose sixth division of vernier coincides with any of the division of main
scale. Thus we have
0 1 2 cm Zero error = [0.00 – (10 – 6) LC]
= [0.00 – 4 × 0.01]
6
0 10 = – 0.04 cm
Jaws in contact Sixth division coinciding \ Correct reading = observed reading – (– 0.04cm)
Figure. 1.19 = observed reading + 0.04 cm
Screw gauge
It is used to measure small lengths like diameter of wire or thickness of sheet etc. It consists of a U-
shaped metal frame.
T R A main scale which graduate in millimeter or half a millimeter. The main scale also
S H
P Q 5
0 Reference
called pitch scale. A cap fits on to the screw and carries on its inner edge H, 50 to 100
Rachet
95 line equal divisions, is called circular or head scale. The object whose length to be
Studs
Main Circular
measured is gripped between the studs P and Q by moving the rachet R.
Scale scale Pitch : It is defined as the linear distance moved by the screw forward or backward
when one complete rotation is given to the circular cap.
U-shaped metal frame
Least count (LC) = Pitch / [Total number of divisions on the circular scale]
Figure. 1.20
Zero error : When the studs P and Q of the screw gauge are brought in contact without apply induce
pressure and if the zero of the circular scale coincides with the reference line, then there is no zero error,
otherwise there will be zero error.

T R
H 5
P Q S
0 Reference
95
line
Spindle
Sleeve
Thimble

Screw gauge with no zero error

Figure. 1.21
Positive zero error : In this case, the zero of the circular scale lies below the reference line as the gap
between studs P and Q reduces to zero.
Let us determine the magnitude of positive error by taking an example. Suppose in a
T R
PQ S
H screw gauge, (when the gap between P and Q is reduced to zero) the zero line of the
5 Reference
0 line
circular scale is 4 division below the reference line. In other words, the 4th division of
the head scale is in line with the line of graduation. Thus,
zero error = + 4 × (LC)
= + 4 × (0.01 mm)
E
Positive zero error (4 division error) i.e., + 0.004 cm
= + 0.04 mm
Figure. 1.22 Zero correction = – Zero error
It must be remembered that the zero correction whether positive or negative should always be added
algebraically to the observed reading to get the correct reading.
www.crackjee.xyz
Units and Measurements 37
Negative zero error : In this case, the zero of the circular scale lies above the reference T
H
R
line when the gap between the studs P and Q become zero. Under this condition, the PQ S 0 Reference
edge of the circular scale lies to the left hand side of the zero of the main scale. That 95 line
is why it called negative zero error.
Let us determine the magnitude of the negative error with the help of an example.
Suppose on reducing the gap between studs P and Q to zero, the zero line of the
circular scale is 3 divisions above the reference line, i.e., 97th division of the circular E
Negative zero error (3 division error) i.e., – 0.003 cm
scale is in line with the reference line. Figure. 1.23
Thus zero error = (97 – 100) × (LC)
= – 3 × 0.01 mm
= – 0.03 mm
Thus zero error = – 0.03 mm
and zero correction = + 0.03 mm.
Backlash error : Sometimes, in a screw gauge, there may be loose fitting between the screw and the
nut. It is either because of wear and tear of the nut as well as that of the screw or due to some
manufacturing defect. In such an instrument, if the screw is adjusted by turning it in one direction and
then in opposite direction, the linear movement of the screw is not proportional to the circular motion.
This implies that for no change in the gap length between the stud and the screw, the circular scale
reading undergoes some appreciable change resulting in an error, is called backlash error.

FORMULAE USED
1. The true value : If a1, a2, ..............., an are the observed value of a quantity, then its true value
is given by

n
a1 + a2 + ....... + an
a=
n
= å ai
i =1

2. Absolute error = true value – obseved value

or Dai = a – ai

3. Mean absolute error

| Da1 | + | Da2 | +.......+ | Dan | 1 n


Da = = å | Dai |
n n i =1

Da
4. Relative error = , and percentage error
a

Da
= ´ 100
a
5. Error in computed quantity
(i) If ±Dx and ±Dy be the absolute errors in X and Y respectively and

if Z = X + Y, then maximum possible error in Z; Dz = ± ( Dx + Dy )

(ii) If Z = X – Y, then Dz = ± ( Dx + Dy )

(iii) If Z = XY, then Dz = ± é Dx + Dy ù


Z êX Y úû
ë

X Dz é Dx Dy ù
(iv) If Z = , then = ±ê + ú
Y Z ëX Y û
www.crackjee.xyz
38 MECHANICS

(v) If Z = X n, then Dz = ± n é Dx ù
Z êXú
ë û

KX aY b Dz é Dx Dy Dw ù
(vi) If Z = , then = ± êa +b +c
Wc Z ë X Y W úû
The absolute error has the same unit as the quantity itself, but fractional error has no unit.

XY
6. If Z = , then Dz = ± é Dx + Dy + ( Dx + Dy ) ù
X +Y Z êX Y X + Y úû
ë
7. Least count of vernier callipers
L.C. = Length of one division of main scale – length of one division of vernier scale

Length of one division of main scale


or L.C. =
number of divisions on Vernier scale
8. Least count of a screw gauge

Pitch
L.C. =
[total number of divisions on the circular scale]

EXAMPLES BASED ON INSTRUMENTS AND ERRORS


Example 14. It is claimed that two cesium clocks, if allowed There are two significant figures in 0.13. Hence P should also be rounded
to run for 100 years, free from any disturbance, may differ by off to 2 significant figures
only about 0.025 s. What does this imply for the accuracy of the \ P = 3.763 = 3.8 Ans.
standard cesium clock in measuring a time interval of 1 s?
Example 16. Which of the following is the most precise device
[NCERT]
for measuring length?
Sol. Error in 100 years = 0.02 s
(a) a Vernier callipers with 20 divisions on the sliding scale
\ Error in 1 sec
coinciding with 19 main scale division
Dt 0.02 (b) a screw gauge of pitch 1 mm and 100 divisions on the
=
t 100 ´ 365 ´ 2.5 ´ 24 ´ 60 ´ 60 circular scale
0.02 (c) an optical instrument that can measure length within a
=
3.5576 ´ 10 9 wavelength of light ? [NCERT]
= 0.0063 × 10 –9 = 0.63 × 10 –11 Sol.
So, there is an accuracy of 1s is 10 –11s. (a) Least count of Vernier callipers = 1 SD – 1 VD

Example 15. A physical quantity P is related to four 19 1 1


= 1 SD – SD = SD = mm
a 3b 2 20 20 20
observations a, b, c and d as follows : P = .
cd = 0.005 cm
The percentage errors of measurement in a, b, c and d are 1%, 3%, (b) Least count of screw gauge
4% and 2% respectively. What is the percentage error in the
quantity P? If the value of P calculated using the above relation pitch
turns out to be 3.763, to what value should you round off the result? =
no. of division on circular scale
a 3b 2
Sol. Given P = 1
cd
= mm = 0.001 cm
The maximum possible percentage error in P is given by 100
(c) Wavelength of light, l = 10–5 cm = 0.00001 cm
DP é Da Db 1 Dc Dd ù
× 100 = ± ê3 +2 + + × 100 Since most precise device should have minimum least count,
P ë a b 2 c d úû
optical instrument is the most precise one.
1
= ± [3 × 1% + 2 × 3% + × 4% + 2%] Example 17. Answer the following :
2
(a) You are given a thread and a metre scale. How will you
= ± 13%
estimate the diameter of the thread?
or DP = ± 0.13
www.crackjee.xyz
Units and Measurements 39
(b) A screw gauge has a pitch of 1.0 mm and 200 divisions on Example 19. The shadow of a tower standing on a level plane is
the circular scale. Do you think it is possible to increase found to be 100 m longer when sun's altitude is 30° than when it is
the accuracy of the screw gauge arbitrarily by increasing 60°. Find the height of the tower.
the number of divisions on the circular scale? Sol. If h is the height of tower, then
(c) The mean diameter of a thin brass rod is to be measured
by Vernier callipers. Why is a set of 100 measurements of
the diameter expected to yield a more reliable estimate than
a set of 5 measurements only? [NCERT]
Sol.(a) Meter scale can not measure small diameter of thread. No.
of turns of the thread to be wound to get turns closely
one another.
Let l– measured length of windings on the scale which
contains n no. of turns.
l
\ Diameter of thread = .
n
(b) Least count
pitch
= Figure. 1.24
no. of division in circular scale
i.e. least count decreases when no. of division on the circular d
scale increases. Thereby accuracy would increase. but h =
cot q2 - cot q1
practically, it is impossible to take precise reading due to
low resolution of human eye.
100
(c) Large no. of observations (say 100) gives more reliable =
result, because probability of making random error in cot 30° - cot 60°
positive side of a physical quantity would be same that = 50 3 Ans
of in negative side. Therefore, when no. of observations
is large random errors would cancel each other and hence Example 20. When the planet Jupiter is at a distance of 824.7
result would be reliable. million kilometers from the earth, its angular diameter is
Ex ample 1 8. One mole of an ideal gas at standard measured to be 35.72 s of arc. Calculate the diameter of Jupiter.
temperature and pressure occupies 22.4 L (molar volume). What Sol. The distance of Jupiter from the earth
is the ratio of molar volume to the atomic volume of a mole of s = 824.7 × 106 km
hydrogen? (Take the size of hydrogen molecule to be about 1 Å). Angular diameter q = 35.72''
Why is this ratio so large? [NCERT] æ 35.72 ö p
Sol. Volume of one mole of ideal gas, V g = ç
è ÷´ rad
60 ´ 60 ø 180
= 22.4 litre = 22.4 ´ 10 –3m 3


Radius of hydrogen molecule =
2
= 0.5 Å = 0.5 ´ 10 –10 m
Figure. 1.25
4 3
Volume of hydrogen molecule = pr Diameter of Jupiter D = s q
3
æ 35.72 ö p
÷´
( ) = 824.7 × 106 × ç
4 22 3
= ´ 0.5 ´ 10-10 m3 è 60 ´ 60 ø 180
3 7
= 148217.8 km
= 0.5238 ´ 10 –30 m 3
One mole contains 6.023 ´ 10 23 molecules.
Example 21. In a submarine equipped with a SONAR, the
time delay between generation of a probe wave and the reception
\ Volume of one mole of hydrogen,
of its echo after reflection from an enemy submarine is found to be
VH = 0.5238 ´ 10–30 ´ 6.023 ´ 10 23 m 3 77s. What is the distance of the enemy submarine? (speed of
= 3.1548 ´ 10 –7 m 3 sound in water = 1450 m/s)
Sol. The distance of enemy submarine is given by
Vg 22.4 ´10-3
Now, = = 7.1´104
VH 3.1548 ´10-7 vt
s =
The ratio is very large. This is because the interatomic separation 2
in the gas is very large compared to the size of a hydrogen
1450 ´ 77
molecule. = = 55825 m Ans.
2
www.crackjee.xyz
40 MECHANICS
Example 22.One mole of an ideal gas at STP occupies 22.4 L. Mass of nucleus
What is the ratio of molar volume to the atomic volume of a mole Nuclear mass density =
volume of nucleus
of hydrogen? Why is this ratio so large ? Take the radius of hydrogen
molecule to be 1 Å. A/ N
Sol. Radius of a hydrogen molecule or r =
4 3
pr
r = 1 Å = 10–10 m 3
Atomic volume of 1 mole of hydrogen
A
or r =
4 3 4 4
= N × pr = 6.023 × 1023 × p × (10–10)3 N p (r0 A1/ 3 )3
3 3 3
= 25.2 × 10–7 m3 3
=
Molar volume = 22.4 L = 22.4 × 10–3 m3 4pNr03

Molar volume 22.4×10 –3 3


\ = = 0.89 × 104 ; 10 4 =
atomic volume 25.2 ´ 10 –7 4p ´ 6.02 ´ 10 23
´ (1.2 ´ 10 -15 )3
This ratio is so large because the actual size of the gas molecules is = 2.3 × 1017 kg /m3
negligible small in comparision to the intermolecular separation.
Example 23. The unit of length convenient on the unclear Example 24. In four complete revolution of the cap, the
scale is a fermi: 1f = 10 –15 m. Nuclear sizes obey roughly the distance travelled on the pitch scale is 2 mm. If there are 50 divisions
following empirical relation r = r0A1/3; where r is the radius of the on the circular scale, then calculate the least count of the screw
nucleus, A its mass number and r0 is a constant equal to about 1.2 gauge.
f. Show that the rule implies that nuclear mass density is nearly distance travelled on pitch scale
constant for different nuclei. Estimate the mass density of sodium Sol. Pitch =
number of rotations
nucleus.
Sol. Given radius of nucleus, 2mm
= = 0.5 mm
4
r = r0 A1/ 3
pitch
Least count =
Mass number number of divisions on circular scale
Mass of the nucleus =
Avogadro's number 0.5mm
= = 0.01 mm
50
A
=
N

In Chapter Exercise 1.2

1. Times for 20 oscillations of a pendulum is measured as t1 4. The farthest objects in our universe discovered by modern
= 39.6 s; t2 = 39.9 s; t3 = 39.5 s. What is the precision in the astronomers are so distant that light emitted by them
measurement? What is the accuracy of the measurement? takes billions of years to reach the earth. These objects
Ans. ± 0.2 s [NCERT Exemplar] (known as quasars) have many puzzling features which
2. A physical quantity x is related to four measurable have not yet been satisfactorily explained. What is the
quantities a, b, c and d as follows distance in km of a quasar from which light takes 3.0
x = a2b3c5/2d–2 billion years to reach us? [NCERT]
Ans. 284 × 1022 km
The percentage error in the measurement of a, b, c and d
5. In an experiment, refractive index of glass was observed
are 1%, 2%, 3% and 4% respectively. What is the percentage
to be 1.45, 1.56, 1.54, 1.44, 1.54 and 1.53. Calculate (i) mean
error in quantity x? If the value of x calculated on the basis
value of refractive index (ii) mean absolute error (iii)
of the above relation is 2.763, to what value should you
fractional effort (iv) percentage error. Express the result
round-off the result? Ans. x = 2.8 [NCERT Exemplar]
in terms of absolute error and percentage error.
3. Each side of a cube is measured to be 7.203 m. What are
Ans.(i) 1.51 (ii) ; 0.04 (iii) 0.03 (iv) 3%
the total surface area and the volume of the cube to
m = 1.51 ± 0.04, 1.51 ± 3%.
appropriate significant figures ? Ans. 311.3m2, 373.7 m3.
www.crackjee.xyz
Units and Measurements 41
EXAMPLES FOR JEE-(MAIN AND ADVANCED)

Example 1. The speed of light c, gravitational constant G and a+b+c = 0


Plank’s constant h are taken as the fundamental units in a system. – a – 3b + 2c = 0
Find the dimensions of length and time in this new system of unit. and – 2a – 2c = 1
Sol. Dimension of After solving above equations, we get
c = [LT–1] ....(i)
G = [M –1L3T –2] ....(ii) 5 1 1
a = – ,b= and c =
and h = [ML2T –1] ....(iii) 6 2 3
From equation (i), Therefore required relation is
c3 = [L 3T –3 ]
And from equations (ii) & (iii), we have d 1/ 2 E1/ 3
Gh = [L5T –3] T = k Ans.
p 5/ 6
Gh
\ = L2
c3
Example 4. In the gas equation æç p + 2 ö÷ (V – b) = Rq, where
a
1/ 2 1/ 2 - 3/ 2 è ø V
which gives L = G h c
Again from equation (i), q is absolute temperature, p is pressure and V is volume, what are
dimensions of constants a?
length Sol. According to principle of homogeneity of dimensions;
T =
speed Dimensions of b = Dimensions of V
G1/2 h1/ 2 c - 3/ 2 = [L3]
=
c a
1/2 1/ 2 -5/ 2 Dimensions of = Dimensions of p
= G h c Ans. V2

Example 2. By the method of dimension test the accuracy of Hence, dimensions of a = (Dimensions of p) × (Dimensions of V2)
= [ML–1T –2] [L3] 2
mg l 3
the equation δ = , where d is the depression produced in = ML 5 T –2 Ans.
4bd 3Y
the middle of a bar of length l , breath b and depth d, when it is α æ αZö
Example 5. Pressure P varies as P = β exp ç – ÷ , where Z
loaded in the middle with mass m. Y is the Young’s modulus of the è kβq ø
material of the bar.
Sol. The dimensions of left hand side of the equation denotes the distance, kβ is Boltzmann’s constant, q is absolute
LHS d = [L] temperature and a, b are constant. Find dimensions of b.

mg l 3 Sol. We know that power of exponent is a dimensionless number


and RHS =
4bd 3Y aZ
0 0 0
\ kbq = M L T
é M.LT -2 .L3 ù
= ê 3 -1 -2 ú = L
êë L.L .ML .T úû é kbq ù
or a = [M0L0T 0] ê Z ú
Dimensions of LHS = dimensions of RHS, therefore the above equation ë û
is dimensionally correct.
Example 3. Given that the period T of oscillation of a gas bubble é ML2T -2 ù
= [M0L0T 0] ê L ú
from an explosion under water depends upon p, d and E, where p is ëê ûú
the static pressure, d is the density of water and E is the total
= [MLT –2]
energy of explosion, find dimensionally a relation of T.
a
Sol. Suppose, T = k p a d b Ec Dimensions of = Dimensions of P.
b
Substituting dimensions of all quantities in above equation,
[M 0 L 0T 1 ] = [ML–1T –2]a [ML–3]b [ML2T –2] c dimensions of α
\ Dimensions of b = dimensions of P
or [M 0 L 0T 1 ] = [Ma + b + c L– a – 3b + 2c T– 2a – 2c]
Equating the powers of M, L and T, we have
www.crackjee.xyz
42 MECHANICS

where Du = 0.1 cm and Dv = 0.1 cm


[ MLT -2 ]
\ Dimensions of b = = [M0L2T0]
[ ML-1T -2 ] Df é 0.1 0.1 0.1 + 0.1 ù
\ = ±ê + + ú
Example 6. If two resistors of resistances R1 = (4 ± 0.5) W and f ë 10 10 10 + 10 û
R2 = (16 ± 0.5) W are connected (i) in series and (ii) in parallel;
find the equivalent resistance in each case with limits of percentage = ±0.03
error.
and D f = ±0.03 f
Sol. (i) In series, the equivalent resistance
R = R1 + R2 = 4 + 16 = 20 W = ±0.03 ´ 5 = ±0.15cm
DR é DR1 + DR2 ù Thus the focal length of lens f = (5.00 ± 0.15)cm Ans.
\ × 100 = ± ê ú × 100
R ë R1 + R2 û Example 8.While measuring the length of the rod by vernier
callipers the reading on main scale is 6.4 cm and the eight division
é 0.5 + 0.5 ù
= ±ê ú × 100 = ± 5% on vernier is in line with marking on main scale division. If the
ë 4 + 16 û
least count of callipers is 0.01 and zero error –0.04 cm, find the
\ Equivalent resistance in series R = (20 ± 5%)W
length of the rod.
(ii) In parallel, the equivalent resistance
Sol. Lenght of the rod = observed reading – zero error
R1R2 4 ´ 16
R = R +R = = 3.2 W = (Main scale division + Vernier scale division × LC) – Zero error
1 2 4 + 16
= (6.4 + 8 × 0.01) – (– 0.04)
DR é DR1 DR2 DR1 + DR2 ù = 6.4 + 0.08 + 0.04 = 6.52 cm Ans.
\ × 100 = ± ê + + ú
R ë R1 R2 R1 + R2 û × 100 Example 9. The length of a cube is measured with the help of
a vernier callipers. The observations are shown in figure. Find
é 0.5 0.5 0.5 + 0.5 ù length of the cube with these observations.
= ± ê + + × 100 =20.625 %
ë 4 16 4 + 16 úû
\ Equivalent resistance in parallel = (3.2 ± 20.625%) W
Example 7. Graph of position of image vs position of point object
from a convex lens is shown. Find, focal length of the lens with
possible error. [IIT-JEE 2006]

Figure. 1.27
Sol. LC of the vernier callipers = 1 cm / 10 = 0.1 cm
Main scale reading = 9.4 cm
Vernier scale reading coinciding with main scale = 5
Length of the cube = Main scale divisions + Vernier scale divisions × LC
= 9.4 + 5 × 0.01 = 9.45 cm Ans.
Example 10. The circular head of a screw gauge is divided
into 200 divisions and move 1 mm ahead in one revolution. Find
the pitch and least count of the screw gauge. If the same instrument
has a zero error of –0.05 mm and the reading on the main scale in
measuring diameter of a wire is 6 mm and that on circular scale is
45, find the diameter of the wire.
Figure. 1.26 Sol. Pitch = 1 mm
Sol. We know that Number of divisions on circular scale = 200
1 1 1 1 1 pitch
= - = - LC =
f v u 10 -10 number of divisions on circular scale
\ f = 5 cm 1 mm
1 = = 0.005 mm = 0.0005 cm Ans.
1 1 200
Also, = -
f v -u Diameter of the wire
uv = (Main scale reading + Circular scale reading × LC) – Zero error
or f= = 6 mm + 45 × 0.005 – (–0.05)
u+v
= 6 mm + 0.225 mm + 0.05 mm
Df é Du Dv Du + Dv ù
\ = ±ê + + = 6.275 mm Ans.
f ë u v u + v úû
www.crackjee.xyz
Units and Measurements 43

Mechanics
MCQ Type 1 Exercise 1.1
Level - 1 (Only one option correct)
Units and Dimensions 8. Which one of the following does not have the same
1. Which of the following statements is correct about a scalar dimensions
quantity: (a) work and energy
(i) it remain conserved in a process (b) angle and strain
(ii) can never take negative sign
(c) relative density and refractive index
(iii) does not vary from one place to another in space
(iv) has same value for observers with different orientation (d) plank constant and energy
of axis
(a) (i) (b) (ii) 9. The density of a material in CGS system is 8 g / cm3. In a
(c) (iii) (d) (iv) system of a unit in which unit of length is 5 cm and unit of
2. Which of the following is not the unit of time mass is 20 g. The density of material is :
(a) Micro second (b) Leap year (a) 8 (b) 20
(c) Lunar month (d) Parallactic second
(c) 50 (d) 80
3. Temperature can be expressed as a derived quantity in terms
of any of the following 10. In a new system the unit of mass is α kg, unit of length is β
(a) length and mass (b) mass and time m and unit of time is γ s. The value of 1 J in this new system
(c) length, mass and time (d) none of these is [AMU B.Tech. 2012]
4. With the usual notations, the following equation g2/ab2 (b)
(a) ga/b2
1
S1 = u + a (2t − 1) is (c) ag2/b2
abg (d)
2
(a) only numerically correct
(b) only dimensionally correct 11. A boy recalls the relation almost correctly but forgets
(c) both numerically and dimensionally correct where to put the constant c (speed of light). He writes;
(d) neither numerically nor dimensionally correct m0
m= , where m and m0 stand for masses and v for
5. Which of the following readings is the most accurate 1 − v2
(i) 4000 m (ii) 40 × 102 m speed. Right place of c is
(iii) 4 × 103 m (iv) 0.4 × 104 m m0
cm0
(a) (i) (b) (ii) m=
(a) m=
(b)
(c) (iii) (d) (iv) 1− v 2 c 1 − v2
6. If unit of length and force are increased 4 times. The unit m0 m0
m=
(c) m=
(d)
of energy: 2 2
c −v v2
(a) is increased by 4 times 1−
(b) is increased by 16 times c2
(c) is increased by 8 times 12. The equation of state of some gases can be expressed as
(d) remain unchanged  a 
7. Which one of the following is a set of dimensionless physical  P + 2  (V – b) = RT. Here P is the pressure, V is the
 V 
quantities : volume, T is the absolute temperature and a, b, R are
(a) strain, specific gravity, angle constants. The dimensions of a are :
(b) strain, work, couple (a) ML5 T–2 (b) ML–1T2
(c) work, angle, specific gravity
(c) M0L3T0 (d) M0L6T–2
(d) work, energy, frequency

Answer 1 (d) 2 (d) 3 (d) 4 (a) 5 (a) 6 (b)


Key 7 (a) 8 (d) 9 (c) 10 (a) 11 (d) 12 (a)
www.crackjee.xyz
44 Mechanics
13. A spherical body of mass m and radius r is allowed to fall 22. A physical quantity x depends on quantities y and z as
in a medium of viscosity h . The time in which the velocity follows : x = Ay + B tan Cz, where A, B and C are constants.
of the body increases from zero to 0.63 times the terminal Which of the following do not have the same dimensions :
velocity (v) is called time constant (t). Dimensionally t can x and B (b)
(a) C and z–1
be represented by y and B / A (d)
(c) x and A
mr 2  6π mrη  23. The time dependence of a physical quantity P is given by
(a) (b)  
6πη 2  P = P0 exp (αt2) where α is a constant and t is time. The
 g 
constant α :
m
(c) (d) none of the above (a) is dimensionless
6πη rv
(b) has dimensions of T–2
14. The dimensions of universal gravitational constant are
(c) has dimensions as that of P
(a) M–2L2T–2 (b) M–1L3T–2

(d) has dimensions equal to the dimensions of PT–2
(c) ML–1T–2 (d) ML2T–2
24. If L and R denote inductance and resistance respectively,
p F then the dimensions of L / R is :
15. The frequency of vibration of string is given by f =
2 µ
(a) M0 L0 T0 (b) M0L0T1
Here p is number of segments in the string and  is the
(c) M2L0T2 (d) MLT2
length. The dimension formula for µ will be :   
25. The dimensions of the quantity E × B where E represents
(a) [M0LT–1] (b) [M1L2T1] 
–1 0 the electric field and B the magnetic field may be given as:
(c) [ML T ] (d) [M0L2T–1]
16. Dimensions of coefficient of viscosity are (a) MT–3 (b) M2LT–5A–2
(a) M L2 T–2 (b) M L2 T–1 (c) M2LT–3A–1 (d) MLT–2A–2
(c) M L–1 T–1 (d) M LT  A
17. In the formula X = 3 YZ2 , X and Z have dimensions of 26. The electric field is given by E = iˆ + Byjˆ + Cz 2 kˆ . The
3
x
capacitance and magnetic induction respectively. The
SI units of A, B and C are respectively:
dimensions of Y in MKSA system are :
[AMU B.Tech. 2013]
(a) [M–3L–2T–2A–4] (b) [ML–2]
3
–3 –2
(c) [M L A T ] 4 8 (d) [M–3L2A4T4] (a) N − m , V/m2, N/m2-C
18. The physical quantities not having same dimensions are C
(b) V-m2, V/m, N/m2-C
(a) speed and (m0e0)-1/2
(b) torque and work (c) V/m2, V/m, N-C/m2
(c) momentum and Planck’s constant (d)
V/m, N-m3/C, N-C/m
(d) stress and Young’s modulus
=
27. What are the dimensions of A/B in the relation F A x + Bt 2
19. Inductance L can be dimensionally represented as
, where F is the force, x is the distance and t is time?
(a) M L2 T–2A–2 (b) M L2 T–4A–3 [AMU B.Tech. 2013]
(c) M L–2 T–2A–2 (d) M L2 T4A3
(a) ML2T–2 (b) L-1/2T2
20. If the time period (T) of vibration of a liquid drop depends
on surface tension (S), radius (r) of the drop and density (r) (c) L–1/2T–1 (d) LT–2
of liquid, then the expression of T is 28. The potential energy of a particle is given by the expression
x
(a) = k ρ1/ 2 r 3 / S
= k ρr 3 / S (b)
T T U ( x) = – αx + β sin . A dimensionless combination of the
γ
= k ρr 3 / S1/ 2
(c)
T (d) none of these constants a, b and g is : [KVPY - 2012]
α 2
21. If energy E, velocity v and time T are chosen as fundamental α
(a) (b)
βγ βα
units, the dimensions of surface tension will be :
(a) [Ev–2T–2] (b) [Ev–1T–2] γ αγ
(c) (d)
(c) [Ev2T–1] (d) [E2v–1T–1] αβ β

Answer 13 (d) 14 (b) 15 (c) 16 (c) 17 (c) 18 (c) 19 (a) 20 (a)


Key 21 (a) 22 (d) 23 (b) 24 (b) 25 (b) 26 (a) 27 (b) 28 (d)
www.crackjee.xyz
Units and Measurements 45

29. The value of resistance is 10.845 Ω and the value of current is (a) 11 % (b) 14 %
3.23 A. The potential difference is 35.02935 volt. Its value (c) 10 % (d) 19 %
in significant number would be : 34. In a vernier callipers N division of vernier coincide with (N
(a) 35 V (b) 35.0 V – 1) divisions of main scale in which length of a division
(c) 35.03 V (d) 35.029 V in 1 mm. The least count of the instrument in cm is:
Instruments and Errors N (b)
(a) N–1
1
30. The least count of a stop watch is s. The time of 20 1
5 (c) (d) (1 / N) – 1
oscillations of a pendulum is measured to be 25 s. What is 10N
the maximum percentage error in this measurement ? 35. The resistance of a metal is given by R = V/I, where V is
(a) 8 % (b) 1 % potential difference and I is the current. In a circuit the
(c) 0.8 % (d) 16 % potential difference across resistance is V = (10 ± 0.5) V and
current in resistance, I = (2 ± 0.2) A. The value of resistance
31. The refractive index of water measured by the relation µ =
in Ω with percentage error is:
real depth
is found to have values of 1.34, 1.38, 1.32 (a) 5 ± 10 % (b) 5 ± 15 %
apparent depth
and 1.36; the mean value of refractive index with percentage (c) 5 ± 20 % (d) 5 ± 25 %
error : 36. One centimetre on the main scale of a vernier callipers
(a) 1.35 ± 1.48 % (b) 1.35 ± 0 % is divided into 10 equal parts. If 10 divisions of vernier
(c) 1.36 ± 6 % (d) 1.36 ± 0 % coincide with 8 small divisions of the main scale, the least
count of callipers is :
32. A wire has a mass 0.3 ± 0.003 g, radius 0.5 ± 0.005 mm and
length 6 ± 0.06 cm. The maximum percentage error in the (a) 0.01 cm (b) 0.02 cm
measurement of its density is : (c) 0.05 cm (d) 0.005 cm
(a) 1 (b) 2 37. While measuring the length of the rod by vernier callipers
(c) 3 (d) 4 the reading on main scale is 6.4 cm and the eight division
on vernier is in line with marking on main scale division.
a 3b 2 d
33. A physical quantity X is given by X = , the If the least count of callipers is 0.01 and zero error
c1/ 2 –0.04 cm, the length of the rod is
percentage error in the measurement a, b, c and d are 1 %,
(a) 6.50 cm (b) 6.48 cm
3 %, 2 % and 4 % respectively. The maximum percentage
(c) 6.52 cm (d) 6.60 cm
error in X is :

Answer 29 (b) 30 (c) 31 (a) 32 (d) 33 (b)


Key 34 (c) 35 (b) 36 (b) 37 (c)

Level - 2 (Only one option correct)

Units and Dimensions (c) MLT–2 A (d) ML2T–1A2

1. Dimensional formula of magnetic flux is ML2


4. Which of the following units denotes the dimensions ,
(a) ML2T–2A–1 (b) ML0T–2A–2 Q2
where Q denotes the electric charge :
(c) M0L–2T–2 A–3 (d) ML2T–2A3
2. E, M, J and G denote energy, mass, angular momentum Wb
(a) weber (Wb) (b)
and gravitational constant respectively. The dimensions of m2
EJ 2 H
are that of : (c) henry (H) (d)
M 5G 2 m2
5. If the constant of gravitational constant (G) and Plank’s
(a) angle (b) length constant (h) and the velocity of light (c) be chosen as
(c) mass (d) time fundamental units. The dimensions of the radius of gyration
3. Using mass [M], length (L), time (T) and current [A] as is :
fundamental quantities, the dimensions of permitivity is : h1/2 c–3/2G1/2 (b)
(a) h1/2c3/2G1/2
(a) ML–2T2A (b) M–1L–3T4A2 h1/2 c–3/2G–1/2 (d)
(c) h–1/2 c–3/2G1/2
www.crackjee.xyz
46 Mechanics

1 divisions in line with the main scale as 35. The diameter of


6. The dimensions of is that of the wire is
ε0 µ0
(a) 3.73 mm (b) 3.67 mm
(c) 3.38 mm (d) 3.32 mm
(a) velocity (b) time 8. The period of oscillation of a simple pendulum is given by
(c) capacitance (d) distance 
T = 2π , where l is about 100 cm and is known to have
Instruments and Errors g
7. Two full turns of the circular scale of a screw gauge cover 1 mm accuracy. The period is about 2 s. The time of 100
a distance of 1 mm on its main scale. The total number of oscillations is measured by a stop watch of least count 0.1
divisions on the circular scale is 50. Further, it is found
s. The percentage error in g is :
that the screw gauge has a zero error of – 0.03 mm. While
measuring the diameter of a thin wire, a student notes the (a) 0.1% (b) 1%
main scale reading of 3 mm and the number of circular scale (c) 0.2% (d) 0.8%

Answer 1 (d) 2 (a) 3 (b) 4 (c)


Key 5 (a) 6 (a) 7 (c) 8 (c)

Mechanics
MCQ Type 2 Exercise 1.2
Multiple correct options (b) pressure
1. Which of the following are dimensionless? (c) energy density
(a) acceleration due to gravity (d) angular momentum
(b) strain 8. Which of the following group have different dimensions ?
(c) mach number (a) potential difference, EMF, voltage
(d) refractive index (b) pressure, stress, Young’s modulus
2. Which of the following are dimensionless constant? (c) heat, energy, work done
(a) Gravitational constant (d) dipole moment, electric flux, magnetic field
(b) Reynold’s number 9. Which of the following pairs have the same dimension?
(c) Mach number (a) electric flux and q/∈0 (b) electric flux and µ0i
(d) permeability
3. Select the correct statement(s) : h h
(c) and electric flux (d) and magnetic flux
(a) a dimensionally correct equation may be correct e e
(b) a dimensionally correct equation may be incorrect 10. If L, C and R represent inductance, capacitance and
(c) a dimensionally incorrect equation may be correct resistance respectively, then which of the following have
(d) a dimensionally incorrect equation may be incorrect dimensions of time?
4. Which of the following pairs have same dimensions : (a) L/R (b) CR
(a) torque and work
(b) angular momentum and work (c) LC (d) LC/R
(c) energy and Young’s modulus 11. A book with many printing errors contains different formulas
(d) light year and wavelength for the displacement of a particle undergoing periodic
5. The pair(s) of physical quantities that have the same motion :
dimensions, is (are) 2πt
(i) y = a sin
(a) Reynolds number and coefficient of friction T
(b) Latent heat and gravitational potential (ii) y = a sin vt
(c) Curie and frequency of a light wave a t
(d) Planck’s constant and torque (iii) y = sin
t a
dx 1 a 2πt 2πt
6. The expression = sin −1 ; where x and a stand (iv) y = (a √ 2)(sin + cos )
for distance :
2
a −x 2 a x T T
(a) mathematically correct (b) mathematically incorrect where a = maximum displacement of the particle, v = speed
(c) dimensionally correct (d) dimensionally incorrect of the particle, T = time period of motion. Rule out the wrong
7. The dimensions ML–1T–2 are corresponding to formulas on dimensional ground.
(a) modulus of elasticity (a) i (b) ii (c) iii (d) iv
www.crackjee.xyz
Units and Measurements 47
12. If the dimensions of length are expressed as Gxcyhz; where 1 1 1 1
G, c and h are the universal gravitational constant, speed (a)
=x = ,y (b)
=x = ,z
2 2 2 2
of light and Planck’s constant respectively, then
1 3 3 1
(c)
=y = ,z (d) y=− ,z =
2 2 2 2

Answer 1 (b, c, d) 2 (b, c) 3 (a, b, c, d) 4 (a, d) 5 (a, b, c) 6 (b, d)


Key 7 (a, b, c) 8 (a, b, c) 9 (a, d) 10 (a, b, c) 11 (b, c) 12 (b, d)

Mechanics
Reasoning Type Questions Exercise 1.3
Read the two statements carefully to mark the correct option out of the options given below:
(a) Statement - 1 is true, Statement - 2 is true; Statement - 2 is correct explanation for Statement - 1.
(b) Statement -1 is true, Statement - 2 is true; Statement - 2 is not correct explanation for Statement - 1.
(c) Statement - 1 is true, Statement - 2 is false.
(d) Statement - 1 is false, Statement - 2 is true

1. Statement - 1 6. Statement - 1
mass
Dimensional constants are the quantities whose values are In the equation momentum, P = x, the dimensional
area
constant. formula of x is LT – 2.
Statement - 2 Statement - 2

Dimensional constants are dimensionless. Quantities with different dimensions can be multiplied.
7. Statement - 1
2. Statement - 1
If y = A sin (ωt + φ), then dimensions of A are equal to
Number of significant figures in 0.005 is one and that in
dimensions of y.
0.500 is three.
Statement - 2
Statement - 2
sin (ωt + φ) is dimensionless.
This is because zeros are not significant.
8. Statement - 1
3. Statement - 1
Units of Rydberg constant R is m–1.
L / R and CR both have same dimensions.
Statement - 2
Statement - 2 1  1 1 
= R
It follows from Bohr’s formula − ,
L / R and CR both have dimensions of time. λ  n2 n2 
 1 2 
4. Statement - 1 where the symbols have their usual meaning.
Angle and strain are dimensionless. 9. Statement - 1
Statement - 2 Now a days a standard metre is defined in terms of the
wavelength of light.
Angle and strain have no unit.
Statement - 2
5. Statement - 1
Light has no relation with length.
The dimensional formula for relative velocity is same as
that of the change in velocity. 10. Statement - 1
In y = A sin(ωt − kx), ( ωt − kx ) is dimensionless.
Statement - 2
Statement - 2
| Relative velocity | = | change in velocity |
Because dimensions of w= [M0L0T]

Answer 1 (c) 2 (c) 3 (a) 4 (c) 5 (a)


Key 6 (d) 7 (a) 8 (a) 9 (c) 10 (c)
www.crackjee.xyz
48 Mechanics

Mechanics
Passage & Matrix Exercise 1.4
Passages
Passage for Questions. 1 & 2 : called the plasma frequency. To sustain the oscillations, a time
Resistor is used in electric circuit, which opposes the flow of charge. varying electric field needs to be applied that has an angular
When two resistors are put in series in the circuit, they offer total frequency ω, where a part of the energy is absorbed and a part
resistance R = R1 + R2, and when they placed in parallel, they offer the
resistance R = R1R2 / R1 + R2. In any experiment the value of resistances of it is reflected. As ω approaches ωp all the free electrons are set
are found as ; R1 = 100 ± 3 Ω and R2 = 200 ± 4 Ω. to resonance together and all the energy is reflected. This is the
1. The equivalent resistance when resistors are connected in series : explanation of high reflectivity of metals. [IIT-JEE 2011]
(a) 300 Ω (b) 300 ± 7 Ω 3. Taking the electronic charge as ‘e’ and the permittivity as
(c) 300 ± 3 Ω (d) 300 ± 4 Ω ‘ε0’. Use dimensional analysis to determine the correct
2. The equivalent resistance when resistors are connected in parallel
expression for ωp.
(a) 66.7 ± 12 Ω (b) 66.7 ± 7 Ω
(c) 66.7 ± 1.8 Ω (d) none of these Ne mε0
(a) (b)
mε0 Ne
Passage for Questions. 3 & 4 :
A dense collection of equal number of electrons and positive ions Ne2 Ne2
(c) (d)
is called neutral plasma. Certain solids containing fixed positive mε0 mε0
ions surrounded by free electrons can be treated as neutral plasma.
Let ‘N’ be the number density of free electrons, each of mass 4. Estimate the wavelength at which plasma reflection
‘m’. When the electrons are subjected to an electric field, they will occur for a metal having the density of electrons
are displaced relatively away from the heavy positive ions. If the N ≈ 4 × 1027 m–3. Taking ε0 = 10–11 and mass m ≈ 10–30,
electric field becomes zero, the electrons begin to oscillate about where these quantities are in proper SI units.
(a) 800 nm (b) 600 nm
the positive ions with a natural angular frequency ‘ωp’ which is
(c) 300 nm (d) 200 nm

Matrix Matching
5. Column I Column II
A. Curie (p) MLT–2
B. Light year (q) M
C. Dielectric constant (r) Dimensionless
D. Atomic weight (s) T
E. Decibel (t) ML2T2
(u) MT–3
(v) T–1
(w) L
6. Column - I Column - II
A. Force (p) T–1
B. Angular velocity (q) MLT–2
C. Work (r) ML–1T–2
D. Surface tension (s) ML–1T–1
(t) MT–2
7. Column - I Column - II
A. Angular momentum (p) ML–1T–1
B. Torque (q) MT–2
C. Surface tension (r) ML2T–1
D. Coefficient of viscosity (s) ML2T–2

Answer 1 (a) 2 (d) 3 (c) 4 (b) 5 A → p, q ; B→ r, s ; C→ r, s ; D→ r, s


Key 7 A → q ; B→ p ; C→ t ; D→ r 7 A→r; B→s;C→q;D→p
www.crackjee.xyz
Units and Measurements 49
8. Column - I Column - II
A. Energy (p) M1L2T–1
B. Moment of inertia (q) M1L2T0
C. Angular acceleration (r) M1L2T–2
D. Angular momentum (s) M0L0T–2
9. Column - I Column - II
A. Spring constant (p) M1L2T–2
B. Pascal (q) M0L0T–1
C. Hertz (r) M1L0T–2
D. Joule (s) M1L–1T–2
10. Column - I Column - II
A. Capacitance (p) ohm - second
B. Inductance (q) coul2 Joule–1
C. Magnetic induction (r) coulomb - volt–1
(s) newton (ampere - metre)–1
(t) volt - second (ampere)–1
11. Column I Column II
A. Magnetic field intensity (p) Wbm–1
B. Magnetic flux (q) Wb/m2
C. Magnetic potential (r) Wb
D. Magnetic induction (s) Am–1
12. Column I Column II
(A) Capacitance (p) volt (ampere)–1
(B) Magnetic induction (q) volt-sec (ampere)–1
(C) Inductance (r) newton(ampere)–1 (metre)–1
(D) Resistance (s) coulomb2 (joule)–1
13. Column I Column II
(A) Distance between earth & stars (p) micron
(B) Inter-atomic distance in a solid (q) angstrom
(C) Size of the nucleus (r) light year
(D) Wavelength of infrared laser (s) fermi
(t) kilometre
14. Column - I Column - II [IIT 2007]
A. GMeMs (p) (volt)(coulomb) (metre)
G - universal gravitation constant
Me - mass of the earth
Ms - mass of the sun
3RT
B. (q) (kilogram) (metre)3 (second)–2
M
R - universal gas constant
T - absolute temperature
M - molar mass
F2
C. (r) (metre)2 (second)–2
q2 B2
F - force
q - charge
B - magnetic field
GM e
D. (s) (farad)(volt)2(kg)–1
Re
G - universal gravitational constant
Me - mass of the earth
Re - radius of earth
15. Match List I with List II and select the correct answer using the codes given below the lists: [JEE Adv. 2013]
Column - I Column - II
A. Boltzmann constant (p). [ML2T-1]
B. Coefficient of viscosity (q) [ML–1T–1]
C. Planck constant (r) [MLT–3K–1]
D. Thermal conductivity (s) [ML2T–2K–1]
www.crackjee.xyz
50 Mechanics

8 A → r ; B→ q ; C→ s ; D→ p 9 A→r ; B→s ; C→ q ; D→ p 10 A→q,r ; B→p,t ; C→ s


Answer
11 A → s ; B→ r ; C→ p ; D→ q 12 A → s ; B→ r ; C→ q ; D→ p 13 A → r ; B→ q ; C→ s ; D→ p
Key
14 A → p, q ; B→ r, s ; C→ r, s ; D→ r, s 15 A → s ; B→ q ; C→ p ; D→ r

Mechanics
Best of JEE (Main & Advanced) Exercise 1.5
JEE- (Main) Main scale reading :58.5 degree
1. The physical quantities not having same dimensions are Vernier scale reading : 09 divisions
(a) stress and Young’s modulus [AIEEE 2003] Given that 1 division on main scale corresponds to 0.5
(b) speed and (m0e0)–1/2 degree. Total divisions on the vernier scale is 30 and match
(c) torque and work with 29 divisions of the main scale. The angle of the prism
(d) momentum and Planck’s constant. from the above data is: [AIEEE 2012]
2. Out of the following pairs, which one does NOT have (a) 58.77 degree (b) 58.65 degree
identical dimensions? [AIEEE 2005] (c) 59 degree (d) 58.59 degree.
(a) moment of inertia and moment of a force 8. Resistance of a given wire is obtained by measuring the
(b) work and torque current flowing in it and the voltage difference applied
(c) angular momentum and Planck’s constant across it. If the percentage errors in the measurement of the
(d) impulse and momentum. current and the voltage difference are 3% each, then error
3. A body of mass m = 3.513 kg is moving along the x-axis in the value of resistance of the wire is : [AIEEE 2012]
with a speed of 5.00 ms–1. The magnitude of its momentum (a) zero (b) 1%
is recorded as [AIEEE 2008] (c) 3% (d) 6%
(a) 17.6 kg m s–1 (b) 17.565 kg m s–1 9. The dimensions of (m0e0)–1/2are: [AIEEE 2012, 2011]
(c) 17.56 kg m s–1 (d) 17.57 kg m s–1 (a) [L1/2T–1/2] (b) [L–1T]
(c) [LT ] –1 (d) [L1/2T1/2]
4. Two full turns of the circular scale of a screw gauge cover
a distance of 1 mm on its main scale. The total number of 10. A student measured the length of a rod and wrote it as
divisions on the circular scale is 50. Further, it is found 3.50 cm. Which instrument did he use to measure it?
that the screw gauge has a zero error of – 0.03 mm. While [JEE -Main 2014]
(a) A screw gauge having 50 divisions in the circular scale
measuring the diameter of a thin wire, a student notes the
and pitch as 1 mm.
main scale reading of 3 mm and the number of circular scale
divisions in line with the main scale as 35. The diameter of (b) A meter scale
the wire is [AIEEE 2008] (c) A vernier calliper where the 10 divisions in vernier
(a) 3.32 mm (b) 3.73 mm scale matches with 9 divisions in main scale and main
(c) 3.67 mm (d) 3.38 mm scale has 10 division in 1 cm.
5. In an experiment, the angles are required to be measured (d) A screw gauge having 100 divisions in the circular
using and instrument. 29 divisions of the main scale exactly scale and pitch as 1 mm.
coincide with the 30 divisions of the vernier scale. If the 11. The current voltage relation of diode is given by
smallest division of the main scale is half-a-degree (= 0.5°), I = (e1000V/T – 1) mA, where the applied voltage V is in
then the least count of the instrument is : [AIEEE 2009] volt and temperature T is in kelvin. If a student makes an
(a) one degree (b) half degree error measuring ± 0.01 V while measuring the current of 5
(c) one minute (d) half minute mA at 300 K, what will be the error in the value of current
6. A screw gauge gives the following reading when used to in mA? [JEE-Main 2014]
measure the diameter of a wire. [AIEEE 2011] (a) 0.05 mA (b) 0.2 mA
Main scale reading : 0 mm (c) 0.02 mA (d) 0.5 mA
Circular scale reading : 52 divisions L
12. The period of oscillation of a simple pendulum is T = .
Given that 1 mm on main scale corresponds to 100 divisions g
of the circular scale. Measured value of L is 20.0 cm known to 1 mm accuracy
The diameter of wire from the above data is : and time for 100 oscillations of the pendulum is found to
(a) 0.52 cm (b) 0.052 cm be 90 s using wrist watch of 1 s resolution. The accuracy
(c) 0.026 cm (d) 0.005 cm in the determination of g is [JEE Main 2015]
7. A spectrometer gives the following reading when use to (a) 3% (b) 1%
measure the angle of a prism. (c) 5% (d) 2%

Answer 1 (d) 2 (a) 3 (a) 4 (d) 5 (c) 6 (b) 7 (b)


Key 8 (d) 9 (c) 10 (c) 11 (b) 12 (a)
www.crackjee.xyz
Units and Measurements 51

JEE- (Advanced) uncertainty of ± 0.01 mm. Take g = 9.8 m s–2(exact). The


13. A cube has a side of length 1.2 × 10–2m. Calculate its Young’s modulus obtained from the reading is close to
volume. [IIT-JEE 2003] (a) (2.0 ± 0.3) × 1011 Nm–2 [IIT-JEE 2007]
(b) (2.0 ± 0.2) × 1011 Nm–2
(a) 1.7 × 10–6 m3 (b) 1.73 × 10–6 m3
(c) (2.0 ± 0.1) × 1011 Nm–2
–6
(c) 1.70 × 10 m 3 (d) 1.732 × 10–6 m3 (d) (2.0 ± 0.05) × 1011 Nm–2
αz
α − kθ 19. Students I, II and III perform an experiment for measuring
14. In the relation P = e the acceleration due to gravity (g) using a simple pendulum.
β
They use different lengths of the pendulum and / or record
P is pressure, Z is distance, k is Boltzmann constants and θ time for different number of oscillations.
is the temperature. The dimensional formula of β will be The observations are shown in the table.
[IIT-JEE 2004] Least count for length = 0.1 cm
0 2
(a) [M L T ] 0 (b) [M L T1]
1 2 least count for time = 0.1 s [IIT-JEE 2008]
(c) [M1L0T–1] (d) [M0L2T–1] Student Length of the Number of Total time Time
pendulum(cm) oscillations (n) for (n) period
15. A wire has a mass 0.3 ± 0.003g, radius 0.5 ± 0.005 mm and oscillations (s) (s)
length 6 ± 0.006 cm. The maximum percentage error in the
I 64.0 8 128.0 16.0
measurement of its density is [IIT-JEE 2004]
(a) 1 (b) 2 II 64.0 4 64.0 16.0
(c) 3 (d) 4. III 20.0 4 36.0 9.0

16. Screw gauge shown in the figure has 50 divisions on its If EI, EII and EIII are the percentage errors in g, i.e.,
circular scale and in one complete rotation of circular scale
the main scale moves by 0.5 mm. The diameter of a sphere  ∆g 
 × 100  for students I, II and III, respectively, then
is measured using this screw gauge. Two positions of screw  g 
gauge are shown in the figure. The diameter of sphere is
(a) EI = 0 (b) EI is minimum
[IIT-JEE 2006]
(c) EI = EII (d) EII is minimum
20. A Vernier callipers has 1 mm marks on the main scale. It‘s
20 equal divisions on the Vernier scale which match with
16 main scale divisions, For this Vernier callipers, the least
(a) 1.25 mm (b) 1.20 mm count is [IIT-JEE 2010]
(c) 2.25 mm (d) 2.20 mm (a) 0.02 mm (b) 0.05 mm
17. A student performs an experiment for determination of (c) 0.1 mm (d) 0.2 mm
 4π2 l  21. A student uses a simple pendulum of exactly 1 m length to
= g  , l ≈ 1m, and he commits an error of ∆l. For T, determine g, the acceleration due to gravity. He uses a stop
 T2 
  watch with the least count of 1 second for this and records
he takes the time of n oscillations with the stop watch of 40 second for 20 oscillations. For this observation, which
least count ∆Τ and he commits a human error of 0.1s. For of the following statements (s) is (are) true?[IIT-JEE 2010]
which of the following data, the measurement of g will be (a) Error ∆T in measuring T, the time period, is 0.05 second
most accurate ? [IIT-JEE 2006] (b) Error ∆T in measuring T, the time period, is 1 second.
∆l ∆Τ n Amplitude of oscillation
(c) Percentage error in the determination of g is 5%
(a) 5 mm 0.2 s 10 5 mm (d) Percentage error in the determination of g is 2.5%
(b) 5 mm 0.2 s 20 5 mm 22. The density of a solid ball is to be determined in an
(c) 5 mm 0.1 s 20 1 mm experiment. The diameter of the ball is measured with
(d) 1 mm 0.1 s 50 1 mm a screw gauge, whose pitch is 0.5 mm and there are 50
divisions on the circular scale. The reading on the main scale
18. A student performs on experiment to determine the Young’s is 2.5 mm and that on the circular scale is 20 divisions. If
modulus of a wire, exactly 2 m long, by Searle’s method. the measured mass of the ball has a relative error of 2% the
In a particular reading, the student measures the extension relative percentage error in the density is [ IIT-JEE 2011]
in the length of the wire to be 0.8 mm with on uncertainty
(a) 0.9% (b) 2.4%
of ± 0.05 mm at a load of exactly 1.0 kg. The student also
measures the diameter of the wire to be 0.4 mm with an (c) 3.1% (d) 4.2%

Answer 13 (a) 14 (a) 15 (d) 16 (b) 17 (d)


Key 18 (b) 19 (b) 20 (d) 21 (a, c) 22 (c)
www.crackjee.xyz
52 Mechanics

 4MLg  (a) due to the errors in the measurements of d and l are


23. In the determination of Young’s modulus  Y =  the same.
 πld 2  (b) due to the error in the measurement of d is twice that
by using Searle’s method, a wire of length L = 2 m and due to the error in the measurement of l.
diameter d = 0.5 mm is used. For a load M = 2.5 kg, an (c) due to the error in the measurement of l is twice that
extension l = 0.25 mm in the length of the wire is observed. due to the error in the measurement of d.
Quantities d and l are measured using a screw gauge and a (d) due to the error in the measurement of d is four times
micrometer, respectively. They have the same pitch of 0.5 that due to the error in the measurement of l.
mm. The number of divisions on their circular scale is 100. 24. The dimensional formula of magnetic flux is[IIT-JEE 2012]
The contributions to the maximum probable error of the Y (a) [ML2T–2A–1] (b) [ML2T–2A–2]
measurement [IIT-JEE 2012] (c) [MLT A ] –3 –1 (d) [ML0T–2A–1]

Answer 23 (a) 24 (a)


Key
www.crackjee.xyz
Units and Measurements 53

In Chapter Exercise
In Chapter Exercise -1.1 x =
1 1
,y= ,z=–
1
2 2 2
1. Length, mass and time are chosen as base quantities in Putting values of x, y and z in Eq. (i), we get
mechanics because
(i) Nothing is simpler than length, mass and time. m = kc1/2h1/2G–1/2
(ii) All other quantities in mechanics can be expressed in ch
or m = k
terms of length, mass and time G
(iii) Length, mass and time cannot be derived from one (b) Let L ∝ cxhyGz
another. or L = kcxhyGz
2. (a) Plane angle has unit as radian but has no dimensions. where, k is a dimensionless constant.
(b) Strain has neither unit nor dimensions Substituting dimensions of each term, we get
(c) Gravitational constant (G) = 6.67 × 10–11 N-m2/kg2
[M0LT0] = [MT–1]x × [ML2T–1]y × [M–1L3T–2]z
(d) Reynold number is a constant which has no unit.
= [My – zLx + 2y + 3zT–x –y – 2z]
3. Here, force (F) = [MLT–2] = 100 N … (i)
On comparing powers of same terms, we get
Length (L) = [L] = 10 m … (ii)
y–z = 0
Time (t) = [T] =100 s … (iii)
x + 2y + 3z = 1
Substituting values of L and T from Eqs (ii) and (iii) in Eq.
–x – y – 2z = 0
(i) we get
After solving above equation, we get
M × 10 × (100)2 = 100
3 1 1
M × 10 x = – ,y= ,z=
or = 100 2 2 2
100 × 100 Putting values of x, y and z, we get
or M = 100 × 1000 kg
L = kc–3/2h1/2G1/2
4. The given expression is P = EL2m–5G–2
Dimension of (E) = [ML2T–2] hG
= k
(L) = [ML2T–1] c3
(m) = [M] (c) Let T ∝ cxhyGz
or T = kcxhyGz
(G) = [M–1L3T–2]
Substitution dimensions of each term in the given expres- where, k is a dimensionless constant
sion, Substituting dimensions of each term in Eq. (ix), we
(P) = [ML2T–2] × [ML2T–1]2 × [M]–5 × [M–1L3T–2]–2 get
= [M1 + 2 – 5 + 2 L2 + 4 – 6 T2 –2 + 4] [M0L0T] = [LT–1]x × [ML2T–1]y × [M–1L3T–2]z
= [M0L0T0] = [My – z Lx + 2y + 3zT–x –y –2z]
Therefore, P is a dimensionless quantity. On comparing powers of same terms, we get
5. Dimensions of (c) = [LT–1] y–z = 0
Dimension of Planck;s constant (h) = [ML2T–1] x + 2y + 3z = 0
Dimension of gravitational constant (G) = [M–1L3T–2] –x – y – 2z = 1
(a) Let m ∝ cxhyGz After solving above equation, we get
or m = kcxhyGz … (i) 5 1 1
x= − ,y=
,z=
where, k is a dimensionless constant of proportionality. 2 2 2
Substituting dimensions of each term in Eq. (i), we Putting values of x, y and z in Eq., we get
get T = kc–3/2h1/2G1/2
[ML0T0] = [LT–1]x × [ML2T–1]y × [M–1L3T–2]z hG
T = k
= [My – z Lx + 2y + 3zT –x – y – 2z] c5
Comparing powers of same terms on both sides, we 6.
Dimensions of energy = [ML2T–2]
get Let M1, L1, T1 and M2, L2, T2 are units of mass, length and
y–z = 1 time in given two systems.
x + 2y + 3z = 0 ∴ m1 = 1 kg , L1 = 1 m, T1 = 1s
– x – y – 2z = 0 m2 = α kg , L2 = β m, T2 = γ s
After solving above equation, we get Using,
www.crackjee.xyz
54 Mechanics

[M1L12T12 ] In Chapter Exercise -1.2


n2 = n1 1. Given, t1 = 39.6 s
[M 2 L22T22 ]
2 –2 t2 = 39.9 s
M  L  T  t3 = 39.5 s
= 5 1  ×  1  ×  1 
 M 2   L 2   T2  Least count of measuring instrument = 0.1s
2 −2 [As measurement have only one decimal place]
 1  1  1 
= 5  kg  ×  m  ×  s  precision in the measurement
 α  β   γ 
= Least count of the measuring instrument = 0.1 s
1 1 1 Mean value of time for 20 oscillations
= 5 × × 2 × −2
α β γ t +t +t 39.6 + 39.9 + 39.5
t= 1 2 3
= = 39.7 s
5γ 2 3 3
n2 = new unit of energy Absolute errors in the measurements
αβ2
7. From Kelper’s third law ∆t1 = t – t1 = 39.7 – 39.6 = 0.1s
T2 ∝ r3 or T∝r3/2 ∆t2 = t – t2 = 39.7 – 39.9 = –0.2s
and T is a function of R and g ∆t3 = t – t3 = 39.7 – 39.5 = 0.2s
| ∆t1 | + | ∆t2 | + | ∆t3 |
Let T ∝ r3/2Ragb Mean absolute error =
3
or T = kr3/2Ragb ..... (i)
0.1 + 0.2 + 0.2 0.5
where, k is a dimensionless constant of proportionality. = = = 0.17 ≈ 0.2
Substituting the dimensions of each term in Eq. (i), we get 3
(rounding-off upto one decimal place)
[M0L0T] = k[L]3/2[L]a[LT–2]n
∴ Accuracy of measurement = ± 0.2 s
= k[La + b + 3/2T–2b]
2. The given physical quantity
On comparing the powers of same terms, we get
x = a 2b3c5/2d–2
a + b + 3/2 = 0 ..... (ii)
Maximum percentage error in x.
–2b = 1 ⇒ b = –1/2
∆x
From Eq. (ii), we get × 100
x
a – 1/2 + 3/2 = 0 ⇒ a = –1
Substituting the values of a and b in Eq. (i), we get = ±  2  ∆a × 100  + 3  ∆b × 100  + 5  ∆c × 100  + 2  ∆d × 100  
T = kr3/2R–1g–1/2 a
   b  2 c   d 
 5 
k r3 = ±  2(1) + 3(2) + (3) + 2(4)  %
or T =  2 
R g
n2 − n1  15 
8. Given n = − D = ±  2 + 6 + + 8
x2 − x1  2 
n ( x2 − x1 ) = ± 23.5%
∴ D = − ∴ percentage error in quantity x = ±23.5%
( n2 − n1 )
Mean absolute error in x = ± 0.235
dimensions of n × dimensions of x = ± 0.24 (round-off upto two significant digits)
Dimensions of D =
dimensions of n2 or n1 The calculate value of x should be round-off upto two
T −1L−2 × L significant digits.
= = L2T–1 Ans.
L−3 ∴ x = 2.8
9. We can write, F = k m a ωb r c 3. Total surface area = 6 × ( 7.203)2
b = 311.299254 m2
or [ M ][ L ][T ]−2 = [ M ]a [T −1 ] [ L ]c The result should be rounded off to four significant figures,
On comparing the dimensions on both sides, we get so it becomes 311.3 m2
a = 1, b = 2 and c = 1 Volume of the cube = ( 7.203)3 = 373.714754 m3
Thus F = kmω2r. Ans. = 373.7 m3 Ans.
10. Given, F = a x + bt 2 4. Here t = 3.0 billion years
The dimensions of a x = dimensions of bt2 = 3.0 × 109 × 365.25 × 24 × 60 × 60 s
= dimensions of F = MLT–2 Speed of light, c = 3 × 105 kms–1
Distance of quasar
MLT −2
∴ dimensions of a = 1/2
= ML1/2T −2 ct = 3 × 105 × 3.0 × 109 × 365.25 × 24 × 60 × 60
=
L = 2.84 × 1022 km.
MLT −2 5. (i) Mean value of refractive index
b = = MLT −4
T 2 1.45 + 1.56 + 1.54 + 1.44 + 1.54 + 1.53
m =
a −1/2 2 6
Thus dimensions of = L T Ans.
b = 1.51.
www.crackjee.xyz
Units and Measurements 55
(ii) Absolute errors in measurements are : ∆µ 0.04
∆µ1 = 1.51 – 1.45 = 0.06 (iii) Fractional error = = = 0.02649
µ 1.51
∆µ2 = 1.51 – 1.56 = – 0.05
= 0.03
∆µ3 = 1.51 – 1.54 = – 0.03
∆µ4 = 1.51 – 1.44 = 0.07 ∆µ
(iv) Percentage error = × 100 =3%
∆µ5 = 1.51 – 1.54 = – 0.03 µ
∆µ6 = 1.51 – 1.53 = – 0.02 Thus, µ = 1.51 ± 0.04
Mean absolute error or µ = 1.51 ± 3% Ans.
 ∆µ + ∆µ 2 + ∆µ3 + ∆µ 4 + ∆µ5 + ∆µ6 
∆µ =  1 
 6 
= 0.0433 = 0.04.

Exercise 1.1 Level -1


1. (d) Scalar quantity can be negative and may have any value 13. (d) Time constant has the dimensions of time.
in the process. m1m2 Fr 2
14. (b) F = G 2 ; = ∴ G = M −1L3T −2
2. (d) Parallactic second is the unit of distance. r m1m2
3. (d) Temperature is a fundamental quantity, so it can not
F MLT −2
be expressed in terms of other quantities. 15. (c) The dimension =
of m = = ML−1
4. (a) On LHS, St is the distance while on RHS, u is the speed, f 22 T −2 L2
so it is not dimensionally correct. It is numerically dv F
correct only. 16. (c) F = η A ; ∴ η
= = ML2T −2
dy  dv 
5. (a) Reading 4000 has for significant figures, which are A 
 dy 
largest in the given values.
6. (b) The work done = force × displacement dimensions of X
17. (c) Dimensions of Y =
∴ unit, u1 = Fs dimensions of Z2
and u2 = 4F × 4s = 16u. M −1L−2T 4 A2
∆ density of substance =
7. (a) Strain = ; sp. gravity = ; ( MT −2 A−1 )2
 density of water
= M −3 L−2T 8 A4 .
arc distance
angle = 18. (c) The dimensions of momentum = MLT–1
radius
The dimensions of plank’s constant = ML2T–1.
8. (d) The dimensions of Plank constant = ML2T–1
Energy = ML2T–2. potential
19. (a) Inductance = = ML2T −2 A−2 .
9. (c) n1u1 = n2u2 charge
3 20. (a) Time period, T = k S a r b ρ c
u1  M  L 
∴ n2 = n1 = 8 1   2  M0L0T1 = [MT–2]a [L]b [ML–3]c
u2  M 2   L1 
= [M]a + c [L]b–3c [T]–2a
3
 1  5 ∴ –2a = 1 or a = –1/2
= 8     = 50.
 20   1  Also a + c = 0 or c = 1/2
 M L2T − 2  and b–3c = 0 or b = 3c = 3/2
1 1 1  2 −2
10. (a) n2 = n1  = 1 1 ×  1  ×  1  ρr 3
 M L2 T − 2  α β  γ  Thus T =k .
 2 2 2  S
= g2/ab2 force
2 2
11. (d) In, 1 – v , v should be dimensionless, so it should be 21. (a) Surface tension = length
energy energy
v2 = =
1− length × length (speed/time)2
c2 .
a = EV–2T–2.
12. (a) The dimensions of = dimensions of P 22. (d) The dimensions of Ay = dimensions of x.
V2
∴ dimensions of a = ML–1T–2 × L6 x
∴ dimension of A =
= ML5T–2. y
www.crackjee.xyz
56 Mechanics

23. (b) 2 ∆x ∆a ∆b ∆d 1 ∆c
= P P0 eαt ; =
αt 2 1 33. (b) × 100 = 3 × 100 + 2 × 100 + × 100 + × 100
x a b d 2 c
1
∴ dimensions of α= = T −2 . 1
t2 = 3 × 1% + 2 × 3% + 4 % + × 2%
2
L ML2T −2 A−2 = 14%
24. (b) = 2 −3 −2 = T value of 1division of main scale 1 1
R ML T A = = mm cm.
  34. (c) L.C.=
25. (b) E × B → (MLT–3A–1) × (MT–2A–1) = M2LT–5A–2. number of division on main scale N 10 N
26. (a) 27. (b) 28. (d) V 10
35. (b) R = = = 5Ω
29. (b) The significant number in the potential, V = iR; should I 2
be the minimum of either i or R. So corresponding to ∆R ∆V ∆I
Also, × 100 = × 100 + × 100
i = 3.23 A, we have only three significant numbers in R V I
V = 35.02935 V. Thus the result is V = 35.0 V. 0.5 0.2
Error and Instrument = × 100 + × 100 = 15%
10 2
1 100 Thus, R = 5 ± 15% Ω.
30. (c) The percentage error = × = 0.8%
5 25 1
31. (a) The mean value of refractive index, 36. (b) The value of 1 division of main scale = 10 = 0.1 cm
1.34 + 1.38 + 1.32 + 1.36 8 × 0.1
= µ = 1.35
4 The value of 1 division of vernier scale =
10
and
= 0.08 cm
| (1.35 − 1.34) | + | (1.35 − 1.38) | + | (1.35 − 1.32) | + | (1.35 − 1.36) |
∆µ = Thus L.C. = 0.1 – 0.08
4
= 0.02 cm
= 0.02
37. (c) Length of the rod = observed reading – zero error
∆µ 0.02
Thus × 100 = × 100 =
1.48 = (Main scale division + Vernier scale division × LC)
µ 1.35
– Zero error
M M
32. (d) Density, =
ρ = = (6.4 + 8 x 0.01) – (– 0.04)
V πr 2  = 6.4 + 0.08 + 0.04
∆ρ  ∆M 2∆r ∆  = 6.52 cm
∴ × 100 =  + +  × 100
ρ  M r  
 0.003 0.005 0.06 
=  +2 + × 100 = 4
 0.3 0.5 6 

Exercise 1.1 Level -2


= BA
1. (a) Magnetic flux, φ ∆i
4. (c) We have e = L   , L → inductance
= MT –2A–1 × L2  ∆t 
= ML2T–2A–1. e ML2 T −3 A −1 ML2 ML2
2. (a) The dimensions of ∴ L = = = =
 ∆i  (A / T) (AT)2 Q2
EJ 2 ( ML2T −2 ) ( ML2T −1 )2  
 ∆t 
= =1
M 5G 2 ( M 5 ) ( M −1L3T −2 )2 5. (a) The dimensions of radius of gyration
So it represents dimensions quantity like angle. = dimensions of L
3. (b) We know that, = G½ h½ c–3/2.
1 q1q2 1 1
= 6. (a) =
4π ∈ r2 ∈0 µ0
0
(M L A T ) × MLT −2 A −2
−1 −3 2 4
1 q1q2
∴ = 4π ∈ r2 = L2 T −2 = LT −1
0
⇒ velocity
1 (AT)2
= × 0.5
MLT −2 L2 7. (c) L.C. = = 0.01 mm
50
= M–1L–3T 4A2.
The diameter of the wire
www.crackjee.xyz
Units and Measurements 57
= 3 + 35 × 0.01 – (– 0.03) ∆g
× 100 =  ∆ ∆T 
= 3.38 mm. and  +2 × 100
g   T 

8. (c) T = 2π  0.1 0.1 
g = 100 + 2 2 × 100  × 100
 
∴ g = 4π2  = 0.2%
T2

Exercise 1.2
∆ L 7.
(a,b,c) Modulus of elasticity,
1. (b,c,d) Strain, e
= = = 1 f
 L = = MLT −2 / L=
E 2
ML−1T −2
v0 LT −1 e
Mach number == = 1
v LT −1 Pressure,
c LT −1 F MLT −2
Refractive index = = = 1 P
= = = ML−1T −2
v LT −1 A L2
2. (b,c) Similar to question 10. Energy density
3. (a,b,c,d) Energy ML2 T −2
= U = = ML−1T −2
(a) Any physical relation. Vol L3

rg 8. (a,b,c) Heat, energy and work done have same dimensions.
(b) tan θ = , is dimensionally correct. but it is
v2 9. (a,d) The dimension of electric flux
physically wrong equation. F MLT −2 2
a =φ EA
= A
= . L= ML3 T −3 A −1
(c) s n = u + (2n − 1) is dimensionally incorrect, but q AT
2
q AT
it is correct equation. = and = ML3 T −3 A −1 .
(d) Obvious. ∈0 M L−3 A 2 T 4
− 1

4. (a,d) The dimensions of torque and work, are 10. (a,b,c) L ML2 T −2 A −2
2 –2 = = T
= ML T . R ML2 T −3 A −2
The dimensions of light year and wavelength both are −1 −2 4 2 2 −3 −2
= L. CR = ( M L T A ) × ( ML T A ) = T
1/ 2
5. (a,b,c) = LC = ( ML2 T −2 A−2 ) × ( M −1 L−2 T 4 A2 )  T
(a) Reynolds number and coefficient of friction are
dimensionless. t
11. (b,c) In sin vt, vt must be dimensionless. Similarly is not
(b) Latent heat and gravitational potential both have a
dimensionless.
same unit. i.e., Joul/kg.
12. (b,d) lenght  = Gx cy hz
(c) Curie and frequency both are per second.
or M0LT0 = [M–1L3T–2]x [LT–1]y [ML2T–1]z
6. (b,d) L HS is dimensionless and so RHS must be. But
After comparing, we get
1 −1 x = 1/2, y = –3/2, z = 1/2.
dimensions of RHS are = L .
L

Exercise 1.3
1. (c) Dimensional constants are not dimensionless. Quantities with different dimensions can be multiplied.
2. (c) Zeros after a digit are significant. 7. (a) sin (ωt +φ) is the ratio of sides of a triangle, so it is
3. (a) Statement-2 is the explanation of statement-1. dimensionless.
4. (c) Angle is dimensionless, but it has unit radian. 1 1
8. (a) In LHS, has unit = m −1 .
5. (a) Statement-2 is the explanation of statement-1. λ m
mass 9. (c) Light has well defined relation with length.
6. (d) P = x
area 10. (c) Angle has no dimensions, but ω has dimensions T–1.

P × area MLT −1 2
∴ =x = L
×= L3 T −1
mass M
www.crackjee.xyz
58 Mechanics

Exercise 1.4
Passage (Questions 1 & 2) (C) Induced emf, e = L ∆i/∆t
1. (b) The equivalent resistor e ∆t
R = R1 + R2 ∴ L = = volt– sec (ampere)–1
= 100 + 200 = 300 Ω ∆i
Also, ∆R = ∆R1 + ∆R2 = ± (3 + 4) = ±7 Ω V
(D) Resistance, R = = volt (ampere)–1
Thus R = 300 ± 7 Ω.
R1R2 100 × 200 13. A → r ; B→ q ; C→ s ; D→ Ip
2. (d) = R = = 66.7 Ω Given in the theory of the chapter.
R1 + R2 100 + 200
Error and Instrument
∆R ∆R ∆R ∆R + ∆R2 A → p, q ; B→ r, s ; C→ r, s ; D→ r, s
14.
Also, = 1 + 2 + 1
R R1 R2 R1 + R2 (A) (p,q) The unit of GMeMs = Fr2 = Nm2
= kg m3s–2
3 4 3+ 4
= + + = 0.073 Also volt × coulomb × metre = joule × metre
100 200 (100 + 200)
= Nm2 = kg m3 s–2
∴ ∆ R = 0.073 × 66.7 = 4.9 Ω 3RT 2
R = 66.7 ± 4.9 Ω
Thus, (B) (r,s) The unit of = vms
M = m2s–2.
Passage (Questions 3 & 4)
Also (farad) (volt)2 (kg)–1 = (joule) kg–1
3. (c) e = [AT], ω = [T–1] = kg × ms–1 × mkg–1 = m2s–2.
N = [L–3], ∈o = [M–1 L–3 A2 T4] F2
We do not want Ampere [A] in the expression. This (C) (r, s) F = qvΒ ⇒ v2 = = m2s–2.
is only possible when ∈0 occurs as square. Therefore q2B2
options a and b are incorrect. 2GM
(D) (r,s) ve =
Ne 2 L−3 A2T 2 R
= = T −2 T −1
=
m ∈o M M −1L−3 A2T 4 ⇒
2GM
ve2
= m 2s –2 .
=
R
Ne 2 c 15. A → s ; B→ q ; C→ p ; D→ r
4. (b) ωp = = 2πν= 2π
m ∈o λ
R PV ML−1T −2 × L3
m ∈o Boltzmannn constant == =
λ = 2πc N nTN K
Ne 2
= ML2T −2K −1
−30 −11
22 10 × 10
= 2 × × 3 × 108 = 600 nm F MLT −2
7 4 × 1027 × (1.6 × 10−19 ) 2 Coefficient of viscosity = = = ML−1T −1
6πrv L × LT −1
Passage (Q5 – 11) Are given in the theory of the chapter. E ML2T −2
Planck constant
= = =
−1
ML−2T −1
12. A → s ; B→ r ; C→ q ; D→ p v T
Q2
(b) (A) Energy, E = H ML2T −2 × L
2C Thermal conductivity = =
tA∆T T × L2 × K
∴ C = Q /2E = (Joule)–1 coulomb2
2
= MLT–3K–1
(B) Force, F = Bil (c) is the correct option.
F
∴ B = = newton (ampere)–1 (metre)–1
i

Exercise 1.5
1. (d) 5. (c) 30 Divisions of vernier scale coincide with 29 divisions
2. (a) of main scales
3. (a) Momentum, p = m × v 29
= (3.513) × (5.00) = 17.565 kg m/s Therefore 1 V.S.D = MSD
30
= 17.6 (Rounding off to get three significant figures)
29
0.5 Least count = 1 MSD – 1VSD = 1 MSD– MSD
4. (d) Least count of screw gauge = mm = 0.01mm 30
50
Reading = [Main scale reading + circular scale 1 1
= MSD = × 0.5° = 1 minute.
reading × L.C] – (zero error) 30 30
= [3 + 35 × 0.01] – (–0.03) = 3.38 mm
www.crackjee.xyz
Units and Measurements 59

1 [ ML2T −2 θ−1 ][θ] 


6. (b) L.C. = mm a = = MLT −2 
100 [ L]  
Diameter of wire = MSR + CSR × L.C. Also, dimensional formula of P = [ML–1 T–2]
1 α α
= 0 + × 52 and dimensionally P = b=
100 β P
= 0.52 mm = 0.052 cm
7. (c) Reading of Vernier = Main scale reading MLT −2
\ [b] = = M 0 L2T 0
+ Vernier scale reading × least count. ML−1T −2
m
Main scale reading = 58.5 15. (d) r =
Vernier scale reading = 09 division  π r2
least count of Vernier = 0.5°/30 ∆ρ ∆m 2∆r ∆
= + +
0.5° ρ m r 
Thus R = 58.5° + 9 ×
30 Putting the values
R = 58.65 Dl = 0.06 cm, l = 6cm Dr = 0.005 cm; r = 0.5 cm,
V V ± ∆V m = 0.3 gm; Dm = 0.003 gm
8. (d) R = ⇒ R ± ∆R =
I I ± ∆I ∆ρ 4
  we get =
 ∆R  V 1 ± ∆V / V 
 ρ 100
R 1 ± =  
 R  I ∆I 
 1 ±  ∆ρ
 I  \ 4%.
× 100 =
ρ
 ∆R   ∆V   ∆I 
  = + = (3 + 3)% = 6% 16. (d) d = 2 + 25 × 0.01 – 0.05
 R   V   I 
= 2.20 mm
9. (c) 10. (c)
11. (b) At I = 5mA ∆g ∆ ∆T
17. (d) = +2
5 = (e1000V/T –1) ⇒ e1000V/T = 6 g  T
= Now
dI
dV dV
d 1000V / T
= [e –1]
T
e (
1000 1000V / T
) Dl and DT are least and number of readings are
maximum in option (d), therefore the measurement
1000 1000V / T 
= or dI  e  × (dV ) of g is most accurate with data used in this option.
 T  mg L
1000 ×
= × 6 × 0.01 =0.2mA 18. (b) We know that Y = D 2 
300 π
4π2 L 4
12. (a) g=
T2 4mgL 4 × 1× 9.8 × 2
= ⇒Y =
( ) (
πD  π 0.4 × 10−3 2 × 0.8 × 10−3
)
2
∆g ∆L  ∆T 
= = 2 
g L  T 
∆L 0.1 ∆T 0.01 = 2.0 × 1011 N/m2

= = , ∆Y 2∆D ∆
L 20 T 0.9 Now = +
Y D 
 ∆g   ∆L   ∆T 
100=
 g  100  L  + 2 × 100 × =
  3% [\ the value of m, g and L are exact]
   T 
0.01 0.05
13. (a) V = 3= (1.2 × 10–2 m)3 = 1.728 × 10–6 m3 = 2 × + = 2 × 0.025 + 0.0625
⇒ V = 1.7 × 10–6 m3. 0.4 0.8
Note :  has two significant figures. Hence V will also = 0.05 + 0.0625 = 0.1125
have two significant figures. ⇒DY = 2 × 1011 × 0.1125 = 0.225 × 1011
14. (a) Unit of k is joules per kelvin or dimensional formula = 2.0 × 1011 N/m2
of k is [ML2T–2q–1] Note : we can also take the value of Y from options
given without calculating it as it is same in all options.
Note : The power of an exponent is a number.
αz \ Y =( 2 ± 0.2 ) × 1011 N/m 2
Therefore, dimensionally = M ° L°T °
kθ 19. (b) The time period of a simple pendulum is given by

\ a = × a dimension less quantity   
z T = 2π \ T 2 = 4π2 ⇒ g = 4π2
\ Dimensional formula of
g g T2
www.crackjee.xyz
60 Mechanics

∆g ∆ ∆T l
⇒ × 100 = × 100 + 2 × 100 ∴ g = 4π2
g  T T2
Case (i) ∆g ∆l ∆T
∴ × 100 = × 100 + 2 × 100
Dl = 0.1 cm, l = 64cm, DT = 0.1s, T = 128s g l T

∆g
∴ × 100 = 0.3125 ∆g  1 
g ∴ × 100 =0 + 2   × 100 = 5
g  40 
Case (ii)
22. (c) Diameter D = M.S.R. + (C.S.R) × L.C.
Dl = 0.1 cm, l = 64cm, DT = 0.1s, T = 64s
∆g 0.5
∴ × 100 = 0.46875 D = 2.5 + 20 ×
g 50

D = 2.70 mm
Case (iii)
The uncertainty in the measurement of diameter
Dl = 0.1 cm, l = 20cm, DT = 0.1s, T = 36s
∆D = 0.01 mm.
∆g We know that
∴ × 100 = 1.055
g Mass M M
ρ = = =
Volume V 3
∆g 4 D
Clearly, the value of × 100 will be least in case (i). ð 
g 3 2
20. (d) 20 divisions on the vernier scale ∆ρ ∆M ∆D
= 16 divisions of main scale ∴ × 100 = × 100 + 3 × 100
ρ M ∆
∴ 1 division on the vernier scale
0.01
16 16 =2+3× × 100 = 3.1%
= divisions of main scale = × 1mm = 0.8 mm 2.70
20 20
23. (a) The maximum possible error in Y due to l and d are
We know that least count = 1MSD – 1VSD
= 1 mm – 0.8 mm = 0.2 mm ∆Y ∆l 2∆d
= +
21. (a,c) As the length of the string of simple pendulum is Y l d
exactly l m (given), therefore the error in length The least count
∆l = 0. Pitch
Further the possibility of error in measuring time is 1s =
Number of divisions on circular scale
in 40s.
∆t ∆T 1 0.5
∴ = = = mm = 0.005 mm
t T 40 100
40 ∆l 0.005 mm 1
The time period T = = 2 seconds Error contribution of l = = =
20 l 0.25 mm 50

∆T 1 ∆T 1 2∆d 2 × 0.005 mm 1
∴ = ⇒ = ⇒ ∆T= 0.05sec Error contribution of d = = =
T 40 2 40 d 0.5 mm 50

l l 24. (a)
We know that T = 2π ⇒ T 2 =π
4 2
g g
www.crackjee.xyz
www.crackjee.xyz
62 MECHANICS
Definitions Explanations and Derivations
2.1 SCALAR QUANTITY OR SCALAR

A physical quantity which has magnitude only is called scalar quantity. Example : speed, distance,
mass, pressure, electric current, surface tension etc.

2.2 VECTOR QUANTITY OR VECTOR

A physical quantity which has magnitude and direction and must obey law of vector addition is called
vector quantity. Example : displacement, velocity, angular velocity, force, torque, angular momentum,
momentum, impulse etc.
Surface tension is a scalar quantity
Surface tension is represented by direction, but it is not a vector quantity. Any quantity which has
unified direction is treated as scalar. Surface tension always directs along the tangent of the surface of
the liquid, therefore it is a scalar quantity.
Figure. 2.1
Note:
1. Scalar quantity may be negative. e.g. charge, electric current, potential energy, work etc.
2. Scalar quantity may have direction. e.g., pressure, electric current, surface tension etc.
uuur ur
3. Small element of length dl , small element of surface d A and small angular displacement are
treated as vectors.
Electric current is a scalar quantity
Electric current is always associated with direction, but it is not a vector quantity. It
does not obey the law of vector addition for its addition.
The resultant of i1 and i2 is (i1 + i2) by Kirchhoff’s law. The result does not depend on
angle between currents i1 and i2.
Polar vector
Figure. 2.2
A vector which has translational effect, that is its tail and head lie on a line, is called
polar vector. Example : velocity, force, momentum etc.
Axial vector
A vector which has rotational effect and act along axis of rotation is called axial vector.
Example : angular velocity, torque, angular momentum, angular acceleration etc.
Direction of axial vector is given by right hand screw rule.
Tensor
A physical quantity which can neither be treated as scalar nor as vector is called a
tensor. Example : moment of inertia. It has different values about different axes, but
never negative.
An important note on angular displacement
A vector quantity must be commutative in addition. If it is not commutative, then it can
not in general be represented by a parallelogram operation and is thus not a vector.
With this mind, consider the angle of rotation of a body about some axis. We can
associate a magnitude (degrees or radians) and a direction (the axis and a sense of
clockwise or anticlockwise) with this quantity. However, the angle of rotation can not
be considered a vector because it does not obey commutative law of vector addition
Figure. 2.3
i.e., q1 + q2 ¹ q2 + q1 . To understand this, place a book on the floor as in figure (a). Now give the two
successive 90° angular displacements, first about the x-axis and then about y-axis (see figure), it gives
us q1 + q2 .
www.crackjee.xyz
Vectors 63
Now, with the book in the same initial position as in figure (b), give two 90° angular displacements in
the reverse order (that is, first about y-axis and then about x-axis), it gives us q2 + q1 . It can be seen
from the figure that (q1 + q2 ) is not same as (q2 + q1 ) . Thus we can say that large anuglar displacement
is not a vector quantity.

Figure. 2.4
Position vector and displacement vector
A vector which gives the position of an object with reference to some specified point in a
system is called position vector. Displacement vector refers to the change of position vectors.
Thus
displacement vector = final position vector – initial position vector
ur uur ur
or Δr = r2 - r1
Fixed and free vectors
The vector whose initial point is fixed, is called fixed or localised vector. Example : position
vector of a particle; because its initial point lies on the origin. A vector whose initial point
is not fixed, is called a free or a non-localised vector. Example : displacement vector,
velocity vector etc.
Modulus of a vector
The modulus of a vector means the length of the vector. It therefore has no sign and no
direction.
ur ur
Modulus of vector A is represented as | A | or A.
Figure. 2.5
Zero vector or null vector
A vector whose magnitude is zero and has any arbitrary direction is called zero vector. It is represented
ur
by 0 . The need of a zero arises in the situations :
ur uur ur uur ur
(i) If A = B , then A – B = 0
ur ur
(ii) If m = – l, then (l + m) A = 0 .
ur ur ur ur ur ur ur
Properties of zero vector : A + 0 = A ; l 0 = 0 ; 0 A = 0
Unit vector
A vector whose magnitude is one unit, is called unit vector. A unit vector in the direction of vector
uur
ur A
A is represented by  , and is given by .
A
ur
\ Any vector can be expressed as A = A Â
î , ĵ and k̂ are unit vectors along x, y and z-axes. Figure. 2.6
www.crackjee.xyz
64 MECHANICS
Equal vectors
Two vectors are said to be equal if they have same magnitude and same direction.

Figure. 2.7
Negative of a vector
The negative of a vector is defined as vector having same magnitude but an opposite direction.

Figure. 2.8
Multiplication of a vector by a scalar
ur
When a vector is multiplied by a scalar l, we get a new vector which is l times the vector A i.e
ur ur
l A . The direction of resulting vector is that of A . If l has negative value then vector becomes
ur ur
–l A , whose direction is opposite of A . The unit of resulting vector is the multiplied units of l
ur
and A . For example when mass is multiplied with velocity, we get momentum. The unit of
momentum is obtained by multiplying unit of mass and velocity.

Figure. 2.9
uur
ur A
Similarly, we can have vector A divided by a scalar l. The resulting vector becomes . The
l
ur ur
magnitude of A decreases by l and direction is as that of A . Or l = –2, the resulting vector
becomes twice in magnitude and opposite in direction, with as follows:

Figure. 2.10
Collinear or parallel vectors
The vectors which act along the same line or along parallel lines are called collinear vectors.

Figure. 2.11
(a) Like or parallel vectors.

Figure. 2.12
(b) Unlike or antiparallel vectors.
ur uur uur ur
If A and B be two collinear vectors, then there exists a scalar k such the B = k A ; the absolute
value of k being the ratio of the length of the two collinear vectors.
www.crackjee.xyz
Vectors 65
Coplanar and concurrent vectors
ur uur ur
Vectors started from same point are known as concurrent vectors. In fig. 2.13 A , B and C are
coplanar and concurent vectors.
2.3 VECTORS OPERATIONS
The possible vectors operations are :
(i) Addition or subtraction of vectors.
(ii) Multiplication of vectors.
Note: Division of a vector by a vector is not possible.
Figure. 2.13
The addition or subtraction of vectors can be done by following two methods :
(i) Analytical method.
(ii) Geometrical method.
Geometrical method is more suitable for addition of more number of vectors while analytical method
is suitable for addition of less number of vectors.
2.4 ADDITION OR SUBTRACTION OF TWO VECTORS
Geometrical method
(a) Triangle law of vector addition : If two non-zero vectors can be represented by the two sides
of a triangle taken in same order, then their resultant is represented by third side of the
uur uur
triangle taken in the opposite order. Consider two vectors A and B at an angle q between
them.

Figure. 2.14
r r uur uuur
(i) Finding A + B : First draw vector A ( OP ) in the given direction. Then draw vector
uur uuur uur r uuuur
B ( PQ ); starting from the head of the vector A . Then close the triangle. R ( OQ )
will be their resultant (fig. 2.15).

Figure. 2.15
r r uur uuur
(ii) Finding A - B : First draw vector A ( OP ) in the given direction. Then draw
r uuur uur
vector –B ( PQ ). starting from head of the vector A . Then close the triangle,
r uuuur
R ( OQ' ) be their resultant (see fig. 2.16)

Figure. 2.16
www.crackjee.xyz
66 MECHANICS
(b) Parallelogram law of vector addition : If two non-zero vectors can be represented by the two
adjacent sides of a parallelogram, then their resultant is represented by the diagonal of the
parallelogram passing through the point of intersection of the vectors. Suppose two vectors
r uur
A and B as shown in fig. 2.17.
r ur r uuur ur uuuur
(i) Finding A + B : Draw vectors A ( OP ) and B ( OQ ) starting from a common point
uuur
O in the given directions. Then complete parallelogram. The diagonal OS will represent
Figure. 2.17 their resultant.

Figure. 2.18
r ur r uuur ur uuuur
Figure. 2.19 (ii) Finding A – B : Draw vectors A ( OP ) and – B ( OQ' ) starting from a common
uuuur
point O. Then complete the parallelogram. The diagonal OS ¢ will represent their resultant.

Analytical method:
r ur
Finding A + B :
uur uur
It is clear from the geometry of the figure that resultant of A and B is equal to the
uur uur uur
resultant of ( A + B cos q) and B sin q. By Pythagorous theorem, we have
\ R2 = (A + B cos q)2 + (B sin q)2 = A2 + B2cos2 q + 2AB cos q + B2sin2 q
or R = A2 + B 2 + 2 AB cos q
uur uur
Figure. 2.20
If a is the angle which resultant R makes with A , then
B sin q
tan a =
A + B cos q
Special cases :
(i) For q = 0°; Rmax = A2 + B 2 + 2 AB = A + B
(ii) For q = 180°, Rmin = A2 + B 2 - 2 AB = A – B
Thus resultant of two vectors R can be; ( A – B ) £ R £ ( A + B )

(iii) If A = B, R = A2 + A2 + 2 AA cos q

= 2 A2 (1 + cos q)

2 2
= 2 A ´ 2 cos q / 2
= 2A cos q/2
q
Figure. 2.21 and a=
2
uur uur uur
(iv) R = A – B
uur uur
= A + (– B )
www.crackjee.xyz
Vectors 67
uur uur uur uur
The subtraction of B from A means addition of B to A with an angle (180° – q).

\ R= A2 + B 2 + 2 AB cos (180° - q)

= A2 + B 2 - 2 AB cos q
B sin(180° - q) B sin q
and tan a = = .
A + B cos(180° - q) A - B cos q
uur uur uur uur
Note: For q = 90°, | A + B | = | A - B | .
Resolution of a vector into two perpendicular components
uur r r
Consider a vector R in xy- plane which makes an angle q with + x-axis. Let R x and R y
are the components along x and y-axes respectively, then
Rx
= cosq
R
Þ Rx = R cosq
Ry
and = sinq
R
Þ Ry = R sinq Figure. 2.22
ur uur uur
We can also write ; R = R x + R y
uur
or R = Rx î + Ry ĵ (2D – Vector)

Rectangular components of 3D-vector


uur
If R makes an angle a with x-axis, b with y-axis and g with z-axis, then
Rx
cos a = = l,
R
Ry
cos b = = m,
R
Rz Figure. 2.23
and cos g = =n
R
uur
where l , m, n are called direction cosines of the vector R
Squaring and adding, we get

Rx2 + R y2 + Rz2
2 2 2
cos a + cos b + cos g =
R2

As Rx2 + R y2 + Rz2 = R2 ,

\ cos 2 a + cos 2 b + cos 2 g = 1

Writing the position vector Figure. 2.24


The position of a point from any reference point, such as the origin O of the cartesian coordinate
uuuur ur
system, is uniquely specified by the vector OP = r , called the position vector of point P relative
ur
to O. The coordinates of point P being (x, y, z). We can write r = x î + y ĵ + z k̂ and

r= x2 + y 2 + z 2 .
www.crackjee.xyz
68 MECHANICS

2.5 ADDITION OR SUBTRACTION OF MORE THAN TWO VECTORS


uur uur uur
Consider three vectors A , B and C as shown in fig. 2.27.
uur uur uur uur
Finding R = A + B + C :

Geometrical method
Polygon law of vector addition : If a number of vectors are represented by the sides of an open
polygon taken in the same order, then their resultant is represented by the closing side of the
Figure. 2.25 polygon taken in opposite order.
r r r
Here in the fig. 2.26 R (closing side of polygon) represents the resultant. of vectors A , B and
r
C.
Analytical method
In this method first we have to resolve all the vectors into
two perpendicular axes. Then by using Pythagorous
theorem, their resultant can be obtained. Let Rx and Ry be
the sum of the components along x-axis and y-axis, then
their resultant,

R= Rx 2 + R y 2

Ry
and tan a = Figure. 2.27
Rx

where Rx = A cos 0° + B cos q1 + C cos (q1 + q2),


and Ry = A sin 0° + B sin q1 + C sin (q1 + q2).
Figure. 2.26

Note:
r r
D C If a number of vectors makes a closed polygon, their resultant will be zero. Here vectors A , B ,
r r r
C , D and E make closed polygon.
E r r r r r r
B \ A+B+C+D+E = 0

A
Figure. 2.28 uur uur
Condition of collinearity of vectors A and B
uur uur
Let A = A1 î + A2 ĵ + A3 k̂ and B = B1 î + B2 ĵ + B3 k̂
uur
Obviously, to be collinear (or parallel), the direction cosines of vector A must be equal to the
uur
respective direction cosines of vector B , i.e., we should have

A1 B A B A B
= 1 , 2 = 2 and 3 = 3
A B A B A B

A1 A2 A3 A
which gives = = = .
B1 B2 B3 B
www.crackjee.xyz
Vectors 69
FORMULAE USED
1. The resultant of two vectors with angle q between them is given by;

R = A2 + B 2 + 2 AB cos q
Rmin = A – B, Rmax = A + B, R
B
é B sin q ù
and tan a = ê ú.
ë A + B cos q û a q
For A = B, R = 2Acos q/2 and a = q/2. A
ur
2. If A x, A y and A z are the rectangular components of A and ˆi, ˆj and k̂ are the unit vectors along
x- , y- and z-axis respectively, then
ur
A = Ax ˆi + Ay ˆj + Az kˆ

ur
and A =| A |= Ax2 + Ay2 + Az2

ur
A Axˆi + Ay ˆj + Az kˆ
ˆ
A= r =
A Ax2 + Ay2 + Az2
uur
3. Resolution of a vector : If a vector R makes angle q with the positive x-axis, then components
uur
of R :
R x = Rcosq
and R y = Rsinq R
uur Ry
Also R = Rxˆi + Ry ˆj = R cos qˆi + R sin qˆj

and R = Rx2 + R y2 . q
Rx

EXAMPLES BASED ON ADDITION OR SUBTRACTION OF VECTORS


Example 1. Read each statement below carefully and state with Example 2.Which of the following quantities are independent
reasons, if it is true or false : [NCERT] of the choice of orientation of the coordinate axes :
(a) The magnitude of a vector is always a scalar. r r r r r r r r
a + b, 3a x + 2a y , a + b - c , the angle between b and c, λa, where
(b) Each component of a vector is always a scalar.
(c) The total path length is always equal to the magnitude of the l is a scalar ?
displacement vector of a particle. Sol. A vector and its magnitude do not depend on the choice of the
(d) The average speed of a particle (defined as total path length orientation of the axes, but component of a vector depends on the
divided by the time taken to cover the path) is either greater r r r r r
orientation of the axes. Thus a + b , a + b - c , the angle between the
or equal to the magnitude of average velocity of the particle
over the same interval of time. r
vector, la are independent of orientation of axis, while 3a x + 2a y
(e) Three vectors not lying in a plane can never add up to give a
null vector. depends on the orientation of the axes.
Sol. (a) True : The magnitude of a vector is a pure number, and so is Example 3. State with reasons, whether the following algebraic
operations with scalar and vector physical quantities are
a scalar.
meaningful. [NCERT]
(b) False : Component of a vector is also a vector.
(a) adding any two scalars,
(c) False : They are equal only along a straight line without changing
(b) adding a scalar to a vector of the same dimensions,
the direction, otherwise total path length will be greater than
(c) multiplying any vector by any scalar,
magnitude of acceleration.
(d) multiplying any two scalars,
Average velocity (e) adding any two vectors,
(d) True : £ 1. (f) adding a component of a vector to the same vector.
average speed
(e) True : This is because the resultant of the two vectors will not lie Sol.
in the plane of the third vector and so cannot be cancel out to give (a) No, because only the scalars of same dimensions can be added.
zero resultant. (b) No, because a scalar cannot be added to a vector.
www.crackjee.xyz
70 MECHANICS
(c) Yes, multiplying a vector with a scalar gives the scalar (number) uur uur
times the vector quantity which makes sense and one gets a The modulus of (OS - PS ) has been taken because the LHS is
r always positive but the RHS may be negative if OP < PS. Thus
bigger vector. For example, when acceleration A is multiplied from (3) we have,
r r ur ur ur ur
by mass m, we get a force F = m A. | A + B | > | | A | – | B || …… (4)
(d) Yes, two scalars multiplied yield a meaningful result, for example ur ur
multiplication of rise in temperature of water and its mass gives If the two vectors A and B are acting along a straight line in
the amount of heat absorbed by that mass of water. opposite directions, then
(e) No, because the two vectors of same dimensions can be added. ur ur ur ur
(f) Yes, because both are vectors of the same dimensions. | A + B | = | | A | – | B || …… (5)
Considering (4) and (5) together, we get,
Example 4. Establish the followin g vector inequalities ur ur ur ur
geometrically or otherwise: [NCERT] | A + B | ³ | | A | – | B ||
r r r r ur ur ur ur
(a) A+ B £ A + B (c) | A – B |£| A |+| B |
r r r r ur ur ur ur
In Fig, | A | = OP and | – B | = OT = PR and ( A - B ) = OR
(b) A+ B ³ A - B
r r r r From DOPS we note that OR < OP + PR.
(c) A- B £ A + B ur ur ur ur
or | A – B | < | | A | + | – B ||
r r r r ur ur ur ur
(d) A- B ³ A - B or, | A – B | < | | A | + | B || …… (6)
If the two vectors are acting along the straight line but in opposite
When does the equality sign above apply? direction, then,
ur ur ur ur ur ur
Sol. Consider two vectors A and B be represented by the sides | A – B | = | | A | + | B || …… (7)
uuur uuur
OP and OQ of a parallelogram OPSQ. According to parallelogram Considering (6) and (7) together, we get,
ur ur ur ur
ur ur uuur | A – B |£| A |+| B |
law of vector addition, ( A + B ) will be represented by OS as shown
ur ur ur ur ur ur
(d) | A – B | ³ | | A | – | B ||
in the adjoining figure. Thus, OP = | A |, OQ = PS = | B |and OS = |
ur ur In Fig, from DOPS we have,
A + B |. OR + PR > OP or, OR > | OP – PR |
ur ur ur ur
(a) to prove | A + B | £ | A | + | B | or, OR > | OP – OT | …… (8)
(Since OT = PR)
Q S uuur uuur
The modulus of (OP - OT ) has been taken because LHS is
A +B
B positive and RHS may be negative if OP <OT.
From (8),
O A ur ur ur ur
P | A – B |>| A |–| B | …… (9)
ur ur
–B A –B If the two vectors A and B are along the same straight line in
the same direction then,
ur ur ur ur
T R | A – B |=| A |–| B | …… (10)
We know that the length of one side of triangle is always less Considering (9) and (10) together, we get,
than the sum of the lengths of the other two sides. Hence from ur ur ur ur
| A – B | ³ | | A | – | B ||
DOPS, we have
r r r r
OS < OP + PS or, OS < OP + OQ or, Example 5. Given a + b + c + d = 0, which of the following
ur ur ur ur statements are correct ? [NCERT]
| A + B |£| A |+| B | …… (1)
ur ur r r r r
(a) a, b, c and d must each be a null vector..
If the two vectors A and B are acting along the same straight
line and in the same direction then, r r r r
ur ur ur ur (b) The magnitude of ( a + c ) equals the magnitude of (b + d) .
| A + B |=| A |+| B | …… (2) r
ur ur ur ur (c) The magnitude of a can never be greater than the sum of
(b) | A + B | ³ | | A | – | B || r r r
the magnitudes of b, c, and d.
From DOPS, we have,
OS + PS > OP or, OS > | OP – PS | or, r r r r r r
(d) b + c must lie in the plane of a and d if a and d are not
OS > | OP – OQ | …… (3)
r r
(Since PS = OQ) collinear, and in the line of a and d , if they are collinear ?
www.crackjee.xyz
Vectors 71
r r r r r
Sol. (a) a, b, c and d need not each be a null vector. The resultant of v = vb2 + vc2 + 2vbvc cos120°
four non-zero vectors even in different planes can be zero resultant.
r r r r æ 1ö
252 + 102 + 2 ´ 25 ´ 10 ´ ç - ÷
(b) a + b + c + d = 0, = è 2ø
r r r r r r r r
(
\ (a + c ) = - b + d ) or (a + c ) = ( b + d) , hence given
r
= 21.8 km/h Ans.
r
Let the resultant velocity v makes an angle a with the direction of vb ,
statement is correct.
then
r r r r r r r r
(c) (
As a + b + c + d = 0, \ a = - b + c + d . ) vc sin120° 10 3 / 2 ( )
r r r r tan a = =
Thus magnitude of a is equal to the magnitude of (b + c + d) . vb + vc cos120° 25 + 10 ´ ( -1/ 2)
The given statement is correct.
3
r r r r r r r r
(d) ( ) ( )
As a + b + c + d = 0, \ b + c = - a + d and hence plane of
=
4
r r r r æ 3ö
( ) ( )
b + c and a + d must be same. or a = tan -1 ç ÷ .
è 4 ø
Ans.

Example 6. A person moves 30 m north, then 20 m east and Example 8. Two billiard balls are rolling on a flat table. One
finally 30 2 m south-west. What is his displacement from the has the velocity components v x = 1 m/s, v y = 3 m/s and the other
initial position ? has components v'x = 2 m/s and v'y = 2 m/s. If both the balls start
Sol. If ˆi , ˆj are the unit vectors along east and north respectively, then moving from the same point, what is the angle between their paths
r ?
s1 = 30 ˆj, Sol. If q and q¢ are the angles made by resultant velocities of first and
r second ball respectively from the x-axis, then
s2 = 20 ˆi ,
r vy 3
and (
s3 = 30 2 cos 45° ˆi + sin 45° ˆj ) tan q =
vx
=
1
= 3

= -30ˆi - 30ˆj or q = 60°


The total displacement v 'y 2
r r r r and tan q ¢ = = =1
s = s1 + s2 + s3 Figure. 2.29 v 'x 2

= 30ˆj + 20ˆi - 30ˆi - 30ˆj = -10ˆi. or q ¢ = 45°


Angle between the paths of the balls
i.e., the resultant displacement is 10 m along west. Ans.
= q – q ¢ = 60° – 45° = 15° Ans.
Example 7. A motorboat is racing towards north at 25 km/h and Example 9.Two vectors, both equal in magnitude, have their
the water current in that region is 10 km/h in the direction of 60° resultant equal in magnitude of the either vector. Find the angle
east of south. Find the resultant velocity of the boat. between the vectors.
Sol. The velocity of motorboat, vb = 25 km/h, due north velocity of Sol. Let q is the angle between the vectors
\ A2 = A2 + A2 + 2 AA cos q
water current, vc = 10 km/h, 60° east of south.
1
which gives cos q = –
2
or q = 120°. Ans.
Example 10. On an open ground, a motorist follows a track
that turns to his left by an angle of 60° after every 500 m. Starting
from a given turn, specify the displacement of the motorist at the
third, sixth and eig hth turn. Co mpare the magnitude o f the
dis place ment with the total path length c overe d by the motor ist
in each case. [NCERT]
Sol. In this question,
E 60°
the path is a regular D
60°
hexagon ABCDEF of
side length 500 m. In 60°
Figure. 2.30 Fig, F G C
Let motorboat starts moving from O, as shown in fig. 2.25. From the 60°
r r 60° 60° 500 m
figure, the angle between vb and v c is 120°. The resultant velocity of b
r r A
boat is the resultant of vb and v c. Thus B
500 m
www.crackjee.xyz
72 MECHANICS
Let the motorist start from A. r
\ v = v cos a iˆ + v cos bˆj + v cos gkˆ
Third Turn
The motorcyclist will take the 3rd turn at D. Displacement vector at D é1 1 ˆ 1 ˆù
= AD = 20 ê ˆi + j + kú
ë 2 2 2 û
Magnitude of this displacement
= 500 + 500 = 1000 m = 10ˆi + 10 2 ˆj + 10kˆ Ans.
Total path length from A to D = AB + BC + CD
Example 13. Check whether three vectors of magnitude 1,
= 500 + 500 + 500 = 1500 m
2 and 4 can give zero resultant.
Sixth Turn
Sol. Choose any two of them, let A = 1 and B = 4, then
The motorcyclist will take the 6th turn at A.
A ~ B = 3 and A + B = 5.
\ Displacement vector is null vector.
Total path length = AB + BC + CD + DE + EF The third vector C = 2 does not lie between (A – B) and (A + B),
therefore they can not give zero resultant.
= 500 + 500 + 500 + 500 + 500 + 500 = 3000 m
Eighth Turn Example 14. Check whether three vectors of magnitude 2,
The motorcyclist takes the 8th turn at C. 3 and 5 be in equilibrium?
\ Displacement vector = AC, which is represented by the diagonal Sol. Choose any two of them, let A = 2 and B = 3 then
of the parallelogram ABCG . A ~ B = 1 and A + B = 5.
The third vector C = 5, lies between (A – B) and (A + B). Therefore
\ [(500)2 + (500)2 + 2 ´ (500) ´ (500) cos 60°] the given vectors can give zero resultant.
r r r r
2 2
[(500) + (500) + 250000] = 866.03m Example 15. Four forces P , 2 P , 3 P and 4 P act along sides
=
of a square taken in order. Find their resultant.
tan b = 500 sin 60°/{500 + 500 cos 60°]
Sol. The forces are drawn along the sides of a square as shown in
= (500 3 / 2) /{500(1 + 1/ 2)} = 1/ 3 the fig. 2.31. It is clear from the figure that :

= tan 30° or, b = 30°


R = (2 P ) 2 + (2 P )2 = 2 2 P
Example 11. The sum of the magnitudes of two forces acting at
a point is 18 N and the magnitude of their resultant is 12 N. If the and it makes angle
resulta nt makes an angle of 90° with th e force of smaller a = 180° + 45°
magnitude, what are the magnitude of the two forces ? = 225° with x-axis.
Sol. It is clear from geometry of the figure that the resultant of
ur uur uur
P and R is equal to Q .
\ P2 + R2 = Q 2
or Q2 – P2 = R 2
= 122 = 144
or (Q + P) (Q – P) = 144 …(i)
Given P + Q = 18 …(ii)
\ 18 (Q – P) = 144
or Q – P = 8 …(iii)
Now from equation (ii) and (iii), we get
uur ur uur
P = 5N Example 16. If A + B = C and A2 + B2 = C2, then prove
and Q = 13N Ans. uur ur
Example 12. A bird moves with velocity 20 m/s in a direction that A and B are perpendicular to each other.
making an angle of 60° with the eastern line and 60° with uur ur uur
vertical upward. Represent the velocity vector in rectangular Sol. We have, A + B = C
form. uur ur uur
Sol. Let eastern line be taken as x-axis, northern as y-axis and vertical \ |A + B | = |C |

upward as z-axis. Let the velocity v makes angle a, b and g with x, or A2 + B2 + 2AB cosq = C2
y and z-axis respectively, then a = 60°, g = 60°. given A2 + B2 = C2
We have cos2a + cos2b + cos2g = 1 \ C2 + 2AB cosq = C2 Þ AB cos q = 0
or cosq = 0
1
or cos2 60 + cos2 b + cos2 60 = 1 or cos b = \ q = 90°.
2
www.crackjee.xyz
Vectors 73
In Chapter Exercise 2.1
ur ur ur ur
1. If A = B - C , then, determine the angle between A and 5. If the resultant of the vectors 3ˆi + 4ˆj + 5kˆ and 5ˆi + 3ˆj + 4kˆ
ur
B. makes an angle q with x-axis, then find cos q. Ans. 0.574
é A2 + B 2 - C 2 ù 6. At what angle do the two forces (P + Q) and (P – Q) act so
Ans. q = cos -1 ê ú
ë 2 AB û
2. In a regular hexagon ABCDEF, that the resultant is 3P 2 + Q 2 . Ans. 60°.
uuur uuur uuur uuur uuur uuuur
prove that AB + AC + AD + AE + AF = 6 AO . 7. Two forces equal to 2 P and P, respectively act on a particle;
E D if the first is doubled and the second increased by 12 N the
direction of the resultant is unaltered. Find the value of P.
Ans. 12 N.
F O C r r
8. The angle of inclination between two vectors P and Q
r r
A B is q. If P and Q are interchanged in position, show that
3. When t = 0, a particle at (1, 0, 0) moves towards (4, 4, 12) the resultant will be turn through an angle f, where
with a constant speed of 65 m/s. The position of the particle
is measured in metre and time in second. Assume constant f éP -Qù q
tan = tan .
velocity, find the position of the particle for t = 2 s. 2 êë P + Q úû 2
Ans. (31iˆ + 40jˆ + 120k)
ˆ metre
r r r 9. Two forces (P + Q) and (P – Q) make an angle 2a with one
4. The resultant vector P and Q is R. On reversing the another and their resultant makes an angle q with the
r r
direction of Q , the resultant vector becomes S. bisector of the angle between them. Show that

(
Show that : R 2 + S 2 = 2 P 2 + Q2 . ) P tan q = Q tan a.

2.6 PRODUCT OF TWO VECTORS

The way in which two vectors enter into combination in physics, we come across are two distinct
kinds of vector products :
(1) Scalar or dot product :
uur uur
The scalar product of two vectors A and B is defined as the product of the magnitudes
uur uur
of A and B and cosine of the angle between them. Thus
uur uur
A . B = AB cos q.
As A, B and cos q all are scalars, so their product is a scalar quantity.
Geometrical interpretation of scalar product
uur uur
We have, A .B = AB cos q
= A (B cos q)
or = (A cos q) B
uur uur
(a) A .B = A × B cos q
= AB cos q
uur uur
(b) A .B = A cos q × B
Figure. 2.31
= AB cos q
Properties of scalar product
uur uur uur uur
(i) The scalar product is commutative i.e., A . B = B . A .
uur uur uur uur uur uur uur
(ii) The scalar product is distributive over addition i.e., A .( B + C ) = A . B + A . C .
www.crackjee.xyz
74 MECHANICS
uur uur uur uur
(iii) If A and B are perpendicular to each other, then A . B = AB cos 90° = 0.
uur uur
(iv) If A and B are parallel having same direction, then
uur uur
A .B = A B cos 0° = AB.
uur uur uur uur
(v) If A and B are antiparallel, then A . B = A B cos 180° = – AB.
uur uur
(vi) The scalar product of two identical vectors A . A = A A cos 0° = A2.

(vii) If î , ĵ and k̂ are mutually perpendicular unit vectors, then

î . î = (1) (1) cos 0° = 1


\ î . î = ĵ . ĵ = k̂ . k̂ = 1

î . ĵ = (1) (1) cos 90° = 0

\ î . ĵ = ĵ . k̂ = k̂ . î = 0
uur uur
(viii) If A = A1 î + A2 ĵ + A3 k̂ and B = B1 î + B2 ĵ + B3 k̂ , then
uur uur
A . B = A1 B1 + A2 B2 + A3 B3 .

Applications of scalar product


uur r
(i) Work done : If a force F causes displacement s , then work done
uur r
W = F . s = Fs cos q.
uur uur
(ii) Angle between the vectors : For two vectors A and B , we have
uur uur
A . B = AB cos q
uur uur
A .B
\ cos q =
AB
A1 B1 + A2 B2 + A3 B3
or cos q = .
A12 + A22 + A32 B12 + B22 + B32

Figure. 2.33 (iii) Component or projection of one vector along other vector :
uur uur
(a) Component of vector A along vector B is given by

AB cos q ˆ
A cos q B̂ = B
B
uur uur
æ A .B ö
ˆ
= ç ÷B
è B ø
uur uur
(b) Component of vector B along vector A ; is given by

Figure. 2.34 AB cos q ˆ


B cos q  = A
A
uur uur
æ A .B ö ˆ
= çç ÷÷ A .
è A ø
www.crackjee.xyz
Vectors 75
(2) Vector or cross product
The vector product of two vectors is defined as the vector whose magnitude is
equal to the product of the magnitudes of two vectors and sine of angle between
them and whose direction is perpendicular to the plane of the two vectors and is
uur uur
given by right hand rule. Mathematically, if q is the angle between A and B ,
then
uur uur
A × B = AB sin q n̂ .
uur uur
where n̂ is a unit vector perpendicular to the plane of vectors A and B .

Geometrical interpretation of vector product


uur uur
Suppose two vectors A and B are represented by the sides OP and OQ of a
uur uur
parallelogram, as shown in fig. 2.37. The magnitude of vector product A × B is Figure. 2.35
uur uur
| A × B | = AB sinq
= A (B sinq)
= area of rectangle OPTS
= area of parallelogram OPRQ
Thus the magnitude of the vector product of two vectors is equal to the area of
the parallelogram formed by the two vectors as its adjacent sides.
Moreover, the area of parallelogram OPRQ
= 2 × area of triangle OPQ.

1 Figure. 2.36
\ Area of triangle OPQ = (area of parallelogram OPRQ).
2

1 uur uur
or = | A × B |.
2
Properties of vector product
(i) Vector product is not commutative. It is anticommutative i.e.,
uur uur uur uur
A × B = – ( B × A ). Figure. 2.37
(ii) Vector product is distributive i.e.,
uur uur uur uur uur uur uur
A × (B + C ) = A × B + A × C .
(iii) Vector product of two parallel vectors
uur uur
A × B = AB sin (0° or 180°) n̂ = 0.
(iv) Vector product of two identical vectors
uur uur
A × A = AA sin 0° n̂ = 0.
(v) The vector product of two mutually perpendicular vectors
uur uur
| A × B | = AB sin 90°
= AB.

(vi) For unit vectors î , ĵ and k̂ Figure. 2.38

î × î = (1) (1) sin 0° n̂ = 0


\ î × î = ĵ × ĵ = k̂ × k̂ = 0
and î × ĵ = (1) (1) sin 90° k̂

= k̂
www.crackjee.xyz
76 MECHANICS

Similarly ĵ × k̂ = î and k̂ × î = ĵ
\ ĵ × î = – k̂ , kˆ ´ ˆj = -iˆ , î × k̂ = – ĵ
Vector product of consecutive two gives next and positive in anticlockwise sense.
Application of vector product
uur
1. Moment of force or torque : Let a force F is acting on a body which is free to rotate about
ur
O, and let r is the position vector of any point P on the line of action of the force. Since,
torque = force × perpendicular distance of its line of action from O, so
t = F(r sin q)
or t = Fr sin q uur ur
Its direction is perpendicular to the plane containing F and r , and given by right hand
screw rule.
ur uur
We know that F r sin q is the magnitude of the cross product r × F . So in vector notation,
we can write
r r r
τ= r ´F
Figure. 2.39 If we draw a set of three coordinate axes through O, as shown in figure 2.39, we will have
ur
r = x î + y ĵ + z k̂ ,
ur
where x î , y ĵ and z k̂ are the rectangular components of r along the three axes respectively.
uur
Similarly, F = Fx î + Fy ĵ + Fz k̂ .
ˆi ˆj kˆ
ur ur uur
\ t = r × F = x y z
Fx Fy Fz
r
or τ = ˆi ( Fz y - Fy z ) + ˆj( Fx z - Fz x) + kˆ ( Fy x - Fx y ).

2. Couple : We know that couple is a combination of two equal and opposite forces whose line
uur uur
of actions are different. Let F and – F be two forces acting at points P and Q and let the
r r
position vectors of P and Q with respect to O be r1 and r2 respectively.
uur uur
The moment of couple with respect to O = Vector sum of moment of forces F and – F . i.e.,
ur r uur r uur
t = – r1 × F + r2 × F
ur r r uur
\ t = ( r2 – r1 ) × F
r r ur
From the figure r2 – r1 = a
ur ur uur
\ t = a × F
= F a sin q n̂ (Here n̂ = k̂ )
ur
and | t | = F a sinq
or t = Fd (a sin q = d)
uur uur
Figure. 2.40 3. Sine of angle between the vectors : If q is the angle between vectors A and B , then we
have
uur uur
| A × B | = A B sinq
uur uur
| A ´B|
\ sin q = .
AB
uur uur
4. Unit vector along the direction perpendicular to plane of A and B
uur uur
ur A ´B
n = uur uur .
| A ´B |
www.crackjee.xyz
Vectors 77
2.7 GEOMETRICAL INTERPRETATION OF SCALAR TRIPLE PRODUCT
ur ur ur
Let us take three vectors A , B and C which form a parallelopiped as shown in fig. 2.41.

Figure . 2.41
It is clear from the fig. 2.45 that the area of base of parallelopiped = BC sin f. If q is the angle made by
ur ur ur
the vector B ´ C with A , then height of parallelopiped h = Acos q. Therefore volume of parallelopiped
ur ur
V = (BC sin f) (A cos q) = | B ´ C | Acos q
ur ur ur
or we can write V = A.(B ´ C) .
ur ur ur
Thus, we see that scalar triple product A.(B ´ C) represents the volume of parallelopiped, with the
three vectors forming its three edges.

Note:
ur ur ur ur ur ur ur ur ur ur ur ur
1. It can be easily proved that A.(B ´ C) = A.(C ´ B) = B.( A ´ C) = C.(A ´ B)
2. The order of vectors in scalar triple product is immaterial. Therefore we can simply write it as
ururur
[ ABC] .
ururur
The scalar triple product [ ABC] can directly calculated by using a determinant. For vectors
ur ur ur
A = A1iˆ + A2ˆj + A3ˆj , B = B1iˆ + B2 ˆj + B3kˆ and C = C1ˆi + C2 ˆj + C3kˆ , we have
A1 A2 A3
ururur
[ ABC] = B1 B2 B3
C1 C2 C3
= A1 (B2C3 – C2B3) + A2 (B3C1 – B1C3) + A3(B1C2 – C1 B2).

More about scalar triple product


ururur
1. In case the three vectors are coplanar, their scalar triple product is zero. i.e., [ ABC] = 0.
ururur ur ur ur
2. If two of the vectors be equal, the scalar triple product is zero i.e., [ AAB] = [ A ´ A].B = 0.
ur ur
3. If two vectors are parallel, the scalar triple product is zero. Let A and B are parallel, we can have
ur ur ururur ur ur ur
B = k A , where k is a scalar. Then [ ABC] = (k A ´ A )B = 0 .
4. ijkˆ ] = (ˆi ´ ˆj).kˆ = 1.
The scalar triple product of the orthogonal vector triad is unity i.e. [ ˆˆ
ur ur ur ur ur ur ur ur ur
Vector triple product : If A , B and C are three vectors, then A ´ (B ´ C) , B ´ (C ´ A) and
ur ur ur
C ´ (A ´ B) are the examples of vector triple product. You can prove the following relations by
previous knowledge
ur ur ur ur ur ur ur ur ur
A ´ (B ´ C) = ( A.C)B – ( A.B)C
ur ur ur ur ur ur ur ur ur
B ´ (C ´ A) = (B.A)C – (B.C) A
ur ur ur ur ur ur ur ur ur
and C ´ (A ´ B) = (C.B) A – (C.A)B .
www.crackjee.xyz
78 MECHANICS

FORMULAE USED
ur ur
1. Scalar product : A.B = AB cos q. B
ur ur
(i) If q = 90°, A.B = 0.
ur ur
(ii) Angle between A and B is given by
ur ur
A.B
cos q = .
AB q
ur ur A
(iii) If A = A1ˆi + A2 ˆj + A3kˆ and B = B1ˆi + B2 ˆj + B3kˆ , then
ur ur
A.B = A 1B 1 + A 2B 2 + A 3B 3.
ur ur ur ur
(iv) A.B = B.A .
(v) ˆi.iˆ = ˆj.jˆ = k.k
ˆ ˆ = 1.
uur r
(vi) Work done, W = F.s
2. Vector product :
ur ur A ×B
A´B
(i) h = ur ur
ˆ
|A´B|
ur ur
ur ur |A´B|
(ii) Angle between A and B , sin q = .
AB
ur ur q
(iii) If A = A1ˆi + A2 ˆj + A3kˆ and B = B1ˆi + B2 ˆj + B3kˆ , then

ˆi ˆj kˆ
ur ur A ur ur B
A ´ B = A1 A2 A3 A ´ B = AB sin qηˆ .
B1 B2 B3

= ˆi ( A2 B3 - A3 B2 ) + ˆj( B1 A3 - B3 A1 ) + kˆ ( A1B2 - A2 B1).


ur ur r
(iv) For parallel vectors A ´ B = AB sin 0ηˆ = 0
ur ur ur ur
(v) A ´ B = - B ´ A .
(vi) ˆi ´ ˆj = kˆ , ˆj ´ kˆ = ˆi , kˆ ´ ˆi = ˆj .
ur uur uur
(vii) Moment of force or torque, τ = r ´ F . B
ur ur
(viii) Area of a parallelogram which has adjacent sides A and B ;
ur ur q
A =| A ´ B |
A

1 ur ur
(ix) Area of triangle A= |A´B|
2 q
A
ur ur
(x) Condition of collinearity of two vectors A and B ;
( A1ˆi + A2 ˆj + A3kˆ ) × ( B1ˆi + B2ˆj + B3kˆ ) = 0
A1 A2 A3 A
which gives, = = = .
B1 B2 B3 B
ur ur ur
(xi) Condition of coplanarity of three vectors A , B and C
A1 A2 A3
ur ur ur
A.(B ´ C) = B1 B2 B3 = 0
C1 C2 C3
Or A 1(B 2C 3 – C 2B3) + A2(B 3C1– B 1C 3) + A 3(B1C 2 – C 1B 2) = 0.
www.crackjee.xyz
Vectors 79
EXAMPLES BASED ON DOT PRODUCT AND CROSS PRODUCT
Example 17. A point P lies in the x-y plane. Its position ur ur
Here A . B = ( î + ĵ + k̂ ).( î + ĵ – k̂ )
can be specified by its x, y coordinates or by a radially directed
ur = 1 × 1 + 1 × 1 + 1 × (–1) = 1
vector r = (x î + y ĵ ), making an angle q with the x-axis. Find
ur
and A = 12 + 12 + 12 = 3,
a vector ˆir of unit magnitude in the direction of vector r and
B = 12 + 12 + ( -1)2 = 3
a vector ˆiq of unit magnitude normal to the vector ˆir and lying 1 1
Now cos q = =
in the x-y plane. 3 3 3
ur
Sol. The unit vector in the direction of vector r is given by
-1 æ 1 ö
ur or q = cos ç ÷ Ans.
ˆ ˆ è3ø
ˆi = r = xi + yj = x ˆi + y ˆj ur
r
r r r r Example 19. Find the components of a = 2 î + 3 ĵ along the
From the geometry of figure, we have
x = r cos q and y = r sin q directions of vectors ( î + ĵ ) and ( î – ĵ ). [NCERT]
ur ur
\ ˆi = cos q î + sin q ĵ .
r Sol. Let b = ( î + ĵ )and c = ( î – ĵ )
ur ur
The component of a along b
ur ur
æ a .b ö
a cos q b̂ = ç ˆ
÷b
è b ø
(2ˆi + 3ˆj). (ˆi + ˆj) (ˆi + ˆj)
12 + 12 12 + 12
2 ´ 1 + 3 ´ 1 (ˆi + ˆj)
=
2 2
Figure. 2.42
5 ˆ ˆ
Let the unit vector normal to the unit vector ˆir be given by = (i + j) Ans.
2
ur ur
ˆi = x ' ˆi + y ' ˆj The component of a along c is given by
q
ur ur
where x ' and y ' are to be determined by using the fact that ˆiq is æ a .c ö
a cos q ĉ = ç ÷ cˆ
perpendicular to ˆir , so è c ø

ˆi . ˆi = 0, (2ˆi + 3ˆj) . (ˆi – ˆj) (ˆi - ˆj)


r q =
12 + 12 12 + 12
or (cos q î + sin q ĵ ) . ( x ' ˆi + y ' ˆj ) = 0, 1
= – (ˆi - ˆj) . Ans.
or x ' cos q + y ' sin q = 0 2
sin q Example 20. Find the area of the triangle formed by the tips
\ x' = – y ' . …(i) ur ur
cos q of the vectors a = î – ĵ – 3 k̂ , b = 4 î – 3 ĵ + k̂ and
Since îq is a unit vector, so ur
c = 3 î – ĵ + 2 k̂ .
x '2 + y '2 = 1 Sol. Let ABC is the triangle formed by the tips of the given vectors,
From equation (i), then
2
sin q
y '2 + y '2 = 1
cos 2 q
2 2 2
\ y '2 (sin q + cos q) = cos q
or y ' = cos q
and x ' = – sin q
Hence ˆi = – sin q î + cos q ĵ .
q
ur ur Figure. 2.43
Example 18. Given two vectors A = î + ĵ + k̂ and B = uuur ur ur
ur ur AB = b – a
î + ĵ – k̂ . Find angle between vectors A and B .
ur ur = (4 î – 3 ĵ + k̂ ) – ( î – ĵ – 3 k̂ )
æ A.B ö
Sol. We know that cos q = çç ÷.
÷ = 3 î – 2 ĵ + 4 k̂
è AB ø
www.crackjee.xyz
80 MECHANICS
uuur ur ur From equations (i) and (ii), we have
and AC = c – a uuur uuur
r OQ + PR
= (3 î – ĵ + 2 k̂ ) – ( î – ĵ – 3 k̂ ) A =
2
= 2 î + 5 k̂ = 2i - ˆj + 3kˆ
ˆ
uuur uuur uuuur uuur
Now AB × AC = (3 î – 2 ĵ + 4 k̂ ) × (2 î + 5 k̂ ) r OQ - PR
and B =
ˆi ˆj kˆ 2
= 3 -2 4 = ˆi + 2ˆj - kˆ
2 0 5 Area of parallelogram is given by
r r
= A´B
= î (–10 – 0) + ĵ (8 – 15) + k̂ (0 + 4)
= – 10 î – 7 ĵ + 4 k̂ ˆi ˆj kˆ
uuuur uuur = 2 -1 3
and | AB × AC | = ( -10) 2 + ( -7)2 + (4)2
1 2 -1
= 165 = 12.8 = ˆi (1 - 6 ) + ˆj ( 3 + 2) + kˆ ( 4 + 1)
1 uuur uuur
\ Area of DABC = 2 | AB × AC | = -5iˆ + 5ˆj + 5kˆ

=
1
× 12.8 = 6.4 sq unit. Ans. = ( -5 ) 2 + 5 2 + 5 2 = 5 3 sq. unit Ans.
2
Example 21. The diagonals of a parallelogram are given by Example 22. Prove that the four points (4ˆi + 5ˆj + kˆ ) ,
ˆ Find the area of the
the vectors 3iˆ + ˆj + 2kˆ and ˆi - 3jˆ + 4k. – (ˆj + kˆ ) , (3ˆi + 9ˆj + 4kˆ ) and 4( - ˆi + ˆj + kˆ ) are coplanar..
parallelogram. Sol. Let given vectors be the position vectors of four points A, B, C
r r and D with reference to an origin O. It is clear that the given points
ur
Sol. If A and B represent the adjacent sides of the parallelogram, then ur ur
will be coplanar if the vectors B A , B C and CD are coplanar..
diagonals uuur uuur uuur
BA = OA - OB = (4ˆi + 5ˆj + kˆ ) – [ -(ˆj + kˆ )] = 4iˆ + 6ˆj + 2kˆ
uuur uuur uuur
and BC = OC - OB = (3iˆ + 9 ˆj + 4kˆ ) – [ - ( ˆj + kˆ )] = (3ˆi + 10ˆj + 5kˆ )
uuur uuur uuur
and CD = OD - OC = -4ˆi + 4ˆj + 4kˆ – (3iˆ + 9 ˆj + 4kˆ ) = -7ˆi - 5ˆj

uuur uuur uuur 4 6 2


Figure. 2.44 Now [BA BC CD] = 3 10 5
uuur r r
OQ = A+B ...(i) -7 -5 0

= 3ˆi + ˆj + 2kˆ = 4(0 + 25) + 6(– 35 – 0) + 2(– 15 + 70) = 0


uuur r r uuur uuur uuur
and PR = A-B Since scalar triple product [BA BC CD] is zero, therefore given points
= ˆi - 3ˆj + 4kˆ ...(ii) are coplanar.

In Chapter Exercise 2.2


1. Calculate the values of r r
5. Show that the vectors A = 3ˆi - 2ˆj + kˆ , B = ˆi - 3ˆj + 5kˆ and
(
(i) ˆj. 2iˆ - 3ˆj + kˆ ) and (ii) ( 2ˆi - ˆj). (3iˆ + kˆ ). r
C = 2ˆi + ˆj - 4kˆ form a right angle triangle.
Ans. (i) –3 (ii) 6. 6. Find the cosine of the ang le between the vectors
r r
2. Find the value of a for which the vectors 3ˆi + 3ˆj + 9kˆ and A = 3ˆi + ˆj + 2kˆ and B = 2ˆi – 2ˆj + 4kˆ . Find also the unit
ˆi + aˆj + 3kˆ are parallel. r ur
Ans. a = 1. vector perpendicular to both A and B .
3. Find the value of m so that the vector 3ˆi - 2ˆj + kˆ is 3 1 ˆ ˆ ˆ
Ans. ; (i – j – k)
21 3
perpendicular to the vector 2iˆ + 6ˆj + mkˆ . Ans. 6.
r r r
4. Find a vector of magnitude 18 which is perpendicular to 7. Write down the values of (i) [ˆˆ i jkˆ ] (ii) a ´ (b ´ c ) .
rr rr r
both the vectors 4ˆi - ˆj + 3kˆ and 2ˆi + ˆj - 2kˆ . Ans. (i) 1 ; (ii) (a.c)b – (a.b)c
r r r r r r r r r
Ans. -6ˆi + 12ˆj + 12kˆ . 8. Show that a ´ (b ´ c ) + b ´ (c ´ a) + c ´ (a ´ b ) = 0 .
www.crackjee.xyz
Vectors 81
MISCELLANEOUS EXAMPLES FOR JEE-(MAIN AND ADVANCE)
Example 23. The position vectors of four points A, B, C and Sol. The situation is shown in the fig. 2.44. The bird flies in a
ur ur ur direction perpendicular to line P1 P2. Let it reaches the point P2 in
D are a = 2 î + 3 ĵ + 4 k̂ , b = 3 î + 5 ĵ + 7 k̂ , c = î + 2 ĵ time t.
ur
+ 3 k̂ and d = 3 î + 6 ĵ + 9 k̂ respectively. Examine whether d
t = …(i)
uuur uuur v
vectors AB and CD are collinear.
Sol.
uuur ur ur
Vector AB = b – a

= (3ˆi + 5ˆj + 7kˆ ) - (2iˆ + 3ˆj + 4kˆ ) = iˆ + 2ˆj + 3kˆ


uuur ur ur
and CD = d – c

= (3ˆi + 6ˆj + 9kˆ ) - (ˆi + 2ˆj + 3kˆ ) = 2ˆi + 4ˆj + 6kˆ


For collinearity of vectors, we have the condition Figure. 2.45
a1 a2 a3 where d = A sinq
= = .
b1 b2 b3 uur ur
|A ´B |
=
a1 1 a2 1 a 3 1 B
Here = , = and 3 = = ; i.e., condition of collinearity
b1 2 b2 2 b3 6 2 r r
|A´B|
is satisfied. The two vectors are thus collinear. Ans. \ t = ...(ii)
vB
uur ur
Example 24. If A = 3 î + 4 ĵ and B = 7 î + 24 ĵ , find a uur
A = (– 1 – 4) î + (2 + 1) ĵ + (0 – 5) k̂
ur uur
vector having the same magnitude as B and parallel to A .
= – 5 î + 3 ĵ –5 k̂
Sol. The required vector is = B Â ur
and B = (1 + 1) î + (1 – 2) ĵ + (4 – 0) k̂
B = 72 + 242 = 25
uur = 2 î – ĵ + 4 k̂ ,
A 3ˆi + 4ˆj
and  = =
A 32 + 42 B = 2 2 + ( -1) 2 + 4 2 = 21 = 4.58

1 ˆi ˆj kˆ
=
5
(3 î + 4 ĵ ) uur ur
A × B = -5 3 -5
1 2 -1 4
\ B Â = 25 ×
5
(3 î + 4 ĵ )
= î (12 – 5) + ĵ (– 10 + 20) + k̂ (5 – 6)
= 15 î + 20 ĵ Ans.
= 7 î + 10 ĵ – k̂
Example 25. A bird is at a point P whose coordinates are
(4m, –1m, 5m). The bird observes two points P1 and P2 having uur ur
and | A × B | = 7 2 + 102 + ( -1) 2
coordinates (–1m, 2m, 0m) and (1m, 1m, 4m) respectively. At
time t = 0, it starts flying in a plane of three positions, with a = 12.25 m2
Now from equation (ii), we get
constant speed of 5 m/s in a direction perpendicular to the
uur ur
straight line P 1 P 2 till it sees P 1 and P 2 collinear at time t. |A ´B | 12.25
Calculate t. t = = = 0.54 s. Ans.
vB 5 ´ 4.58
www.crackjee.xyz
82 Mechanics

Mechanics
MCQ Type 1 Exercise 2.1
Level - 1 (Only one option correct)

Addition or Subtraction of Vectors    Β
 (a) A = 2 B (b) Α=
1. A vector A makes an angle 240° with the positive x-axis, 2
 
its components along x-axis and y-axis are : (c) | A | = | B | (d) AB = 1
A A A 8. How many minimum number of coplanar vectors having
(a) A and 3 (b) – and 3 different magnitudes can be added to give zero resultant:
2 2 2
A A A (a) 2 (b) 3
(c) – and − 3 (d) − and 3A (c) 4 (d) 5
2 2 2
9. How many minimum number of vectors in different planes
2. Two billiard balls are rolling on a flat table. One has the
can be added to give zero resultant :
velocity components v x = 1 m/s, v y = 3 m/s and the (a) 2 (b) 3
' (c) 4 (d) 5
other has components v'x = 2 m/s and v y = 2 m/s. If   
10. Given A + B + C = 0. Which of the following statements
both the balls start moving from the same point, The angle is not correct:
between their paths is   
(a) A , B and C each must be a null vector
(a) 30° (b) 45°   
(b) the magnitude of A equals the magnitude of B + C
(c) 60° (d) 15° 
(c) the magnitude of A can never be greater than the sum
3. Two vectors, both equal in magnitude, have their resultant  
equal in magnitude of the either vector. The angle between of the magnitudes of B and C
  
the vectors is (d) A , B and C must lie in the same plane
(a) 90° (b) 120° 11. Can the resultant of 2 vectors be zero:
(c) 180° (d) zero (a) yes, when the 2 vectors are same in magnitude and
4. The resultant of two unit vectors equal in magnitude is equal direction
to either of them, then their difference is (b) no
(a) 1 (b) 3 (c) yes, when the 2 vectors are same in magnitude but
(c) 3 (d) 2 opposite in sense.
  (d) yes, when the 2 vectors are same in magnitude making
5. Given vector R= 2ˆi + 3ˆj . The angle between R and y-axis
is: 2π
an angle of with each other
3 2 3
(a) tan −1 (b) tan −1     
2 3 12. Two vectors A and B are such that A + B = C and
    
2 2 A+B= C . Then the vectors A and B are
(c) sin −1 (d) cos −1
3 3 [AMU B.Tech. 2014]
6. The length of a second’s hand in a watch is 1cm. The change (a) parallel (b) perpendicular
in velocity in 15 sec is : (c) anti-parallel (d) null vectors
   
π 13 Four forces P , 2 P , 3 P and 4 P act along sides of a square
(a) zero (b) cm/s
30 2 taken in order. Their resultant is
π π (a) 2 2P (b) 2P
(c) cm/s (d) 2 cm/s
30 30 (c) P / 2 (d) Zero
     
7. The angle between two vectors A and B is θ. Resultant 14. The value of ( A + B) × ( A − B) is :
 θ  (a) 0 (b) A2 – B2
of these vectors R makes an angle with A which of    
2 (c) A×B (d) – 2( A × B)
the following is true :

Answer 1 (c) 2 (d) 3 (b) 4 (b) 5 (b) 6 (d) 7 (c)


Key 8 (b) 9 (c) 10 (a) 11 (c) 12 (a) 13 (a) 14 (d)
www.crackjee.xyz
Vectors 83
       
15. If | V1 + V2 | = | V1 – V2 | and V1 and V2 are finite, (a) B (b) C
   
then: (c) B . C (d) B × C
     
(a) V1 is parallel to V2 21. The condition for A + B to be perpendicular to A – B
 
(b) V1 = V2 is that    
 (a) | A | = | B | (b) A = B
     
(c) | V1 | = | V2 | (c) B = 0 (d) | A + B | = | A – B |
    
(d) 22. The angle between ( A × B ) and ( A + B ) is :
V1 and V2 are mutually perpendicular
π
( ) (
16. The vector that is added to ˆi – 5jˆ + 2kˆ and 3iˆ + 6jˆ – 7kˆ ) (a) 0 (b)
4
to give a unit vector along the x-axis is : π
(c) (d) p
(a) 3iˆ + ˆj + 5kˆ (b) ˆi + 3jˆ + 5kˆ 2

(c) –3iˆ – ˆj + 5kˆ (d) 3iˆ + ˆj – 5kˆ 23. Find the component of vector =A (2ˆi + 3ˆj) along the
  direction (ˆi – ˆj) ]
17. The two vectors A and B that are parallel to each other
are : 1 1
  (a) − (iˆ – ˆj ) (b) − (iˆ + ˆj )
(a) A = 3ˆi + 6ˆj + 9kˆ B =ˆi + 2ˆj + 3kˆ 2 2
1 ˆ ˆ 1 ˆ ˆ
  (c) (i – j ) (d) (i + j )
(b) A = 3iˆ – 6jˆ + 9kˆ B = ˆi + 2jˆ + 3kˆ  2 2 
  =
24. If A ˆiA cos θ + ˆjA sin θ , then another vector B which is
(c) A = 2iˆ + 3jˆ + 3kˆ B = ˆi + 2jˆ – 3kˆ 
  perpendicular to A can be expressed as :
(d) A = 2iˆ + 6jˆ – 9kˆ B = ˆi – 2jˆ – 3kˆ
(a) ˆiB cos θ − ˆjB sin θ (b) ˆiB sin θ − ˆjB cos θ
   
18. The condition for A + B = A – B is that
 (c) iB cos θ + jB cos θ (d) ˆiB cos θ + ˆjB sin θ
(a) A = B (b) B =0    
(c) A = 0 (d) B is unit vector 25. If P . Q = 0 and P × R = 0, what is the angle between
 
Dot and Cross Product Q and R :
 
19. If A = 5ˆi + 7ˆj − 3kˆ and B = 2ˆi + 2ˆj − akˆ are perpendicular (a) 0° (b) 90°
(c) 120° (d) 180°
vectors, the value of a is: 2  2
  2
(a) – 2 (b) 8 ( ) ( )
26. The condition that a ⋅ b = a ⋅ b is satisfied when
(c) – 7 (d) – 8    
     (a) a  b (b) a≠b
20. Three vector A , B and C satisfy the relation A . B = 0    
   (c) a ⋅ b = 1 (d) a ⊥ b
and A C = 0. The vector A is parallel to :

Answer 15 (d) 16 (c) 17 (d) 18 (b) 19 (d) 20 (d) 21 (a)


Key 22 (c) 23 (a) 24 (b) 25 (b) 26 (a)

Level - 2 (Only one option correct)


Addition or Subtraction of Vectors 3.
From figure, the correct relation is :
       C N
1. If the magnitudes of vectors A , B and C are 12, 5 and (a) A+B + E = 0
O
      B
13 units respectively and A + B = C , the angle between (b) C − D =− A
D
A
vectors A and B is :     M E P
(c) B +E−C = −D
(a) 0 (b) p
π (d) all of these
π
(c) (d) 4. A bird moves with velocity 20 m/s in a direction making
2 4
  an angle of 60° with the eastern line and 60° with vertical
ˆ ˆ
2. If A= 3ˆi + 4ˆj and B= 7 i + 24 j , the vector having the same
  upward. The velocity vector in rectangular form.
magnitude as B and parallel to A is (a) 10iˆ + 10 ˆj + 10kˆ (b) 10iˆ + 10 2 ˆj + 10kˆ
(a) 15i + 20j (b) 3i + 4j
(c) 7i + 24j (d) none of these −20iˆ + 10 2 ˆj + 20kˆ (d) 10 2iˆ + 10 ˆj + kˆ
(c)
www.crackjee.xyz
84 Mechanics

5. The x and y components of A are 4 m and 6 m, respectively. 1      
  (c) | b × c | + | c × a | + | a × b |
( )
The x and y components of A + B are 10 m and 9 m 6
 1      
respectively. The magnitude of vector B is : ( ) ( )
(d)  a ⋅ b + b ⋅ c + ( c ⋅ a ) 
6    
(a) 19 m (b) 27 m 9. Given
= | A 2 | 3 and | A1 + A 2 | =
| A1 | 2,= 3 . Find the value
(c) 45 m (d)
50 m    
( )(
of A1 + 2 A 2 ⋅ 3A1 − 4 A 2 : )
Dot and Cross Product
(a) 64 (b) 60
6. Three particles A, B and C start from the origin at the same (c) 62 (d) 61
time, A with velocity a along x-axis, B with velocity b along 10. The area of the triangle having vertices at P(1, 3, 2), Q(2,
y-axis and C with velocity c in xy- plane along a line x = y. – 1, 1) and R (– 1, 2, 3) is :
The magnitude of c, so that they always remain collinear is 1 1
(a) 107 (b) 105
a+b 2 2
(a) (b)
ab
2 (c) 110 (d) None
ab 2 ab 11. If the vectors ( iˆ + ĵ + k̂ ) and 3 î form two sides of a
(c) (d)
a+b a+b triangle, the area of the triangle is :
7. The vector having magnitude equal to 3 and perpendicular
 
to the two vectors A = 2ˆi + 2ˆj + kˆ and B = 2ˆi − 2ˆj + 3kˆ is: (a) 3 (b)
2 3

( )
± 2ˆi − ˆj − 2kˆ (b)
(a) (
± 3ˆi + ˆj − 2kˆ ) (c) (d)
3
3 2
2 
(c) (3ˆi − ˆj − 3kˆ )
− ( 3ˆi + ˆj − 3kˆ ) (d) 12. The components of a = 2 î + 3 ĵ along the direction of
8.
The area of a triangle bounded by vectors a, b and c is : vector ( î + ĵ ) is
1    (a) iˆ + ˆj (b)
1 ˆ ˆ
(i + j )
(a) | a + b + c |
6 2
1       5 5 ˆ ˆ
(c) (iˆ + j ) (d) (i − j )
(b) | a ⋅ b + b ⋅ c + c ⋅ a | 2 2
6

Answer 1 (c) 2 (a) 3 (d) 4 (b) 5 (c) 6 (d) 7 (a)


Key 8 (c) 9 (a) 10 (a) 11 (c) 12 (c)

Mechanics
MCQ Type 2 Exercise 2.2
Multiple correct options 4. The following sets of three vectors act on a body, whose
   resultant can be zero. These are :
1. The resultant of A and B makes an angle α with A and
 (a) 10, 10, 10 (b) 10, 10, 20
β with B then :
(c) 10, 20, 20 (d) 10, 20, 40
(a) α < β (b) α < β if A < B
(c) α < β if A > B (d) α = β if A = B 5. The x-component of the resultant of several vectors :
2. The magnitude of resultant of three unit vectors can be : (a) is equal to the sum of the x-components of the vectors
(a) zero (b) 1 (b) may be smaller than the sum of the magnitudes of the
vectors
(c) 3 (d) 3 2
 (c) may be greater than the sum of the magnitudes of the
3. The magnitude of the vector product of two vectors A and vectors

B may be : (d) may be equal to the sum of the magnitudes of the
(a) greater than AB (b) equal to AB vectors
(c) less than AB (d) equal to zero
www.crackjee.xyz
Vectors 85
6. Which of the following vectors is/are perpendicular to the    
(c) | a – b | | a | + | b |
vector 5kˆ ?    
(d) | a – b | | a | – | b |
   
(a) 6iˆ
4ˆi + 3ˆj (b) 10. Given a + b + c + d = 0, which of the following
statements are correct :
(c) 3ˆi + 4ˆj    
7kˆ (d) (a) a , b , c and d = 0 must each be a null vector

  (b) The magnitude of ( + c ) equals the magnitude of
7. For two vectors A and B , identify the correct relation :  
    ( b + d )
(a) A+ B = B + A 
    (c) The magnitude of a can never be greater than the sum
(b) A . B = B . A   
    of the magnitude of b , c and d
(c) A ×B =B ×A      
    (d) b + c must lie in the plane of a and d if a and d
(d) ( A – B ) × ( B – A ) = 0  
  are not collinear, and in the line of a and d , if they
8. If A = B , the angle between the vectors is 0°. Now if A are collinear
= B, the angle between vectors may be : 11. The incorrect expression(s) in the following is/are
        
(a) 0° (b) 90° (a) A × ( B × C ) + B × ( C × A ) + C × ( A × B )= 0
(c) 180° (d) 30°         
(b) A . ( B . C ) + B . ( C . A ) + C .( A . B ) = 0
9. Which of the following is/are correct :      
      
(a) | a + b | | a | + | b | (c) A .( B × C ) + B .( C × A ) + C . ( A × B ) = 0
            
(b) | a + b | | a | + | b | (d) A .( B + C ) + B . ( C + A ) + C .( A + B ) = 0

Answer 1 (c, d) 2 (a, b, c) 3 (b,c,d) 4 (a, b, c) 5 (b, d) 6 (a, b, d)


Key 7 (a, b, d) 8 (a, b, c, d) 9 (a, c, d) 10 (b, c, d) 11 (b, c, d)

Mechanics
Reasoning Type Questions Exercise 2.3
Read the following questions and give your answer using the following options (a, b, c and d) :
(a) Statement - 1 is true, Statement - 2 is true; Statement - 2 is correct explanation for Statement - 1.
(b) Statement - 1 is true; Statement - 2 is true; Statement - 2 is not correct explanation for Statement - 1.
(c) Statement - 1 is true, Statement - 2 is false.
(d) Statement - 1 is false, Statement - 2 is true.
1. Statement - 1 4. Statement - 1
     
Two vector are said to be like vectors if they have same If A . B = B . C , then A may not equal to C .
direction but different magnitude. Statement - 2
Statement - 2 The dot product of two vectors involves cosine of the angle
Vector quantities always have a fixed direction. between two vectors.
2. Statement - 1 5. Statement - 1
Vector product of two vectors is an axial vector. If  = B̂ , then Â × B̂ = 0.
Statement - 2 Statement - 2
   If angle between  and B̂ is 0°, then their cross product
ω= v × r .
is a null vector.
3. Statement - 1 6. Statement - 1
         
A × B is perpendicular to A + B . A × B is perpendicular to both A + B as well as A – B
Statement - 2 Statement - 2
    
      A + B as well as A – B lie in the plane containing A and
A + B lies in the plane containing A and B , but A × B   
  B , but A × B lies perpendicular to the plane containing
lies perpendicular to plane containing A and B .  
A and B .
www.crackjee.xyz
86 Mechanics
7. Statement - 1 8. Statement - 1
The sum of two vectors can never be zero. The scalar product of two vectors can be zero.
Statement - 2 Statement - 2
Sum of two equal and opposite vectors is zero. If two vectors are perpendicular to each other, their scalar
product will be zero.

Answer 1 (c) 2 (c) 3 (a) 4 (a)


Key 5 (b) 6 (a) 7 (d) 8 (a)

Mechanics
Passage & Matrix Exercise 2.3
Passages
Passage for Questions. 1 - 3 :  
  3. The unit vector perpendicular to the plane of A and B is
Given two vectors A = 2ˆi + 6ˆj + 4kˆ and B =ˆi − 2ˆj + 8kˆ
1 1
1. The magnitude of their resultant is
(a)
13 5
( 28ˆi − 6ˆj − 5kˆ ) (b) 5 13
( 6ˆi − 28ˆj − 5kˆ )
(a) 5 (b) 8
1

2.
(c) 9   
The value of A ⋅ A × B is
(d) 13
5
(
(c) 28ˆi − 6ˆj − 5kˆ ) (d) none of these

(a) 5 (b) 13
(c) 3 (d) zero

Matrix Matching
   
4. If | A | = 2 and | B | = 4, then match the relations in column I with the angle θ between A and B in column II.
Column - I Column - II
 
A. A⋅ B =0 (p) θ = 0°
 
B. A⋅ B =+8 (q) θ = 90°
 
C. A⋅ B =4 (r) θ = 180°
 
D. A⋅ B =–8 (s) θ = 60°
   
5. If | A | = 2 and | B | = 4, then match the relations in column I with the angle θ between A and B in column II.

Column - I Column - II
 
A. | A × B |= 0 (p) θ = 30°
 
B. | A ⋅ B |=+8 (q) θ = 45°
 
C. | A × B |= 4 (r) θ = 90°
 
D. | A ⋅ B |=4 2 (s) θ = 0°

Answer 1 (d) 2 (d) 3 (a) 4 A → q, B → p, C → s, D → r,


Key 5 A → s, B → r, C → p, D → q,
www.crackjee.xyz
Vectors 87

In Chapter Exercise
In Chapter Exercise -2.1
  
5. The resultant vector = (3ˆi + 4ˆj + 5kˆ ) + (5ˆi + 3ˆj + 4kˆ )
1. Given, A = B − C
` = 8ˆi + 7ˆj + 9kˆ
  
\ C = B − A 8

cos q = = 0.574 Ans.
 
If θ is the angle between A and B , then 8 + 7 2 + 92
2

B 2 + A2 − 2 AB cos θ
C2 = 6. We know that R2 = A2 + B 2 + 2 AB cos θ

2 2 2
or ( 3P 2 + Q 2 ) = ( P + Q )2 + ( P − Q )2 + 2 ( P + Q )( P − Q ) cos θ
B + A −C
\ cos q = Ans.
2 AB
      R
R =
2. AB + AC + AD + AE + AF P
      7.
= (
AB + AB + BC + AB + BC + CD ) ( )  
    
(
+ AB + BC + CD + DE + CD ) ( ) 2P

  P sin θ


tanα =
As
DE = − AB 2 P + P cos θ
   
\ R = 3 AB + BC + CD ( ) =
( P + 12 ) sin θ
  4 P + ( P + 12 ) cos θ
= 3 × AD = 3 × 2 AO
 After simplifying, we get
= 6 AO Ans.
3. The distance travelled in 2 second = 65 × 2 = 130 m P = 12 N Ans.
 
8. If α is the angle which resultant R makes with P , then
(4, 4, 12)
2α + φ = θ
\ φ = ( θ − 2α )
R
φ θ 
r or =  − α 
r0 2 2 
0
(1, 0, 0) φ θ 
 or tan = tan  − α  ...(i)
r0 = 1iˆ 2 2 
 Q sin θ
r = 130 rˆ where tan α = ...(ii)
P + Q sin θ
( 4 − 1) ˆi + 4ˆj + 12kˆ 
= 130 ×  
32 + 42 + 122

= 30ˆi + 40ˆj + 120kˆ


  
\ R = r0 + r = 31ˆi + 40ˆj + 120kˆ Ans.

4. We have R2 = P 2 + Q 2 + 2 PQ cos θ ...(i)


After solving equations (i) and (ii), we get
and S2 = P 2 + Q 2 − 2 PQ cos θ ...(ii)
φ P −Q θ
Adding equations (i) and (ii), we get tan =   tan .
2  P + Q  2
R 2 + S 2 = 2 ( P 2 + Q 2 ) .
www.crackjee.xyz
88 Mechanics
9. The angle which the resultant makes with P + Q will be −ˆi + 2ˆj + 2kˆ
(α – θ). =
3

Q)
(P – R Thus, required vector will be = 18nˆ

 −ˆi + 2ˆj + 2kˆ 
  = 18  
 3 
(P + Q)

( P − Q ) sin 2α = −6ˆi + 12ˆj + 12kˆ Ans..


Thus tan(α – θ) =
( P + Q ) + ( P − Q ) cos 2α   2
( A × B ) = A × B=
 
5. ( AB sin θ )2
sin ( α − θ ) ( P − Q ) sin 2α
or = = A2 B 2 (1 − cos 2 θ )
cos ( α − θ ) ( P + Q ) + ( P − Q ) cos 2α
= A2 B 2 − A2 B 2 cos 2 θ
( P + Q ) sin ( α − θ ) = ( P − Q ) sin ( α + θ )
or   2
= A2 B 2 − ( A ⋅ B )
P sin ( α + θ ) − sin ( α − θ )  = Q sin ( α + θ ) + sin ( α − θ ) 
or  
6. Angle between A and B is given by
 
or
P × 2cos α sin θ =
Q × 2sin α cos θ A B
cos q =
\ P tan q = Q tan a AB

In Chapter Exercise -2.2 3 × 2 + 1 × ( −2 ) + 2 × 4


=
2
3 + 12 + 22
2
22 + ( −2 ) + 42
1. (i) ˆj ⋅ ( 2ˆi − 3ˆj + kˆ ) =
1 × ( −3) =−3
3
( 2ˆi − ˆj) ⋅ ( 3ˆi + kˆ ) = 2 × 3 + ( −1) × 0 + 0 × 1 = Ans.
(ii) 21
= 6 Ans.  
The unit vector perpendicular to A and B is given by
2. For vectors to be parallel, the angle between them should
 
be 0° or 180°. Thus
 
A×B
n̂ =   =
(
3ˆi + ˆj + 2kˆ × 2ˆi − 2ˆj + 4kˆ ) ( )
A × B = AB sin 0°nˆ =0 A×B (
3ˆi + ˆj + 2kˆ × 2ˆi − 2ˆj + 4kˆ ) ( )
or ( 3ˆi + 3ˆj + 9kˆ ) × ( ˆi + 9ˆj + 3kˆ ) = 0
8ˆi − 8ˆj − 8kˆ ˆi − ˆj − kˆ
= = Ans.
After solving, we get a = 1 Ans. 2 2 3
82 + ( −8 ) + ( −8 )
3. (3ˆi − 2ˆj + kˆ ) ⋅ ( 2ˆi + 6ˆj + mkˆ ) = 0
7. (i)   (
ˆi ˆj kˆ  = ˆi ⋅ ˆj × kˆ = ˆi ⋅ ˆi = )
1
or 3 × 2 − 2 × 6 + 1 × m = 0
        
m = 6 Ans (ii) ( )
a × b × c = ( a ⋅ c ) b − a ⋅ b c Ans. ( )
          
4. A vector perpendicular to A and B is given by 8. ( )
a × b × c + b × (c × a) + c × a × b ( )
             
A×B = ( a ⋅ c ) b − ( a ⋅ b ) c + ( b ⋅ a ) c − ( b ⋅ c ) a
n̂ =  
A×B
     
+ ( c ⋅ b ) a − ( c ⋅ a ) b = 0 Ans.
=
( 4ˆi − ˆj + 3kˆ ) × ( −2ˆi + ˆj − 2kˆ )
( 4ˆi − ˆj + 3kˆ ) × ( −2ˆi + ˆj − 2kˆ )

Exercise 2.1 Level -1


240° 2. (d) If θ and θ′ are the angles made by resultant velocities
x of first and second ball respectively from the x-axis,
then
A A
1. (c) Ax = A cos 240° = − vy 3
2 tan θ = = = 3
vx 1
A
and Ay = A sin 240° = − 3
2 or θ = 60°
www.crackjee.xyz
Vectors 89
13. (b) The forces are drawn along the sides of a square as
v 'y 2
and tan θ′ = = = 1 shown in the fig. 2.31. It is clear from the figure that:
vx' 2
= (2 P)2 + (2 P)2 = 2 2 P
R
or θ′ = 45°
and it makes angle
Angle between the paths of the balls
α = 180° + 45°
= θ – θ′ = 60° – 45° = 15° Ans.
= 225° with x-axis.
3. (b) Let θ is the angle between the vectors
\ A2 = A2 + A2 + 2AA cos θ
1
which gives cos θ = –
2
or θ = 120°. Ans.
2 2 2
4. (b) 1 = 1 + 1 + 2 × 1× 1× cos θ
1
∴ cos θ = − .
2
           
Now R = 12 + 12 − 2 × 1× 1cos θ 14. (d) ( A + B ) × ( A − B ) = A × A + B × A − A × B − B × B

     
2 2  1 = – A × B − A × B =−2( A × B) .
= 1 + 1 − 2 × 1× 1×  −  = 3
 2    
15. (d) | V1 | + | V2 |=| V1 − V2 |
5. (b) From the figure as shown, tan θ = 2/3.
y
or V12 + V22 + 2V1V2 cos θ = V12 + V22 − 2V1V2 cos θ

or cos θ = 0
R
 3
∴ θ = 90°
2 x

6. (d) The second's needle gets rotated by 90° in 15 seconds, 16. (c) If A is the required vector, then

so ∆= 
v v 2 + v 2 − 2vv cos 90° A + (iˆ − 5 ˆj + 2kˆ) + (3iˆ + 6 ˆj − 7 kˆ) = iˆ or ˆj or kˆ
2π π 2
= 2 v= 2 ωr= 2× ×1 = cm/s. 
60 30 ∴ A =−3iˆ − ˆj + 5kˆ
B sin θ
7. (c) tan θ/2 =
A + B cos θ 17. (d) For parallel vectors
∴ A = B. A1 A2 A3
= =
8.
(b) For three vectors in a plane B1 B2 B3

      
A+ B +C = 0 18. (b) If A + B = A − B

9. (c) The resultant of any three vectors will be cancel out or B=0.
by fourth vector.  
      19. (d) A.B = 0
10. (a) As A + B + C = 0, so A = −( B + C ) . They must be in a
   (5iˆ + 7 ˆj − 3kˆ). (2iˆ + 2 ˆj − akˆ) =
or 0
plane. Also | A=| | B + C |
11. (c) Two equal and opposite vectors will cancel each other. ∴ 5 × 2 + 7 × 2+ 3a = 0

12. (a)
or a=–8
www.crackjee.xyz
90 Mechanics
     
20. (d) As A.B = 0, so A and B are perpendicular, 24. (b) For perpendicular, A. B = 0,
     
also A.C = 0, so A and C are perpendicular. i.e., A is ∴
= ˆ sin θ − ˆj B cos θ.
B iB
  
perpendicular to the plane of B and C . So will be    
  25. (b) If P. Q = 0 , then Q is perpendicular to P
parallel to B × C .
 
    and P × R = 0.
21. (a) ( A + B ).( A − B) =
0 
  
        So R is parallel to P . Thus Q and R must be
or A. A + B. A − A.B − B.B =0 perpendicular.
∴ A = B.   
26. (a) (a.b)2 = (a 2 .b 2 )
   
22. (c) ( A × B) will be perpendicular to the plane of A and B
or (ab cos θ)2 = a2 b2
23. (a) The required result is , or cos θ = 1
  
( A.B ) B [(2iˆ + 3 ˆj ).(iˆ − ˆj )] ∴ θ = 0°,
= (i − j )
B2 ( 2)2

−(iˆ − ˆj )
= .
2

Exercise 2.1 Level -2


1. (c) 132 = 122 + 52 + 2 × 12 × 5 cos θ 4 + Bx = 10, ∴ Bx = 6.
13
∴ θ = 90° 5 Also 6 + By = 9 , ∴ By = 3.

BA 12
2. (a) The required vector is = B Aˆ = B=
Thus, Bx2 + B y2 = 62 + 32 = 45 .
A
6. (d) The situation is shown in figure. y
2 2 (3iˆ + 4 ˆj )
= 7 + 24 ×
32 + 42 bt − ct sin 45° ct sin 45°
= bt ct
ct cos 45 ° at − ct cos 45° 45°
= 15iˆ + 20 ˆj . o at x
2 ab
3. (d) Use law of vector addition. ∴ c= .
a+b
4. (b) Let eastern line be taken as x-axis, northern as y-axis 7. (a) The required vector is,
and vertical upward as z-axis. Let the velocity v makes  
( A × B) [(2iˆ + 2 ˆj + kˆ) × (2iˆ − 2 ˆj + 3kˆ)]
angle α, β and γ with x, y and z-axis respectively, then = 3   = 3  
α = 60°, γ = 60°. | A× B | | A× B |

We have cos2α + cos2β + cos2γ = 1 (8iˆ − 4 ˆj − 8kˆ)


= 3 = 2iˆ − ˆj – 2kˆ .
2 2 2
1 8 + 4 +8   
or cos2 60 + cos2 β + cos2 60 = 1 or cos β = 8. (c) The area of the triangle of sides a, b and c can be
2
written as;

v v cos α ˆi + v cos βˆj + v cos γkˆ
∴ =      
a×b b×c c×a
A = = =
1 1 ˆ 1 ˆ 2 2 2
= 20  ˆi + j+ k
2 2 2       
a×b b×c c×a
∴ 3A = + +
10ˆi + 10 2 ˆj + 10kˆ
= 2 2 2


5. (c) A= 4iˆ + 6 ˆj. If Bx and By are the components of vector 1      
A = | a × b | + | b × c | + | c × a | .
or
6  
along x and y axes, then
www.crackjee.xyz
Vectors 91
 
9. (a) If θ is the angle between A1 and A2 , then 1 ˆ 1
= | (i − 4 ˆj − kˆ) × (−2iˆ − ˆj + kˆ) | = 107 .
2 2
A2 = A12 + A22 + 2 A1 A2 cos θ
11. (c) The area of the triangle
or 32 = 22 + 32 + 2 × 2 × 3 cos θ 1 ˆ ˆ ˆ 3
A
= | (i + j + k ) × (3iˆ) | = .
∴ cos θ = – 1/3. 2 2
     
Now, ( A1 + 2 A2 ) . (3 A1 − 4 A2 ) 12. (c) Let b = ( ˆ + ĵ )and c = ( î – ĵ )
 
        The component of a along b
= 3 A1. A1 + 6 A2 . A1 − 4 A1 . A2 − 8 A2 . A2
 
 a .b  ˆ
  a cos θ b̂ =   b
= 3 A12 + 2 A1. A2 − 8 A22  b 
= 3 × 22 + 2 × 2 × 3× (–1/3) – 8 × 32 = – 64
(2ˆi + 3ˆj). (ˆi + ˆj) (ˆi + ˆj)
=
10. (a) Two sides of a triangle are; 12 + 12 12 + 12

PQ = (2iˆ − ˆj + kˆ) − (iˆ + 3 ˆj + 2kˆ) = iˆ − 4 ˆj − kˆ 2 × 1 + 3 × 1 (ˆi + ˆj)
=
 2 2
and PR =(−iˆ + 2 ˆj + 3kˆ) − (iˆ + 3 ˆj + 2kˆ) =−2iˆ − ˆj + kˆ
5 ˆ ˆ
= (i + j)
Thus area of the triangle 2
1  
=A | PQ × PR |
2

Exercise 2.2
1. (c,d) From the figure, it is clear that if A > B, α < β. And if  
8. (a,b,c,d) The angle between A and B may have any value.
A = B, α = β. A

R   
 9. (a,c,d) The resultant R of a and b is such that
B
    
| A − B |≤ R ≤| A + B | , so options (a, c, d) are correct.
2. (a,b,c) If unit vectors are along the same direction, then
resultant be zero. The resultant may be zero when each    
10. (b,c,d) a + b + c + d =0
one has angle 120° from other.
       
    or a + c =−(b + d ) , so | a + c | =−|b+d |
3. (b,c,d) | A × B | = A B sin θ. As sin θ ≤1, so | A × B | can
       
not be greater than A B. Also a =−(b + c + d ), so a = | b + c + d |

4. (a,b,c) If A, B and C are the magnitudes of three vectors, 11. (b,c,d)


then for their resultant to be zero,         
A × ( B × C ) + B × (C × A) + C × ( A × B)
(A – B) ≤ C ≤ (A + B).

5. (b,d) If all the vectors are in the same direction, then (d)            
= ( A. C ) B − ( A.B)C + ( B. A) C − ( B.C ) A
will be correct, otherwise (b) will be correct.      
+(C.B ) A − (C. A) B = 0.
6. (a,b,d) The dot product of 5kˆ with 4iˆ + 3 ˆj , 6iˆ and 3iˆ + 4 ˆj
is zero, so these are perpendicular vectors to 5kˆ . The options (b, c, d) are incorrect.
   
7. (a,b,c) A × B =− B × A , so option (c) is not correct.
www.crackjee.xyz
92 Mechanics

Exercise 2.3
1. (c) Vector quantity may have any direction, so statement-2 5. (b) Aˆ × Bˆ = AB sin 0° = 0.
is not correct.    
 
6. (a) A + B and A − B are in the plane of A and B . But
2.
(c) The vector product of two vectors is perpendicular to    
the plane of given vectors, so it is axial vector. A × B is perpendicular to the plane of A and B .
   7. (d) The sum of two equal and opposite vectors is equal to
Also V = ω× r , so statement-2 is incorrect.
zero.
3. (a) Statement-2 is the explanation of statement-1
8. (a) If θ = 90°, then AB cos θ = 0.
4. (a) AB cos θ1 = BC cos θ2, so A cos θ1 = C cos θ2.
A = C, only if θ1 = θ2, otherwise A C.

Exercise 2.4
Passage (Questions 1 to 3)  
  A× B 56iˆ − 12 ˆj − 10kˆ 1
3. =
(a) ηˆ = = (28iˆ − 6ˆ ˆj − 5kˆ)
1. (d) A + B = (2iˆ + 6 ˆj + 4kˆ) + (iˆ − 2 ˆj + 8kˆ) | A× B | 2 2 2 13 5
56 + 12 + 10
= 3iˆ + 4 ˆj + 12kˆ Matching (Q 4 &5)
  4. A → q, B → p, C → s, D → r,
∴ | A + B |= 32 + 42 + 122 = 13
    5. A → s, B → r, C → p, D → q,
2. (d) A.(
= A × B ) A.( AB sin= θ ηˆ ) A( AB sin θ)cos90
= ° 0.
www.crackjee.xyz
www.crackjee.xyz
94 MECHANICS

Definitions, Explanations and Derivations

3.1 CONCEPT OF A POINT OBJECT


If the position of an object changes by distances much greater than its size in a considerable interval of
time, then the object can be regarded as a point object.
Example : A car under a journey of several kilometers can be regarded as a point object. Moon can be
regarded as a point object for studying its motion around the earth.

3.2 REST AND MOTION ARE RELATIVE TERMS


A person in a car is at rest with respect to the driver of the car but he is in motion with respect to the
observer outside the car. Thus an object may be at rest w.r.t. one object and the same time it may be in
motion relative to another object. Hence rest and motion are relative terms. No object in the universe is
in a state of absolute rest or motion.

3.3 MOTION
One dimensional motion (1D) : The motion of an object whose position changes with time along a
straight line, may be any one of the coordinate axes is known as one dimensional motion or rectilinear
motion.
Example : Motion of a car, a train along a straight track, motion of a freely falling body etc.
Two dimensional motion (2D) : If an object moves in such a way that it covers two directions
simultaneously, its motion is known as two dimensional motion.
Example : All motions on curved path in a plane are two dimensional. (see fig. 3.1)
Three dimensional motion (3D) : The motion of an object is said to be three dimensional if all the three
coordinates specifying its position change with time.
Figure. 3.1 Motion in 2D Example : Motion of a kite, motion of a fly etc. (see fig 3.2)

3.4 MOTION PARAMETERS

Distance
It is the actual length of path traversed by a moving particle. It is a scalar quantity. Its SI unit
is metre (m).
Displacement
It is the shortest distance between the initial and final position of the particle. It is a vector
uur uur
quantity. Its SI unit is metre (m). If r1 and r2 are the position vectors of a particle at time
ur uuur uur uur
t1 and t2 respectively, then its displacement in this interval Dt = t2 – t1 is s = r21 = r2 - r1
Figure. 3.2 Motion in 3D (see fig. 3.3)
More about distance and displacement
1. Distance can never be negative and can not decrease with time. Displacement may be zero or
negative.
2. Until particle changes the direction of motion, displacement is equal to distance. Otherwise
displacement will be less than distance. Thus
displacement
£1
distance
Speed
It is the distance travelled by object in unit time. It is a scalar quantity. Its SI unit is m/s.
Figure. 3.3
Average speed
The average speed is the total distance travelled by the object in any time interval divided by that time
interval.
www.crackjee.xyz
Motion in a Straight Line 95
total distance travelled
Average speed =
time interval
Suppose an object travels a distance Ds, in time Dt, then its average speed
Ds
vav = < v > = .
Dt
Instantaneous speed
The speed of object at a particular instant is called instantaneous speed. The limiting value of average
speed when time interval Dt approaches to zero, gives the instantaneous speed at any instant t. Thus
Ds ds
vinst = lim = .
Dt ®0 Dt dt
Velocity
It is the displacement covered by the object per unit time. It is a vector quantity. Its unit is m/s.
Average velocity
The average velocity is the displacement covered by object in any time interval divided by that time
interval.
displacement
\ Average velocity =
time interval
ur
Suppose an object travels a displacement D s in time interval Dt, then its average velocity is
ur
ur ur Ds
v av = < v > = .
Dt
The direction of average velocity is that of the direction of displacement.
Instantaneous velocity
The velocity of an object at a particular instant of time is called instantaneous velocity. It is equal to
the limiting value of average velocity of the object when the time interval approaches zero. Thus
ur ur
ur Ds ds .
Instantaneous velocity v= lim =
Dt ® 0 Dt dt
More about speed and velocity
1. The instantaneous velocity in magnitude is equal to instantaneous speed.
2. A particle may have constant speed but variable velocity. In uniform circular motion speed
remains constant while velocity changes because of change in direction of motion.
3. If particle is moving along a straight line without changing the direction, its average velocity will
be equal to its average speed. Otherwise average velocity will be less than average speed. Thus
average velocity
£1
average speed

Uniform motion and non-uniform motion


If an object covers equal distance in equal time interval or object moves with constant speed then it
is said to be in uniform motion. For uniform motion
distance = speed × time
s = vt
In uniform circular motion
2pR = v t
where R is the radius of path and t is the time to complete the circle.
A body is said to be in non-uniform motion if its speed changes with time. Ex. Motion under gravity,
car starts from rest etc.
www.crackjee.xyz
96 MECHANICS

Note:
1. In uniform motion average speed is equal to the instantaneous speed.
2. In a uniform motion along a straight line without change in direction, the average velocity is
equal to instantaneous velocity.
3. In uniform motion along a straight line without change in direction of motion,
r r
d |v| dv
= 0 and =0
dt dt
r r
d |v| dv
4. If body moves uniformly but its direction of motion changes, then = 0, but ¹0.
dt dt

Calculating average speed and average velocity


1. A body covering different distances with different speeds : Suppose a body covers distances s1,
s2,....., sn with speed v1, v2,....., vn respectively, then
Total distance travelled s
Average speed = =
total time taken t
( s1 + s2 + ... + sn )
=
(t1 + t2 + ... + tn )
( s1 + s2 + ... + sn )
vav =
Figure. 3.4 s1 s2 s
+ + .... + n
v1 v2 vn
Special case : If s1 = s2 = s
2s 2 v1v2
vav = =
s s v1 + v 2
+
v1 v2
It is the harmonic mean of the individual speeds.
2. A body moving with different speeds in different time interval : Suppose a body travels with
speeds v1, v2, .........., vn in time intervals t1, t2, .............., tn respectively, then
total distance travelled s
Average speed = =
total time taken t
s1 + s2 + ..... + sn
=
t1 + t2 ..... + tn
(v1t1 + v 2 t 2 + ...... + v n t n )
=
(t1 + t 2 + ..... + t n )
Special case : If t1 = t2 = .... = tn = t, then
v1 + v2 .... + v n
vav =
n
It is the arithmetic mean of the individual speeds.
3. For continuously changing speed with time, the average speed is defined as
t2

ò vdt
s t1
vav = = .
t t2

ò dt
t1
www.crackjee.xyz
Motion in a Straight Line 97
Acceleration
The rate of change of velocity of an object with time is called acceleration. It is a vector quantity. It
can be negative. Negative acceleration is called retardation or deceleration. Its SI unit is m/s2.
Average acceleration
For an object moving with variable velocity, the average acceleration is defined as the ratio of the
ur ur
change of velocity of the object to the time of change of velocity. Suppose v 1 and v2 are the
velocities of an object at time t1 and t2 respectively, then its average acceleration ;
ur ur ur
ur v 2 - v1 D v
aav = = .
t 2 - t1 Dt
Acceleration occurs due to change of velocity of the object. The velocity of the object may change
due to change in its magnitude or may due to change in direction of motion or due to change in both
magnitude and direction.
Instantaneous acceleration
The acceleration of any object at any instant is called its instantaneous acceleration. It is equal to the
limiting value of the average acceleration of the object when time interval approaches to zero . Thus
r r
r Dv dv
a = lim =
Dt ® 0 Dt dt
r
r ds
As v= ,
dt
r r
r d æ d sö d 2 s
\ a= ç ÷ = 2 .
dt è dt ø dt
Acceleration can also we written as
dv dv ds dv
a= = . =v
dt dt ds ds
r
r r dv
In vector form, a = v. .
ds
3.5 EQUATIONS OF MOTION
Consider a body moving along a straight line with constant acceleration a. Let its initial velocity be u,
after covering a displacement s its velocity becomes v.

Figure. 3.5
(i) First equation of motion
By the definition, the acceleration
dv
a =
dt
[Here we can drop the vector sign with displacement, velocity and acceleration, because
motion is along the straight line]
Above equation can be written as;
dv = adt …(i)
Integrating equation (i), we get
v t
ò dv = ò adt
u 0
v t
vu = at0
or (v – u) = a(t – 0)
or v = u + at …(1)
www.crackjee.xyz
98 MECHANICS
(ii) Second equation of motion
By the definition, the instantaneous velocity v is given by
ds
v = or ds = vdt
dt
From equation (i), we have v = u + at
\ ds = (u + at) dt …(ii)
Integrating equation (ii), we get
s t

ò ds = ò (u + at )dt
0 0
t
s 1 2
s0 = ut + at
2 0
1 2
or s = ut + at . …(2)
2
(iii) Third equation of motion
By the definition, acceleration a is given by
dv dv ds dv
a = = ´ =v
dt dt ds ds
or vdv = ads …(iii)
Integrating equation (iii), we have
v s v
v2
ò vdv =
ò ads or
2
u
= a | s |0s
u 0
\ (v2 – u2) = 2a(s – 0)
2 =
or v u 2 + 2as. …(3)
(iv) Fourth equation of motion
By the definition of velocity, we have
ds
v = or ds = vdt
dt
or ds = (u + at) dt …(iv)
Let at t = n – 1, displacement travelled = sn – 1 and at t = n, displacement travelled = sn.
Thus the displacement travelled in nth second snth = sn – sn–1
Integrating equation (iv), we have
sn n

ò ds = ò (u + at ) dt
sn -1 ( n -1)
n
s 1 2
| s |sn = ut + at
n -1 2 n -1
é 1 2ù é 1 2ù
or sn – sn –1 = êun + an ú - êu (n - 1) + a(n - 1) ú
ë 2 û ë 2 û
a
or snth = u+ (2n - 1) . …(4)
2
1
Calculating distance from s = ut + at 2
2
Case 1. When acceleration and velocity are along same direction.
In such cases the displacement calculated by second equation of motion is equal to the
distance travelled.
Case 2. When acceleration and velocity are opposite to each other.
www.crackjee.xyz
Motion in a Straight Line 99
In this case, if t0 is the time at which the velocity becomes zero, then

0 = u – at0 Þ t0 = æç ö÷ .
u
\
è aø

Figure. 3.6
(a) For the motion t £ t0, the displacement calculated by second equation of motion is equal to
distance travelled. Thus we have
2
1 2 u 1 æ uö
s = ut0 - at0 = u ´ - a ç ÷
2 a 2 è aø
u2
or AB = .
2a
u
(b) For the motion t > t0: Let t = 1.5 t0 = 1.5
a
1 2
Displacement s = ut - at
2
2
æ uö 1 æ uö
= u ç1.5 ÷ - a ç1.5 ÷
è aø 2 è aø
Figure. 3.7
æ u2 ö
or AC = 0.375 ç a ÷
è ø
Distance, s = AB + BC
AB is the distance travelled in the duration t0. BC is the distance travelled in the duration
t – t0 .
2 1
Here AB = u calculated previously and BC = v B (t - t0 ) + a (t - t0 )
2
2a 2
As vB = 0,
1 1
\ BC = a (1.5t0 - t0 ) 2 = a ´ (0.5t0 ) 2
2 2
2
1 æ uö u2
= a ´ 0.52 ´ ç ÷ = 0.125
2 è aø a
Thus total distance s = AB + BC
u2 u2 u2
= + 0.125 = 0.625 .
2a a a

Thus if body travels for time t and it changes the direction of motion after t0, where t > t0, then
1
displacement is given by s = ut - at 2
2
1
and distance is given by s = ut0 - at0 2 + 1 a(t - t0 )2
2 2
In case when acceleration is not constant
Let the time of motion be t and body changes direction of motion after time t0 (t0 < t).
t

The displacement is given by, s = ò v dt


0
t0 ( t -t 0 )
The distance travelled is given by s = ò v dt + ò v dt
0 t0
www.crackjee.xyz
100 MECHANICS
Motion in presence of air resistance
Suppose a body is thrown vertically upward with initial velocity u. Let it experiences a constant
retardation ‘a’ due to air. In upward journey, its retardation is (g + a) and in downward journey its
acceleration becomes (g – a). Let t1 be the time of upward journey and t2 be the time of downward
journey, then:
Upward journey : From first equation
v = u – (g + a) t1
As v = 0
u
\ t1 = . …(i)
( g + a)
Height attained by the body;
v2 = u2 – 2 (g + a)h
h = u2 – 2(g + a) h
u2
or h = . …(ii)
2( g + a)
Downward journey: u = 0
1 2
Figure. 3.8 \ h = 0 + ( g - a)t2
2
u2 1
or = ( g - a )t2 2
2( g + a) 2
u
or t2 = . …(iii)
g 2 - a2
To understand simply, let g = 10 m/s2 and a = 1 m/s2
u u
\ t1 = = s
10 + 1 11
u u
and t2 = =
2
10 - 1 2 99 s.
Now it is clear that t2 > t1 i.e., time of descend will be greater than time of ascend.

Stopping distance :
The distance travels before stopping when breaks are applied is called stopping distance.
It depends on the initial velocity of the vehicle and the braking capacity or deceleration (–a) that is
caused by braking.
If u is the initial velocity of the vehicle, then using third equation of motion,
v2 = u2 + 2as, we have
0 = u2 + 2 (–a)s
æ u2 ö
\ s = ç ÷
è 2a ø
Clearly, if speed of which is doubled, its stopping distance will be four times.
Reaction time : Reaction time is the time a person takes to observe, think and act. For example, if
a person is driving and suddenly a boy appears on the road, then time elapsed before he applied the
brakes of the car is the reaction time. The reaction time roughly is about 0.2s.

FORMULAE USED
total distance s s1 + s2 + ...... + sn
1. (i) Av. speed = = =
total time t t1 + t2 + ..... + t n
For body cover different distances with different speeds,
s1 + s2 + ...... + sn
vav =
æ s1 s2 sn ö
ç + + ..... + ÷
è v1 v2 vn ø
www.crackjee.xyz
Motion in a Straight Line 101
(ii) For a body having different speeds in different time
v1t1 + v2t2 + ..... + vntn
vav =
t1 + t2 + ....... + tn
t2

ò vdt
t1
vav =
(t2 - t1)
2. Equations of motion
1
(i) v = u + at (ii) s = ut + at 2
2
th a
(iii) v2 = u2 + 2as (iv) sn = u + (2n - 1)
2
3. Motion under gravity
1 2
(i) v = u + gt (ii) h = ut + gt
2
th g
(iii) v 2 = u 2 + 2 gh (iii) hn = u + (2 n - 1)
2
4. For a freely falling particle under gravity, g is taken as positive and for body thrown upward, g
is taken negative.
5. For a particle just drop, u = 0.
6. Rising of balloon is not motion under gravity, in such a case its acceleration is positive in
upward direction.

PROBLEM SOLVING STRATEGY


Average velocity
1. If particle is going in a straight line without change in direction of motion, then
Average velocity = average speed
2. If body changes direction of motion, then find displacement and distance separately. In this
case
average velocity < average speed
Motion with constant acceleration
Identify the relevant concepts : In majority of problems, you can use the constant acceleration
equations. Occasionally, however, you will encounter a situation in which the acceleration is not
constant. In such a case you need a different approach, which you will encounter is next section.
Setup the problem using the following steps :
Step I First decide which direction of motion is positive. It is often easiest to place the particle at
the origin at t = 0; then x = 0.
Step II (i) Remember that your choice of positive axis direction automatically determines the
positive direction for x-velocity and x-acceleration, if x is positive to the right of the
origin, then vx and ax are also positive towards the right.
(ii) You may choose cartesion coordinates system for the motion parameters. Students
always confused in putting the signs in case of motion in vertical direction. According
to this system, if particle is moving up then, displacement = +y, velocity = +v, acceleration
= –g.
When particle moves down, displacement = –y, velocity = –v and acceleration = –g.
Figure 3.9 Motion in 1D
Step III Make a list of all the known quantities, such as x, u, a and t. Look out the informations
given; as car starts with constant acceleration, here u = 0. A particle starts falling, u = 0. A
particle thrown vertically up v = 0.
Execute the solution
Choose an equation of motion that contains only one unknown. Then substitute the known values
and compute the unknown quantity. Sometimes you will have to solve two simultaneous equations
for two unknown quantities.
www.crackjee.xyz
102 MECHANICS

EXAMPLES BASED ON AVERAGE VELOCITY AND CONSTANT ACCELERATION


Example 1. A cyclist travels from centre O of a circular park of s
v1 + v2
radius 1 km and reaches point P. After cycling 1/4 th of the \ = t , ...(i)
circumference along PQ, he returns to the centre of the park QO. 2 1
If the total time taken is 10 minute, calculate [NCERT] s
v2 + v3
= t , ...(ii)
2 2

v3 + v4 s
= t , ...(iii)
2 3

v1 + v4 3s
and = t +t +t ...(iv)
2 1 2 3
Doing (i) – (ii) + (iii), we get
Figure. 3.10
(i) net displacement v1 + v4 é1 1 1 ù
(ii) average velocity and = s êt - t + t ú ...(v)
2 ë 1 2 3û
(iii) average speed of the cyclist.
Now from equations (iv) and (v), we get
Sol. (i) The net displacement becomes zero.
(ii) As net displacement is zero, so average velocity 1 1 1 3
- + = t +t +t .
t1 t2 t3 1 2 3
Net displacement
vav = = 0. Example 3. A point traversed half the distance with a velocity
time taken
vo. The remaining part of the distance was covered with velocity v1
(iii) Total distance covered = OP + Arc PQ + OQ for half the time, and with velocity v2 for the other half of the time.
2 pr Find the mean velocity of the point averaged over the whole time of
= r+ +r motion.
4
Sol. Since direction of motion is not changing
2 ´ 22 ´ 1 25
= 1+ +1 = km total distance
7´4 7 \ average or mean velocity = .
total time
Time taken = 10 min = 1/6 h.
Let t0 and t be the time of motion of first half of distance and rest half of
total distance covered the distance respectively, then
Average speed =
time taken

æ 25 ö
çè ÷ø
7 Figure. 3.12
= = 21.43 km / h Ans.
æ 1ö s/2 + s/2
çè ÷ø vav =
6 t0 + t
Example 2. If a point moves in a straight line with uniform s/2 vt v t s
acceleration and covers successive equal distances in times t1, t2, t3, where t0 = , and 1 + 2 =
v0 2 2 2
then show that
3 s
1 1 1 or t =
- + = . v1 + v2
t1 t 2 t 3 t1 + t 2 + t 3
Sol. Let s be the successive equal distances and v1, v2, v3 the initial s
velocities for the successive distances and v4, the final velocity in the \ vav = s / 2 s
+
third distance. Since the acceleration is constant, so velocities in the v0 (v1 + v2 )
three intervals and in the total time are
v1 + v2 v2 + v3 v3 + v4 v +v 2v0 (v1 + v2 )
, , and 1 4 . = . Ans.
2 2 2 2 2v0 + v1 + v2
Example 4. A man walks on a straight road from his home to a
market 2.5 km away with a speed of 5 km / h. Finding market closed,
Figure. 3.11 he instantly turns and walks back home with a speed of 7.5 km/h.
We know that average velocity What is the [NCERT]
(a) magnitude of average velocity,
distance (b) average speed of the man over the interval of time
vav = ,
time (i) 0 to 30 min (ii) 0 to 50 min (iii) 0 to 40 min ?
www.crackjee.xyz
Motion in a Straight Line 103
Sol. The time taken in ongoing journey from A to B is Example 6. A particle is moving at a speed of 5 m/s along east.
2.5 1 After 10 s its velocity changes and becomes 5 m/s along north.
= = hr = 30 min What is the average acceleration during this interval?
5 2
Sol.
– v1
Figure. 3.13 N
Time taken in returning journey from B to A
v 2 = 5 m/s v
2.5 1 – v2 W E
= = hr = 20 min 2
v
7.5 3 1

(i) In the interval 0 to 30 min; v1 = 5 m/s q S


displacement Figure. 3.15
Average velocity = Average acceleration is given by
time
r r
r v2 - v1
2.5 a =
= = 5km / h Dt
1/ 2
(ii) In the interval 0 to 50 min; 52 + 52 1
The distance travelled in 20 min. in return journey is 2.5 km. \ a = = m/s 2
10 2
displacement 0 The direction of acceleration is the direction of change of velocity,
Average velocity = = =0
time 1 1 ur ur
+ v 2 - v1 . Thus
2 3

distance 2.5+2.5 v1 5
Average speed = = = 6 km/h tan q = v = 5 = 1
time 1 1 2
+
2 3 or q = 45° North-West. Ans.
(iii) In the interval 0 to 40 min; Example 7. Let a body falls from height h. After collision with
The distance travelled in 10 min in return journey the ground it rises to height h¢. Suppose Dt is the time of collision,
10 then find the average acceleration during contact.
= ´ 7.5 = 1.25 km
60
Sol.

Figure. 3.16
Velocity before collision v1 = 2 gh .
Figure. 3.14
Velocity after collision v2 = - 2 gh ' .
net displacement
Average velocity =
time The change in velocity Dv = - 2 gh ' - 2 gh .
2.50 - 1.25
= 1 1 = 1.875 km/h Dv - 2g ( h ' + h )
+ Thus acceleration a= = .
2 6 Dt Dt
Example 8. An athlete runs a distance of 1500 m in the following
total distance manner. (i) Starting from rest, he accelerates himself uniformly at
Average speed =
total time 2m/s2 till he covers a distance of 900 m. (ii) He, then runs the
remaining distance of 600 m at the uniform speed developed.
2.50 +1.25 Calculate the time taken by the athlete to cover the two parts of
= = 5.625 km/h.
1 1 the distance covered. Also find the time, when he is at the centre of
+
2 6 the track.
Example 5. A particle starts moving along x axis, with constant Sol. The situation is shown in figure.
velocity of 4 m/s. After 2 s from the start of motion of the first
particle, another particle starts in the same direction, with the
same position with constant velocity of 6 m/s. Calculate the time at Figure. 3.17
which second particle will catch the first particle. For the motion between t = 0 to t1; s = 750 m. We know that
Sol. If t is the time, the first particle takes, till meeting, then for the 1 2
second particle the time will be (t–2). For the meeting, the displacement s = ut + at
2
of both the particles must be same. Thus
1 2
4t = 6 (t – 2) 750 = 0 + ´ 2 ´ t1
or t = 6s Ans. 2
\ t1 = 27.4 s.ns.
www.crackjee.xyz
104 MECHANICS
(i) For the motion fromt = 0 to t = t2; s = 900 m Sol.
1
900 = 0 + ´ 2 ´ t2 2
2
\ t2 = 30 s. Ans.
(ii) Let v is the velocity of the athlete at t = t2, then Figure. 3.18
v = 0 + 2 × 30 Let the full speed of man is v, and he takes time t to pick the train. In this
= 60 m/s.
æ 1 ö
For the motion between t2 and t3; s = 600 m. time his distance from bus becomes ç 9 + at 2 ÷ . He covers this distance
è 2 ø
If t is the time of motion, then
with constant speed v, then we have
distance
t = 1
speed vt = 9 + at 2
2
600 1
= = 10 s. Ans. or ´ 2 ´ t 2 - vt + 9 = 0
60 2
Example 9. A driver takes 0.20 s to apply the brakes after he or t 2 - vt + 9 = 0
sees a need for it. This is called the reaction time of the driver. If he
is driving car at a speed of 54 km/h and the brakes cause a v ± v2 - 4 ´ 1 ´ 9 v ± v 2 - 36
\ t= =
deceleration of 6.0 m/s2, find the distance travelled by the car after 2 2
he sees the need to put the brakes. t will have real solution, if
Sol. During the reaction time car continues to move with constant v2 – 36 ³ 0
speed of 54 km/h or 15 m/s. or v ³ 6 m/s.
Thus, v = 6 m/s is enough to pick the train
\ Distance travelled during this time
v 6
s1 = 15 × 0.20 = 30 m The time t= +0= = 3 s. Ans.
2 2
There after brakes start decelerating the car, the distance travelled till
The distance travelled by man = vt = 6 × 3 = 18 m. Ans.
stop is s2, then
Example 12. Two balls are thrown simultaneously, A vertically
v2 = u2 – 2as
upwards with a speed of 20 m/s from the ground, and B vertically
or 0 = 152 – 2 × 6 × s2 downwards from height of 40 m with the same speed along the
same line of motion. At what point do the two balls collide ? Take
152 g = 9.8 m/s2.
\ s2 = = 18.75 m.
12 Sol. Suppose the balls collide at a height y from the ground after time
Total distance travelled = s1 + s2 = 3.0 + 18.75 = 21.75 m. Ans. t from the start.
For downward motion of ball B
Example 10. A body covers 12 m in 2nd and 20 m in 4th second.
1 2
How much distance will it cover in 4 second after the 5th second ? 40 – y = 20 t + gt ........(i)
2
Sol. Given : s2nd = 12 m, s4th = 20 m. For upward motion of ball A
a 1
We known that sn = u + (2 n - 1) y = 20t - gt 2 ........(ii)
2 2
a 3a
\ 12 = u + (2 ´ 2 - 1) = u + …(i)
2 2

a 7a
and 20 = u + (2 ´ 4 - 1) = u + …(ii)
2 2
Solving equations (i) and (ii), we get
a = 4m/s2and u = 6 m/s.
Now distance covered in 4 second after 5th second = s9 – s5

æ 1 2ö æ 1 2ö
= çè 6 ´ 9 + ´ 4 ´ 9 ÷ø - çè 6 ´ 5 + ´ 4 ´ 5 ÷ø
2 2
Figure. 3.19
= 136 m. Ans. Adding equations (i) and (ii), we get
Example 11. A man is s = 9 m behind the door of a train when 40 = 40t or t=1s
it starts moving with acceleration a = 2m/s2. The man runs at full Now from equation (ii), we get
speed. How far does he have to run and after what time does he get 1
into the train ? What is his full speed ? y = 20 ´ 1 - 9.8 ´ 12 = 15.1 m Ans.
2
www.crackjee.xyz
Motion in a Straight Line 105

In Chapter Exercise 3.1

1. A proton moves along the x-axis according to the equation covered during acceleration be 0.5 km, find the time lost
in the journey. Ans. 1 minute
x - 50t
t= , where x is in metres and t is in seconds. 5. A body falling freely under gravity passes two points 30
10 m apart in 1s. Find, from what point above the upper
Calculate the average velocity of the proton during the point it began to fall ? Take g = 9.8 m/s2. Ans. 32.1 m
first 3.0 s of its motion. Ans. 80 m/s.
6. A car moving with constant acceleration covered the
2. A body travels, with uniform acceleration for time t1 and distance between two points 60.0 m apart in 6.00 s. Its
with uniform acceleration a2 for time t2. What is the average speed as it passes the second point was 15.0 m/s.
acceleration ?
(a) What was the speed at the first point ?
a1t1 + a2 t 2 (b) What was the acceleration ?
Ans.
t1 + t 2 (c) At what prior distance from the first point was the
car at rest ?Ans. (a) 5.00 m/s (b) 1.67 m/s2 (c) 7.50 m.
3. A body travels 200 cm in the first 2 second and 220 cm in
the next 4 second. What will be the velocity at the end of 7. Two bodies are projected vertically upwards from one
the 7th second of the start ? Ans. 10 cm/s point with the same initial velocities v0, the second t sec
after the first. How long after will the bodies meet ?
4. A train moving with a velocity of 30 km/h has to slow
down to 15 km/h due to repairs along the road. If the v0 t
distance covered during retardation be 1 km and that Ans. t = + .
g 2

3.6 STUDY OF MOTION BY GRAPHS


Our previous knowledge reveals that, the motion of uniformly accelerated body can be studied by
1
v = u + at and s = ut + at 2 . The equation v = u + at is an inclined straight line between v and t and
2
1 2
s = ut + at is a parabola between s and t.
2
From the above discussion it is clear that the curve of a uniformly accelerated body be either a straight
r r
line between v and t and parabola between s and t, no circle no ellipse.

Figure. 3.20
The following points must be remembered regarding with graphs :
1. Usually independent variable is taken on the x-axis and dependent variable is taken on the
y-axis.
2. Usually the previous direction of motion is taken as positive. A body thrown up; its upward
journey is taken as positive and return journey negative. If a body thrown down; its downward
journey is taken as positive and return journey is taken as negative. Sometimes the graph may be
drawn according to cartesian coordinate system.
3. When body moves along a straight line without change in direction its distance-time and
displacement time graphs remain identical. Similarly its speed-time and velocity-time-graphs
remain identical.
4. As distance and speed can never be negative, so no part of their graphs can be below time axis.
5. Any line perpendicular to time axis indicates, that quantity under consideration is changing
without spending the time, which is not possible. During collision a very large change in velocity
occurs in very short time. Also no proper informations about motion during collision are available.
Therefore in such cases time of collision is neglected.
www.crackjee.xyz
106 MECHANICS
About slope
The slope of the curve at any point is defined by tan q, where q is the angle made from x-axis. In terms
of differentiation, tan q = dy/dx. The following points should be remembered regarding with the slope:
1. A straight line graph has a single slope. If line makes angle q < 90°, the slope tan q = +ve, and if
line makes an angle q > 90°, the slope tan q = –ve. For line parallel to x-axis, q = 0, tan q = 0
(see figure).

y y y

q
q
x x x
O O O
Zero slope Positive slope Negative slope
2. A curved graph has variable slopes. In a curve with a trough, the slope increases with increasing
value of x. In a curve with crest upward, slope decreases with increasing value of x (see figure).
y y
q2
q2
q1
q1
O x O x
q1 < q2 q1 > q2

Graphs showing rest :


r r r r
For a particle is at rest, its position will not change with time and so s1 = s2 . Also v = 0 and a = 0 .
r r
The following graphs represents rest on s - t , v - t and ar - t :
s v a

t t t
O O O
Graphs showing constant velocity :
The slope of displacement-time curve gives velocity, so for constant velocity, it must be a
r
straight line. For v = constant, ar = 0 . The following graphs represents constant velocity on
r r r
s - t , v - t and a - t :

s v a

t t t
O O O
Graphs showing constant acceleration :
For constant acceleration motion of a particle, the dis-time curve must be of increasing slope
(increasing velocity), so it is a parabolic curve with trough upward.
r
dv r
As acceleration is constant, so = a (constant). It must be a straight line between velocity-
dt
r r r
time. The following graphs represents constant acceleration on s - t , v - t and a - t :
s v a

t t t
O O O
www.crackjee.xyz
Motion in a Straight Line 107
Graphs showing constant retardation :
For constant retardation motion of a particle, the disp-time curve must be of decreasing slope
(decreasing velocity), so it is a parabolic curve with crest upward.
As acceleration is negative and so it must be a straight line with negative slope between
r r
velocity-time. The following graphs represents constants retardation on s - t , vr - t and a - t :

s v a

O t
t t
O O
Following graphs cannot exist in practice :
• Distance travelled by a particle cannot decrease with time. Also it never negative.
• Speed also can never be negative.
• A quantity cannot change without spending time.
speed
distance

O t
O t
distance/ speed

any quantity

t t
O O

Quantities calculated from different types of graphs :


r
r
ds
1. Slope of displacement-time graph gives the velocity at the point. i.e., v = .
dt
r
r dv
2. Slope of velocity-time graph gives the acceleration at the point. i.e., a = .
dt
3. The area under the speed-time or velocity-time graph gives v
the displacement/ distance. In the following vr - t graph, the
displacement and distance may be calculated as follows :
displacement = area A – area B, A
distance = area A + area B. t
O B
4. The area under acceleration-time graph gives the change in velocity.i.e.,
r
r Dv
a = Dt a
r r
or Dv = a ´ Dt
r A
= area of a – t graph
t
= area A – area B O B
r r r
If vi = 0, v f = area of a - t graph. Dt
www.crackjee.xyz
108 MECHANICS

EXAMPLES BASED ON GRAPHS


Example 13. The position-time (x-t) graphs for two children A Example 15. A ball is dropped and its displacement versus time
and B returning from their school O to their homes P and Q graph is as shown in given figure. (Displacement x is from ground
respectively are shown in figure. Choose the correct entries in the and all quantities are +ve upwards). [NCERT Exemplar]
brackets below: [NCERT] x
(a) (A/B) lives closer to school than (B/A)
(b) (A/B) starts from the school earlier than (B/A)
(c) (A/B) walks faster than (B/A)
(d) A and B reach home at the (same/different) time.
(e) (A/B) overtakes on the road (once/twice)

t
Figure. 3.23
(a) Plot qualitatively velocity versus time graph.
(b) Plot qualitatively acceleration versus time graph.
Sol. From the given graph x is positive upwards. Ball is dropped from
a height and its velocity increases in downward direction due to gravity
pull. In this condition v is negative but acceleration of the ball is equal to
Figure. 3.21 acceleration due to gravity i.e., a = –g.
Sol. (a) It is clear from the graph that OP < OQ, \ A lives closer to When ball rebounds in upward direction its velocity is positive but
the school than B. acceleration is a = –g
(b) As A starts from t = 0 while B starts little later. So A starts the (a) The velocity-time graph of the ball is shown in fig (i).
school earlier than B.
v
(c) The slope of x – t graph for motion of B > slope of x – t graph of
A. Hence B walks faster than A.
(d) The value of t corresponding to positions P and Q of there homes
is same, so A and B reach home at the same time. O t
(e) It is clear from the graph that B overtake A once on the road.
Example 14. A train moves from one station to another in two
hours time. Its speed time-graph during the motion is shown in
figure. Figure (i)
(i) Determine the maximum acceleration during the journey (b) The acceleration-time graph of the ball is shown in fig (ii).
(ii) Also calculate the distance covered during the time interval
a
from 0.75 hour to 1 hour.

O t

–g

Figure (ii)
Example 16. Suggest a suitable physical situation for each
of the following graphs? [NCERT]
Figure 3.22 Sol. Figure (a): The x-t graph shows that initially x is zero i.e. at
rest, then it increases with time, attains a constant value and again
Sol.
reduces to zero with time, then it increases in opposite direction till it
(i) As the part BC of the graph has maximum slope, so acceleration is
again attains a constant value i.e. comes to rest. The similar physical
maximum in this duration.
situation arises when a ball resting on a smooth floor is kicked which
\ Maximum acceleration a = slope of line BC rebounds from a wall with reduced speed. It then moves to the opposite
v2 - v1 (50 - 20) wall, which stops it.
= =
t2 - t1 1.00 - 0.75
x x a
= 120 km/h2. Ans.
(ii) Distance travelled in the duration 0.75 to 1 hour.
= Area of trapezium BCEF A
t t t
B
1
= [20 + 50] × (1.00 – 0.75)
2
= 8.75 km. Ans. (a) (b) (c)
www.crackjee.xyz
Motion in a Straight Line 109
Figure (b): The velocity changes sign again and again with passage of Distance travelled by scooter in 15 s
time and every time some speed is lost. The similar physical situation = area of rectangle OCFE
arises when a ball is thrown up with some velocity, returns back and = 15 × 30 = 450 m.
falls freely. On striking the floor, it rebounds with reduced speed each Thus difference between distance travelled by them
time it strikes against the floor. = 450 m – 337.5 m
Figure (c): Initially body moves with uniform velocity. Its acceleration
= 112.5 m. Ans.
increases for a short duration and then falls to zero and thereafter the
(ii) Let after time t from start car will catch up the scooter. In time t the
body moves with a constant velocity. The similar physical situation
arises when a cricket ball moving with a uniform speed is hit with a bat distance travelled by them are equal.
for very short interval of time. 1
Distance travelled by car = × 15 × 45 + 45 (t – 15).
Example 17. A woman starts from her home at 9.00 am, walks 2
with a speed 5km/h on straight road up to her office 2.5 km away, Distance travelled by scooter = 30 t
stays at the office up to 5.00 pm and returns home by an auto with
a speed of 25 km/h. Choose suitable scales and plot the x-t graph 1
\ × 15 × 45 + 45 (t – 15) = 30t
of her motion. [NCERT] 2
Sol. Time taken in reaching office = distance/ speed = 2.5/5 = 0.5 hr. which gives t = 22.5 s. Ans.
Time taken in returning from office = 2.5/25 = 0.1 hr. = 6 minutes Distance travelled by car or scooter in 22.5 s
It means the woman reaches the office at 9.30 am and returns home at = 30 × 22.5
5.06 pm. The x-t graph of this motion will be as shown below: = 675 m Ans.
So the car catches the scooter when both are at 675 m from the
3 starting point.
x (in km)

Example 19. A body falls from some height and returns back
2 to initial position. Draw displacement ( rs ) – time (t), distance (s) –
r r
time (t), velocity ( v ) – time, speed-time and acceleration ( a ) –
time (t) graphs for the motion of the body.
1
2h
Sol. Let body falls from height h. It takes time g
to strike the

9.00 11.00 1.00 3.00 5.00 5.06 ground, its velocity just before strike is 2gh . Neglecting time of
t (in hour ) collision, we have following graphs:
Figure 3.24 distance (s)
Example 18. As soon as a car just starts from rest in a certain s
direction, a scooter moving with a uniform speed overtakes the car. 2h
Their velocity-time graphs are shown in figure. Calculate
(i) The difference between the distances travelled by the car
and the scooter in 15 s. h h
(ii) The distance of car and scooter from the starting point at
that instant.
t O t
O 2h /g 2 2h /g
v(m/s) 2h/g 2 2h/g
v speed (v)
60 2gh

A Car B
45
O t 2gh
E Scooter 2h/g 2 2h/g
30 F G

– 2gh
O t
15 2h/g 2 2h /g
C D t(s)
O 5 10 15 20 25 30

Figure 3.25
Sol.
(i) The distance travelled by car in 15 s
= area of DOAC
1
= × 15 × 45 = 337.5 m
2 Figure 3.26
www.crackjee.xyz
110 MECHANICS
Example 20. A body is thrown up and returns back to its initial The slope of line OP,
position. Draw displacement ( s ) – time (t), distance (s) – time (t), vmax
a = t1
velocity ( v ) – time (t) and speed (s) – time (t) graphs for the
Þ vmax = at1 …(i)
motion of the body. The slope of line PQ,
u vmax
Sol. Let the body be thrown with initial velocity u, it takes time g to
b = t -t
1
u2 Þ vmax = b (t – t1). …(ii)
reach the highest position. It goes to a height h = . Neglecting air
2g From equations (i) and (ii), we get
resistance we have following graphs: at 1 = b (t – t1)
distance (s)
u2 bt
s g which gives t1 = . …(iii)
a +b
u2
u2 Substituting value of t1 in equation (i), we get
2g
2g
abt
(i) vmax = . Ans.
a +b
t t uur
O u 2u O u 2u (ii) Total displacement s = area of v - t graph
g g g g
v speed (v) 1
= ´ vmax ´ t
u 2

1 abt 2
= ´ ´ t = abt . Ans.
u 2 a+b 2(a + b)
O u t
2u Example 22. The distance (s) between two stations is to be
g g
covered in minimum time. The maximum value of acceleration or
–u retardation of a car can not exceed a and b respectively. Find the
O u 2u t
g time of motion.
g
Sol. To cover the distance in minimum time the car must get the maximum
possible acceleration a and then retard to maximum possible value b.
Let t1 is the time up to which car accelerates and t is the required time of
motion. The velocity-time graph of motion of car can be drawn as in figure
3.29.

Figure 3.27
Example 21. A car accelerates from rest at a constant rate a for
some time, after which it decelerates at a constant rate of b to
come to rest. If the total time elapsed is t second, then calculate;
(i) the maximum velocity attained by the car, and
(ii) the total displacement travelled by the car in terms of a, b
and t.
Sol. Let vmax be the maximum velocity attained and t1 be the time at
which maximum velocity will occur. The velocity vs time graph can be Figure 3.29
drawn as follows:
abt 2
We have already calculated that s = .
2(a + b)
Solve above equation for t, we have

2 s ( a + b) æ 1 1ö
t= = 2s ç + ÷ . Ans.
ab è a bø
Example 23. A particle of mass m moves on x-axis as follows:
It starts from rest at t = 0 from the point x = 0 and comes to rest at
t = 1 and the point x = 1. No other information is available about its
motion at intermediate times (0 £ t £ 1). Discuss about the
Figure 3.28 acceleration of the particle.
www.crackjee.xyz
Motion in a Straight Line 111
Sol. Let a and b are the acceleration and retardation of the particle v
during the motion.
The velocity-time graph of motion of a particle is shown in figure 3.30. P Q
5t

a a

O t 25 t
t 25 – 2 t t
Figure 3.31
Given the average velocity in whole time of motion
Figure 3.30 72 ´ 5
vav = = 20 m/s.
We have, from the graph (already calculated) 18
The average velocity from the graph can be obtained as
abt 2
s = x= total displacement
2(a + b ) vav =
total time
Given x = 1m, t = 1s uur
area of v - t graph
a 2 (1) 2 =
total time
let | a | = | b |, then 1 =
2(a + a) 1
which gives a = 4 ´ [25 + (25 - 2t )] ´ 5t
\ 20 = 2
25
When | a | > | b | then | a | will be greater than 4, and | b | will be less
than 4. 1
´ [50 - 2t )] ´ 5t
Example 24. A car starts moving rectilinearly, first with = 2
acceleration a = 5 m/s2 (the initial velocity is equal to zero), then 25
uniformly, and finally, decelerating at the same rate a comes to a or 200 = 50t – 2t2
stop. The total time of motion equals t = 25 s. The average velocity or t2 –25t + 100 = 0
during that time is equal to <v> = 72 km/h. How long does the car (t – 20)(t – 5) = 0
move uniformly? t = 5 s or 20 s
Sol. Let t be the time upto which car accelerates or decelerates. The But t = 20 is not possible
maximum velocity attained in this duration is 5 t. The time upto which \ t = 5 s.
car move uniformly = 25 – 2t. The velocity – time graph of the motion of The time upto which car moves uniformly
car is drawn as in figure 3.31. = 25 – 2t = 25 – 2 × 5
= 15 s. Ans.

In Chapter Exercise 3.2

1. Figure shows the velocity-time graph for the motion of a v (m/s)


certain body. Determine the nature of this motion. Find 20
acceleration and write the variation of displacement with 10
time.
0
2 4 6 8 10 t(s)
- 10
v(m/s) - 20

Ans: 100 m , 60 m
10
8 A 3. The speed-time graph of a particle moving along a fixed
6 direction is shown in figure.
4 Speed (m/s)
2 B
0 t(s) 12
5 10 15
C

O t(s)
5 10
Ans. a = 0.64 m/s2, s = 7t – 0.32 t2. Find :
2. The velocity - time graph of an object moving along (i) distance travelled by the particle between 0s to 10s,
(ii) average speed between this interval,
straight line is shown in the figure. Find the net distance
(iii) the time when the speed was minimum.
covered by the object in time interval between t = 0 to (iv) the time when speed was maximum.
t = 10 s. Also find the displacement in time 0 to 10 s. Ans. (i) 60 m (ii) 6 m/s (iii) 0s and 10 s. (iv) 5s.
www.crackjee.xyz
112 MECHANICS

4. Figure shows the position - time graphs of three cars A, B (ii) Are the three cars ever at the same point on the road?
and C. On the basis of the graphs, answer the following
(iii) When A passes C, where B is ?
questions :
(i) Which car has the highest speed and which the lowest? (iv) How far did car A travel between the time it passed
cars B and C ?
x(km) (v) What is the relative velocity of car C with respect to
car A ?
14
C B A
(vi) What is the relative velocity of car B with respect to
12
10 car C ?
8
Ans : (i) C has the highest speed and A has the lowest speed
6
4 (ii) No (iii) 6 km from the origin (iv) 6 km
2
t(h) (v) 7 km/h (vi) - 2 km/h
0 0.2 0.4 0.6 0.8 1.0 1.2 1.4 1.6

3.7 RELATIVE VELOCITY


Consider the motion of the car moving towards right and two observers O1 and O2 are coming from
opposite directions as shown in figure 3.32.
Observer O1 finds that car is moving slower while observer O2 finds that car is moving
faster in comparison to when observer is at rest. The motion of same object looks
different for two different observers. To understand such observations, there is a
Figure 3.32
need of the concept of relative velocity.
r r
Consider two objects A and B moving with constant velocities v A and v B in one dimension, say
along x-axis. Let objects start from origin, their positions xA and xB at time t are given by :

vA q
vB

180° – q
q
v AB

vA –vB

Figure 3.33
xA = vA t …(i)
and xB = vB t …(ii)
Then, the displacement from object A to object B is given by
xBA = xB – xA
= (vB – vA) t. …(iii)
Equation (iii) can easily be understood. It tells us that as seen from object A, object B has a velocity vB
– vA. We can say that the velocity of the object B relative to the object A is;
ur ur ur
v BA = v B – vA . …(1)
Similarly, velocity of the object A relative to object B is:
ur ur ur
vAB = vA – v B . …(2)
r ur
The result of v BA or vAB depends on the angle between their directions of motion.
ur ur
Now consider two objects A and B moving with velocities vA and v B respectively, having an angle q
between their directions of motion as shown in fig. 3.33.
www.crackjee.xyz
Motion in a Straight Line 113
The relative velocity of object A with respect to object B is given by
ur ur ur
vA B = v A – v B

or vAB = v A2 + vB 2 + 2v A vB cos(180° - q)

v A 2 + v B 2 - 2v A v B cos q .
=
ur ur
Suppose the relative velocity vAB makes an angle a with vA , then
vB sin(180° - q)
tana =
v A + vB cos(180° - q)
vB sin q
or tana = .
v A - vB cos q
x(m) x(m)
B A

A B
vA > vB
vA = vB

t(s) t(s)
Position-time graph of two Position- time graph of two
objects with equal velocities. objects with unequal velocities.
Figure 3.34
Special cases :
1. When both the objects are moving along same direction, we have q = 0°

\ vAB = v A 2 + v B 2 - 2v A v B cos q
= v A – v B.
Thus the relative velocity of object A with respect to object B is equal to the difference between
magnitudes of their velocities.
If vA = vB, vA – vB = 0. The relative velocity vAB or vBA becomes zero.
2. When the objects are moving in opposite directions, we have q = 180°.

\ vAB = v A 2 + v B 2 - 2v A v B cos q
= vA + vB .
Thus relative velocity of object A with respect to B is equal to the sum of magnitudes of their
velocities.
Position time graph of two objects with velocities in opposite directions.

Figure 3.35
Relative acceleration
The treatment that we have done for relative velocity, can be done for relative acceleration also. Thus
relative acceleration of the object A with respect to B is given by :
www.crackjee.xyz
114 MECHANICS
ur ur ur
aAB = aA – a B , …(1)

and relative acceleration of object B with respect to A is given by :


ur ur ur
aBA = a B – aA . …(2)

FORMULAE USED
1. Relative velocity of particle A w.r.t. particle B
r r r
v AB = v A - vB

2. Relative velocity of particle B w.r.t. particle A


r r r
vBA = vB - vA

3. When particles move in the same direction, the velocity of approach/ separation
v AB = vA - vB

4. When the particle B moves in opposite of A, velocity of approach


v AB = vA + vB

5. Relative distance, x = relative velocity × time


6. If two trains of lengths l1 and l2 with initial separation x0,
coming from opposite directions, the time to cross them

( x0 + l1 + l 2 )
t= (v1 + v2 )

7. If first train is to be over taken by second train, then time to cross them

( x0 + l1 + l 2 )
t=
(v2 - v1 )

PROBLEM SOLVING STRATEGY


Relative velocity
Identify the concepts : Whenever you see the phrase like velocity relative to, velocity with
respect to, velocity of approach, velocity of separation, its likely that the concepts of relative
velocity will be helpful.
In many problems you asked the time to cross the objects going in the same direction or coming from
opposite directions, the concepts of relative velocity will be used.
r r
Setup the problem : To get the relative velocity (vA – vB ) , the velocity of both the particles must
be in the same frame of reference, otherwise they have to be converted into same frame of reference.
Execute the solution : Solve the problem by using equations of relative velocity. If the velocities
are not along the same direction, you will need to use the vector form of the equation. In case if
objects velocity makes some angle with line of relative motion, then
vA vB
r r r
(v AB ) x = v Ax - vBx
qA qB
A B
or (v AB ) x = v A cos q A + vB cos q B. Figure 3.36
www.crackjee.xyz
Motion in a Straight Line 115
EXAMPLES BASED ON RELATIVE VELOCITY
Example 25. A police van moving on a highway with a speed of Example 27. The engineer of a train moving at a speed v1
30 km/h fires a bullet at a thief’s car speeding away in the same sights a freight train a distance d ahead of him on the same track
direction with a speed of 192 km/h. If the muzzle speed of the moving in the same direction with a slower speed v2. He puts on the
bullet is 150 m/s, with what speed does the bullet hit the thief’s brakes and gives his train a constant deceleration a. Find the
minimum value of d at which brakes are applied so as to avoid
car? [NCERT]
collision.
5 25 Sol. Collision will be avoided if speed of the train v1 becomes equal to
Sol. Speed of police van = 30 × = m/s. v2 in travelling a relative distance d. Therefore final relative speed of
18 3
trains becomes zero. The initial relative speedv12 = v1 – v2, By third
The muzzle velocity, that is velocity of bullet with respect to van equation of motion, we have
v122 = u122 – 2 a12s
0 = (v1 – v2)2 – 2 (a – 0) d

(v1 - v2 )2
or d= .
2a
Figure 3.37
[vbullet]van = [vbullet]ground – [vvan]ground (v1 - v2 )2
The collision can be avoided if d ³ .
or [vbullet ]ground = [vbullet ]van + [vvan]ground 2a

25 Example 28. A car travelling at 60 km/h overtakes another car


= 150 + travelling at 42 km/h. Assuming each car to be 5.0 m long, find the
3
time taken during the overtake and the total road distance used for
475 the overtake.
= m/s Sol. The velocity of car which is overtaking
3
5 50
5 160 v1 = 60 × = m/s
Speed of thief’s car = 192 × = m/s 18 3
18 3
Now velocity of bullet with respect to the thief’s car 5 35
and the velocity of car to be overtaken, v2 = 42 × = m/s.
18 3
[vbullet]car = [vbullet]ground – [vvan]ground
The relative velocity between them v12 = v1 – v2
475 160
= – 50 35
3 3 = –
3 3
= 105 m/s.
= 5 m/s
Hence the speed of the bullet with which it hits the thief’s car = 105 m/s.
The distance travelled by car 1 in overtaking car 2 = 10 m
Example 26. A bird is tossing (flying to and fro) between two 10
cars moving towards each other on a straight road. One car has a \ Time taken in overtaking this distance = 5 = 2 s.
speed of 18 km/h while the other has the speed of 27 km/h. The bird
starts moving from first car towards the other and is moving with
the speed of 36 km/h and when the two cars were separated by 36
km. What is the total distance covered by the bird?
[NCERT Exemplar]
Sol. Speed of first car = 18 km/h
Figure. 3.38
Speed of second car = 27km/h
The distance travelled by car 1 in this duration
\ Relative speed of each car w.r.t. each other
= 18 + 27 = 45 km/h 50
Distance between the cars = 36 km s= × 2 = 33.3 m.
3
Distance between the cars 36
\ Time of meeting the cars (t) = = The total road distance used for overtaking
Relative speed of car 45
= s + 5 = 33.3 + 5 = 38.5 m. Ans.
4
h = 0.8 h.
Example 29. On a two lane road, car A is travelling with a
=
5 speed of 36 km/h. Two cars B and C approach car A in opposite
Speed of the bird (vb) = 36 km/h directions with a speed of 54 km/h each. At a certain instant, when
\ Distance covered by the bird = vb × t the distance AB is equal to AC, both 1 km B decided to overtake A
= 36 × 0.8 before C does. What minimum acceleration of car B is required to
= 28.8 km Ans. avoid an accident? [NCERT]
www.crackjee.xyz
116 MECHANICS
Sol. At the instant when car B decides to overtake car A, the velocities Solving equations (ii) and (iii), we get
of cars are ;
3
5 v = 40 km/h and T = h. Ans.
20
vA = 36 × = 10 m/s
18 Example 31. The speed of a motor launch with respect to still
5 water is 7 m/s and the speed of stream is v = 3 m/s. When the launch
v B = 54 × = 15 m/s began travelling upstream, a float was dropped from it. The launch
18
travelled 4.2 km upstream, turned about and caught up with the
5 float. How long is it before the launch reaches the float ?
and v C = – 54 × = – 15 m/s Sol. The speed of the motor launch in still water = 7 m/s.
18
Its speed when it moves upstream
= 7 – 3 = 4 m/s.

Figure. 3.39
Velocity of car B relative to A, vBA = vB – vA = 15 – 10 = 5 m/s
Velocity of car C relative to A, vCA = vC – vA = – 15 – 10 = – 25 m/s
Time the car C requires to just cross A Figure. 3.40
1000 Distance moved = 4200 m
1000
= = = 40 s. 4200
vCA 25
Time takent1 = = 1050 s.
4
In order to avoid accident, car B must overtake A in this time, so
The distance moved by float in downstream direction in this duration
1 = speed × time
1000 = uBA t + a t2 = 3 × 1050 = 3150 m
2 BA
The distance to be covered by the motor launch
1 = 4200 + 3150 = 7350 m
or 1000 = 5 × 40 + a × 402 The speed of the motor launch in downstream direction with respect to
2 BA
the float
\ aBA = 1 m/s2
= 7 m/s.
Thus the minimum acceleration that car B requires to avoid an accident is
1 m/s2. 7350
\ Time taken,t2 = = 1050 s
Example 30. Two towns A and B are connected by a regular bus 7
service with a bus leaving in either direction every T min. A man Total time taken, t = t1 + t2
cycling with a speed of 20 km/h in the direction A to B notices that = 1050 + 1050 = 2100 s Ans.
a bus goes past him every 18 min in the direction of his motion, and Example 32. On a foggy day two drivers spot each other when
every 6 min in the opposite direction. What is the period T of the they are just 80 m apart. They are travelling at 72 km/h and 60 km/
bus service and with what speed (assumed constant) do the buses h respectively. Both of them applied brakes retarding their cars at
ply on the road? [NCERT] the rate of 5 m/s2. Determine whether they avert collision or not.
Sol. Let speed of each bus = v km/h. Sol. Method-I
Speed of the fist car, v 1 = 72 km/h = 20 m/s,
The distance between the nearest buses plying on either side and speed of the second car,
= vT km. …(i) v 2 = 60 /km/h
For buses going from town A to B : 50
Relative speed of bus in the direction of motion of man, = (v – 20). = m/s.
3
Buses plying in this direction go past the cyclist after every 18 min. If s1 and s2 are the distances moved by the cars before stop, then
18 0 = 202 - 2 ´ 5 ´ s1
Therefore separation between the buses = (v – 20) × .
60
\ s 1 = 40 m,
From equation (i), 2
æ 50 ö
18 and 0 = ç ÷ - 2 ´ 5 ´ s2
è 3ø
(v – 20) × = vT. …(ii)
60
\ s 2 = 27.78 m
For buses coming from B to A : The distance approaches by the cars
The relative velocity of bus with respect to man = (v + 20) s = s1 + s2
= 40 + 27.78
Buses coming from town B past the cyclist after every 6 min therefore
= 67.78 m.
6 As this distance is less than the initial distance between them, so the
(v + 20) × = vT.. …(iii) collision will be averted.
60
www.crackjee.xyz
Motion in a Straight Line 117
Method-II Now using second equation for relative motion
The initial separation between the cars 1
s = 80 m. [scoin]elevator = [ucoin]elevatort + [a ] t2
The initial velocity of approach of the cars 2 coin elevator
50 110 1
u = u1 + u2= 20 + = m / s. or 2=0+ (g + a) t2
3 3 2
The relative acceleration between them 1
2
or 2= (9.8 + 1) t2
a = a1 + a 2 = 5 + 5 = 10 m / s 2
The final velocity of approach v = 0.
2´2
If s is the distance of approach before stop, then or t= = 0.61 s. Ans.
10.8
0 = u 2 - 2 as
2 Example 34. A balloon is rising vertically upwards with uniform
0 = æç
110 ö acceleration 15.7 m/s2. A stone is dropped from it. After 4 s another
- 2 ´ 10 ´ s
è 3 ÷ø stone is dropped from it. Find the distance between the two stones
\ s = 67.22 m. 6 second after the second stone is dropped.
Example 33. An elevator, in which a man is standing, is moving Sol. Consider motion of stones with respect to the balloon. At the
upward with a constant acceleration of 1m/s2. At some instant instant of release of stones, the initial velocity of both stones w.r.t. the
when speed of elevator is 10 m/s, the man drops a coin from a balloon is zero. The acceleration of stone w.r.t. the balloon
height of 2 m. Find the time taken by the coin to reach the floor. [astone]balloon = g – (– a) = g + a
(g = 9.8 m/s2) where a is the acceleration of balloon which is = 15.7 m/s2
Sol. Analysing the motion of coin with respect to the observer standing 1
in the elevator. As the coin releases from rest inside elevator, its velocity Now s1 = 0+ [a ] t2
with respect to ground is equal to the velocity of elevator. i.e.,10 m/s. 2 stone balloon 1
where t1 = (4 + 6) = 10 s
1
\ s1 = (g + a) × 102
2
1
= (9.8 + 15.7) × 102 m
2
1
and s2 = 0 + (g + a) t22
2
where t2 = 6s
Figure. 3.41 1
ur ur \ s2 = (9.8 + 15.7) × 62
\ [ucoin]observer = [u coin ]ground – [uelevator]ground 2
The distance between s1 and s2 :
= 10 – 10 = 0
s = s1 – s2
The acceleration of coin with respect to the observer in the elevator:
ur ur ur 1
[a coin ]elevator = [a coin ]ground – [a elevator ]ground = (9.8 + 15.7) [102 – 62]
2
= g – (– a) = g + a. = 816 m. Ans.

In Chapter Exercise 3.3


1. A jet airplane travelling at the speed of 500kmh–1 ejects (i) both cars are travelling eastwards and
its products of combustion at the speed of 1500kmh –1 (ii) car A is travelling eastwards and car B is travelling
relative to the ground. What is the speed of the latter westwards.
with respect to Jet plane? [NCERT] Ans. (i) 15 km/h eastwards (ii) 105 km/h eastwards
Ans. –2000 km/h 5. Two buses start simultaneously towards each other from
2. A jet plane travelling at the speed of 450 km/h ejects the towards A and B which are 480 km apart. The first bus
burnt gases at the speed of 1200 km/h relative to the jet takes 8 hours to travel from A to B while the second bus
plane. Find the speed of the burnt gases w.r.t. a stationary takes 12 hours to travel from B to A. Determine when and
observer on earth. Ans. 750 km/h where the buses will meet. Ans. 4.8 h, 288 km from A.
3. A burglar’s car had started with an acceleration of 2 m/s 2. 6. A train is moving along a straight line with a constant
A police vigilant party came after 5 s and continued to acceleration ‘a’. A boy standing in the train throws a ball
close the burglar’s car with a uniform velocity of 20 m/s.
forward with a speed of 10 m/s, at an angle of 60° to the
The time taken in which the police van will over take the
horizontal. The boy has to move forward by 1.15 m inside
burglar’s car, in s, is [Integer] Ans. 5
the train to catch the ball back at the initial height. The
4. Two cars A and B are moving with velocities of 60 km/h
and 45 km/h respectively. Calculate the relative velocity acceleration of the train, in m/s2, is [IIT 2011]
of A w.r.t. B, if [Integer] Ans. 5 m/s2
www.crackjee.xyz
118 MECHANICS

3.8 MOTION WITH VARIABLE ACCELERATION


If acceleration is not constant; either of the displacement, velocity or acceleration is given in terms of
time or otherwise. You have to start the problem from the general equations of motion given in terms of
differentiation. These are :
dv dv
acceleration, a= =v ,
dt ds
ds
and velocity, v= .
dt

3.9 PROBLEMS BASED ON MAXIMA AND MINIMA


In many problems, you may be asked, the distance of closest approach, shortest distance, Minimum
MISCELLANEOUS TOPICS

time or maximum time. These problems can be solved


easily by using method of differentiation. Let we have
to find maxima or minima of y, which is a function of x.
This can be execute in following ways:
dy d 2y
(i) For maxima, = 0 and < 0.
dx dx 2
dy d2y
(ii) For minima, = 0 and >0
dx dx 2

Figure. 3.42

Note:
d2y
In practice either maxima or minima occurs at a time. So there is no need to find . The required
dx 2
dy
result will be obtain by simply putting =0.
dx

EXAMPLES BASED ON VARIABLE ACC. AND MAXIMA AND MINIMA


Example 35. An experiment on the take off performance of an
aeroplane shows that the acceleration varies as shown in figure d 2s 5t
\ 2 = .
3.43, and that it takes 12 s to take off from a rest position. Calculate dt 6
the distance along the runway covered by the aeroplane.
Integrating again, we get
Sol. From 0 to 6 second the acceleration varies linearly with time,
therefore we have ds 5t 2
= + c2
da 5 dt 12
= +
dt 6

d é d 2s ù 5 ds
or ê ú + at t = 0, v= = 0 , \ c2 = 0
dt ëê dt 2 ûú = 6 dt

Figure. 3.43 ds 5t 2
\ dt
= …(ii)
d 3s 5 12
or = + … (i)
dt 3 6 Integrating once more, we get
Integrating equation (i) w.r.t. time, we get
5t 3
2
5 s = + c3 .
d s 36
= t + c1
dt 2 6
As t = 0, s = 0 , \ c3 = 0
2
d s 5 3
at t = 0, =0, \ c1 = 0 .t
dt 2 s = …(iii)
36
www.crackjee.xyz
Motion in a Straight Line 119
The distance travelled from 0 to 6s, from equation (ii), we get Example 37.The motion of a body is given by the equation
5 dv(t)
s1 = ´ (6)3 = 30 m = 6.0 – 3v(t)
36 dt
The velocity at t = 6 s, where v (t) is the speed in m/s and t in second.
5 2 5 If the body was at rest at t = 0; then test these corrections of the
v = t = ´ 6 2 = 15 m/s following results
12 12
(a) the terminal speed is 2.0 m/s
Now from, 6 s to 12 s
(b) the magnitude of initial acceleration is 6.0 m/s2
u = 15 m/s, a = 5m/s2
(c) the speed varies with time as v(t) = 2 (1 – e–3t) m/s
1 1 (d) the speed is 1.0 m/s when the acceleration is half the initial
\ s2 = ut + at 2 = 15 ´ 6 + ´ 5 ´ 62 = 180 m.
2 2 value.
Therefore total distance travelled on runway = 30 + 180 = 210 m.Ans. dv(t)
Sol. Given = 6.0 – 3v(t)
Example 36. The velocity of a particle moving in the positive dt
(a) The terminal speed is the constant speed when acceleration is
direction of the x-axis varies as v = a x , where a is a positive
zero. Thus
constant. Assuming that at the moment t = 0 the particle was located
0 = 6.0 – 3 v
at the point x = 0, find:
(a) the time dependence of the velocity \ v = 2 m/s
(b) At t = 0, v = 0; therefore initial acceleration a = 6. 0 – 0 = 6.0 m/s2
(b) the mean velocity of the particle averaged over the time that
the particle takes to cover the first 5 metre of the path. (c) We have,
dv(t)
Sol. (a) Given, v=a x = 6.0 – 3v(t)
dt
\ 2
v2 = a x . \ dv = [6.0 – 3v (t)] dt
Differentiating above equation, we have
dv
or = dt
d 2
dt
( )
v =
d ( a 2 x)
dt
or 2v
dv
dt
= a2
dx
dt
6.0 - 3v(t )
Integrating both sides of above equation, we get
dx v t
=v dv
Since
dt ò
0 6.0 - 3v (t )
= ò dt
0

\ dv a2
2 = a 2 or dv = dt v
dt 2 ln(6.0 - 3v )
= t
-3 0
v a2 t
\ ò 0
dv =
2 ò dt
0
ln (6.0 - 3 v) – ln 6.0 = – 3t
2
a t
or v = . …(i) æ 6.0 - 3v ö
ln ç = -3t
è 6.0 ÷ø
2 or
(b) Let t be the time to cover the first s meter of the path, then from
equation (i), we have æ vö
or ln ç 1 -
è ÷ = -3t
ds 2
a t 2ø
=
dt 2 or v = 2 (1– e –3t) m/s.
s a2 t 6.0
\ ò 0
ds =
2 ò 0
t dt (d) Acceleration half that of initial =
2
= 3.0 m/s2

This gives 3 = 6 – 3 v(t)


a 2t 2
or s = or v (t) = 1 m/s
4
We have seen that alternatives (a), (b), (c) and (d) all are correct.
2 s
\ t = Example 38 A body of mass m is thrown straight up with velocity
a
Now the mean velocity of the particle v0. Find the velocity v ' with which the body comes down if the air
t 2 2
drag equals kv2, where k is a constant and v is the velocity of the
v dt a t
ò0
=
2
4 = a t
body.
Sol. The net retarding force on the body
vav = t t 4
ò 0
dt
m
dv
dt
2
= - (mg + kv )

a2 2 s a
= ´ = s. Ans. dv æ k 2ö
4 a 2 \ = - çè g + v ÷ø …(i)
dt m
www.crackjee.xyz
120 MECHANICS
Integrating both sides of above equation, we get
dv dv
We have = v h
dt dy v'

ò dy
vdv
dv æ k 2ö òæ kv 2 ö
=
\ v = - çè g + v ÷ø 0 çg - 0
dy m è m ÷ø

vdv
é kv '2 ù
æ k 2ö êg - ú 2kh
or çè g + v ÷ø = - dy or– ln ê m ú =
m m
ëê g ûú
Integrating both sides of above equation, we get
-1
0 é kv '2 ù
vdv h êg - ú 2kh
ò æ
v0 ç g +
k 2ö =
v ÷
- dy
ò …(ii)
or ln ê
ëê g
m ú
ûú
=
m
…(iv)
è m ø 0
From equation (iii) and (iv), we get
k 2 -1
On substituting g + v = z é kv 2 ù é kv '2 ù
m êg + 0 ú êg - ú
ê m ú = ê m ú
æ k ö
d ç g + v2 ÷ ëê g ûú êë g úû
è m ø dz
=
dv dv é kv02 ù é g ù
êg + ú ê 2ú
or ê m kv '
ú = êg - 0 ú
k dz æ mö
or ´ 2v = or vdv = ç ÷ dz êë g úû êë m ûú
m dv è 2k ø
Substituting these values in equation (ii), we get v0
After solving we get, v ' = Ans.
0 kv 2
m dz
h 1+ 0
2k ò z =
- dy
ò mg
v0 0 Example 39. Two bodies start moving in the same straight line
at the same instant of time from the same origin. The first body
0 - 2k
or lnz v = | y |0h moves with a constant velocity of 40 m/s, and the second starts from
0 m rest with a constant acceleration of 4 m/s2. Find the time that
0
elapses before the second catches the first body. Find also the
æ k 2ö -2k greatest distance between them prior to it and time at which this
or ln çè g + m v ÷ø = h
occurs.
v0 m
Sol. The distance travelled by first body in time t, s1 = 40 t.
é æ kv 2 ö ù 2k The distance travelled by second body in time t,
or - ê lng - ln ç g + 0 ÷ ú = h
êë è m ø úû m 1
s2 = ´ 4 ´ t 2 =2t 2 .
2
æ kv02 ö The separation between them at any time t
çg + m ÷ s = s1 – s2 = 40 t – 2t2 …(i)
è ø 2k
or ln = h …(iii) The second body will catch the first if s = 0
g m
\ 0 = 40 t – 2t2
m é k 2ù which gives t = 20 s.
or h = 2k ln ê1 + mg v0 ú ds
ë û =0
For s to be greatest,
When body comes down from height h, we have dt

dv d
m = ( mg - kv 2 ) or (40t - 2t 2 ) = 0 or 40 – 2 × 2t = 0
dt dt
or t = 10 s.
dv æ kv 2 ö Thus greatest distance between them
\ v = çg - m ÷
dy è ø s = 40 × 10 – 2 × 102 = 200 m. Ans.
Example 40. Two ships are 10 km apart on a line from south to
vdv north. The one farther north is moving towards west at 40 km/h
æ kv 2 ö and other is moving towards north at 40 km/h. What is the distance
or çg - m ÷ = dy
è ø of closest approach and how long do they take to reach it?
www.crackjee.xyz
Motion in a Straight Line 121
Sol. In time t the distance travelled by each ship is 40 t. Initially ships The total time of motion
were at A and B.
2( H - h) 2h
t = t1 + t2 = + .
40t g g
D B
N
dt
x For t to be minimum, =0
W E dh
C 10 km
æ 1 1

40t S dt d çæ 2ö 2 1/ 2 æ 2 ö 2
or = ( H - h ) + ( h ) 2÷
dt çè èç g ø÷ èç g ø÷
A dh ÷ø
Figure. 3.44
After time t, they are at C and D, let separation between them after time 1/ 2
æ 2ö 1
t is x. or 0= ç ÷ ´ ( H - h) -1/ 2 ´ ( -1)
è gø 2
\ x2 = (40 t)2 + (10 – 40t)2 … (i)
dx 1/ 2
For x to be minimum =0. æ 2ö 1
dt + ç ÷ ´ h-1/ 2
Differentiating equation (i) with time, we get è gø 2
dx H
2x = 2 ´ 40t ´ 40 + 2(10 - 40t ) ´ ( -40)
dt After solving, we get, h =
2
or 0 = 3200 t – 800 + 3200 t
h 1
1 or = . Ans.
which gives t = hr H 2
8
Now from equation (i), the closest approach Example 42. From point A located on a highway as shown in
figure. 3.46, one has to get by car as soon as possible to point B
2 2
æ 1ö æ 1ö located in the field at a distance l from the highway. It is known
xmin = çè 40 ´ ÷ø + çè10 - 40 ´ ÷ø that car moves in the field h time slower on the highway. At what
8 8
distance from point D one must turn off the highway?
= 52 + 52 = 5 2 km. Ans.

Note: The value of closest approach does not depend on the


speed of objects provided both have same speed.

Example 41. A body falling freely from a given height H hits an


inclined plane in its path at a height h. As a result of this impact the
direction of velocity of the body becomes horizontal. For what value
of (h/H) will the body take maximum time to reach the ground?
Sol. Time taken in falling height (H – h)
Figure. 3.46
1 2 Sol. Suppose x distance from D the car turns off the highway.
(H – h) = 0 + gt1
2 Let v be the speed of car on highway, then its speed on field will
be v/h. The time of motion,
2( H - h)
or t1 = .
g æ AD - x ö x2 + l2
t = ç
è ÷+ …(i)
v ø v/h
dt
t to be minimum, =0
dx
dt d é AD - x h 2 ù
or = dx ê + ( x + l 2 )1/ 2 ú
dx ë v v û
[AD is constant]
1 h 1 2 2 -1/ 2
or 0 = - + ´ (x + l ) ´ 2x
Figure. 3.45 v v 2
After impact with the inclined plane the vertical component of velocity or h2 x 2 = x 2 + l 2
becomes zero. Let t2 be the time taken in falling height h, then
l
1 2 2h \ x = . Ans.
h=0+ gt2 or t2 = 2
h -1
2 g
www.crackjee.xyz
122 MECHANICS

In Chapter Exercise 3.4

take to cover that distance ?


by x = t 3 - 4t 2 + 3t , where x is in metre and t in second.
(a) What is the object¢s displacement between t = 0 and t = æ 2 ö 2 v0
1. The position of an object moving along an x axis is given Ans. s = ç 3a ÷ v03/2, t = 2
4s? è ø a
(b) What is its average velocity for the time interval 3. The distance between two moving particles at any time is
from t = 2s to t = 4s ? a. If v be their relative velocity and v 1and v 2 be the
Ans. (a) + 12 m (b) + 7 m/s. components of v along and perpendicular to a. Find the
2. A point moves rectilinearly with deceleration whose time when they are closest to each other, and the minimum
modulus depends on the velocity v of the particle as distance between them.

a = a v , where a is a positive constant. At the initial av1 av2


Ans: 2 ,
moment the velocity of the point is equal to v 0. What v v
distance will it transverse before it stops? What will it

MISCELLANEOUS EXAMPLES FOR JEE-(MAIN AND ADVANCE)


Example 1. A particle starts moving with constant velocity of 10 The distance at which second particle will cross the first particle
m/s. Simultaneously another particle starts with constant s = 10 ´ 20 = 200 m. Ans.
acceleration of 1 m/s2 in the direction of first particle. Find;
Example 2. A bird flies for 4 s with a velocity v = (t – 2) m/
(a) the time at which distance between them is greatest and the
s in a straight line, where t = time in second. Calculate the
greatest distance between them,
displacement and distance covered by the bird.
(b) the time and distance at which second particle will cross the
Sol. The velocity-time and speed-time graphs of motion of the
first particle.
bird are as follows:
Sol.
Velocity (m/s) Speed (m/s)
(a) In time t; v=t–2
v =| t – 2|
The distance travelled by first particle = 10 t.
2 2

A
+ B + + B
0 time (s) 0 time (s)
A – 2 4 2 4
Figure. 3.47
The distance travelled by second particle –2
(a) Variation of velocity with time (b) Variation of speed with time
1 2 1 t2
= at = ´ 1 ´ t 2 = . Figure. 3.48
2 2 2
From the graph, displacement s = area A – area B = 0
Thus distance between them at any time t,
And distance, s = Area A + Area B
t2
s = 10t - . 1 1
2 = ´ 2 ´ 2 + ´ 2 ´ 2 = 4 m.
2 2
ds
The distance s to be greatest, = 0, Example 3. Each second a rabbit moves half the remaining
dt
distance from his nose to the head of a lettuce. Does he ever get to
d æ t2 ö the lettuce? What is the limiting value of his average velocity?
or ç10t - ÷ = 0 Draw graphs showing his velocity and position as time increases.
dt è 2ø
Sol. Let us take x0 is the initial distance of rabbit from the lettuce. His
or 10 – t = 0 distance at any moment x;
\ t = 10 s Ans. A t t = 0, x = x0
The greatest distance between them
1
x0 æ 1ö
102 t = 1, x = = x0 ç ÷
è 2ø
= 10 ´ 10 - 2
2
2
= 50 m Ans. x0 æ 1ö
(b) For distance between them to be zero, s = 0, t = 2, = x0 ç ÷ x =
4 è 2ø
t2 –––––––––––––
or 10t - = 0
2 æ 1ö
n
or t = 20 s. Ans. t = n, x = x0 ç ÷
è 2ø
www.crackjee.xyz
Motion in a Straight Line 123
For upward motion, the velocity is given by the equation
x
v = u – gt
x0
= 37.8 – 9.8 × t
v The speed decreases linearly and becomes zero at;
0 = 37.8 – 9.8 t
Þ t = 3.9 s
O t O t
Thus, the ball reaches the highest point again after time t
= 4.3 + 3.9 = 8.2 s from the start of the motion.
(iii) At the highest point, the speed of ball becomes zero. It again
starts falling. At any time its speed is given by v = 0 + 9.8 t.
The speed increases linearly with time from 0 to 37.8 m/s in the
next time interval of 3.9s. The total time of motion from start now
Figure. 3.49 becomes = 4.3 + 3.9 + 3.9 = 12.1 s.
To reach the lettuce, x = 0
The motion of the ball is shown by graph as in figure 3.50.
n
æ 1ö
or 0 = x = x0 ç ÷
è 2ø

which gives n = ¥
Thus rabbit practically will never reach the lettuce.

dx
The velocity of a body is the slope of x - t plot, that is = v. Slope of
dt
the plot at each instant is negative, which is drawn in the figure. Since
time to reach the lettuce is infinite.

displacement x0
Therefore average velocity = = =0 Ans.
time ¥
Example 4. A ball is dropped from a height of 90 m on a floor. At
each collision with the floor, the ball loses one-tenth of its speed.
Plot the speed-time graph of its motion between t = 0 to 12 s. Figure. 3.50
[NCERT] Example 5. Two stones are thrown up simultaneously from the
Sol. The time taken by the ball to fall a height 90 m : edge of a cliff 200 m high with initial speeds of 15 m/s and 30 m/s.
Verify that the following graph correctly represents the time
1 variation of the relative position of the second stone with respect to
(i) 90 = ut + g t2
2 the first. Neglect air resistance and assume that the stones do not
rebound after hitting the ground. Take g = 10 m/s2. Give equations
1 for the linear and curved parts of the plot. [NCERT]
or 90 = 0 + × 9.8 × t2
2 [JEE (Main) 2015]

which givest = 4.3 s


Now velocity just before collision with the floor
v = u + gt
= 0 + 9.8 × 4.3 = 42 m/s
The velocity between 0 to 4.3 s
v = gt = 9.8 t.
In this time velocity varies linearly with time from 0 to 42 m/s
during downward motion.
(ii) After first collision with the floor, speed lost by ball

1
= × 42 = 4.2 m/s
10

9
Thus ball rebound with a speed of v = × 42 = 37.8 m/s.
10 Figure. 3.51
www.crackjee.xyz
124 MECHANICS
Sol. Time to hit the ground by stones can be calculated as : 1
For second stone, 200 = – 30 t + × 10 × t22
2
\ t2 = 10 s
The positions of stones at anytime t, taking O as the origin are;
1
x1 = 200 + (15 t – × 10 × t2) …(i)
2
1
and x2 = 200 + (30 t – × 10 × t2) …(ii)
2
(a) The relative position of second stone w.r.t first is given by;
For t £ 8 s,x2 – x1= 15 t. …(iii)
Figure. 3.52 (b) After 8 s when first stone stops falling, so x1 = 0
1 1
We have h = ut + g t2. \ x2 = 200 + (30 t – × 10 × t2)
2 2
1 and x2 – x1 = 200 + 30 t – 5 t2 …(iv)
For first stone, 200 = – 15 t + × 10 × t12
2 The equation (iii) is a straight line between (x2 – x1) and t and
\ t1 = 8 s equation (iv) is parabolic.
www.crackjee.xyz
Motion in a Straight Line 125

Mechanics
MCQ Type 1 Exercise 3.1
Level - 1 (Only one option correct)
Average Velocity, Motion with Constant 7. A point moves with uniform acceleration and v1, v2 and v3
Acceleration denote the average velocities in three successive intervals of
time t1, t2 and t3. Which of the following relations is correct
1. If a car covers 2/5th of the total distance with v1 speed and
(a) v1 – v2 : v2 – v3 = t1 – t2 : t2 + t3
3/5th distance with v2 , then average speed is
(b) v1 – v2 : v2 – v3 = t1 + t2 : t2 + t3
1 v1 + v2
(a) v1v2 (b) (c) v1 – v2 : v2 – v3 = t1 – t2 : t1 – t3
2 2
(d) v1 – v2 : v2 – v3 = t1 – t2 : t2 – t3
2v1v2 5v1v2
(c) (d) 8. Speed of two identical cars are u and 4u at a specific instant.
v1 + v2 3v1 + 2v2
The ratio of the respective distances in which the two cars
2. A particle moving in a straight line covers half the distance
are stopped from that instant is
with speed of 3m/s. The other half of the distance covered
(a) 1 : 1 (b) 1 : 4
in two equal time intervals with speed of 4.5 m/s and
(c) 1 : 8 (d) 1 : 16
7.5 m/s respectively. The average speed of the particle
during this motion is 9. Two balls A and B of same mass are thrown from the top
(a) 4.0 m/s (b) 5.0 m/s of the building. A thrown upward with velocity v and B,
(c) 5.5 m/s (d) 4.8 m/s thrown down with velocity v, then
3. Which are of the following represents uniformly accelerated (a) velocity A is more than B at the ground
motion: (b) velocity of B is more than A at the ground
t−a t−a (c) both A and B strike the ground with same velocity
(a) x= (b) x= (d) none of these
b b
10. A particle displacement x of a particle moving in one
x−a
(c)
t= (d) =x t +a dimension under constant acceleration is related to time
b
4. The velocity of a body depends on time according to t as=t x + 3 . The displacement of the particle when its
equation v = 20 + 0.1t2. The body is undergoing velocity is zero is:
(a) uniform acceleration (b) uniform retardation (a) zero (b) 3 units
(c) non uniform acceleration (d) zero acceleration (c) 3 units (d) 9 units
5. A particle experiences a constant acceleration for 20 sec 11. A particle moves a distance x in time t according to equation
after starting from rest. If it travels a distance s1 in 10 sec x = (t + 5)–1. The acceleration of the particle is proportional
and distance s2 in the next 10 sec, then to :
s2 (a) (velocity)3/2 (b) (distance)2
s1 = s2
(a) (b) s1 =
3 (c) (distance)–2 (d) (velocity)2/3
s2 s 12. A frictionless wire AB is fixed on a sphere of radius R. A
(c)
s1 = (d) s1 = 2
2 4 very small spherical ball slips on this wire. The time taken
6. A particle is dropped vertically from rest from a height. The by this ball to slip from A to B is :
time taken by it to fall through successive of 1 meter each
will then be:
2
(a) all equal, being equal to second
g
(b) in the ratio of square roots of integers 1, 2, 3, ...
(c) in the ratio of the difference in the square roots of 2 gR cos θ
(a) 2 gR
(b)
integersi.e. ( 1 − 0), ( 2 − 1), ( 3 − 2), ( 4 − 3) g cos θ g

1 1 1 1 R gR
(d) in the ratio : : : (c)
2 (d)
1 2 3 4 g g cos θ

Answer 1 (d) 2 (a) 3 (c) 4 (c) 5 (b) 6 (c)


Key 7 (b) 8 (d) 9 (c) 10 (a) 11 (a) 12 (c)
www.crackjee.xyz
126 Mechanics
13. A particle had a speed of 18 m/s at a certain time, and 2.4 s later moves in the same direction with a constant velocity v. The
its speed was 30 m/s in the opposite direction. The average two bodies meet after a time t. The value of t is.
acceleration of the particle in the duration is : 2v v
(a) (b)
(a) 20 m/s2 in the direction of initial velocity a a
(b) 20 m/s2 in the direction opposite to the initial velocity v v
(c) 5 m/s2 in the direction of initial velocity (c) (d)
2a 2a
(d) 5 m/s2 in the direction opposite to the initial velocity.
21. A point moves in a straight line so that its displacement x at
14. A stone is thrown vertically upward. On its way up it passes
time t is given by x2 = 1 + t2. Its acceleration at any time t
point A with speed of v, and point B, 3 m higher than A, with
is
speed v/2. The maximum height reached by stone above point
1 –t
B is (a) (b) 3
3
(a) 1 m (b) 2 m x x
(c) 3 m (d) 5 m 1 t2 1 1
(c) − (d) − 2
15. Two diamonds begin a free fall from rest from the same height, x x 3 x x
1.0 s apart. How long after the first diamond begins to fall 22. Two balls are dropped to the ground from different heights.
will the two diamonds be 10 m apart ? One ball is dropped 2s after the other but they both strike
(a) 1.0 s (b) 1.5 s the ground at the same time. If the first ball takes 5s to
(c) 2.0 s (d) 2.5 s reach the ground, then the difference in initial heights is
16. A body is projected vertically upwards. If t1 and t2 be the (g = 10 ms–2)
times at which it is at height h above the projection while (a) 20 m (b) 80 m
ascending and descending respectively, then h is (c) 170 m (d) 40 m
1 23. The time taken by a block of wood (initially at rest) to
(a) gt1t2 (b) gt1t2
2 side down a smooth inclined plane 9.8 m long (angle of
(c) 2gt1t2 (d) 2hg inclination is 30°) is

17. A ball is dropped from a bridge 122.5 m above a river. After


the ball has been falling for two second, a second ball is
thrown straight down after it. What must its initial velocity 30°
be so that both hit the water at the same time?
(a) 49 m/s (b) 55.5 m/s 1
(a) s (b) 2 s
(c) 26.1 m/s (d) 9.8 m/s 2
(c) 4 s (d) 1 s
18. Two boys are standing at the ends A and B of a ground
24. If a ball is thrown vertically upwards with speed u, the
where AB = a. The boy at B starts running in a direction
distance covered during the last t seconds of its ascent is
perpendicular to AB with velocity v1. The boy at A starts
1 1
running simultaneously with velocity v and catches the other (a) gt 2 (b) ut − gt 2
boy in a time t, where t is 2 2
a a (c) (u – gt)t (d) ut
(a) 2 2 (b) 25. A ball is thrown vertically upwards. It was observed at a
v + v1 v + v1
height h twice, with a gap of time interval ∆t. The initial
a a
(c) (d) velocity of the ball is
v − v1 v 2 − v12 g ∆t 2
19. A body A starts from rest with an acceleration a1. After
(a) 8 gh + g 2 (∆t )2 (b) 8 gh + ( )
2
2 seconds, another body B starts from rest with an 1
acceleration a2. If they travel equal distances in the 5th (c) 8 gh + g 2 (∆t )2 (d) 8 gh + 4 g 2 (∆t )2
2
seconds, after the start of A, then the ratio a1 : a2 is equal to
26. Two bodies begin to fall freely from the same height but the
(a) 5 : 9 (b) 5 : 7
second falls T second after the first. The time (after which
(c) 9 : 5 (d) 9 : 7
the first body begins to fall) when the distance between the
20. A body A moves with a uniform acceleration a and zero
bodies equals L is
initial velocity. Another body B, starts from the same point

Answer 13 (b) 14 (a) 15 (b) 16 (a) 17 (c) 18 (d) 19 (a)


Key 20 (a) 21 (c) 22 (b) 23 (b) 24 (a) 25 (c)
www.crackjee.xyz
Motion in a Straight Line 127

T L (a) 12 m/s (b) 18 m/s


1 +
(a) T (b) (c) 27 m/s (d) 36 m/s
2 2 gT
33. The displacement-time graph for two particles A and B are
L 2L straight lines inclined at angles of 30° and 60° with the time
(c) (d) T +
gT gT axis. The ratio of velocities of vA : vB is
27. A bus starts from rest and moves with an acceleration of (a) 1 : 2 (b) 1 : 3
1 m/s2. A boy, who is 48 m behind the bus run after with a
constant speed of 10 m/s. The boy can catch the bus (c) 3 : 1 (d) 1 : 3
(a) only once, after 8 s form start Graphical Questions
(b) only once, after 12 s from start 34. The acceleration of the body travelling along a straight line
(c) twice after 8s and 12 s from start changes with time as shown in the figure. What does the
(d) never area under the graph measure ?
Motion with Variable Acceleration
28. The initial velocity of particle is u and the acceleration at
the time t is at, a being a constant. Then the v at the time t
is given by
(a) v=u (b) v = u + at
1 (a) the distance travelled from time t1 to time t2
(c) v = u + at2 (d) v= u + at 2 (b) the average acceleration for the period under
2
29. Starting from rest, acceleration of a particle is a = 2(t – 1). consideration
The velocity of the particle at t = 5s is (c) the average velocity for the period under consideration
(d) the velocity at time t2
(a) 15 m/s (b) 25 m/s
(c) 5 m/s (d) none of these 35. The velocity versus time curve of a moving point is as given
30. The deceleration experienced by a moving motor-boat after below. The maximum acceleration is
dv
its engine is cut off, is given by = −kv3 , where k is a
dt
constant. If v0 is the magnitude of the velocity at cut-off,
the magnitude of the velocity at a time t after the cut-off
is [AMU B.Tech. 2002]
v
(a) 0 (b) v0e–kt
2
(2v0 kt + 1)
v0 (a) 1 cm/s2 (b) 2 cm/s2
(c) (d) v0
2 (c) 3 cm/s2 (d) 4 cm/s2
31. A self-propelled vehicle of mass m whose engine delivers 36. Displacement-time curve of a particle moving along a
P straight line is shown in figure. Tangents at A and B make
constant power P has an acceleration a = (assume that
mv angles 45° and 135° with positive x-axis respectively. The
there is no friction). In order to increase its velocity from average acceleration of the particle during t = 1 to t = 2s
v1 to v2, the distance it has to travel will be x(m )
3P m 3 3
(a) (v22 − v12 ) (b) (v2 − v1 )
m 3P A 45° B
135°
m m
(c) (v22 − v12 ) (d) (v2 − v1 )
3P 3P
32. The acceleration a in m/s2 of a particle is given by O t(s)
t= 1 t= 2
a = 3t2 + 2t + 2 where t is the time. If the particle starts out
with a velocity u = 2 m/s at t = 0, then the velocity at the (a) – 2 m/s2 (b) 1 m/s2
end of 2 second is (c) –1 m/s2 (d) zero

Answer 26 (b) 27 (c) 28 (d) 29 (a) 30 (a) 31 (b)


Key 32 (b) 33 (d) 34 (d) 35 (d) 36 (a)
www.crackjee.xyz
128 Mechanics
37. The position-time relation of a particle moving along the v v
x-axis is given by x = a – bt + ct2 where a, b and c are positive
numbers. The velocity-time graph of the particle is
(c) (d)
O t1 t2 t O t1 t2 t
(a) (b)
41. The graph of displacement vs time is

(c) (d)
t
Its corresponding velocity – time graph will be
38. A particle starts from rest at t = 0 and moves in a straight
line with an acceleration as shown below. The velocity of
the particle at t = 3s is
(a) (b)

(a) 2 m/s (b) 3 m/s (c) (d)


(c) 4 m/s (d) 6 m/s
39. Acceleration-time graph of a body is shown. The
corresponding velocity-time graph of the same body is:
42. An object is moving with a uniform acceleration which
is parallel to its instantaneous direction of motion. The
displacement(s)–velocity (v) graph of this object is:

(a) (b)
(a) (b)

(c) (d)
(c) (d)

43. A graph between the square of the velocity of a particle and


40. A batsman hits a sixer and the ball reaches out of the the distance moved is shown in the figure. The acceleration
cricket ground. Which of the following graphs describes of the particle in kilometer per hour squared is
the variation of the cricket ball’s vertical velocity v with v2
time t1 (the time of hitting the bat and time t2 (the time of 3600
touching the ground)? [AMU B.Tech. -2007]
v v
900
(a) (b) s(km)
0 0.6
t t2 t
O t1 t2 O t1
(a) 2250 (b) 22.5
(c) – 2250 (d) 225
Answer 37 (c) 38 (b) 39 (c) 40 (d)
Key 41 (b) 42 (c) 43 (c)
www.crackjee.xyz
Motion in a Straight Line 129
44. The velocity-time graph of a body is shown in figure. The 47. Two trains, each 50 m long are travelling in opposite
slope of the line is ‘m’. The distance travels by body in time direction with velocity 10 m/s and 15 m/s. The time of
Ts crossing is
(a) 2s (b) 4s
(c) 2 3s (d) 4 3s
48. A massless string of length l passes over a frictionless
pulley whose axis is horizontal. Two monkeys hang from
mv 2 v2 the ends of the string at the same distance /2 from the
(a) (b)
2T 2T pulley, the monkeys start climbing upward simultaneously.
First monkey climbs with a speed v relative to the string
v2
(c) 2mv2 (d) and the second with a speed 2v. Both monkeys have equal
2m
masses. Then the time taken by the first and second monkey
45. A particle starts from rest at t = 0 and undergoes an
is meeting each other are respectively
acceleration a in ms–2 with time t in seconds which is as
shown    
(a) and (b) and
a 3v 3v 2v 4v
3    
(c) and (d) and
0 t 4v 2v v 2v
1 2 3 4
–3 49. A ball A is thrown up vertically with a speed u and at the
same instant another ball B is released from a height h. At
Which one of the following plot represents velocity v in time t, the speed of A relative of B is
ms–1 versus time t in seconds (a) u (b) 2u

(u 2 − gt )
v v
6 6 u − 2 gt (d)
(c)
4 4 50. An elevator is moving upward with a constant speed of 10
(a) (b) m/s. A man standing in the elevator drops a coin from a
2 2
height of 2.5 m, the coin reaches the floor of the elevator
t
0 1 2 3 4
t 0 1 2 3 4 after a time (g = 10 m/s2) :
v 1 1
6 (a) s (b) s
4 v 2 2
6
2 4 (c) 2 s (d) 2s
(c) t (d) 51. Two trains A and B, each of length 400 m, are moving on
0 2
1 2 3 4
two parallel tracks in the same direction (with A ahead of
–2 t
0 1 2 3 4 B) with same speed 72 km/h. The driver of B decided to
–4
overtake A and accelerates by 1 m/s2. If after 50s, B just
brushes part A, calculate the original distance between
Relative Motion A and B: [AMU B.Tech.-2012]
46. Two trains A and B, each of length 100 m, are running on (a) 750 m (b) 100 m
parallel tracks. One overtakes the other in 20 s and one
(c) 1250 m (d) 2250 m
crosses the other in 10 s. The velocity of trains are :
(a) 5 m/s, 5 m/s (b) 10 m/s, 15 m/s
(c) 15 m/s, 5 m/s (d) 15 m/s, 30 m/s

Answer 44 (d) 45 (a) 46 (c) 47 (b)


Key 48 (a) 49 (c) 50 (b) 51 (c)
www.crackjee.xyz
130 Mechanics

Level - 2 (Only one option correct)


Average Velocity, Motion with Constant 7. A particle moves with a velocity (3i + 4j) m/s from origin.
Acceleration The displacement of particle along line x = y after two
seconds will be:
1. An electron moving along the x axis has a position given by 7
x = 20t e–t m, where t is in second. How far is the electron (a) 10 m (b)
2
from the origin when it momentarily stop ?
(c) 7 √ 2 m (d) none of these
(a) 20 m (b) 20 e m
20 8. A target is made of two plates, one of wood and the other
(c) m (d) zero
e of iron. The thickness of the wooden plate is 4 cm and that
2. A stone is dropped from a height h, simultaneously another of iron plate is 2 cm. A bullet fired goes through the wood
stone is thrown up from the ground which at a height 4h, first and then penetrates 1 cm into iron. A similar bullet fired
the two stones cross each other after time with the same velocity from opposite direction goes through
iron first and then penetrates 2 cm into wood. If a1 and a2
h h
(a) (b) be the retardations offered to the bullet by wood and iron
2g 8 g plates respectively, then
(c) 8hg (d)
2hg a1 = 2a2 (b)
(a) a2 = 2a1
3. A drunkard walking in a narrow lane takes 5 steps forward a1 = a2
(c) (d) data insufficient
and 3 steps backward, followed again by 5 steps forward
and 3 steps backward, and so on. Each step is 1 m long and 9. A parachutist after bailing out falls 50 m without friction.
requires 1 s. The time when he will fall into the pit 13 m When parachute opens, it decelerates at 2 m/s2. He reaches
away from him is : the ground with a speed of 3 m/s. At what height, did he
(a) 13 s (b) 37 s bail out :
(c) 40 s (d) 42 s (a) 111 m (b) 293 m
4. A car moves on a straight track from station A to the station (c) 182 m (d)
91 m
B, with an acceleration a = (b – cx), where b and c are
constants and x is the distance from station A. The maximum
Motion with Variable Acceleration
velocity between the two stations is 10. Starting from rest a particle moves in a straight line with
(a) b / c (b) b/c acceleration a = {2 + |t – 2|} m/s2. Velocity of particle at
(c) c / a (d) b /c the end of 4 s will be
5. Balls are thrown vertically upward in such a way that the (a) 16 m/s (b) 20 m/s
next ball is thrown when the previous one is at the maximum
(c) 8 m/s (d) 12 m/s
height. If the maximum height is 4.9 m, the number of balls
thrown per minute will be :
Graphical Questions
(a) 60 (b) 40
11. An experiment on the take-off performance of an aeroplane
(c) 50 (d) 120
shown that the acceleration varies as shown in the figure,
6. Four persons K, L, M and N are initially at the corners of
and that 12 s to take-off from a rest position. The distance
a square of side of length d. If every person starts moving
along the runway covered by the aeroplane is
with velocity v, such that K is always headed towards L, L
towards M, M is headed directly towards N and N towards
K, then the four persons will meet after :
d 2d
(a) sec (b) sec
v v
(a) 210 m (b) 2100 m
d d (c) 21000 m (d) none
(c) sec (d) sec
2v 2v

Answer 1 (c) 2 (b) 3 (b) 4 (a) 5 (a) 6 (a)


Key 7 (c) 8 (b) 9 (b) 10 (d) 11 (a)
www.crackjee.xyz
Motion in a Straight Line 131
12. The displacement of a body is given to be proportional to 14. A ball is dropped vertically from a height d above the
the cube of time elapsed. The velocity-time graph of motion ground. It hits the ground and bounces up vertically to a
of the body is: height d/2. Neglecting subsequent motion and air resistance.
v v Its velocity v varies with height h above the ground as

(a) (b)
(a) (b)
t t
v v

(c) (d) (c) (d)

t t
Relative Motion
13. A ball is thrown horizontally from a height with a certain 15. A person walks up a stalled escalator in 90 s. When standing
velocity at time t = 0. The ball bounces repeatedly from the on the same escalator, now moving, he is carried in 60 s.
ground with coefficient of restitution less than 1 as shown. The time it would take him to walk up the moving escalator
Neglecting air resistance and taking the upward direction will be
as positive, which figure qualitatively depits the vertical (a) 27 s (b) 72 s
component of the balls velocity (vy) as a function of time (c) 18 s (d) 36 s
(t) : [KVPY -2013] 16. On a two lane road, car A is travelling with a speed of
36 km/h. Two cars B and C approach car A in opposite
directions with a speed of 54 km/h each. At a certain instant,
when the distance AB is equal to AC, both 1 km B decided
to overtake A before C does. The minimum acceleration of
car B is required to avoid an accident is
vy vy (a) 5 m/s2 (b) 4 m/s2
(c) 2 m/s2 (d) 1 m/s2
(a) (b) 17. A car, starting from rest, accelerates at the rate f through
t t a distance s, then continuous at constant speed for time t
f
and then decelerates at the rate come to rest. If the total
vy vy distance traversed is 5 s, then : 2
1 2 1 2
s=
(a) ft (b) s = ft
4 2
(c) (d)
t t 1 2
(c) s = ft (d)
s = ft
6

Answer 12 (b) 13 (b) 14 (a) 15 (b) 16 (d)


Key 17 (b)
www.crackjee.xyz
132 Mechanics

Mechanics
MCQ Type 2 Exercise 3.2
Multiple Correct Options (c) The average velocity of a particle is zero in a time
interval. It is possible that the instantaneous velocity
1. A particle in one-dimensional motion. Choose the correct is never zero in the interval
(a) with zero speed at an instant may have non-zero (d) The average velocity of a particle moving on a straight
acceleration at that instant line is zero in a time interval. It is possible that the
(b) with zero speed may have non-zero velocity instantaneous velocity is never zero in the interval,
(c) with constant speed must have zero acceleration (infinite accelerations are not allowed).
(d) with positive value of acceleration must be speeding 6. Consider the motion of the tip of the minute hand of a clock,
up in one hour:
2. The position-time (x–t) graphs for two children A and (a) the displacement is zero
B returning from their school O to their homes P and Q (b) the distance covered is zero
respectively are shown in figure. Choose the correct(s) (c) the average speed is zero
answers (d) the average velocity is zero.
7. The motion of a body is given by the equation
dv(t )
= 6.0 − 3v(t ) , where v(t) is speed in m/s and t in sec.
dt
If body was at rest at t = 0
(a) The terminal speed is 2.0 m/s
(b) The speed varies with the time as v(t) = 2( – e–3t)m/s
(a) A lives closer to the school than B (c) The speed is 0.1 m/s when the acceleration is half the
(b) A starts from the school at earlier than B initial value
(c) A move faster than B (d) The magnitude of the initial acceleration is 6.0 m/s2.
(d) A and B reach home at same time. 8.
A particle of mass m moves on the x-axis as follows: It starts
  from rest at t = 0 from the point x = 0, and come to rest at
3. Let v and a denote the velocity and acceleration
t = 1at the point x = 1. No other information is available
respectively of a body. Select the wrong statement(s)?
 about its motion at intermediate times [0 ≤ t ≤ 1]. If α
(a) a can be non zero when v = 0. denotes the instantaneous acceleration of the particle, then
 
(b) a must be zero when v = 0. (a) α cannot remain positive for all t in the interval 0 to 1

(c) a may be zero when v ≠ 0 (b) | α | cannot exceed 2 at any point in its path

(d) the direction of a must have some correlation with (c) | α | must be ≥ 4 at some point or points in its path

the direction of v . (d) α must change sign during the motion, but no other
  assertion can be made with the given information.
4. Let v and a denote the velocity and acceleration
respectively of a body in one-dimensional motion. Select 9. The figure shows the velocity (v) of a particle plotted against
the wrong statements? time (t):
 
(a) | v | must decreases when a < 0
(b) speed must increase when >0
 
(c) speed will increases when both v and a are < 0
 
(d) speed will decrease when v < 0 and a > 0.
5. Pick the correct statements:
(a) average speed of a particle in a given time is never less (a) The particle changes its direction of motion at some
than the magnitude of the average velocity point

dv (b) The acceleration of the particle remains constant.
(b) It is possible to have a situation in which ≠0
 dt (c) The displacement of the particle is zero
d| v|
but =0 (d) The initial and final speed of the particle are the same.
dt

Answer 1 (a,c, d) 2 (a, b, d) 3 (b,c,d) 4 (a, b, d) 5 (a, b)


Key 6 (a, d) 7 (a, b, d) 8 (a, c) 9 (a, b, c, d)
www.crackjee.xyz
Motion in a Straight Line 133

10. The velocity of a particle is zero at t = 0 12. A body falls from some height. The velocity displacement
(a) the acceleration may be zero at = 0 graph is best represent in
(b) the acceleration must be zero at = 0
(c) if the acceleration is zero from t = 0 to t = 2s, the speed (a) (b)
is also zero in this interval.
(d) if the speed is zero from t = 0 to t = 2s, the acceleration
is also zero in this interval.
11. The speed versus time graph are shown in figure. Which (c) (d)
graph(s) are possible:

(a) (b)

(c) (d)

Answer 10 (a, c, d) 11 (a, b, d) 12 (a, c)


Key

Mechanics
Reasoning Type Questions Exercise 3.3
Read the following questions and give your answer using the following options (a, b, c and d) :
(a) Statement - 1 is true, Statement - 2 is true; Statement - 2 is correct explanation for Statement - 1.
(b) Statement - 1 is true; Statement - 2 is true; Statement - 2 is not correct explanation for Statement - 1.
(c) Statement - 1 is true, Statement - 2 is false.
(d) Statement - 1 is false, Statement - 2 is true.

1. Statement - 1 4. Statement - 1
The average and instantaneous velocities have same value A body, whatever its motion is always at rest in a frame of
in a uniform motion. reference which is fixed to the body itself.
Statement - 2
Statement - 2
In uniform motion, the velocity of an object increases
The relative velocity of a body with respect to itself is zero.
uniformly.
5. Statement - 1
2. Statement - 1
A body may be accelerated even when it is moving A body can have acceleration even if its velocity is zero at
uniformly. a given instant of time.
Statement - 2 Statement - 2
When direction of motion of the body is changing, the body A body is momentarily at rest when it reverses its direction
must have acceleration. of motion.
3. Statement - 1 6. Statement - 1
The average velocity of the object over an interval of time For a non-uniform motion the magnitude of
is either smaller than or equal to the average speed of the instantaneous velocity is equal to instantaneous speed.
object over the same interval. Statement - 2
Statement - 2 A particle in nonuniform motion may move along a curved
Velocity is a vector quantity and speed is a scalar quantity. path.
www.crackjee.xyz
134 Mechanics
7. Statement - 1 Statement - 2
 
For one dimensional motion the angle between acceleration If the observer and the object are moving at velocities v1 and v 2
and velocity must be zero. respectively with reference to a laboratory frame, the velocity of
 
the object with respect to the observer is v 2 − v1.
Statement - 2
(a) Statement - 1 is true, Statement - 2 is true; Statement
One dimensional motion is always on a straight line.
- 2 is correct explanation for Statement - 1.
8. Statement - 1[IIT-JEE-2008]
(b) Statement -1 is true, Statement - 1 is true; Statement - 2
For an observer looking out through the window of a fast moving
is not correct explanation for Statement - 1.
train, the nearby objects appear to move in the opposite direction
(c) Statement - 1 is true, Statement - 2 is false.
to the train, while the distant objects appear stationary.
(d) Statement - 1 is false, Statement - 2 is true.

Answer 1 (c) 2 (a) 3 (a) 4 (a)


Key 5 (a) 6 (b) 7 (d) 8 (b)

Mechanics
Passage & Matrix Exercise 3.4
Passages
Passage for Questions. 1 to 3 : v (m/s)
From the top of a multi-storeyed building, 39.2 m tall, a boy
projects a stone vertically upwards with an initial velocity of 9.8 +10

m/s such that it finally drops to the ground.


0 t
1. The stone reach the ground in 1 2 3 4 5 6 7 8 9 10 11 12
(a) 1 s (b) 4 s –10
(c) 2 s (d) 3 s
2. The stone will pass through the point of projection
(a) 2 s (b) 3 s 4. Average velocity in whole time of motion is
(c) 4 s (d) 5 s (a) 2.22 m/s (b) 3.33 m/s
(c) 4.32 m/s (d) zero
3. The velocity before striking the ground is
5. Average speed in whole time of motion is
(Take g = 10 m/s2)
(a) 3.33 m/s (b) 4.44 m/s
(a) 14.2 m/s (b) 22.4 m/s (c) 6.67 m/s (d) zero
(c) 29.4 m/s (d) 34.2 m/s
6. The acceleration from 10 to 12 s is
Passage for Questions. 4 to 6 :
The velocity-time graph of a body moving along a straight line (a) 5 m/s2 (b) 10 m/s2
2
(c) 12 m/s (d) zero
is given below.

Matrix Matching

7. represent displacement, velocity and acceleration; t the time, then match the columns :

Column I Column II
s

A. Zero velocity (p)

B. Constant velocity (q)

t
www.crackjee.xyz
Motion in a Straight Line 135

C. Constant acceleration (r)


t

(s)

t
v

(t)

t
8.
Column I gives a list of possible set of parameters measured in some experiments. The variations of the parameters in the form of graphs
are shown in column II. Match the set of parameters given in Column I with the graphs given in Column II.
Column I Column II

A. Potential energy of a simple pendulum (p)


(y-axis) as a function of displacement
(x-axis).

B. Displacement (y-axis)as a function of time (q)


(x-axis) for one dimensional motion at zero
or constant acceleration when the body
is moving along the positive x-direction.

C. Range of a projectile (y-axis) as a function (r)


of its velocity (x-axis) when projected at a
fixed angle.

D. The square of the time period (y-axis) of a (s)


simple pendulum as a function of its length
(x-axis)
9. A particle is going along a straight line with constant acceleration a, having initial velocity u. Then the match the columns :
Column I Column II
v

u = + ve and a = + ve
A. (p)
t
0

B. u = – ve, and a = + ve (q)

t
0
v

C. u = + ve, and a = – ve (r)


t
0
www.crackjee.xyz
136 Mechanics

u = – ve, and a = – ve
D. (s)

t
0

10. Column I gives some physical situation and Column II, the graphical representation. Match the columns.
Column I Column II
a

A. A ball hits the wall and return back and then stops. (p)

t
x

B. A ball thrown upward and rebound again and (q)


again with an inelastic collision
t

C. A cricket ball hits by a bat. (r)

D. A particle start moving with constant acceleration. (s)

1 (b) 2 (a) 3 (c) 4 (b) 5 (c) 6 (a)


Answer
7 A → q ; B → p, t ; C → r, s. 8 A → (p); B → (q, s); C → (s) ; D → (q)
Key
9 A→q;B→p;C→s;D→r 10 A→ p ; B→r ; C→q ; D→ s

Mechanics
Best of JEE-(Main & Advanced) Exercise 3.5
JEE- (Main) time t/2 sec [AIEEE -2004]
1. A ball is thrown from a point with a speed ' v0 ' at an (a) at h/2 from the ground
elevation angle of θ. From the same point and at the same (b) at h/4 from the ground
'v ' (c) depends upon mass and volume of the body
instant, a person starts running with a constant speed 0
2 (d) at 3h/4 from the ground
to catch the ball. Will the person be able to catch the ball?
The relation between time t and distance x is t = αx2 + βx
3.
If yes, what should be the angle of projection θ ?
where α and β are constants. The retardation
[AIEEE 2004] [AIEEE -2005]
(a) No (b) Yes, 30° (a) 2 αv3 (b) 2 βv3
(c) Yes, 60° (d) Yes, 45° (c) 2 αβv3 (d) 2 β3v3
2. A body is released from the top of a tower of height h. It where v is the velocity
takes t sec to reach the ground. Where will be the ball after
www.crackjee.xyz
Motion in a Straight Line 137
4. The velocity of a particle is v = v0 + gt + ft2. If its position JEE- (Advanced)
is x = 0 at t = 0, then its displacement after unit time (t = 1)
7. The given graph shows the variation of velocity with
is [AIEEE 2007]
displacement. Which one of the graph given below
v0 + g/2 + f (b)
(a) v0 + 2g + 3f correctly represents the variation of acceleration with
v0 + g/2 + f/3 (d)
(c) v0 + g + f displacement. [IIT-JEE 2005]
5. A body is at rest at x = 0. At t = 0, it starts moving in the
positive x-direction with a constant acceleration. At the
same instant another body passes through x = 0 moving in
the positive x-direction with a constant speed. The position
of the first body is given by x1(t) after time t and that of the
second body x2(t) after the same time interval. Which of the
following graphs correctly describes (x1–x2) as a function (a) (b)
of time t ?  [AIEEE -2008]

(a) (b)
(c) (d)

8. Two identical discs of same radius R are rotating about their


(c) (d) axes in opposite directions with the same constant angular
speed ω. The discs are in the same horizontal plane. At time
t = 0, the points P and Q are facing each other as shown
6. Consider a rubber ball freely falling from a height h = 4.9 in the figure. The relative speed between the two points P
m onto a horizontal elastic plate. Assume that the duration and Q is vr. In one time period (T) of rotation of the discs,
of collision is negligible and the collision with the plate is vr as a function of time is best represented by
totally elastic. Then the velocity as a function of time and
the height as a function of time will be : [AIEEE 2009] [IIT-JEE 2012]
v y  
+v1
(a) h
P Q
O t R R
–v1
t vr
vr
v
y
+v1 (a) (b)
(b) h
0
O t1 2t1 3t1 4t1
t T t 0 T t
–v1
t
vr
y vr

(c) h (c) (d)


O
t1 2t1 3t1 4t1 t 0 T t 0 T t
t
v y
v1 h
(d)
O t t

Answer 1 (c) 2 (d) 3 (a) 4 (c)


Key 5 (b) 6 (b) 7 (a) 8 (a)
www.crackjee.xyz
138 Mechanics

In Chapter Exercise
In Chapter Exercise -3.1 1 2
A
h = gt ...(i)
x − 50t 2 h
1. Given, t =
10 1
h + 30 = g ( t + 1)2 ...(ii)
and B
or x = 50t + 10t 2 2
Instantaneous velocity, After solving equations, we get 30 m
dx t = 2.56 s, h = 32.1 m Ans.
v = = 50 + 20t
dt 6. The situation is shown in figure. If a is the acceleration,
The average velocity, then
3 3
x 60 m
O
∫ vdt ∫ ( 50 + 20t ) dt 1 2
B A
v = 0 = 0 x = at ...(i)
3 3 2
3 1 2
1 20t 2 and (x + 60) = a ( t + 6 ) ...(ii)
= 50t + 2
3 2 0
uA = at, ...(iii)
= 80 m/s Ans.
2. Starting from t = 0 the change in velocity in the duration 15 = a(t + 6) ...(iv)
( t1 + t2 ) is equal to a1t1 + a2t2 . After solving above equations, we get
uA = 5 m/s, a = 1.67 m/s2,
a t + a t  x = 7.50 m Ans.
Thus average acceleration =  1 1 2 2  Ans.
 t1 + t2  7. It t is the time of motion of first body, then time of motion
1 for second body will be (t–τ). For meeting of the bodies,
3. Given, 200 = u × 2 + a × 22 their displacement must be equal, so
2
1
1 H = v0t − gt 2
and (200 + 200) = u × ( 2 + 4 ) + a ( 2 + 4 )2 2
2
1 2
After solving above equations, we get = v0 ( t − τ ) − g (t − τ)
u = 115 cm/s, a = –15 cm/s2 2
\ v = u + at = 115 – 15 × 7 = 10 cm/s v0 τ
After simplifying, we get, t = + Ans.
Ans. g 2
4. If a1 is the retardation, then
In Chapter Exercise -3.2
152 = 302 – 2a1 × 1
675 ∆v −7
⇒ a1 = km/h2 1. Acceleration, a = = = 0.64 m/s
2 ∆t 11
If a2 is the acceleration, then We can write
dv
= – 0.64
302 = 152 + 2a2 × 0.5 dt
⇒ a2 = 675 km/h2 or dv = –0.64 dt
v−u 15 30 After integrating, we have
Time taken t1 = = = h v t
a1 675 / 2 675
v−u 15
∫ dv = −0.64∫ dt
and t2 = = h 7 0
a2 675
v – 7 = – 0.64 t
Total time taken in the journey or v = 7 – 0.64 t ...(i)
 30 + 15  ds
= t1 + t2 =   × 60 =
4 min. As v = ,
 675  dt
Time to be taken in the journey ds
1.5
\ = 7 − 0.64t
t =
× 60 =
3 min dt
30 After integration, we get
\ Time lost = 4 – 3 = 1 min Ans. s = 7t − 0.32t 2 ...(ii)’]Ans.
5. Suppose body falls from height h from upper point. 1 1
If t is the time taken to fall h height, then 2. Distance = × 6 × 20 + × 2 × 20 + 2 × 10
2 2
= 60 + 20 + 20 = 100 m
www.crackjee.xyz
Motion in a Straight Line 139

1 1 cal motion of the ball from the point of throw till it


Displacement = × 6 × 20 − × 2 × 20 + 2 × 10 reaches back at the initial height.
2 2
= 60 m Ans. uy = + 5 3 m/s, sy = 0

3. (i) Distance = area of speed-time graph ay = – 10m/s2, t = ?
1 10m/s
= × 10 × 12 =60 m 5 3 m/s
2 A
distance 60 B
(ii) Average speed = = = 6 m/s 60°
time 10 u
(iii) Clearly 0 and 10 s. 5m/s
(iv) Clearly 5s. 1 2
4. (i) Slope of x–t curve of car C is greatest and so its speed Applying s = ut + at
2
is highest. The speed of car A is lowest one. 2
(ii) There is no position at which x–t of all the cars 0 = 5 3t − 5t
intersect. ∴ t = 3 sec
(iii) When A passes C, car B is 6 km from the origin. Considering horizontal motion from the perspective
8 of observer B. Let u be the speed of train at the time
(v) & (vi) v=
A = 5 km/h;
1.6 of throw.
14 12 The horizontal distance travelled by the ball
v=
B = 10 km/h; v=
C = 12 km/h = (u + 5) 3 .
1.4 1
The horizontal distance travelled by the boy

vCA = 12 − 5 = 7 km/h and vBC =
10 − 12 =
−2 km/h.
 1 
= u 3 + a ( 3)2  + 1.15
In Chapter Exercise -3.3  2 
As the boy catches the ball therefore
1. [vgas]rocket = vgas – vrocket 3
= –1500 – 500 = 2000 km/h (u + 5) 3 = u 3 + a + 1.15
2
1200 km/h
2. 450 km/h ∴ 5 3 = 1.5a + 1.15 ∴ 7.51 = 1.5a
∴ a ≈ 5 m/s2
  
 v gas  =  v gas  −  v jet 
g g

jet
 
In Chapter Exercise -3.4
\  v gas  =  v gas  +  v jet  1. Given, x = t 3 − 4t 2 + 3t
g jet g
= – 1200 + 450 (a) At t = 0, xi = 0
= 750 km/h Ans. t =4, x f = 43 − 4 × 42 + 3 × 4 = 12 m

3. If t is the time taken, then
\ Displacement = x f − xi =
12 m
1 2
× 2 ( t + 5 ) = 20 t
2 (b) At t = 2s, xi =23 − 4 × 22 + 3 × 2 =−2
After solving, t = 5 s Ans.
  t = 4s, x f = 43 − 4 × 42 + 3 × 4 = 12
4. (i) Relative speed = v A − v B
Displacement, s = x f − xi = 12 − ( −2 ) = 14
= 60ˆi − 45ˆi =
15ˆi km/h
Displacement
(ii) Relative speed = v A − vB Average velocity =
time interval
= 60ˆi − 45ˆi =
15ˆi km/h 14
= = 7 m/s
480 4−2
5. The speed of bush A,
vA = = 60 km/h
8 dv
2. Given − = a v
480 dt
The speed of bush B,
vB = = 40 km/h
12 dv
or = adt
If t is the time of meeting, then v
v At + vBt = 480 v t
−1/2
or 60t + 40t = 480
or ∫ v dv = ∫ ( −a ) dt
v0 0
\ t = 4.8 h.
The distance travelled by car A in this duration, 1/2 v
v
= – at
xA = vAt = 60 × 4.8 1/ 2
v0
= 288 km Ans.
− at
6. 5 From the perspective of observer A, considering verti- v − v0 =
2
www.crackjee.xyz
140 Mechanics
at 3 Suppose particles are moving mutually perpendicular, so
\ v = v0 − ...(i) that
2   
For stop, v to be zero i.e., v 21 = v 2 − v1
at v21 = v12 + v22 =
v ...(i)
0 = v0 −
2
The distance between them at any instant
2 v0
\ t = Ans. x2 = ( a − v1t )2 + ( v2t )2 ...(ii)
a
Again from equation (i), we have After differentiation, we get
2 dx
ds  at  2x = 2 ( a − v1t ) × ( −v1 ) + 2v2t × v2
=  v0 −  dt
dt  2 a
2 (a – v1t) v1
 at 
or ds =  v0 −  dt
 2 v2
On integrating, we get
3 v2t
 at  x
 v0 −  2  at 
3
s =  2
−  v0 − 
=
 −a  3a  2
3×  
 2 
2 v0 dx
At t = , For x to be minimum, = 0,
a dt

−2  a 2 v0 
3 or 0 = 2 ( a − v1t ) × ( −v1 ) + 2tv22
s =  v0 − 

3a  a  av1 av1
 \ t = = Ans.
v12 + v22 v
 2  3/2
=   v0 Ans. After substituting this values in eq. (i), we get
 3a 
av2
x = Ans.
v

Exercise 3.1 Level -1


2 x 3x 6. (c) For first 1m of fall,
+ 5v1v2 1 2
5 5 2
1. (d) The average speed = 2 x / 5 3 x / 5 = . 1 = gt1 , ∴ t1 =
3v1 + 2v2 2 g
+
v1 v2 1 2 4
2. (a) Average speed is given by For 2m of fall, 2 = gt , ∴ t= ,
2 g
vav = 2v0 (v1 + v2 )
4 2 2
2v0 + v1 + v2 ∴ t2 = t – t1 = − = ( 2 − 1)
g g g
2 × 3(4.5 + 7.5)
= =4 m/s For 3 m of fall,
(2 × 3) + 4.5 + 7.5
1 2 2
(c) For uniformly accelerated motion, the s must be
3. 3 = gt , ∴ t = 3 .
2 g
quadratic in t. So,
x−a 2
t2 = ∴ t3 = t – t2 = ( 3 − 2) .
b g
v v v
2 7. (b) t 1 2
t3
3
or x = a + bt 1 t2
4. (c) v = 20 + 0.1 t2 Velocity changes from v1 to v2 in time t1 + t2, so
dv v −v
∴ a = = 0.1 × 2t = 0.2t. a = 2 1 .
dt t1 + t2
Here acceleration depends on time. v3 − v2
Similarly a =
1 t2 + t3
5. (b) s1 = a (10)2
2  v2 − v1   v3 − v2 
1 Thus  t + t  =  .
and s = a (20)2 = 4 s1 1 2  t2 + t3 
2
∴ s2 = s – s1 = 3s1.
www.crackjee.xyz
Motion in a Straight Line 141

2 1 2
8. (d) 0 = u2 – 2a s ⇒ s = u Also h = ut2 − gt2
2a 2
After simplify above equations, we get
s1 u12 u2 1
∴ =
= 2
= 2 1
s2 u2 (4u ) 16 h = gt1t2 .
2
9. (c) If h is the height of the building, then 17. (c) If t is the time taken by the first ball to hit the water,
v 2A = v gh 1
then 122.5 = × 9.8 × t2
2
and vB2 = ( −v)2 + 2 gh .
∴ t = 5s.
Clearly vA = vB. The time of motion for second ball is 3s. So
10. (a) Given t = x + 3 1
122.5 = u × 3 + × 9.8 × 32
or x = (t – 3)2. 2
∴ u = 26.1 m/s.
dx A a B
Velocity v = = 2 (t – 3) 18. (d) From the geometry, we have
dt
a2 + (v1t)2 = (v t )2
or 0 = 2(t – 3) vt v1t
∴ t = 3s a
Thus x = (3 – 3)2 = 0. ∴ t =
v – v12
2
dx d
11. (a) v = = (t + 5) −1 = −1(t + 5) −2 a1
dt dt 19. (a) 0 + (2 × 5 − 1) = 0 + [2 – 3 − 1]
and a =( −1) × (–2)(t + 5) −3 2
a1 5
2
or a= 4(t + 5) −6 = Cv3 ∴ = .
a2 9
∴ a ∝ v3 / 2 1
20. (a) vt = at 2
12. (c) The displacement, s = 2R cosθ, 2
and acceleration, a = g cosθ. 2v
1 ∴ t = .
∴ s = at 2 a
2 dx 1
1 21. (c) = (1 + t 2 ) −1/ 2 × 2t = t (1 + t 2 ) −1/ 2
or 2R cosθ = ( g cos θ)t
2 dt 2
2
d2x 2 −1/ 2  1 2 −3 / 2 1 t2
R a == (1 + t ) + t  −  (1 + t ) =– .
∴ t = 2 . h dt 2  2 x x3
g
v/2
h1 = 1 × 10 × 52 =
B
13. (b) –30 = 18 + a × 2.4 22. (b) 125 m
∴ a = – 20 m/s2. v 3m 2
1 2
2 A h2 = × 10 × 5 = 125 m
14. (a)  v  =  v2 – 2g × 3 2
 
2 ∴ h = h1 – h2 = 80 m.
u
∴ v = 8g . 1 2
23. (b)  = 9.8 a = g sin 30° = × 10 × 5 = 125 m
If h is the further height, then 2
2 1
 v Using  = at 2
0 =  2  − 2 gh 2
1 g
v2 8g or 9.8 = × × t 2
∴ h = = = 1 m. 2 2
8g 8g ⇒ t = 2 s.
1 2 1 24. (a) The time taken by the ball to reach the highest point
s1 =
15. (b) gt and= s2 g (t − 1)2 u
2 2 0 = u –gn ⇒ n = .
1 g
∴ s1 – s2 = 10 = g [t 2 − (t − 1)2 ]
2 1 2
Sn = un – gn
∴ t = 1.5 s. 2
1 2 and 1
16. (a) h = ut1 − gt1 S(n – t) = u (n–t) – g(n–t)2
2 2
www.crackjee.xyz
142 Mechanics

1 2 vA tan 30° 1/ 3 1
∴ s = Sn – S(n–t) = gt . 33. (d) = = = .
2 vB tan 60° 3 3
1 2 1 ∆v v2 − v1
25. (c) h = ut − gt also h = u (t + ∆t) – g (t + ∆t)2. =
2 2 34. (d) a =
∆t t2 − t1
After simplifying above equations, we get
∴ v2 = v1 + a(t2 – t1)
1 
u = 8 gh + g 2 ( ∆t )2 . Here v1 = 0, ∴ v2 = area of a − t between t1 and t2.
2
35. (d) The maximum acceleration will occur in the duration
1 1
26. (b) L = gt 2 − g (t − T )2 30 s to 40s. So
2 2
v2 − v1 60 − 20
T L = a = = 4 m/s2.
⇒ t = + . t2 − t1 40 − 30
2 gt
v2 − v1
1 36. (a) a =
27. (c) × 1 × t 2 = 10t
48 + ∆t
2
tan135° − tan 45°
∴ t = 8 s and 12 s. = = –2 m/s2
1
dv
28. (d) = at 37. (c) x = a – bt + ct2
dt dx
v t ∴ v = = – b + 2c t.
or ∫u dv = ∫0 (at ) dt dt
It represents a straight line with negative intercept on
2
∴ v = u + at y-axis.
2 38. (b) v = area of a – t upto 3s.
dv = 3 × 2 – 3 × 1 = 3 m/s
29. (a) = 2(t –1) 39. (c) According to the acceleration time graph, first velocity
dt
v 5 increases, then becomes constant and thereafter it will
or ∫0 dv = ∫0 2(t − 1) dt increase.
40. (d)
 t2   52 
5 
v 2  − t  = 2  − 5 = 15 m/s.
or
= 41. (a) It is the s − t graph of a body projected upward. It has
2 0  2  uniform acceleration downward.
42. (c) v2 = 2as.
dv
30. (a) = –kv3 It represents a parabola about s-axis.
dt
43. (c) We have v2 = u2 + 2a s
v dv t
or ∫v0 v3 = − k ∫0 dt or 900 = 3600 + 2a × 0.6
∴ a = – 2250.
v0
∴ v = vT
2v02 kt + 1 44. (d) The distance, s =
2
dv P v v
31. (b) = Also, m = , ∴ T = .
dt mv T m
dv P v2
or v = Now s = .
ds mv 2m
s m v2 2 45. (a) From 0 – 2 s: v = 0 + at = 3 × 2 = 6 m/s
or ∫0 ds = P ∫v1 v dv From 2 – 4 s : v = 6 – at = 6 – 3 × 2 = 0.
m 3 3 46. (c) If v1 and v2 are the velocities, then
∴ s = (v2 − v1 ) .
3P (v1 – v2) × 20 = (100 + 100) ...(i)
dv and (v1 + v2) × 10 = (100 + 100) ...(ii)
32. (b) = 3t2 + 2t + 2
dt After solving above equations, we get
v 2 2 v1 = 15 m/s and v2 = 5 m/s.
or ∫2 dv = ∫0 (3t + 2t + 2)dt
47. (b) If t is the time of crossing, then
2 100 = (10 + 15) t
t3 t2
v – 2 = 3 + 2 + 2t ∴ t = 4 s.
3 2
0 48. (a) The speed of approach, = v – (–2v) = 3 v.
∴ v = 18 m/s.
www.crackjee.xyz
Motion in a Straight Line 143

 1
∴ t =
, each one have same time. or 2.5 = × 10 × t 2
3v 2
49. (c) vA = u – gt and vB = gt. 1
vA – vB = (u – gt) – gt = u – 2gt. ∴ t = s.
2
50. (b) For the elevator going with constant velocity, we have 1
1 2 51. (c) x0 = × 1 × 502 =1250 m
h = gt 2
2
Exercise 3.1 Level -2
1. (c) x = 20 t e–t v
dx  de −t  veffective = v cos 45° = .
= 20  t + e −t × 1 2
∴ v =
dt  dt 
K L

or 0 = 20 [t e–t × (–1) + e–t]


∴ t = 1 v s
20
x = 20 × 1 × e −1 = .
M
Thus N d
e
2. (b) If u is the velocity of projection, then s d/ 2 d
Time, t = = = .
0 = u2 – 2g (4h) y veffective v / 2 v

∴ u = 8gh 7. (c) If A is the displacement along the velocity vector, then
y
h-y
Now y = gt ...(i) u
1 2 y=
x B
and h – y = ut −
gt ...(ii)
2 A
x
From above equations, we have  
45°

h h h A = 2v = (6iˆ + 8 ˆj ) .
t =
= =
u 8 gh 8g unit vector along line y = x,
 ˆ ˆ
3. (b) The time taken to move net 2 steps is 8s, and so for 8 steps B = cos 45° i + sin 45° j
he takes 32 s. In last 5 steps he will take 5s and fall into the  iˆ ˆj 
pit. =  + .
4. (a) For maximum velocity, a = 0  2 2 

and so, 0 = b – cx or x = b/c. Thus the displacement along B
dv  
Now, v = b − cx AB cos θ A . B
dx A cos θ = =
B B
v x
ˆ
i ˆj
dv ∫ ( b − cx ) dx
∫v= (6iˆ + 8 ˆj ).( + )
0 0 2 2
=
2 1
v cx 2
= bx − = 7 2 m.
2 2
8. (b) Let a1 and a2 be the retardations offered to the bullet
2
b c (b / c) by wood and iron respectively.
= b× −
c 2 a1 a2
b
or v= B` C
c v2 0
5. (a) The time taken by ball to reach maximum height
1
4.9 = 0 + × g × t 2 C` B` A`
2 v2
0
∴ t = 1 s. u
So number of balls thrown per minute will be 60. Wood Iron
d /2 d 4cm 2cm
6. (a) s = =
cos 45° 2 2 2
For A→ B→ C, v1 − u = 2a1 (4),
www.crackjee.xyz
144 Mechanics
2 2 14. (a) At a height y, the velocity of the ball
and 0 − v1 = 2a2 …(1)
v2 = 0 + 2g (d–y) d-y
2
Adding, we get −u= 2(4a1 + a2 ) or v = 2 g (d − y )
For A ' → B ' → C ' , v22 − u 2 =
2a2 …(2) , It represents a parabola between v and d. y
At y = d, v = 0
and 02 − v22 = 2 a1 …(3)
y = 0, v = 2gd 0
Adding , we get, −u 2= 2 (2a1 + 2a2 ) Just after collision, the speed of the ball
Equating (1) and (2) and solving , we get v = 2 g d / 2 = gd
4a1 + a2 = 2a1 + 2a2 ⇒ a2 = 2a1
Taking downward velocity negative, one find graph of
9. (b) Velocity of the parachute after falling 50 m
option (a) is correct.
u = 2 g × 50 = 10 g m/s
15. (d) If u and v are the speeds of the person and of the escalator,
Thus 32 = u2 – 2 × 2 × h then
or 32 = 100 g – 4 h 50m
 
∴ h = 243 m. = 90, ∴ u =
u 90
The height at which parachute bails out h  
= 243 + 50 = 293 m. and = 60, ∴ v =
10. (d) Acceleration can be written as a = 2 + 2 – t or v 60
a = 4 – t for t ≤ 2s and a = 2 + t – 2 or a = t for t ≥ 2 s If t is the required time, then
Therefore, acceleration time graph of the particle will  
be as shown below t= =   = 36s.
u+v +
a(m / s 2 ) 90 60
16. (d) See examples
4 v

2 17. (b) vmax


f f/2
t (s)
2 4
t
Now since, dv = a dt t1 t 2t1

v f − vi = area under (a-t) graph 1 2 2s


s = ft1 ; ∴ t1 =
1 2 f
or vf – 0 = (4 × 4) – (4) (2)
2 2s
vmax = f t1 = f
= 2 fs .
= 12 m/s f
or velocity of particle at the end of 4s is 12 m/s. 1
11. (a) See examples Thus 5s =
2
[[t + 3t1 ) + t ] × vmax.
12. (b) Given s = k t3 1
ds or 5s = ( 2t + 3t1 ) × vmax
∴ v = = 3 k t2. 2
dt
1 2s 
It represents a parabola between v and t. or 5s =  2t + 3 × 2 fs
13. (b) 2 f 
1 2
∴ s = ft .
2
Exercise 3.2
1. (a,c,d) At the highest point of the projected body, a = g, so
(a) At the highest point of the projected body. option (a) is correct only.
(c) Constant speed will have no change in velocity, so 4. (a,b,d)
acceleration will be zero. Only option (c) is correct . According to it
(d) v = u + at. v = – u – at = – (u + at)
2. (a, b, d)
or |v| = u + at.
On position axis, P is closer to O than Q.
From time axis, it can be said that A starts earlier than 5. (a, b)
B. av velocity
(a) ≤1
3. (b, c, d) av speed
www.crackjee.xyz
Motion in a Straight Line 145
 drawn as shown in figure.
 d | v | v
(b) In circular motion speed is constant   is The displacement in time t,
 dt  α
vmax β
1  αβ  2
constant but there is centripetal acceleration. x=  t
2  α + β t
6. (a, d) Self explanatory. 0 t
Putting x = 1, t = 1 s,
dv
7. (a,b,d) For constant velocity, =0 if α = β,
dt then α = 4,
∴ 0 = 6.0 – 3v ⇒ v = 6/3 = 2m/s or If α > β, then α > 4 m/s2.
At t = 0, v=0 9. (a,b,c,d) From 0 – T, the velocity is negative and from T

∴ a = 6 – 3 × 0 = 6 m/s2. to 2T, the velocity is position. The area of v –t graph
dv will be zero, so displacement will be zero. The slope
Again = 6 – 3v of v–t is same throughout, so acceleration is constant.
dt
10. (a,c,d) If velocity is zero, the acceleration need not be
v dv t
or ∫0 = ∫ dt zero. If initial velocity is zero and acceleration is also
(6 − 3v) 0
zero in interval 0 – 2s, then speed must be zero in this
v interval.
n(6 − 3v) t
= | t |0 11. (a,b,d) Speed can never be zero and so option (c) is not
( −3) 0
possible.
or v = 2(1– e–3t). 12. (a,c) When body falls, its velocity v2 = 2gs, it represents a

8. (a, c) For the data, v − t graph can be parabola b/w v and s and straight line between v2 and s.
Exercise 3.3
1. (c) In uniform motion the speed is same at each instant of 7. (d) One dimensional motion is always along straight line.
motion. But acceleration may be opposite of velocity and so
2. (a) In uniform circular motion, there is acceleration of angle between them will be 180°.
  
constant magnitude. 8. (b) v21 = v2 − v1
3. (a) Because displacement ≤ distance and so average As angular velocity is same for two points, so v21 is
velocity ≤ average speed. very small
    
4. (a) vbody  frame = vbody − v frame = v − v = 0
s

5. (a) At the highest point of the projected body, its


acceleration is not zero.
6. (b) Instantaneous velocity is always equal to instantaneous
speed.
Exercise 3.4
Passage for Questions 1 to 3 2
a (m/s )
1. (b) 2. (a) 3. (c) +5
The time taken by stone to reach the highest point
0 t
0 = 9.8 − gt1 1 2 3 4 5 6 7 8 9 10 11 12
–5
9.8
\ t1 = = 1 s
g
4. (b) Area of graph above time axis
The time taken to pass through the point of projection = 2s. 1
A = × (8 + 4) × 10 =
60m
If t is the time to reach the ground, then 2
Area of graph below time axis
1 2
39.2 = −9.8t + gt
2 1
B = × 4 × 10 =20 m
2
After solving, we get, t = 4s. 39.2 m
Average velocity in whole time of motion
Velocity before striking the ground,
 displacement
v2 = u 2 + 2 gh v =
total time
2
= ( −9.8 ) + 2 × 9.8 × 39.2
area A − area B
v = 29.4 m/s Ans. =
total time
Passage for Questions 4 to 6
60 − 20
= = 3.33 m/s . Ans.
12
www.crackjee.xyz
146 Mechanics
5. (c) Average speed in whole time of motion v2 − v1 10 − 10
From 2 to 6 s; a= = = 0
totaldistance area A + area B t2 − t1 6−2
vav = =
total time total time v2 − v1 −10 − 10
From 6 to 10 s; a = = = −5 m/s2
60 + 20 t2 − t1 10 − 6
= = 6.67 m/s . Ans.
12 v2 − v1 0 − (−10)
From 10 to 12 s; a == = 5 m/s2
6. (a) Acceleration: t2 − t1 12 − 10
v2 − v1 10 − 0
From 0 to 2 s; a= = = 5 m/s2 Acceleration vs time graph is drawn in figure (b).
t2 − t1 2−0
7. to 10. have been explained in the theory of the chapter.
Exercise 3.5
1. (c) Yes, the person can catch the ball when horizontal a negative slope.
velocity is equal to the horizontal component of ball’s 1
Also applying y − y0 = ut + at 2
velocity, the motion of ball will be only in vertical 2
direction with respect to person for that, 1 2 1
vo We get y − h =− gt ⇒ y = h − gt 2
= vo cos θ or θ= 60° 2 2
2 The graph between y and t is a parabola with y = h at
2v t = 0. As time increases y decreases.
2. (d) h = For upward motion.
a y
1 h The ball suffer elastic collision with the horizontal
2
and y = g (t / 2) = elastic plate therefore the direction of velocity is
2 4
reversed and the magnitude remains the same.
h 3h h
The ball is at a height = h − = from Here v = u – gt where u is the velocity just after
4 4
ground. collision.
3. (a) Given t = αx2 + βx As t increases, v decreases. We get a straight line
Differentiating above equation w.r.t. time, we get between v and t with negative slope.
1
dx dx Also = y ut − gt 2
1 = α × 2 x + β 2
dt dt
All these characteristics are represented by graph (b).
or 1 = 2α x v + βv,
7. (a) From the geometry of the figure,
1 v
∴ = 2α x + β ...(i)
v v0
Differentiating again, we get we have,
dv dx dv v v0 v
0 = 2α ( x + v ) + β x − x =
dt dt dt 0 x0 x x0
x
or 2
0 = 2α ( x a + v ) + βa ...(ii)  x0 − x   x
From above equations, we get ∴ v =   v0= 1 −  v0
 x0   x0 
a = – 2αv3. v0
dv
dx Also = −
4. (c) We know that, v = ⇒ dx = v dt dx x0
dt Thus acceleration,
x t
dv  x  v  v02 v02
Integrating, ∫ dx = ∫ v dt a = v = 1 −  v0 ×  − 0  = 2 x −
0 0 dx  x0   x0  x0 x0
t t
 2 gt 2 ft 3  It represents a straight line between a and x with
or x= ∫ (v0 + gt + ft ) dt = v0t + +  negative intersect.
0  2 3 
0 8. (a) At t = 0, the relative velocity will be zero.
gt 2 ft 3 T
or, x =v0t + + At t = , the relative velocity will be maximum in
2 3 4
g f magnitude.
At t = 1, x = v0 + + .
2 3 T
1 At t = , the relative velocity will be zero.
5. (b) x1 – x2 = ut − at 2 2
2 3T
It represents a parabola like in option (b). At t = , the relative velocity will be maximum in
4
6. (b) For downward motion v = –gt magnitude
The velocity of the rubber ball increases in downward At t = T, the relative velocity again becomes zero.
direction and we get a straight line between v and t with
www.crackjee.xyz
www.crackjee.xyz
148 MECHANICS

Definitions, Explanations and Derivations


4.1 INTRODUCTION
In the previous chapter we defined kinematic parameters like position, displacement, velocity and
acceleration for objects moving along a straight line. The directional aspect of these parameters could
be taken by using ( + ) and (–) signs but this is not possible for objects moving in a plane and in a three
dimensions. To understand motion of such objects we have to use the knowledge of vectors.

4.2 POSITION VECTOR AND DISPLACEMENT


r
The position vector r of a particle P located in a plane with reference to the origin of an xy-coordinate
system is given by
r
r = xˆi + yˆj .... (i)

r uur
(a) Position vector r (b) Displacement Dr & average
r
velocity v of a particle
Figure. 4.1
Suppose a particle moves along the path shown in fig. 4.1. The particle is at P at time t and at P¢ at time
t¢, then the displacement is given by
uur ur r
Dr = r ' - r
= ( x ' ˆi + y'ˆj) - ( x ˆi + y ˆj)

= ( x '- x)ˆi + ( y '– y )ˆj


uur
or Dr = Dx ˆi + Dy ˆj. … (ii)

4.3 AVERAGE VELOCITY


uur
The average velocity v of an object can be obtained by dividing displacement Dr by the corresponding
time interval Dt. Thus average velocity
r
r Dr
v av = v =
Dt
Dx ˆi + Dy ˆj Dx ˆ Dy ˆ
= = i+ j
Dt Dt Dt

or v = v x ˆi + v y ˆj .
r
Dr r
Since v = , so the direction of the average velocity is the same as that of Dr , see fig. 4.2.
Dt
Instantaneous velocity
The instantaneous velocity is given by the limiting value of the average velocity as the time interval
approaches zero. Thus instantaneous velocity
www.crackjee.xyz
Motion in a Plane 149
r r
r Dr d r
v = lim =
Dt ® 0 Dt dt

(a) Direction of average velocity (b) Direction of instantaneous velocity


Figure. 4.2
r
The velocity v can be expressed in component form as:
r
r dr
v =
dt
æ Dx Dy ˆ ö
= lim ç iˆ + j÷
Dt ®0 èDt Dt ø

æ Dx ö ˆ æ Dy ö ˆ
= çè lim ÷ i + çè lim ÷j
Dt ®0 Dt ø Dt ®0 Dt ø

r dx ˆ dy ˆ
or v = i+ j
dt dt
r = v x ˆi + v y ˆj. ... (i)
v
The magnitude of vr is

v = vx2 + v y 2 ...(ii) Figure. 4.3


and the direction of vr is given by the angle q
vy
tan q =
vx
æ vy ö
or q = tan -1 ç ÷ .
è vx ø
4.4 AVERAGE ACCELERATION
r
The average acceleration a of an object for any time interval Dt moving in xy plane is the change in
velocity divided by the time interval Dt. Thus average acceleration
r
r r D v D (v x ˆi + v y ˆj)
aav = a = =
Dt Dt

Dvx ˆ Dv y ˆ
= i+ j
Dt Dt

or r = a x ˆi + a y ˆj .
a
Instantaneous acceleration
It is the limiting value of the average acceleration as the time interval approaches zero: Thus instantaneous
acceleration
www.crackjee.xyz
150 MECHANICS

r
Dv æ Dv ˆi Dv y ˆjö
r = lim = lim ç x + ÷
a Dt ® 0 Dt D t ® 0 è Dt Dt ø

æ Dvx ö ˆ æ Dv y ö
ˆ
= çè lim ÷ø i + ç lim ÷j
D t ® 0 Dt è Dt ®0 Dt ø
r a x ˆi + a y ˆj .
or a =

Figure. 4.4

4.5 MOTION IN A PLANE WITH CONSTANT ACCELERATION


r
Suppose an object is moving in xy- plane and its acceleration a is constant. Let initial velocity of the
r
object is u , then velocity at any time
r r r
v = u + at.
In terms of components, we can write :
vx = u x + ax t …(1)
and vy = u y + ayt
Similarly displacement at any time t is given by;
r uur r 1 r
r = r0 + ut + at 2
2
1 2
In components form we can write : x = x0 + uxt + at
2 x
1 2
and y = y0 + uyt + at.
2 y
If x0 and y0 are zero, then
1 2
x = u x t + a xt
2
1 2
y = u yt + ay t ...(2)
2
From equations (1) and (2), we also have
rr r r r r
v.v = (u + at).(u + at )
rr rr rr rr
= u.u + a.ut + u.a + a.at 2
rr r æ r 1 r 2ö
= u.u + 2a. çè ut + at ÷ø
2
rr rr r r r
or v.v = u.u + 2a.(r - r0 )
www.crackjee.xyz
Motion in a Plane 151
In component form we can write :
vx2 = u x2 + 2ax x

v 2y = u 2y + 2a y y ...(3)

4.6 RELATIVE VELOCITY IN TWO DIMENSIONS


The concept of relative velocity introduced in previous chapter can be easily extended to motion in a
uuur uuur
plane. Suppose that two objects A and B are moving with velocities v A and v B with respect to
stationary observer (ground). Then velocity of A relative to that of B is :
r r r
v AB = v A - v B . ...(i)
Similarly, the velocity of B relative to that of A is:
r r r
v BA = v B - v A . ...(ii)

Rain and man


Let us consider rain is falling and a man is running on the horizontal road. The man
experiences the velocity of rain relative to himself. To prevent himself from the rain,
the man should hold his umbrella in the direction of the relative velocity of rain w.r.t.
man.
r
Suppose the velocity of rain is v R directed vertically downward and man is moving
r
along north with a velocity vM . Thus velocity of rain relative to man
r r r
v RM = v R – v M . Fig. 4.5 shows the velocity of rain w.r.t. man.

r v
Let vRM makes angle q with the vertical, then tan q = R south of vertical.
vM

Thus the man should hold his umbrella at an angle q with the vertical towards north
(In the direction of his motion) to protect himself from rain. The velocity with which

rain strikes the umbrella v RM = v R2 + v M


2 .

River- boat Figure. 4.5


r
A river of width b is flowing with velocity v r and a boatman can steer his boat with
r
a velocity v br with respect to river. The velocity of boatman with respect to ground
r r r
évb ù
ë ûground = éë v b ùû river + éë v river ùû ground
r r r
v b = v br + v r .
(a) The velocity of boatman when steers the boat along upstream direction (against flow)
r r r
é vb ù
ë û up = v br - v r

or [ vb ]up = vbr - vr

Thus the time to travel a distance x against the flow


Figure. 4.6
x
.
t = ( vbr – vr )
www.crackjee.xyz
152 MECHANICS

(b) When he steers the boat along downstream direction the velocity of boatman
[vb ]down = vbr + vr .
Thus the time to travel a distance x in the direction of flow
x
t = .
(vbr + vr )
Time to cross the river
Suppose the boatman steers the boat at an angle q with the line A B all the time of his motion.
The velocity of the boatman along the line AB
r r r
v b y = v bry + vry

or vby = vbr cos q + 0


= vbr cos q
And his velocity along the flow
r r r
Figure. 4.7 v bx = v brx + v rx

or vbx = –vbr sin q + vr


= vr – vbr sin q
Thus time to cross the river
displacement
t = velocity along the displacement
b
or t = vbr cos q … (i)

The displacement along the flow (x- axis ) when he reaches on the other bank
x = velocity of boatman along flow × time to
cross the river
= vbx × t

b
= (vr - vbr sin q) ´
vbr cos q

æ vr - vbr sin q ö
or x = b ç v cos q ÷ … (ii)
è br ø

Let he arrive at a point C on the other bank of the river, then his net displacement

s = x2 + y 2

= x 2 + b2 … (iii)
The velocity of boatman with respect to ground

vb = v +v
bx 2 by 2

(a) To cross the river in minimum time:


From equation (i), we can see that time to cross the river is minimum when cos q = +1,or q = 0° :
That is, the boatman steers his boat always at right angles to the direction of flow.
b
\ tmin = vbr … (iv)
Figure. 4.8
www.crackjee.xyz
Motion in a Plane 153
His displacement along the flow (drift ):
b æv ö
x = vx ´ = bç r ÷ … (v)
vbr è vbr ø
(b) To cross the river along shortest path:
It is clear that the shortest path is from A to B.
For the drift , x = 0
b
or (vr - vbr sin q) ´ = 0
vbr cos q
vr
or sin qvbr= …(i)
Figure. 4.9
Hence, to reach at point B just opposite to A , boatman
æv ö
should steer his boat at an angle q = sin -1 r .
çè v ÷ø
br

As sin q >/ 1, so for vr ³ vbr , the boatman can never reach the opposite bank at B. For
sin q = 1; q = 90o , it is impossible to reach the opposite bank, as it is clear from the following fig. 4.10.
Figure. 4.10

Note:
For vr ³ vbr , does not mean that boatman will not reach the other bank. It means he can not reach
the point opposite to starting point. Time to cross the river

b b
t = =
vbr cos q vbr - vr 2
2

b
Since cos q < 1 for the possible angle q, the time to cross the river in this case is greater than .
vbr

FORMULAE USED
r r r
1. The velocity of rain relative to man, vRM = vR - vM
r v
If q is the angle made by vRM from vertical, then tan q = R
vm

2. The velocity with which rain strikes the umbrella vRM = vR2 + vM
2

3. River-boat :
(i) A boatman can cross the river in minimum time, if he always sail the boat right across the flow,
B C
b é Here vr ® v ù
tmin = . ê ú
u ë and vbr ® u û
v b
b
Drift,BC = v ´ .
u u
A
(ii) If boatman wants to reach the opposite bank just in front of starting B
point, then he should sail the boat at angle q, where
v
sin q = ,
u b
b v
and time to cross the river, t = . u q
u cos q
A
www.crackjee.xyz
154 MECHANICS

EXAMPLES BASED ON RAIN AND MAN & RIVER-BOAT PROBLEMS


Example 1. A particle starts from the origin at t = 0 s with a Distance moved along the river in time
velocity of 10.0 ĵ m/s and moves in the xy- plane with a constant 1
t = vr × t = 3 km/h × h = 750 m Ans.
acceleration of (8.0iˆ + 2.0jˆ) m/s2 . The y-coordinate of the particle 4
when x-coordinate is 16: [NCERT] Example 4. A man running along a straight road with uniform
r
Sol. Given that : velocity u = u ˆi feels that the rain is falling vertically down along –
ux = 0; uy = 10 m/s
and ax = 8.0 m/s2; ay = 2.0 m/s2 ĵ . If he doubles his speed, he finds that the rain is coming at an
Let at time t the x-coordinate is 16 m. We have angle q with the vertical. Find the actual direction and speed of
1 2 the rain with respect to the ground.
x = ux + at Sol. Suppose velocity of rain
2 x
r
1 v R = v x ˆi - v y ˆj
or 16 = 0 + ´ 8 ´ t2
2
After simplifying,
t = 2s
The y coordinate at
t = 2 s is
1
a t2
y = uyt +
2 y
1
= 10 × 2 + × 2 × 22 = 24 m Ans.
2
Ex ample 2 . Rain is falling vertically with a speed of
30 ms–1. A woman rides a bicycle with a speed of 10 ms–1 in the
north to south direction. What is the direction in which she should
hold her umbrella ? [NCERT]
Figure. 4.12
and the velocity of the man
r
vrain

v m = u iˆ
q \ Velocity of rain relative to man
Sol. C O vwoman r r r
N S v Rm = v R - v m = ( v x - u ) ˆi - v y ˆj
B
q According to given condition that rain appears to fall vertically,
so ( v x - u ) must be zero.
D A \ vx– u = 0
or vx = u
Figure. 4.11 uur
The rain is falling along OA with speed 30 ms–1 and woman rider is When he doubles his speed, v'm = 2u ˆi
moving along OS with 10 ms–1. i.e. OA = 30 ms–1 and OB = 10 ms–1. The r r uur
Now v Rm = v R - v' m
woman rider can protect herself from the rain if she holds her umbrella
in the direction of relative velocity of rain w.r.t. woman. To do so apply
equal and opposite velocity of woman on the rain i.e. impress the
= ( vxˆi - v y ˆj) - (2uˆi)
velocity 10 ms–1due north on rain which is represented by OC. Now
r r = ( vx - 2u ) ˆi – v y ˆj
the relative velocity of rain w.r.t. woman will be (vrain - vwoman ) ,
represented by diagonal OD of parallelogram OADC.
If ÐAOD = q, then in DOAD, tan q = AD/OA = OC/OA = 10/30 = 0.33
= tan18° 26¢
B = 18°26¢ north of vertical.
Example 3. A man can swim with a speed of 4.0 km/h in still
water. How long does he take to cross a river 1.0 km wide if the
river flows steadily at 3.0 km/h and he makes his strokes normal
to the river current? How far down the river does he go when he
Figure. 4.13
reaches the other bank? r
Sol. Time to cross the river, t = width of river/ speed of man The v Rm makes an angle q with the vertical
r
1 x - componend of v Rm
=
4
h = 15 m. tan q = y - componend of vr
Rm
www.crackjee.xyz
Motion in a Plane 155
Example 6. An aeroplane has to go from a point A to another
( v x - 2u ) point B, 500 km away due 30°east of north. A wind is blowing due
=
-v y north at a speed of 20 m/s. The air -speed of the plane is 150 m/s.
(a) Find the direction in which the pilot should head the plane
u - 2u to reach the point B
=
-v y (b) Find the time taken by plane to go from A to B.
Sol. The given points are shown in the fig. 4.10. The motion of aeroplane
is along the resultant of air- speed of aeroplane and wind velocity. Let
aeroplane should head at an angle q with the line joining A and B, and it
takes time t to reach the point B.
AC = speed of aeroplane × time of motion
= 150 t m
and CB = speed of wind × time
Figure. 4.14 = 20 t m
which gives (a) By sine formula, we have
u
vy =
tan q
Thus the velocity of rain
r
v R = v x iˆ - v y ˆi

u ˆ
= u ˆi - j. Ans.
tanq
Example 5. In a harbour, wind is blowing at the speed of 72 km
/ h and the flag on the mast of a boat anchored in the harbour
flutters along the N- E direction. If the boat starts moving at a
speed of 51 km /h to the north, what is the direction of the flag on
the mast of the boat? [NCERT]
Sol. The speed of the wind is 72 km /h and its direction is along the Figure. 4.16
direction in which flag flutters, i.e., along N -E. When boat starts moving
AC CB
the flag flutters in the relative direction of motion of wind with respect =
to boat. We have, velocity of wind sin 30° sin q
r 150t 20t
v = (72sin 45°ˆi + 72 cos45°ˆj ) km /h or =
r sin 30° sin q
and velocity of boat v b = 51ˆj km/h.
1
The velocity of wind with respect to boat \ sin q =
r r r 15
v wb = v w - vb
-1 æ 1ö
or q = sin çè ÷ø east of the line AB.
15
(b) Again by sine formula we have,
AC AB
=
sin 30° sin[180° - (30° + q)]

ìsin(30° + q) = sin 30° cos q + cos 30° sin q


ï
1 3 1
ïï = ´ 0.99 + ´
í 2 2 15
ï = 0.495 + 0.058
ï
ïî = 0.5527

Figure. 4.15
150t 500 ´ 103
= (72sin 45° ˆi + 72 cos 45° ˆj) - (51 ˆj) km/h or =
sin(30° + q)
sin 30°
= (72sin 45°) ˆi +(72 cos 45° - 51) ˆj km/h.
r 150t 500 ´ 103
Let v wb makes an angle q with east then or
( )
1/ 2 =
0.5527
72 cos 45° - 51 0.081
tan q = = = -0.0016
72 sin 45° 50.92 500 ´ 103
which gives q ; – 0.01° \ t =
0.5527 ´ 2 ´ 150
Hence the flag will flutter almost along east direction = 3015.5 s = 50.3 min Ans.
www.crackjee.xyz
156 MECHANICS
Example 7. A river is flowing due east with a speed 3 m/s. A In case (b),
swimmer can swim in still water at a speed of 4 m/s shown in given Time taken by the swimmer to cross the river
figure. [NCERT Exemplar] b
t2 =
N 7
E Clearly, t1 < t2.
B Hence, the swimmer will cross the river in shorter time in case (a).
Example 8. Two swimmers leave point A on one bank of the
3 m/s river to reach point B lying right across on the other bank. One of
them crosses the river along the straight line AB while the other
swims at right angles to the stream and then walks the distance
A that he has been carried away by the stream to get the point B.
Figure. 4.17 What was the velocity u of his walking if both swimmers reached
(a) If swimmer starts swimming due north, what will be his the destination simultaneously? The stream velocity v0 = 2.0 km/h
resultant velocity (magnitude and direction)? and the velocity v' of each swimmer with respect to water equal
2.5 km/h.
(b) If he wants to start from point A on south bank and reach
Sol.
opposite point B on north bank, then
Suppose width of the river is b. The time taken by swimmer to cross the
(i) which direction should he swim? river along the line AB
(ii) what will be his resultant speed?
b
(c) From two different cases as mentioned in (a) and (b) above, t1 = …(i)
in which case will he reach opposite bank in shorter time? v ' - v02
2

Sol. Speed of the river (vr) = 3 m/s (east)


Speed of swimmer (vs) = 4 m/s (north)
(a) When swimmer starts swimming due north then his resultant
velocity
B vr
v = vr2 + vs2

= (3)2 + (4) 2 vs v
= 25 = 5 m/s Figure. 4.20
A For the other swimmer, which swims at right angle to the stream, the
vr 3
tan q = = = 0.75 Figure. 4.17 time to cross the river
vs 4
b
or q = 36°54¢ t =
v'
(b) To reach opposite points B, the swimmer should swim at an angle
The second swimmer reaches at point C.
q of north. Then he walks the distance CB with velocity u. The time taken to travel
vr B the distance
BC
t =
u
vs v
q where BC = river velocity × time to cross the river
b
A = v0 ×
Figure. 4.19 v'
Resultant speed of the swimmer Thus, total time of the second swimmer takes to reach the point B on the
opposite bank
v = vs2 - vr2
b (v0b / v ')
+ = …(ii)
v' u
= (4) 2 - (3) 2
These two times are equal therefore from equations (i) and (ii), we have
= 16 - 9 = 7 m/s b b v0b
= +
vr 3 2
v ' - v0 2 v' v 'u
tan q = =
v 7
1 1 v
-1 æ 3 ö or = + 0
or q = tan çè ÷ of north v ' - v02
2 v' v 'u

(c) In case (a), Substituting the values of v0 and v', we have
Time taken by the swimmer to cross the river 1 1 2
= +
b b 2
2.5 - 2.0 2 2.5 2.5u
t2 = = s
vs 4 After solving, we get u = 3 km/h. Ans.
www.crackjee.xyz
Motion in a Plane 157

In Chapter Exercise 4.1

æ1ö
1. The position of a particle is given by Ans. At an angle tan -1 ç ÷ with the vertical towards the east.
è 2ø
r
r = 3.0t ˆi + 2.0 t 2 ˆj + 5.0 kˆ , where t is in seconds and the
4. A man in a row boat must get from point A to point B on the
r opposite bank of the river (see figure). The distance BC = a. The
coefficients have the proper units for r to be in metre.
(a) Find v(t) and a(t) of the particle. width of the river AC = b. At what minimum speed u relative to
(b) Find the magnitude and direction of v(t) at t = 3.0 s. the still water should the boat travel to reach the point B? The
r r velocity of flow of the river is v0.
Ans. (a) v(t) = 3.0 ˆi + 4.0t ˆj, a(t)= 4.0 ˆj
C a B
(b) 12.4 m/s, 76° with x-axis.
2. A train is moving with a velocity of 30 km/h due east and a car is
b v0
moving with a velocity of 40 km/h due north. What is the velocity
of car as appears to a passenger in the train ?
A
Ans. 50 km/h, 36° 52' west of north.
v0 b
3. Rain is falling vertically with a speed of 24 m/s. A woman rides a Ans.
bicycle with a speed of 12 m/s in east to west direction. What is a 2 + b2
the direction in which she should hold her umbrella ?

4.7 PROJECTILE MOTION


When a particle is projected obliquely near the earth surface, it moves simultaneously in horizontal and
vertical directions. Motion of such a particle is called projectile motion. Since there is no force acting
in horizontal direction, the velocity remains constant in this direction. In vertical direction gravitational
pull of earth produces the acceleration.
Assumptions used in projectile motion
(i) Neglecting the effect of air resistance on the projectile.
(ii) Assuming the acceleration due to gravity is constant at each point of projectile.
(iii) Neglecting the effect of curvature of earth.
We will discuss the following types of projections in details :

Type 1 : Projectile fired at some Type 2 : Horizontal projection Type 3 : Projectile fired from
angle with the horizontal. some height

Type 4 : Projection on an inclined plane Type 5 :Projection down the inclined plane
Figure. 4.21
www.crackjee.xyz
158 MECHANICS
ANALYSIS OF PROJECTILE OF TYPE 1
Let us consider a particle is projected with initial velocity u at an angle q with the horizontal (called
angle of projection). The velocity u has two rectangular components:
(i) The horizontal component ucosq, which remains constant throughout the motion of particle.
(ii) The vertical component usinq, which changes with time due to effect of gravity. Thus we have
initial velocity
ur
u = u x ˆi + u y ˆj
ur
or u = ucosq ˆi + usinq ˆj

Figure. 4.22
Velocity at any time t : Using first equation of motion in vertical direction, we have
vy = uy – gt
= u sinq – gt
ur
\ Velocity at any time t, v = v ˆi + v ˆj
x y
ur
or v = ucosqˆi + (usinq - gt )jˆ.
Velocity at any height : At any height h
vx = ux = u cosq
Figure. 4.23 …(i)
and vy2 = uy2 – 2gh
= (u sinq)2 – 2gh …(ii)
Squaring (i) and adding with equation (ii), we get
v = u 2 - 2 gh .
Position at any time t
Position of particle at any time t, is given by
ur
r = xˆi + yˆj
where
x = u cosq t …(1)
1 2
and y = u sinq t – gt
2
r 1 2
\ r = u cos q t ˆi + (u sin q t - gt )ˆj
2
2
æ 1 ö
or r = (u cos q t )2 + ç u sin q t - gt 2 ÷
Figure. 4.24
è 2 ø

2
æ gt ö gt sin q
= ut 1+ ç ÷ -
è 2u ø u
www.crackjee.xyz
Motion in a Plane 159
y
and tanf =
x

é 1 2ù
ê u sin qt - 2 gt ú
or f = tan–1 ê ut cos q
ú
ê ú
ë û

é 2u sin q - gt ù
= tan–1 ê .
ë 2u cos q úû
The angle of elevation f of the highest point of the projectile;
u sin q
t =
g
u sin q
2u sin q - g ´
\ tanf = g
2u cos q
tan q
or tanf =
2
Equation of trajectory
We have, x = u cosq t
æ x ö
or t = çè ÷
u cos q ø
1 2
and y = u sinq t – gt
2
2
æ x ö 1 æ x ö
= u sinq çè ÷ – g
u cos q ø 2 çè u cos q ÷ø

gx 2
y = x tan q - .
2u 2 cos2 q
On comparing this equation with general equation of parabola, y = ax ± bx2, we find that path of
projectile is parabolic in nature.

Time of flight (T) : Total time of motion of particle in air is called time of flight.
The displacement in vertical direction (y-axis) becomes zero in whole time of motion. So we have
1 2
y = uy t – at
2 y
1 2
or 0 = uy T – gT
2

2u y 2u sin q
which gives T = = .
g g

Maximum height attained (H): The maximum vertical distance achieved by particle is Figure. 4.25
called maximum height.
At the highest point of projection vy = 0, so we have,
vy2 = uy2 – 2gh
or 0 = uy2 – 2gH
www.crackjee.xyz
160 MECHANICS

u y2 u 2 sin 2 q
which gives H = = .
2g 2g
Horizontal range (R) : The horizontal distance moved by particle in total time of flight is
called horizontal range.
2u y
Horizontal range, R = ux × T = ux ×
g
2u x u y 2u cos q u sin q
= =
g g

u 2 sin 2q
or R = .
g
For maximum range, sin2q = 1 or 2q = 90° or q = 45°.
u2
Thus Rmax = .
g

u2 sin2 45° u 2
Corresponding, H = = .
2g 4g

There are two angles of projection for same range:


Replacing q by (90° – q) in the formula of range, we get

u2 sin 2(90° -q)


R' =
g

u 2 sin(180° - 2q)
=
g

u 2 sin 2q
= = R.
g
Thus, for a given velocity of projection, a projectile has the same range for angle of projection q and
(90° – q).
Time of flight for angle of projection q,
2u sin q
T1 =
g
and time of flight for angle of projection (90° – q),
2u sin(90° - q)
T2 =
g

Figure. 4.26 2u co s q
= .
g
Multiplying T1 and T2, we get
2u sin q 2ucosq
\ T1T2 =
g
×
g

2 æ u 2 sin 2 q ö
or T1T2 = ç ÷
gè g ø

2R
or T1T2 = .
g
www.crackjee.xyz
Motion in a Plane 161
More about projectile motion
1. If t1 is the time taken by projectile to reach a point P at height h and t2 is the time taken from point
P to ground level, then

2u sin q
t1 + t2 = T =
g

g (t1 + t 2 )
or u sinq = .
2
The height of point P,
Figure. 4.27
1
h = u sinq t1 – gt12
2

g (t1 + t 2 ) 1
= t1 – gt 2
2 2 1

1
or h = g t1 t2.
2
2. Change in momentum : Change in momentum between two positions of projectile is given by
uur uur uur ur ur
ΔP = P f - Pi = m(v f - v i ) .
(a) Between point of projection and highest point
ur
v i = u cosq î + u sinq ĵ
ur
and vf = u cosq î
uur
\ ΔP = m [(u cosq î ) – (u cosq î + u sinq ĵ )]

or DP = mu sin q
(b) For the complete projectile motion
r
vi = u cosq î + u sinq ĵ
r
vf = u cosq î – u sinq ĵ
uur
\ ΔP = m [(u cosq î – u sinq ĵ ) – (u cosq î + u sinq ĵ )]

or DP = 2 mu sin q

ANALYSIS OF PROJECTILE OF TYPE 2 : HORIZONTAL PROJECTION


Let a particle be projected horizontally with initial velocity u from height h.
Velocity at any time t
We have, vx = u
and vy = uy + gt
or = 0 + gt
r
\ v = u ˆi - gt ˆj

and v = u 2 + ( g t )2

gt
Also tana = . Figure. 4.28
u
www.crackjee.xyz
162 MECHANICS
Position at any time t
Taking point of projection as the origin, the position vector at any time t
ur
r = x î – y ĵ .
where x = ut
1 2
and y = gt
2

r 1 2ˆ
\ r = utˆi - gt j
2

2
æ1 ö
Displacement s = r= (u t ) 2 + ç g t 2 ÷
è2 ø
Equation of trajectory
x
We have, x = ut or t =
u
1 2
and y = – gt
2
2
1 æ xö
= – gç ÷
2 è uø

1 x2
or y = - g 2
2 u
Time of flight (T)
1
We have, h = uyt + a t2
2 y
1
or h = 0+ g T2
2
2h
which gives T = .
g
Horizontal range (R)
R = ux × T

= u 2h .
g
The average acceleration in total time of flight is g downward.

ANALYSIS OF PROJECTILE OF TYPE 3


Method - I :
Let us consider a particle is projected with initial velocity 100 m/s at an angle 30° with
the horizontal. The height of projection is 100 m.
Time of flight (T)
1
We have y = uy t + g t2
2
1
or 100 = – 100 sin 30° T + × 10 × T2
2
or T2 – 10 T – 20 = 0
Figure. 4.29
www.crackjee.xyz
Motion in a Plane 163

10 ± (-10)2 - 4 ´1´ (-20)


which gives T =
2
= 11.71 s
(consider only positive value)
The horizontal range (R)
R = ux × T = 100 cos 30° × 11.71

3
= 100 × × 11.71
2
= 1014 m.
Method - II :
Taking point of projection as the origin, the coordinates of point of strike are (R, – 100 m).
We have,

gx 2
y = x tanq –
2u 2 cos 2 q
Here y = – 100 m and x=R

10 R 2
\ – 100 = R tan 30° –
2(100)2 cos 2 30°
Figure. 4.30
or R2 – 866 R – 150000 = 0

866 ± (866)2 + 4 ´150000


which gives, R =
2
= 1014 m (consider only positive value)
R
Time of flight, T = = 11.71 s.
u cos q

FORMULAE USED
y
Projectile Type 1

1. ux = u cos q, uy = u sin q u
H
ax = 0, ay = –g . q x
O R
2. Position after time t
1 2
x = u cos q t, y = u sin q t – gt
2
3. Equation of trajectory

gx 2
y = x tan q – .
2u 2 cos 2 q
2u sin q
4. Time of flight, T = .
g

u 2 sin 2 q
5. Maximum height, H = .
2g
www.crackjee.xyz
164 MECHANICS

u 2 sin 2q
6. Horizontal range, R =
g

u2
7. Maximum range, Rmax = , for q = 45°
g

Projectile Type 2 u

1. Position after time t


1 2 h
x = ut, y = gt
2

gx 2 x
2. Equation of trajectory, y = . R
2u 2

3. Velocity after time t, v = u 2 + (gt)2

2h
4. Time to hit the ground, T = g

2h
5. Horizontal range, R = uT = u g
.

EXAMPLES BASED ON PROJECTILE TYPE 1, TYPE 2 AND TYPE 3


Example 9. A bullet fired at an angle of 30° with the horizontal Example 10. A fighter plane flying horizontally at an altitude
hits the ground 3.0 km away. By adjusting its angle of projection, of 1.5 km with speed 720 km/h passes directly overhead an anti-
can one hope to hit a target 5.0 km away? Assume the muzzle aircraft gun. At what angle from the vertical should the gun be
speed to be fixed, and neglect air resistance. [NCERT] fired for the shell with muzzle speed 600 ms–1 to hit the plane? At
what minimum altitude should the pilot fly the plane to avoid
Sol. Here R = 3 km = 3000 m, q = 30°, g = 9.8 m s–2
being hit? (Take g = 10 ms–2) [NCERT]
Sol. Velocity of plane,
u 2 sin 2q
As R= 5
g v = 720 ´ ms–l = 200 ms –l
18
u 2 sin 2 ´ 30° u 2 sin 60 Velocity of shell = 600 ms–l ;
Þ 3000 = =
9.8 9.8 200 1
sin q = =
600 3
3000 ´ 9.8
Þ u2 = = 3464 × 9.8 æ 1ö
3 or q = sin–1 çè ÷ø = 19.47°
3
2

u 2 sin 2q¢
Also, R¢ =
g

3464 ´ 9.8 ´ sin 2q


Þ 5000 =
9.8

5000
i.e. sin 2q¢ = = 1.44
3464
which is impossible because sine of an angle cannot be more than
1. Thus this target cannot be hoped to be hit. Figure. 4.31
Let h be the required minimum height. Using equation
www.crackjee.xyz
Motion in a Plane 165
v2 – u2 = 2a s, we get From equation (ii), we have
(0)2 – (600 cos q)2 = – 2 × 10 × h R = 78.4

600 ´ 600(1 - sin 2 q) u 2 sin ( 2 ´ 45°)


or, h = =
20 9.8
On solving, we get u = 27.72 m/s. Ans.
æ 1ö 8 Example 13. If R is the horizontal range for q inclination and
= 30 ´ 600 ç1 - ÷ = ´ 30 ´ 600 m = 16 km.
è 9ø 9 h is the maximum height reached by the projectile, show that the
Example 11. The ceiling of a long hall is 25 m high. What is
R2
the maximum horizontal distance that a ball thrown with a speed maximum range in given by + 2h.
8h
of 40 m/s can go without hitting the ceiling of the hall ?
Sol. Sol. We know that horizontal range,
Given H = 25 m, u = 40 m/s. u 2 sin 2q
R =
If the ball is thrown at an angle q with the horizontal, then maximum g
height of flight is given by
u 2sin 2θ
2
u sin q 2 and maximum height, h =
2g
H =
2g
2
é u 2sin2θ ù
2
40 sin q 2 ê ú é u 2sin 2θ ù
or 25 = R2 ëê g ûú ê ú
2 ´ 9.8 \ + 2h = +2
8h é u 2sin 2θ ù êë 2g úû
which on solving gives 8ê ú
sinq = 0.554 and êë 2g úû
cosq = 0.833.
The maximum horizontal distance is given by u 4 (2sin θ cosθ)2 u 2sin 2θ
= +
u 2sin 2θ g
u 2 sin 2q 2u 2 sin q cos q g2 ´8
R = = 2g
g g

2 ´ 402 ´ 0.554 ´ 0.833 u2


= = (cos2q + sin2q)
9.8 g
= 150.7 m Ans. u2
Example 12. A boy stands at 39.2 m from a building and throws =
g
= Rmax.
a ball which just passes through a window 19.6 m above the ground.
Calculate the velocity of projection of the ball. Example 14. A hunter aims his gun and fires a bullet directly
Sol.Given H = 19.6 m at a monkey on a tree. At the instant the bullet leaves the barrel of
and R = 39.2 + 39.2 = 78.4 m the gun, the monkey drops. Will the bullet hit the monkey ?

Sol. Suppose the gun situated at O directed towards the monkey at


position M. Let bullet leaves the barrel of the gun with velocity u at an
angle q with the horizontal. Let bullet crosses the vertical line MB at A
after time t. Horizontal distance travelled
OB = x = u cosq t

x
or t =
u cosθ
Figure. 4.32

u 2 sin 2 q
We know that H = ...(i)
2g

2u 2 sin q cos q
and R = ...(ii)
g
H tan q
\ R
=
4
4H 4 ´ 19.6
or tanq = = =1
R 78.4
or q = 45° Figure. 4.33
www.crackjee.xyz
166 MECHANICS
For motion of bullet from O to B, the vertical x = ux ´ t
1 2 = 75 ´ 10 = 750 m
height AB = u sinq t – gt \ Distance through which canon has to be moved
2
= 800 – 750 = 50 m
æ x ö 1 2
= u sinq çè ÷– gt Speed with which canon can move = 2 m/s
u cos θ ø 2
50
\ Time taken by canon =
g t2 2
= x tanq – …(i)
2 t = 25 s
Also MB = x tan q \ Total time takne by a packet to reach on the ground
Now y = MA = MB – AB = t ¢¢ + t + t ¢
= 25 + 10 + 10 = 45 s
æ g t2 ö
= x tanq – ç x tan q - 2 ÷ Example 16. A gun can fire shells with maximum speed v0 and
è ø the maximum horizontal range that can be acheived is

1 2 v 02
= gt R = .
2 g
If a target farther away by distance Dx (beyond R) has to be hit with
1
Thus, in time t the bullet passes through A a vertical distance g t2 the same gun as shown in the figure. Show that it could be achieved
2
by raising the gun to a height at least
below M.
The vertical distance through which the monkey fall in time t, æ Dx ö
h = Dx çè 1 + ÷. [NCERT Exemplar]

1
s = g t2.
2
v0
Thus, the bullet and the monkey will always reach at point A at the same
time. q P
Example 15. A hill is 500 m high. Supplies are to be sent across q
the hill using a canon that can hurl packets at a speed of 125 m/s
v0
over the hill. The canon is located at a distance of 800 m from the
foot of hill and can be moved on the ground at a speed of 2 m/s, so h
that its distance from the hill can be adjusted. What is the shortest
time in which a packet can reach on the ground across the hill?
Take g = 10 m/s2. [NCERT Exemplar]
R Dx
Sol. Given, height of the hill (h) = 500 m
u = 125 m/s
Figure. 4.34
To croos the hill, the vertical component of the velocity should be
Sol. Maximum horizontal range
sufficient to cross such height.
\ uy ³ 2gh v02
R= …(i)
³ g
2 ´ 10 ´ 500
³ 100 m/s Let the gun be raised through a height h from the ground so that it
But u2 = u x2 + u 2y can hit the target.
Horizontal component of initial velocity = v0 cos q
\ Horizontal component of initial velcoity
Vertical component of initial velocity = – v0 sin q
ux = u 2 - u 2y Taking motion in vertical direction,

1 2
= (125)2 - (100) 2 = 75 m/s h = (–v0 sin q)t + gt …(ii)
2
Time taken to reach the top of the hill Taking motion in horizontal direction
2h 2 ´ 500 (R + Dx) = v0 cos q ´ t
t = = = 10s
g 10 ( R + D x)
Time taken to reach the ground from the top of the hill t ¢ = t = 10 s or t = v0 cos q …(iii)
Horizontal distance travelled in 10 s
Substituting value of t in Eq. (ii), we get
www.crackjee.xyz
Motion in a Plane 167
2 2
æ R + D xö 1 æ R + D xö 1 æ nwö
h = (–v0 sin q) ´ ç v cos q ÷ + gç or nh = g ç ÷
è 0 ø 2 è v0 cos q ÷ø 2 è u ø

1 n2 w2
1 ( R + D x)
2
= g
h = –(R + Dx) tan q + g 2 u2
2 v02 cos 2 q
As angle of projection is q = 45° therefore, 2hu 2
or n = . Ans.
gw 2
1 ( R + D x)2
h = –(R + Dx) ´ tan 45° + g Example 18. A shot is fired with a velocity u at a vertical wall
2 v02 cos 2 45°
whose distance from the point of projection is x. Prove the greatest
height above the level of the point of projection at which the bullet
1 ( R + D x)
2
h = –(R + Dx) ´ 1 + g 2
2 v0 (1 / 2) u 4 - g 2 x2
can hit the wall is .
2 gu 2
h = –(R + Dx) +
(R+ D x) 2
[using Eq. (i), R = v20/g] Sol. Let q be the angle of projection. Suppose y is the height at which
R bullet hit the wall, then
1 2 gx 2
= –(R + Dx) + (R + Dx2 + 2RDx) y = x tanq –
R
2u 2 cos 2 q
æ Dx 2 ö
gx 2 sec 2 q
= R – Dx + ç R + R + 2 Dx÷ = x tanq – …(i)
è ø 2u 2

Dx 2
= Dx +
R

æ Dx ö
or h = Dx ç 1 + ÷ .
è Rø
Example 17. A ball rolls off the top of a stairway with a constant
horizontal velocity u. If the steps are h metre high and w metre
wide, show that the ball will just hit the edge of n th step if

2hu 2
n= .
gw 2

Sol.
For nth step, Figure. 4.36
Horizontal displacement ; x = nw and vertical displacement, y = nh if t is
the time of motion then dy gx 2
\ = x sec2q – 2 secq (secq tanq)
x = ut …(i) dq 2u 2
or nw = ut
gx 2
nw = x sec2q – sec2q tanq
\ t = u2
u
é xg tan q ù
= x sec2q ê1 - ú
ë u2 û
For y to be maximum,
dy
= 0
dq
é xg tan q ù
\ x sec2q ê1 - ú =0
ë u2 û

é xg tan q ù
Figure. 4.35 or ê1 - ú = 0
ë u2 û
and vertical displacement
1 2 u2
y = 0+ gt or tanq =
2 gx
www.crackjee.xyz
168 MECHANICS
Substituting this value in equation (i), we get Solving above quadratic equation for t, we have

u2 1 gx 2 é u4 ù g
ymax = x – ê1 + ú u sin q ± u 2 sin 2 q - 4 ´ ´h
gx 2 u 2 ëê x 2 g 2 ûú 2
t =
g

2 2
u gx 2 u2 2u 4 - g 2 x 2 - u 4
= – – =
g 2u 2 2g 2u 2 g
u sin q ± u 2 sin 2 q - 2 gh
or t =
u 4 - g 2 x2 g
or ymax = 2
. Proved
2 gu (u2
t to be real sin2q– 2gh) ³ 0
or u2 sin2q ³ 2gh
Example 19. An aeroplane flies horizontally at a height h at a
or u2 (1 – cos2q) ³ 2gh … (ii)
speed v. An anti-air craft gun fires a shell at the plane when it is
From equation (i),
vertically above the gun. Show that the minimum muzzle velocity
v = u cosq,
2 æ 2 gh ö
required to hit the plane is v + 2 gh at an angle tan–1 ç
ç v ÷÷
. v
è ø \ cosq = ...(iii)
Sol. u
Suppose the muzzle velocity of the shell is u and it is fired at an angle q Now from equation (ii), we have
with the horizontal. To hit the plane, the displacement of bullet along the
motion of plane in time t is equal to the displacement of the plane. Thus æ v2 ö
\ u2 ç1 - 2 ÷ ³2gh
we have ç u ÷
è ø
u cosq t = vt Þ v = u cosq ....(i)
or u2 – v2 ³ 2gh
1 2
and h = u sinq t – gt v 2 + 2 gh
2 or umin =
Substituting this value in equation (ii), we get
g t2
or – u sinq t + h = 0 v v
2 cosq = =
umin 2
v + 2 gh

Figure. 4.38
Figure. 4.37
2gh
and tanq = .
v

In Chapter Exercise 4.2

1. A cricketer can throw a ball to maximum horizontal distance of constants. Find the velocity of the particle at the origin of
160 m. Calculate the maximum vertical height to which he can
a
throw the ball ? Given g = 10 m/s2. [Integer] Ans. 80 m. coordinates. Ans. v0 = ( 1 + b2 )
2c
2. A person observes a birds on a tree 39.6 m high and at a distance
of 35.2 m. With what velocity the person should throw an arrow 4. A cannon fires successively two shells with velocity
at an angle of 45° so that it may hit the bird ? Ans. 41.86 m/s. v0 = 250 m/s, the first at an angle q1 = 60° and the second at an
ur angle q2 = 45° to the horizontal, the azimuth being the same.
3. A particle moves in the x y-plane with constant acceleration a Neglecting the air drag, find the time interval between firings
directed along the negative y-axis. The equation of path of the leading to the collision of the shells.
particle has the form y = bx – cx2, where b and c are positive [Integer] Ans. 11 s
www.crackjee.xyz
Motion in a Plane 169
TOPICS FOR JEE-(MAIN & ADVANCED)
4.8 PROJECTION UP ON AN INCLINED PLANE
Let us consider a particle is projected with velocity u at an angle q with the horizontal on an inclined
plane of inclination a. In this case take x and y-axes along inclined plane and perpendicular to it.
We have, ux = u cos (q - a), ax = – g sina
and uy = u sin (q - a), ay = – g cosa.
Time of flight (T)
The displacement along y-direction becomes zero in total time of flight T. Thus we have,
1
y = uyT + a T2
2 y
1
or 0 = u sin (q - a) – (g cosa) T2
2
2u sin(q - a)
which gives, T = 0 and T=
g cos a
T = 0 corresponds to O. Therefore time of flight
2u sin(q - a)
T = . …(i)
g cos a Figure. 4.39
Range along inclined plane (R)
Using second equation of motion along x-axis, we have
1
R = ux T + a T2
2 x

MISCELLANEOUS TOPICS FOR IIT-JEE


2
é 2u sin(q - a ) ù 1 é 2u sin(q - a ) ù
= u cos (q - a) × ê ú – (gsina) ê ú
ë g cos a û 2 ë g cos a û
After simplifying, we get

u2
R = [sin(2q - a ) - sin a ] …(ii)
g cos2 a
For maximum range, sin (2q - a) = 1
or 2q - a = 90°
a
or q = 45° +
2

u2 u 2 (1 - sin a )
\ Rmax = (1 – sina) =
g cos 2 a g (1 - sin 2 a )

u2
or Rmax = .
g (1 + sin a )
Time taken by projectile to become vy = 0 :
Using first equation of motion, we have
vy = uy + ay t
or 0 = u sin (q - a) – g cosa tA

u sin(q - a) T
or tA = = .
g cos a 2
www.crackjee.xyz
170 MECHANICS
Let it happens at a distance x from O along the inclined plane, then
1
x = ux tA + a t 2
2 x A
2
æTö 1 æTö R
= u cos (q - a) ç ÷ – (g sina) çè ÷ø > 2 .
è 2ø 2 2
Height of A from inclined plane :
By third equation of motion, we have
0 = uy2 + 2 ay y
or 0 = [u sin (q - a)]2 – 2 (g cosa) y

u 2 sin 2 (q - a )
or y = .
2 g cos a

4.9 PROJECTION DOWN THE INCLINED PLANE

(a) (b)
Figure. 4.40
Here we have,
ux = u cos(q + a), ax = g sina
uy = u sin(q + a), ay = –g cosa.
Time of flight
As displacement becomes zero along y-direction in time T,
1
\ 0 = uy T + a T2
2 y
1
or 0 = u sin(q + a) T – (g cosa) T2
2
which gives T = 0
2u sin(q - a)
or T =
g cos a
T = 0 corresponds to O, therefore time of flight
2u sin(q + a )
T = g cos a …(i)

Range along inclined plane (R) :


1
R = ux T + a T2
2 x
2
é 2u sin(q + a ) ù 1 é 2u sin(q + a ) ù
= u cos(q + a) ê ú + g sina ê g cos a ú
ë g cos a û 2 ë û
www.crackjee.xyz
Motion in a Plane 171
After simplifying, we get

u2
R = [sin(2q + a ) + sin a ] …(ii)
g cos2 a
For maximum range, sin (2q + a) = + 1
a
or (2q + a) = 90°or q = 45° –
2

u 2 (1 + sin a ) u 2 (1 + sin a)
\ Rmax = =
g cos 2 a g (1 - sin 2 a)

u2
or Rmax = .
g (1 - sin a )

4.10 MOTION ALONG A CURVED PATH


For a paticle moving along a curved path, the velocity changes due to change in the direciton of motion

v2
of particle, due to which there is an acceleration called normal acceleration. Its magnitude is , where
R
v is the instantaneous speed. R is the radius of curvature at the point under consideration. If particle is
dv
moving with variable speed, it also has tangential acceleration. Its magnitude is . Thus total
dt
acceleration at any point

a = an 2 + at 2

v2 dv
where an = and at = .
R dt
Finding radius of curvature
The r adius of curvature at any point of a cur ve can be obtained by following two Figure. 4.41
ways :
(i) If equation of curve is known : The radius of curvature in such cases can be calculated as :

d2y
1 dx 2
= 3/ 2
,
R é æ dy ö 2 ù
ê1 + ç ÷ ú
è dx ø ú
ëê û

dy
where is the slope of curve at the point under consideration.
dx
(ii) If normal acceleration is known : We know that;

v2
an =
R

v2
\ R =
an Figure. 4.42
By substituting the value of an, we can get radius of curvature at particular point.
www.crackjee.xyz
172 MECHANICS
Radius of curvature of projectile
(i) At point of projection :
From the figure it is clear that
a n = g cosq.
v2
As we know, an =
R
v2 u2
\ R = = .
an g cosq
Figure. 4.43
(ii)At the highest point of projectile :

v = u cosq
v 2 (u cos q ) 2
\ R = =
an g

u 2 cos2 q
or R = .
Figure. 4.44 g

MISCELLANEOUS EXAMPLES FOR JEE-MAIN AND ADVANCED


Example 1. A particle is projected over a triangle from one end
of a horizontal base and grazing the vertex falls on the other end of yR
or tanq = …(i)
the base. If a and b be the base angles and q the angle of projection, x( R - x )
prove that tanq = tana + tanb. From figure;
Sol. Given data are shown in the fig. 4.45. For any point P (x, y), we
have y y
tana + tanb = +
x R- x
g x2
y = x tanq –
2u 2 cos 2 q y ( R - x ) + xy
=
x( R - x )

yR
= …(ii)
x( R - x )

Now from equations (i) and (ii), we get


tanq = tana + tanb Proved
Example 2. A particle is projected horizontally with a speed u
from the top of a plane inclined at an angle q with the horizontal.
How far from the point of projection will the particle strike the
Figure. 4.45 plane?
é gx ù Sol. Consider the motion of the particle along the direction (x-axis) and
= x tanq ê1 - 2 2 ú
ë 2u cos q tan q û perpendicular direction of OB (y-axis). The initial velocities and
accelerations along these directions are shown in the figure. The
é gx ù displacement along y-axis in time T becomes zero.
= x tanq ê1 - 2 ú
ë u (2sin q cos q) û By using second equation of motion along y-axis, we have

é xù 1
= x tanq ê1 - ú y = uy t – a t2
ë Rû 2 y

éR - xù 1
= x tanq ê or 0 = u sinq T – (g cosq) T2
ë R ûú 2
www.crackjee.xyz
Motion in a Plane 173

Figure. 4.47
or 3.0 × 4.0 – g2 t2 = 0
Figure. 4.46
2u sin q 12
which gives, T = or t = .
g cos q g

2u tan q Both the particles have zero initial velocity in vertical direction. Therefore
= they fall equal vertical distances. They lie on same horizontal line.
g
In this duration the displacement along x-axis Therefore we have
1 x1 = 3.0 t and x2 = 4.0 t
R = uxT + a T2 \ x = x1 + x2 = 3.0 t + 4.0 t
2 x
2u tan q 1 7.0 12.0
= u cosq + (g sinq) = 7.0 t = ; 2.5 m. Ans.
g 2 g
2
æ 2u tan q ö Example 4. A ball starts falling with zero initial velocity on a
çè g ÷ø smooth inclined plane forming an angle q with the horizontal.
Having fallen the distance h, the ball rebounds elastically off the
2
= 2u tan q [cosq + 2 tanq sinq] inclined plane. At what distance from impact point will be ball
g rebound for the second time ?

2u 2 tan q é sin 2 q ù Sol.


= ê cos q + ú
g êë cos q úû The velocity of the ball just before hitting the plane is u = 2gh . Since
collision is elastic, so the ball will rebound with the same speed. The
2u 2 tan q é cos2 q + sin 2 q ù velocity component along the plane, ux = u sina and perpendicular to it
= ê ú
g êë cos q úû vy= u cosa. Using second equation of motion along y-axis

2u 2 tan q sec q 1
y = uy t – a t2.
= . Ans. 2 y
g
Let T is the time of flight then in total time T, y becomes zero.
Example 3. Two particles move in a uniform gravitational field
with an acceleration g. At the initial moment the particles were 1
\ 0 = u cosa T – (g cosa) T2
located at one point and moved with velocities u1 = 3.0 m/s and u2 = 2
4.0 m/s horizontally in opposite directions. Find the distance
between the particles at the moment when their velocity vectors
become mutually perpendicular.
Sol.
Supposing point of projection as the origin, the velocities of particles at
time t after the projection
ur
v1 = 3.0 î – g t ĵ ....(i)
ur
and v2 = – 4.0 î – g t ĵ ....(ii)
ur ur ur ur
As v1 and v2 are mutually perpendicular, so v1 . v = 0
2

or (3.0 î – g t ĵ ) . (– 4.0 î – g t ĵ ) = 0
Figure. 4.48
www.crackjee.xyz
174 MECHANICS

2u and d – x = 10 cos 60° × t


Þ T = = 5t … (ii)
g
Now, along the plane Adding equations (i) and (ii), we have
d = 15 t … (iii)
1
AB = ux T + a T2 Let y is the vertical displacement of point of collision from A, then
2 x
2 1 2
æ 2u ö 1 æ 2u ö y = 0+ gt …(iv)
= u sina ç ÷ + (g sina) ç ÷ 2
è g ø 2 è g ø
1
and (10 + y) = 10 sin 60° t + g t2 …(v)
2
4u sin a 2
= Subtracting (iv) from (v), we get
g
10 sin 60° t = 10
We have u = 2gh 2
or t = s
2 3
4( 2 gh ) sin a (ii) The resultant momentum of the objects in horizontal direction
\ AB = = 8 h sina. Ans.
g just before collision,
= m × 10 – 2 m × 10 cos 60° = 0
Example 5. Two towers AB and CD are situated a distance d
\ Velocity of combined object by conservation of momentum,
apart as shown in fig. 4.49. AB is 20 m high and CD is 30 m high
3m × vx = 0 Þ vx = 0
from the ground. An object of mass m is thrown from the top of AB
Thus the combined object falls vertically at a distance x from
horizontally with a velocity of 10 m/s towards CD. Simultaneously
tower AB,
another object of mass 2 m is thrown from the top of CD at an angle
of 60° to the horizontal towards AB with the same magnitude of 2 20
where x = 10 t = 10 × = m Ans.
initial velocity as that of the first object. The two objects move in 3 3
the same vertical plane, collide in mid-air and stick to each other. Example 6. Particles P and Q of mass 20 g and 40 g respectively
(i) Calculate the distance d between the towers. are simultaneously projected from points A and B on the ground.
(ii) Find the position where the objects hit the ground. (g = 9.8 The initial velocities of P and Q make 45° and 135° angles
m/s2) respectively with the horizontal AB as shown in fig. 4.51. Each
particle has an initial speed of 49 m/s. The separation AB is 245 m.
Both particles travel in the same vertical plane and undergo a
collision. After collision P retraces its path. Determine the position
of Q when it hits the ground. How much time after the collision
does the particle Q take to reach the ground? (Take g = 9.8 m/s2)
Sol.
As both the particles have same velocity components in horizontal and
vertical directions, so they travel the equal distances in respective

245
directions. The particle will collide at the middle of AB, i.e., = 122.5
2
Figure. 4.49
m from A towards B.

Figure. 4.51
The height at which they collide is the highest position of their path,
which is
u 2 sin 2 q
H =
2g
Figure. 4.50
Sol. (49) 2 sin 2 45°
=
(i) Let the objects collide after time t. Suppose they collide at a 2 ´ 9.8
horizontal distance x from tower AB, then at time t = 61.25 m
x = 10 t …(i)
www.crackjee.xyz
Motion in a Plane 175
The vertical distance falls by stone in this time t
49
At the highest point each will have velocity vx = 49 cos 45° = m/s 1
2 h–y = 0+ g t2 …(ii)
2
along horizontal direction. Using principle of conservation of momentum
and x = vt …(iii)
along horizontal direction, we have
y
Also = tanq
x
or y = x tan q
= v t tanq …(iv)
Substituting values of t and y in equation (ii), we get
1 2
h – v t tanq = gt
2
Figure. 4.52 2
æv ö 1 æv ö
or h – v ç cot q ÷ tanq = g ç cot q ÷
49 49 èg ø 2 èg ø
20 × 10–3 × – 40 × 10–3 ×
2 2
v2 v2
or h – = cot2q
49 g 2g
= – 20 × 10–3 × + 40 × 10–3 × vQ
2 or 2gh – 2v2 = v2 cot2q
which gives vQ = 0, i.e., after collision the velocity of Q at the highest
2 gh
point becomes zero. So, Q will fall freely under gravity and will hit the or v = . Ans.
2 + cot 2 q
ground at the middle of AB, i.e., 122.5 m from A.
Time taken by Q to reach the ground : Example 8. A large heavy box is sliding without friction down a
smooth plane of inclination q. From a point P on the bottom of the
1 box, a particle is projected inside the box. The initial speed of the
H = 0 + g t2
2 particle with respect to the box is u and the direction of projection
makes an angle a with the bottom as shown in fig. 4.54.
2H
or t =
g

2 ´ 61.25
=
9.8
= 3.54 s. Ans.
Example 7. A stone must be projected horizontally from a point
P, which is h metre above the foot of a plane inclined at an angle q
with horizontal as shown in figure 4.53. Calculate the velocity v of Figure. 4.54
the stone so that it may hit the inclined plane perpendicularly. (a) Find the distance along the bottom of the box between the
point of projection P and the point Q where the particle lands
(assume that the particle does not hit any other surface of
the box. Neglect air resistance).
(b) If the horizontal displacement of the particle as seen by an
observer on the ground is zero, find the speed of the box with
respect to the ground at the instant when the particle was
projected.
Sol.
(a) Consider the motion of the particle along the x and y-axes as
shown in figure. 4.55.
With respect to box, we have
Figure. 4.53
Sol. Take O as the origin and coordinates of point at which stone hits
the plane are (x, y). The velocity component along the plane v cosq and
perpendicular to it is v sinq. As stone hit the plane perpendicularly, its
velocity component v cosq becomes zero. Let t is the time in which it
becomes zero, then
0 = v cosq – g sinq t

v cos q v
which gives, t = = cotq …(i)
g sin q g
Figure. 4.55
www.crackjee.xyz
176 MECHANICS
ur ur ur
u x = [u particle ]x – [u box ]x
= u cosa – 0 = u cosa
ur ur ur
and a x = [a particle ]x – [a box ]x

or ax = g sinq – g sinq = 0
and uy = u sina
ay = – g cosq
(a) Particle will hit the box after time t, then we have
Figure. 4.57
1
y = uy t + a t2 where vbrsin q is the velocity of boat along AB.
2 y The velocity of boat along the direction of flow,
v bx = (vr + vbrcos q)
1
or 0 = u sina – (g cosq) t2 and drift in the direction of flow,
2 x = vbx × t
which gives t = 0
b
= (vr + vbr cosq) ×
2u sin a vbr sin q
or t = Ans.
g cos q
æ ö
Thus distance travelled in time t inside the box æ v ö ç b ÷
PQ = u cosq × t = ç v + cos q÷ ´ ç ÷
è 2 ø v
ç sin q ÷
è2 ø
2u sin a
= u cosa ×
g cos q æ 2 + cos q ö
= bç
è sin q ÷ø
u 2 2 sin a cos a u 2 sin 2a where b is the width of the river. For the drift to be minimum,
= = . Ans.
g cos q g cos q
dx
= 0
(b) Let v is the velocity component of box along horizontal direction. dq
The horizontal displacement as seen by the observer on the ground
d é æ 2 + cos q ö ù
v to be zero, we have or bç ÷ = 0
d q êë è sin q ø úû
v – u cos (q + a) = 0 or v = u cos (q + a).
If velocity of box along the plane is vx, then vx cosq = v or sinq × (–sinq) – (2 + cosq) × (cosq) = 0
or sin2q + 2 cosq + cos2q = 0
or 2 cosq = – (sin2q + cos2q)
or 2cos q = –1
1
\ cosq = –
2
or q = 120°
Hence to minimise drifting boat should move at an angle 120° with the
direction of stream.
æ 2 + cos120° ö
Thus xmin = b çè ÷
sin120° ø
Figure. 4.56
æ 2 -1 / 2ö
\ v u cos(q + a ) = bç
è ÷= 3b . Ans.
vx =
cos q
=
cos q
Ans. 3/2 ø
Example 10. Two boats A and B, move away from a buoy anchored
Example 9. A boat moves relative to water with a velocity which at the middle of a river along the mutually perpendicular straight
is h = 2.0 times less than the river flow velocity. At what angle to lines : the boat A along the river and the boat B across the river.
the stream direction must the boat move to minimise drifting? Having moved off an equal distance from the buoy the boat returned.
tA
Sol. Find the ratio of times of motion of boats t , if the velocity of
Suppose velocity of river flow vr = v, then velocity of boat relative to B
each boat with respect to water is h = 1.2 times greater than the
v stream velocity.
water vbr = . Let boat moves at an angle q with the direction of
2 Sol.
b Suppose the stream velocity is vs = v, then the velocity of each boat with
stream, then time to cross the stream t =
vbr sin q respect to water is vb = 1.2 v. Let each boat travel a distance l . Then for
boat A, time of motion
www.crackjee.xyz
Motion in a Plane 177
Along y-axis the displacement in time t, y = ut
æ 2v0 ö
\ vx = ç
è b ÷ø
ut …(ii)

The rate of change of velocity along x-axis


dvx 2v0u
ax = = (constant) …(iii)
dt b
The distance x travelled is given by second equation of motion
Figure. 4.58
1 2
l l x = uxt + at
+ 2 x
tA =
vb + vs vb - vs
1 æ 2v0 u ö 2
= 0+ t
é l l ù 60l 2 çè b ÷ø
= ê + ú= ....(i)
ë1.2 v + v 1.2 v - v û 11v y
From y = ut, t =
For the boat B, time of motion u
l l 2
tB = + 1 æ 2v0u ö æ y ö
2
vb - vs 2 2
vb - vs 2 \ x = ç ÷
2 çè b ÷ø è u ø
2l
= ....(ii) v0 y 2
vb - vs 2
2 =
ub
2l 3.01l
= = ubx
2
(1.2 v ) - v 2 v or y2 = …(iv)
v0
tA (60l /11v) Equation (iv) is an equation of a parabola, so, the trajectory of the boat
The ratio tB = » 1.8. Ans.
(3.01l / v ) is a parabola OA upto the mid stream. The other half of the trajectory is
of same nature.
Example 11. The current velocity of a river grows in proportional
to the distance from the bank and reaches its maximum v0 in the b
When boat is at the middle of the river, y =
middle. Near the banks, the velocity is zero. A boat is moving along 2
the river in such a manner that it is always perpendicular to the 2
v0 æ b ö v0b
current and the speed of the boat in still water is u. Find the x = ç ÷ = …(v)
distance through which the boat crossing the river will be carried ub è 2 ø 4u
away by the current if the width is b. Also determine the trajectory Above equation gives the drift along x-axis for first half. During second
of the path of the boat. half of motion, it is also x. Thus, total drift along the direction of river
Sol. flow = 2x
Suppose vx is the velocity of the river flow at a distance y from the bank. æ v0 b ö v0b
= 2 ç = . Ans.
Thus according to given condition vx = ky , where k is a constant. è 4u ÷ø 2u
b
Also when y = , v = v0 Example 12. A balloon starts rising from the surface of the
2
earth. The ascension rate is constant and equal to v0. Due to the
b
\ v0 = k wind the balloon gathered the horizontal velocity component
2
vx = ay, where a is a constant and y is the height of ascent. Find how
2v0 the following quantities depend on the height of ascent :
Þ k =
b (a) the horizontal drift of the balloon x (y);
2v0 (b) the total, tangential, and the normal accelerations of the
Now, vx = y …(i)
b balloon.
Sol.
(a) It is given that the ascension rate,

dy dv y
vy = = v0 or = v0
dt dt
\ dy = v0 dt

t
or y = ò v0 dt = v0 t. …(i)
0
Figure. 4.59
www.crackjee.xyz
178 MECHANICS

dx
Also vx = = ay
dt
= a v0t
\ dx = a v0 t dt
t
or x = ò av0t dt
0

Figure. 4.61
av0t 2
= …(ii)
2 Now ax = a sinq
(v0 /ay )
y = av0 ×
From equation (i), we have t = . Substituting this value in equation 2
v0 æv ö
1+ ç 0 ÷
(ii), we get è ay ø
2
av0 æ y ö av02 av02
x = = =
2 çè v0 ÷ø (ay ) 2 + v02 2
æ ay ö
v0 1+ ç ÷
è v0 ø
ay 2
or x = …(iii)
2v0 av0
=
(b) Since velocity in vertical direction is constant, 2
æ ay ö
dv y 1+ ç ÷
è v0 ø
\ ay = =0
dt and at = a cosq
The acceleration in horizontal direction,
1 ay
dv x d (av0t ) = av0 × = a v0
2
ax = = = av0 æv ö ( ay )2 + v02
dt dt 1+ ç 0 ÷
Thus total acceleration, è ay ø

a = ax 2 + a y 2 a 2 v0 y a2 y
= = Ans.
2 2
æ ay ö æ ay ö
= (av0 )2 + 0 = av0 v0 1+ ç ÷ 1+ ç ÷
èv ø 0 èv ø 0
The total acceleration is av0 and directed along horizontal direction.
Let q is the angle that the resultant velocity makes with horizontal, then Example 13. Consider a collection of a large number of particles
Normal acceleration an = a sinq and tangential acceleration at = a cosq , each with speed v. The direction of velocity is randomly distributed
we have in the collection. Show that the magnitude of the relative velocity
ay 2 between a pair of particles averaged over all the pairs in the
x =
2v0 collection is greater than v. [NCERT]
Sol.
ur ur
Let v1 and v 2 are the velocities of any two particles and q is the
angle between them. As each particle has same speed, so
v 1 = v2 = v
The relative velocity of particle 2 w.r.t. 1 is given by
ur ur ur
v 21 = v 2 – v1

Figure. 4.60
2xv0
or y =
a
Differentiating both sides of equation (iii) w.r.t. x, we get
a dy
1 = × 2y ×
2v0 dx

dy v0
or = = tanq
dx ay Figure. 4.62
www.crackjee.xyz
Motion in a Plane 179
2p
or v 21 = v 2 + v 2 - 2v v cos q æ qö
ò çè 2v sin 2 ÷ø d q
0
= 2v 2 (1 - cos q) = 2p
ò dq
= 2 2
2v ´ 2 sin q / 2 0
2p
é - cos θ/2 ù
q 2v ê ú
= 2 v sin . ë (1/2) û 0
2 =
As the velocities of the particles are randomly distributed, so q [θ]20p
varies from 0 to 2p. The magnitude of average velocity when 2p
-4v [ cos(θ/2) ]0
averaged over all such pairs. Thus =
(2p - 0)
2p
2v
ò v21d q = - [cosp – cos0]
p
0
v21 = 2p 4v
= = 1.273 v
ò dq p
0 > v. Proved

4.11 CONSTRAINT RELATIONS


In some devices of mechanics, the connected objects do not have same velocity or
acceleration. The relation between their velocities or accelerations is known as constraint
relation. In the shown device, the velocity of ring and block are not same. Here vring
v
= block .
cos q
Steps to find constraint relations :
Step 1 : Trace the directions of motion of bodies, which are connected
together.
Step 2 : Make geometric relationship between their linear variables or
between linear and angular variables. Figure. 4.63
Step 3 : Differentiate the obtained relations w.r.t. time, to get relationship between
their velocities or accelerations etc.

CONSTRAINT RELATIONS EXAMPLES


v1
Example 14. The ring M1 and block M2, are held in the position shown in figure 4.64. Now
dy the system
s ds is released. If M1 > M2 find ,
or = . v2
when the ring M1 slides down along the smooth fixed vertical rod by the distance h. dt y dt
dy ds
Here = v1 and = v2
dt dt
2 2
For y = h, s = h + l
\ Equation (ii) takes the form

Figure. 4.64 h2 + l 2
v1 = .v2
Sol. h
Let ring has moved down a distance y. From figure (b), we have
v1
or = h 2 + l2 Ans.
2 2
y +l = s 2 ....(i) v2
h
Here l is the length of string which is constant. Example 15. In the arrangement shown in fig. 4.65, the ends
Differentiating equation (i) w.r.t. time, we get P and Q of an inextensible string move downwards with uniform
dy ds speed u and v respectively. Pulley A and B are fixed. Find the
2y +0 = 2s ....(ii) velocity of mass M at the instant shown in the figure 4.65.
dt dt
www.crackjee.xyz
180 MECHANICS
with same velocity v
\ v x sin q = v

or vx = v . Ans.
sin q

Figure. 4.65
Sol.
Let at any instant the block is at a distance y from line AB and length
of string between A and C is l . In DACD, we have

x2 + y2 = l2 .... (i)
Figure. 4.67
dl
Here x remain constants while , the rate of change of length of Example 17. A rod of length l is inclined at an angle q with
dt
the floor against a smooth vertical wall. If the end A moves instan-
string, which is equal to the rate at which it pulls i.e. u.
taneously with velocity v1, what is the velocity of end B at the
Differentiating equation (i) w.r.t. time, we get
instant when rod makes q angle with the horizontal.
d 2 d 2 Sol.
(x + y2 ) = (l ).
dt dt Let at any instant, ends B and A are at a distance x and y respectively from
dy dl the point O.
2y = 2l
dt dt
dy l dl u
or = = .... (ii)
dt y dt cos q
Similar relation can be obtained from DBCD . That is
dy v
= ....(iii)
dt cos q
Adding equations (ii) and (iii), we get
dy (u + v )
=
dt 2 cos q

dy (u + v )
Thus velocity of block, in upward direction is . Ans.
dt 2 cos q Figure. 4.68
Example 16. A block is dragged on a smooth plane with the Thus we have
help of a rope which is pulled with velocity v as shown in figure x2 + y2 = l 2 .... (i)
4.66. Find the horizontal velocity of the block.
Here l is the length of the rod, which is constant. Differentiating
equation (i) with respect to time, we get

d 2 d (l )2 dx dy
(x + y2 ) = or 2 x + 2y =0 .... (ii)
dt dt dt dt
dx dy
where = v2 , and = -v1
dt dt
Now from equation (ii), we have
x(v2 ) + y ( - v1 ) = 0

y
or v2 = v1 = v1 tan q. Ans.
Figure. 4.66 x
Sol. Short-cut method
Example 18. A point A moves uniformly with a velocity v in
Let velocity of block along horizontal direction is vx, then its component
such a way that the direction of its velocity continually points at
along the rope will be v x sin q . Since each point on the rope will move another point B, which in turn, moves along a straight line with a
www.crackjee.xyz
Motion in a Plane 181
uniform velocity u (u < v). At the initial moment u and v are right Substituting the value in equation (i), we get
angles and the points are separated by a distance l . How soon will
the points meet ? æ uT ö
vT - u ç ÷ = l
è v ø
Sol.

(v 2 - u 2 )
or T = l
v

vl
or T = . Ans.
v2 - u 2

Example 19. Three points are located at the vertices of an


equilateral triangle whose side equal to a. They all start moving
simultaneously with velocity v constant in modulus, with first point
heading continually for the second, the second for the third, and
the third for the first. How soon will the points converge?
Sol.
The motions of the points are sketched in the figure. As they start
Figure. 4.69 moving simultaneously symmetrically, they will meet at the centroid
of the triangle.
Let at any instant points A and B are at the positions shown in figure
4.89. The point A moves towards B with velocity v. At the same time
B move away from A with the speed u cos a where a is the
inclination of the line AB with x-axis. The distance between them
decreases at the rate of (v - u cos a) . The initial moment the separation
between them is l. This separation reduced to zero when A and B
meet.
Suppose A and B meet after time T, then

T
ò 0 (v - u cos a)dt = l

T T
or ò0 v dt - ò0 u cos a dt = l
Figure. 4.70
T The velocity of any point towards centroid of triangle O
or vT - u ò0 cos a dt = l .... (i) [Here a is not constant]
3v
Along x-axis, the distance described by B in time T is uT. The velocity = v cos 30° = .
2
of A parallel to x-axis is v cos a . Therefore, distance describes by A
T a/2 a
in time T is ò0 v cos a dt . And its displacement =
cos 30°
=
3
.

When point A and B meet The time taken to converge the points

T
ò0 v cos a dt = uT .... (ii) =
displacement
velocity
=
a/ 3
3v / 2
=
2a
3v
. Ans.

From equation (ii), we have

T
v ò0 cos a dt = uT

T uT
or
ò0 cos a dt =
v
.... (iii)
www.crackjee.xyz
182 MECHANICS

In Chapter Exercise 4.3


1. A boy stands l = 4 m away from a vertical wall and throws a 4. A boat is approaching the shore with a speed of 5 3 m/s. At the
ball. The ball leaves the boy's hand at h = 2 m above the ground
with initial velocity instant when it is at a distance of 30 3 m from the shore, a stone
is to be projected at an elevation of 30° for it to just reach the
v0 = 10 2 m/s at an shore. What should be the speed of the stone relative to the
angle of 45° from the boat ?
horizontal. After Ans. 19.77 m/s
striking the wall v0 5. A rocket is fired vertically and tracked by the radar R as shown in
elastically the ball
45° the figure. At a particular position q = 60°, measured parameters
rebounds. Where
does the ball hit the dq
B are r = 9km and = 0.02 rad / s . Find the velocity of rocket at
ground ? 2m dt
this position.
4m
Ans. 18 m from the wall
2. A ball falls freely from a height h onto an inclined plane forming
an angle a with the horizon. Find the ratio of the successive
ranges of the ball along the plane. Consider the impacts between
the ball and the plane to be absolutely elastic.
Ans. R1 : R2 : R3 : ....... = 1 : 2 : 3......
3. Two guns are projected at each other, one upward at an angle of r
30° and the other at the same angle of depression, the muzzles
being 30 m apart as shown in the figure. If the guns are shot with
velocities of 350 m/s upward and 300 m/s downward respectively.
Find when and where the bullets may meet.
q q = 60°
B
R
Radar
Ans : 360 m/s
m
30 6. Six particles situated at the corners of a regular hexagon of side a
P move at a constant speed v. Each particle maintain a direction
towards the particle at the next corner. Calculate the time the
y particles will take to meet each other.
2a
A 30° Ans :
x
v
Ans. t = 0.0462 s and x = 14 m, y = 8.07 m
www.crackjee.xyz
Motion in a Plane 183

Mechanics
MCQ Type 1 Exercise 4.1
Level - 1 (Only one option correct)
Motion in 2D, River-boat, Rain-umbrella (a) a parabolic path
 (b) a circular path
1. An ion′s position vector is initially ri = 5ˆi − 6ˆj + 2kˆ and
 (c) a straight line path equally to x and y-axis
10 s later it is rf =−2ˆi + 8ˆj − 2kˆ , all in metre.
(d) an elliptical path
The average velocity during 10 s is : 6. A body starts from rest from the origin with an acceleration of
6 m/s2 along the x-axis and 8 m/s2 along the y-axis. Its
(a) ( −0.7ˆi + 1.4ˆj – 0.4kˆ ) m/s distance from the origin after 4 seconds will be
(b) ( 0.7ˆi + 0.8ˆj − 0.4kˆ ) m/s

(a) 56 m
(c) 80 m
(b) 64 m
(d) 128 m
(c) (1.4ˆj − 0.7ˆj + 0.8kˆ ) m/s 7. A particle moves along the parabolic path y = ax2 in such a
way that the x-component of the velocity remains constant,
(d) ( ˆi − 1.4ˆj + 2kˆ ) m/s say c. The acceleration of the particle is:
2. A particle has initial velocity ( 3ˆi + 4ˆj) and has acceleration ackˆ (b)
(a) 2ac 2 ˆj
( 0.4ˆi + 0.3ˆj) . Its speed after 10 s is : 2ac 2 kˆ (d)
(c) a 2 cjˆ
(a) 7 unit (b) 7 2 unit 8. Two particles A and B are separated by a horizontal distance
(c) 8.5 unit (d) 10 unit x. They are projected at the same instant towards each other
3. A smooth square platform ABCD is moving towards right with speeds u 3 and u at angle of projections 30° and 60°
with a uniform speed v. At what angle q must a particle be respectively figure. The time after which the horizontal
projected from A with speed u so that it strikes the point B distance between them becomes zero is :

x x
(a) (b)
u 2u
u v 2x 4x
sin −1   (b)
(a) cos −1   (c) (d)
v u u u
u v 9. A swimmer crosses a 200 m wide channel with straight bank
cos −1   (d)
(c) sin −1   and return in 10 minute at a point 300 m below the starting
v u
point (downstream). The velocity of the swimmer relative
4. The height y and the distance x along the horizontal plane to the bank if he heads towards the bank to the channel all
of a projectile on a certain planet [with no surrounding the time at right angles is.
atmosphere] are given by y = [5t – 8t2] metre and x = 12t
(a) 2 km/h (b) 3 km/h
metre where t is the time in second. The velocity with which
(c) 4 km/h (d) 5 km/h
the projectile is projected, is:
(a) 5 m/s (b) 12 m/s 10. A boat which has a speed of 5 km/h, in still water crosses a
(c) 13 m/s (d) not obtainable from the river of width 1 km along the shortest possible path in 15
data minute. The velocity of the river water in km/h is
5. A particle moves in x-y plane according to law x = a sin ωt and (a) 1 (b) 3
y = b cos ωt where a and b are constants. Then the particle (c) 4 (d) 41
follows:

Answer 1 (a) 2 (b) 3 (b) 4 (c) 5 (d)


Key 6 (c) 7 (b) 8 (a) 9 (b) 10 (b)
www.crackjee.xyz
184 Mechanics
11. A man crosses a 320 m wide river perpendicular to the 17. The angle of projection, for which the horizontal range and
current in 4 minute. If in still water he can swim with a the maximum height of a projectile are equal, is:
5
speed times that of the current, then the speed of the (a) 45° (b) θ = tan–14
3
current, in m/min is: θ = tan–1 (0.25)
(c) (d) none of these.
(a) 30 (b) 40 18. The range of a projectile which is launched at an angle of
(c) 50 (d) 60 15° with the horizontal is 1.5 km. What is the range of the
12. A river is flowing from west to east at a speed of 5m/min. projectile if it is projected of an angle 45° to the horizontal?
A man on the south bank of the river capable of swimming  (AMU B.Tech.-2003)
at 10 m/min in still water wants to swim across the river (a) 1.5 km (b) 3 km
(a) due north to reach in shortest time (c) 6 km (d) 0.75 km
(b) at an angle 30° west of north to reach in minimum
19. An aeroplane is flying horizontally with a velocity of 600
time
km/h at a height of 1960 m. When it is vertically at a point
(c) at an angle 60° west of north to reach along shortest A on the ground, a bomb is released from it. The bomb
path strikes the ground at point B. The distance AB is
(d) at angle 45° west of north to reach along shortest path
(a) 1200 m (b) 0.33 km
Projectile Motion
(c) 3.33 km (d) 33 km
13. A stone is just released from the window of a train moving 20. A body is projected with velocity v1 from point A. At the
along a horizontal straight track. The stone will hit the same time another body is projected vertically upwards with
ground following velocity v2. The point B lies vertically below the highest
(a) straight path (b) circular path v
point. For both the bodies to collide 2 should be:
(c) parabolic path (d) hyperbolic path v1
14. When air resistance is taken into account while dealing with
the motion of the projectile. Of the following properties of
the projectile, the one which shows an increase, is:
(a) range 3
(b) maximum height (a) 2 (b)
2
(c) speed at which it strikes the ground (c) 0.5 (d) 1
(d) the angle of which the projectile strikes the ground 21. A boy throws a ball with a velocity u at an angle θ with
15. At the top of the trajectory of a projectile, the acceleration the horizontal. At the same instant he starts running with
is uniform velocity to catch the ball before it hits the ground.
(a) maximum (b) minimum To achieve this he should run with a velocity of:
(c) zero (d) g u cos θ (b)
(a) u sin θ
16. It was calculated that a shell when fired from a gun with
u 2 tan θ
u tan θ (d)
(c)

a certain velocity and at an angle of elevation radian 22. A cricketer can throw a ball to a maximum horizontal
36
should strike a given target. In actual practice it was found distance of d. How high above the ground can the cricketer
that a hill just prevented in the trajectory. At what angle of throw the same ball ?
elevation should the gun be fired to hit the target:
(a) d/2 (b) d
5π 11π
(a) (b) (c) 2d (d) 5d/2
36 36
23. A particle is projected from the ground with a velocity of 25
7π 13π
(c) (d) m/s. After 2 second, it just clears a wall 5 m height. Then
36 36
the angle of projection of particle is [g = 10 m/s2]:

Answer 11 (d) 12 (a) 13 (c) 14 (d) 15 (d) 16 (d)


Key 17 (b) 18 (b) 19 (c) 20 (c) 21 (a) 22 (a)
www.crackjee.xyz
Motion in a Plane 185
(a) 30° (b) 45°
R = 4 H1H 2 (b)
(a) =R 4 ( H1 − H 2 )
(c) 60° (d) 75°
24. The horizontal and vertical components of the velocity H12
(c)
=R 4 ( H1 + H 2 ) (d)
R=
of a projectile are 10 m/s and 20 m/s, respectively. The H 22
horizontal range of the projectile will be [g = 10 m/s2] Constraint Relation
(a) 5 m (b) 10 m 29. The end A of a rod slides down a smooth wall and its end B
(c) 20 m (d) 40 m slides on a smooth floor. When AB makes an angle α with
25. A cart is moving horizontally along a straight line with the horizontal. A has speed v. The speed of end B will be
constant speed 30 m/s. A projectile is to be fired from the
moving cart in such a way that it will return to the cart after
the cart has moved 80 m. At what speed (relative to the cart)
must the projectile be fired (Take g = 10 m/s2)
(a) 10 m/s (b) 10 8 m/s
40
(c) m/s (d) none of these
3 v
26. For an object thrown at 45° to horizontal, the maximum (a) (b) v tan α
tan α
height (H) and horizontal range (R) are related as
v
(a) R = 16 H (b) R = 8 H v sin α
(c) (d)
cos α
(c) R = 4 H• (d) R = 2H
27. A ball is thrown up at an angle 45° with the horizontal. Then 30. A block is dragged on a smooth plane with the help of a
rope which is pulled by velocity v as shown in figure. The
the total change of momentum by the instant it returns to
horizontal velocity of the block is:
ground is
(a) zero (b) 2 mv
mv
(c) 2 mv (d)
2
28. A stone projected with a velocity u at an angle θ with the
horizontal reaches maximum height H1. When it is projected
π 
with velocity u at an angle  − θ  with the horizontal,
2 
it reaches maximum height H2. The relation between the
v (b)
(a) v/sin θ
horizontal range R of the projectile, H1 and H2 is
v sin θ (d)
(c) v/cos θ
Answer 23 (a) 24 (d) 25 (c) 26 (c)
Key 27 (c) 28 (a) 29 (b) 30 (b )

Level - 2 (Only one option correct)


Motion in 2D, River-boat, Rain-umbrella a/v1 (b)
(a) a/v2
1. A particle starts from the origin at t = 0 s with a velocity av av2
(c) 1 (d)
2
of 10.0 m/s and moves in the xy- plane with a constant v v2
acceleration of m/s2 . The y-coordinate of the 3. Two particles start simultaneously from the same point and
particle when x-coordinate is 16: move along two straight lines, one with uniform velocity
v and other with a uniform acceleration a. If α is the angle
(a) 2 m (b) 24 m
between the lines of motion of two particles then the least
(c) 8 m (d) 16 m
value of relative velocity will be at time given by
2. The distance between two moving particles at any time t
is ‘a’. If v be their relative velocity and v1 and v2 be the v v
(a) sin α (b) cos α
components of v along and perpendicular to ‘a’, then the a a
time when they are closest to each other: v v
(c) tan α (d) cot α
a a
www.crackjee.xyz
186 Mechanics
4. Passengers in the jet transport A flying east at a speed of 8. The equation of trajectory of a particle is given by the equation
800 kmh–1 observe a second jet plane B that passes under y = ax – bx2, where a and b are constant. The horizontal
the transport in horizontal flight. Although the nose of B is range is:
pointed in the 45° north east direction, plane B appears to a (b)
(a) b
the passengers in A to be moving away from the transport ab
(c) (d) a/b
at the 60° angle as shown. The true velocity of B is 9. Two balls are projected simultaneously in the same vertical
plane from the same point with velocities v1 and v2 with
angle θ1 and θ2 respectively with the horizontal. If v1 cos
θ1 = v2 cos θ2, the path of one ball as seen from the position
of other ball is :
(a) parabola
(b) horizontal straight line
(c) vertical straight line
(d) straight line making 45° with the vertical
(a) 586 kmh–1 (b) 400 2 kmh–1 10. A projectile moves from the ground such that its horizontal
(c) 717 kmh–1 (d) 400 kmh–1 displacement is x = Kt and verticle displacement is y = Kt(1
5. A river is flowing with a velocity of 1 m/s towards east – at), where K and α are constants and t is time. Find out
directions. When the boat runs with a velocity of 3 m/s total time of flight (T) and maximum height attained (Ymax)
relative to the river is the direction of the river flow, the its
flag on the boat flutter in north direction. If the boat runs K 1 2K
T=
(a) α, Ymax = (b)
= T = , Ymax
with the same speed but in north direction relative to river, 2α α α
the flag flutters towards north-east direction. The actual 1 K 1 K
velocity of the wind should be : ( i → east direction and T =
(c)
= , Ymax (d) T
= = , Ymax
α 6α α 4α
j → north direction).
11. An object is projected with a velocity of 20 m/s making an
(a) 4 i + 6 j (b) 6 i + 4 j angle of 45º with horizontal. The equation for the trajectory
is h = Ax – Bx2 where h is height, x is horizontal distance,
(c) 4 i – 6 j (d) 6 i – 4 j A and B are constants. The ratio A : B is (g = 10 ms–2)
Projectile Motion (a) 1 : 5 (b) 5 : 1
 (c) 1 : 40 (d) 40 : 1
6. You throw a ball with a launch velocity of = (
v 3ˆi + 4ˆj ) 12. If retardation produced by air resistance of projectile is
m/s towards a wall, where it hits at height h1. Suppose that
 one-tenth of acceleration due to gravity, the time to reach
the launch velocity were, instead, = ( )
v 5ˆi + 4ˆj m/s. If h 2 maximum height
is height, then (a) decreases by 11 percent
(b) increases by 11 percent
(c) decreases by 9 percent
(d) increases by 9 percent
13. An aircraft moving with a speed of 250 m/s is at a height of
6000 m, just overhead of an anti aircraft gun. If the muzzle
velocity is 500 m/s, the firing angle θ should be:
h2 = h1 (b)
(a) h2 < h1
h2 > h1
(c) (d) unanswerable
7. A ball is shot from the ground into the air. At a height of

v 7.6ˆi + 6.1ˆj in m/s.
9.1 m, its velocity is observed to be=
The maximum height the ball will rise is :
(a) 10 m (b) 11 m
(a) 30° (b) 45°
(c) 12.5 m (d) 15 m (c) 60° (d) none of these.

Answer 1 (b) 2 (c) 3 (b) 4 (c) 5 (a) 6 (b) 7 (b)


Key 8 (d) 9 (c) 10 (d) 11 (d) 12 (c) 13 (c)
www.crackjee.xyz
Motion in a Plane 187
14. A particle is ejected from the tube at A with a velocity v at
an angle 30° with the vertical y-axis. A strong horizontal
wind gives the particle a constant horizontal acceleration
a in the x- direction. If the particle strikes the ground at a (c) (d)
point directly under its released position and the downward
y -acceleration is taken as g then

17. A boy throws a ball upward with velocity v0 = 20 m/s


making an angle θ with the vertical. The wind imparts a
horizontal acceleration of 4 m/s2 to the left. The angle at
which the ball must be thrown so that the ball returns to the
boy’s hand is (g = 10 m/s2)
v0 wind

3v 2 3v 2
h=
(a) h=
(b)
2a 2g
v2  3 a  v2  3 g  (a) tan–1(1.2) (b) tan–1(0.2)
h=
(c)  + h=
 (d)  + 
g  2 2g  a  2 2a  (c) tan–1(2) (d) tan–1(0.4)
15. A particle P is projected from a point on the surface of long
18. In figure the angle of inclination of the inclined plane is
smooth inclined plane and Q starts moving down the plane
30°. The horizontal velocity V0 so that the particle hits the
from the same position. P and Q collide after 4 second. The
inclined plane perpendicularly is
speed of projection of P is : (g = 10 m/s2)
V0
(a) 5 m/s (b) 10 m/s 2 gH
(a) V0 = 90°
(c) 15 m/s (d) 20 m/s 5
H
2 gH
V0 =
(b)  90°
7 30°
gH gH
V0 =
(c) V0 =
(d)
5 7
19. A stone is projected from a horizontal plane. It attains
maximum height H and strikes a stationary smooth wall and
16. Which of the following plots correctly represents the falls on the ground vertically below the maximum height.
variation of the magnitude of radial acceleration aR with Assuming the collision to be elastic, the height of the point
time t for a particle projected at t = 0 with speed v0 at an on the wall where ball will strike is:
angle θ above the horizontal?

H
h
(a) (b)
H H
(a) (b)
4 2
3H 7H
(c) (d)
4 8

Answer 14 (d) 15 (b) 16 (c)


Key 17 (d) 18 (a) 19 (a)
www.crackjee.xyz
188 Mechanics
Constraint Relations
20. A racing car travelling along a track at a constant speed of 40 m/s.
A television cameraman is recording the event from a distance
40 m directly away from the track as shown in figure. In order to
keep the car under view, with what angular velocity the camera
should be rotated when q = 45°?

v
(a) 1440 km/h (b) 960 km/h
(c) 1920 km/h (d) 480 km/h
22. In the arrangement shown in the figure the block B
starts from rest and moves towards right with a constant
acceleration. After time t the velocity of A with respect to
 B become v. The acceleration of A is
Camera

A B
(a) 5/2 rad/s (b) 2 rad/s
(c) 3/2 rad/s (d) 1/2 rad/s S
21. A jet plane flying at a constant velocity v at a height
h = 8 km, is being tracked by a radar R located at O directly v 2v
(a) (b)
below the line of flight, if the angle θ is decreasing at the rate t t
of 0.025 rad/s, the velocity of the plane when θ = 60° is : 3v 4v
(c) (d)
t t

Answer 20 (d) 21 (b) 22 (c)


Key

Mechanics
MCQ Type 2 Exercise 4.2
Multiple Options Correct 3. A ball is released from the window of a train moving along
 a horizontal straight track with constant velocity. The path
1. River is flowing with a velocity v R = 4ˆi m / s. A boat is
 as observed:
moving with a velocity of v BR =( −2ˆi + 4ˆj) m / s relative (a) from ground, will be parabolic
to river. The width of the river is 100 m along y-direction. (b) from ground, will be vertical straight line
Choose the correct alternative(s). (c) from train, will be parabolic
(a) The boatman will cross the river in 25 s (d) from train, will be vertical straight line
(b) Absolute velocity of boatman is 2 5 m / s 4. Three balls are projected with same speed as shown in
(c) Drift of the boatman along the river current is 50 m figure. First and third balls are projected from same height h
(d) The boatman can never cross the river. and second ball is projected is ground. In these three cases,
2. A plane is to fly due north. The speed of the plane relative times of flights are T1, T2 and T3 respectively, then choose
to the air is 200 km/h, and the wind is blowing from west correct relations :
u
to east at 90 km/h.

(a) The plane should head in a direction given by 
θ = sin–1 (0.65) u
(b) The plane should head in a direction given by
θ = sin–1 (0.45)
(c) The velocity of plane relative to the ground is 179
u
km/h.
(d) The velocity of plane relative to the ground is 159 
km/h. Case 1 Case 2 Case 3
www.crackjee.xyz
Motion in a Plane 189
T1 = T2 + T3
(a) 8. Two particles are projected from ground with same initial
(b) T2 = T1 + T3 velocities at angles 30° and 60° (with horizontal). Let R1
(c) T1 > T2 and R2 be their horizontal ranges, H1 and H2 their maximum
(d) T1 > T3 heights and T1 and T2 are the time of flights. Then :
5. A body is projected with certain angle from the ground. The H H H1 H 2
(a) 1 > 2 (b) <
motion of the body is described by the equations x = 2t, R1 R2 R1 R2
y = 3t – 4t2. Then: H H H1 H 2
3x (c) 1 > 2 (d) <
(a) equation of trajectory is =y − x2 T1 T2 T1 T2
2
(b) angle of projection is 45° 9. A particle is projected from a point P with a velocity v at
(c) the velocity of projection is 13 m/s an angle θ with horizontal. At a certain point Q it moves at
(d) the acceleration due to gravity is 10 m/s2 right angles to its initial direction. Then:
6. Two second after projection, a projectile is moving at 30° (a) velocity of particle at Q is v sin θ
above the horizontal; after one more second it is moving
(b) velocity of particle at Q is v cot θ
horizontally. Then:
(c) time of flight from P to Q is (v/g) cosec θ
(a) angle of projection is 30°
(d) time of flight from P to Q is (v/g) sec θ
(b) velocity of projection is 20 3 m/s
(c) velocity at any time will be 40 m/s 10. Trajectories are shown in figure are for three kicked
footballs, ignoring the effect of the air on the footballs. If
(d) maximum horizontal range attained = 60 3 m
T1, T2 and T3 are their respective time of flights then:
7. The velocity of a particle moving in a positive direction of
the x-axis varies as v = α x , where α is a positive constant.
Assuming that at the moment t = 0, the particle was located
at the point x = 0:
(a) in the motion, the acceleration is constant
(b) velocity at any time will be α/t
α 2t
(c) velocity at any time will be
2
(d) mean velocity of particle averaged over the time that
T1 > T3 (b)
(a) T1 < T3
α s
the particle takes to cover first s meter is T1 + T3
2 T2 =
(c) T1 = T2 = T3
(d)
2

Answer 1 (a, b, d) 2 (b, c, a) 3 (a, d) 4 (a, c, d)


Key 5 (a, c) 6 (b, d) 7 (a, c, d) 8 (a, c)

Mechanics
Reasoning Type Questions Exercise 4.3
Read the two statements carefully to mark the correct option out of the options given below:
(a) Statement - 1 is true, Statement - 2 is true; Statement - 2 is correct explanation for Statement - 1.
(b) Statement -1 is true, Statement - 2 is true; Statement - 2 is not correct explanation for Statement - 1.
(c) Statement - 1 is true, Statement - 2 is false.
(d) Statement - 1 is false, Statement - 2 is true
1. Statement - 1 When a body is dropped or thrown Statement - 2
horizontally from the same height, it would reach the ground Gravitational force makes the path of projected body always
at the same time. parabolic.
Statement - 2
3. Statement - 1
There is no acceleration in horizontal direction.
2. Statement - 1 The maximum possible height attained by the projected
If there were no gravitational force, the path of the projected u2
body is , where u is the velocity of projection.
body always be a straight line. 2g
www.crackjee.xyz
190 Mechanics
Statement - 2 7. Statement - 1
To attain the maximum height, body is thrown vertically A body is thrown with a velocity u inclined to the horizontal
upwards. at some angle. It moves along a parabolic path and falls
4. Statement - 1 to the ground. Linear momentum of the body, during its
When the range of projectile is maximum, the time of flight motion, will remain conserve.
is the largest. Statement - 2
Statement - 2 Throughout the motion of the body, a constant force acts
Range is maximum when angle of projection is 45°. on it.
5. Statement - 1 8. Statement - 1
A shell fired from a gun is moving along the parabolic path. Two projectiles having same range must have the same time
If it explodes at the top of the trajectory, then no part of the of flight.
shell can fly vertically.
Statement - 2
Statement - 2
Horizontal component of velocity is constant in projectile
The vertical momentum of the shell at the top of the trajec-
motion under gravity.
tory is zero.
6. Statement - 1 9. Statement - 1
A projectile, launched from ground, collides with a smooth A man projects a stone with speed u at some angle. He again
vertical wall and returns to the ground. The total time of projects a stone with same speed such that time of flight
flight is the same had there been no collision. now is different. The horizontal ranges in both the cases
may be same. (Neglect air friction)
Statement - 2
The horizontal range is same for two projectiles projected
with same speed if one is projected at an angle θ with the
horizontal and other is projected at an angle (90° – θ) with
the horizontal. (Neglect air friction)
Statement - 2
The collision changes only the horizontal component of
velocity.

Answer 1 (b) 2 (c) 3 (a) 4 (d) 5 (d)


Key 6 (a) 7 (d) 8 (d) 9 (a)

Mechanics
Passage & Matrix Exercise 4.4
Passages
Passage for (Q. 1 - 2) :
5
A particle is projected at an angle θ with the horizontal such that it just (c) gh
5gh (d)
able to clear a vertical wall of height h at a distance h from point of 2
projection as shown in figure. Passage for (Questions 3 to 5) :
Consider the case of the collision of a ball with a wall. In this case the
u problem of collision can be simplified by considering the case of elastic
collision only. When a ball collides with a wall we can divide its velocity
h
 into two components, one perpendicular to the wall and other parallel to
the wall. If the collision is elastic then the perpendicular component of
h velocity of the ball gets reversed with the same magnitude.
Vcos Vcos
1. The angle of projection θ is :
(a) tan–1 (2) (b) tan −1 3 Vsin Vsin
v
−1 −1
(c) tan ( 2 / 3) (d)
tan ( 3 / 2)
2. The velocity of projection u is :
Velocity just Components of velocity Components of velocity
gh
(a) 2gh (b) before collision just before collision just after collision
www.crackjee.xyz
Motion in a Plane 191
The other parallel component of velocity will remain constant if wall 3. Which of the following relation about the maximum height H of
is given smooth. the three balls from the ground during their motion in air is correct:
Now let us take a problem. Three balls A and B & C are projected from (a) HA = HC > HB (b) HA > HB = HC
ground with same speed at same angle with the horizontal. The balls (c) HA > HC > HB (d) HA = HB = HC
A, B and C collide with the wall during their flight in air and all three 4. If the time taken by the ball A to fall back on ground is 4 seconds
collide perpendicularly with the wall as shown in figure. and that by ball B is 2 seconds. Then the time taken by the ball C
to reach the inclined plane after projection will be –
(a) 6 sec. (b) 4 sec.
A (c) 3 sec. (d) 5 sec.
C
5. The maximum height attained by ball A from the ground is–
B
(a) 10m (b) 15 m
(c) 20 m (d) Insufficient information

Matrix Matching
6. The equation of trajectory of a particle projected from the surface of the planet is given by the equation y = x – x2.
Then match the columns :( suppose, g = 2 m/s2)
Column - I Column - II (magnitude only)
A. angle of projection, tan θ (p) ¼
B. time of flight, T (q) 1
C. maximum height attained, H (r) 2
D. horizontal range, R (s) 4
A particle is projected with some angle from the surface of the planet. The motion of the particle is described by the equation; x = t , y = t − t 2 .
7.
Then match the following columns :
Column - I Column - II
(quantity) (magnitude only)
A. velocity of projection (p) 1
B. acceleration (q) 2
C. time of flight (r) 2
D. maximum height attained (s) 3
1
(t)
4

Answer 1 (a) 2 (d) 3 (a) 4 (c) 5 (c)


Key 6 A→q;B→r;C→p;D→q 7 A→q;B→r;C→p;D→t

Mechanics
Best of JEE-(Main & Advanced) Exercise 4.5
JEE- (Main) (a) yes, 60° (b) yes, 30°
(c) no (d) yes, 45°
1.
A projectile can have the same range R for two angles

of projection. If t1 and t2 are the times of flight in the 3. A particle is moving with velocity= v k ( yiˆ + xjˆ) , where k
two cases, then the product of two times of flight is: is a constant. The general equation of its path is
 [AIEEE 2004] [AIEEE 2010]
R2 2R (a) y2 = x2 + constant (b) y = x2 + constant
(a) (b) (c) y2 = x + constant (d) xy = constant
g g
4. A large number of bullets are fired in all directions with
R 4R 2
(c) (d) same speed v. What is the maximum area on the ground on
2g g which these bullets will spread [AIEEE 2011]
2. A ball is thrown from a point with a speed v0 at an angle of v2 v4
projection θ. From the same point and at the same instant (a)
π (b)
π
g g2
a person starts running with a constant speed v0/2 to catch
the ball. Will the person be able to catch the ball ? If yes, v4 v2
π2
(c) π2
(d)
what should be the angle of projection [AIEEE 2004] g2 g2

Answer Key 1 (b) 2 (a) 3 (a) 4 (b)


www.crackjee.xyz
192 Mechanics

In Chapter Exercise

b
In Chapter Exercise -4.1 vxt ( u cos θ + v0 )
a = =
 u sin θ
1. Given, r = 3.0tˆi + 2.0t 2ˆj + 5.0kˆ
 or au sin θ = ub cos θ + v0b
 dr
v = = 3.0ˆi + 4.0tˆj + 0=
(a) ( 3.0ˆi + 4.0ˆj) m/s v0b
dt \ u = ...(i)
 ( a sin θ − b cos θ )
 d 2r

a = 2
0 + 4.0ˆj =
= 4.0ˆj m/s 2 u to be minimum, du/d θ = 0
dt
 d  vb 
(b) At, t = 3.0 s, v = 3.0ˆi + 4.0 × 3ˆj= 3.0ˆi + 12.0ˆj or  0 = 0
d θ  a sin θ − b cos θ 
\ v = 3.02 + 12.02 =
12.4 m/s
(a sin θ − b cos θ) × 0 − (a cos θ + b sin θ) = 0
12.0 a
and tan θ = = 4, \ θ = 76° Ans. or tan q = −
3.0 b

2. Given v train = 30iˆ b
–vtrain \ cos q =
 a + b2
2
and v car = 40jˆ
   On substituting these values in equation (i), we get
[vcar ]train = [vcar ] − [v ]train [ vcar ]train v v0b
 car umin = Ans.
ˆ ˆ
= 40 j − 30i a 2 + b2
2
or vcar = 402 + ( −30 ) =50 km/h In Chapter Exercise -4.2
30
tan θ = u2
40 1. Rmax = = 160 m
g
\ θ = 36° 52′ Ans.
 u2 R
3. Given, v rain = 24ˆj m / s ^i H max = = max
160
= = 80 m
 2g 2 2
and v woman = 12 ( −iˆ ) m/s
2. If u is the velocity of arrow, then
  
[ v rain ]woman = v rain − v woman u x = u cos 45° =
u
= 24ˆj − ( −12ˆi ) 2
^j
u
= 24ˆj + 12ˆi and u y = u sin 45° =
2
an


m

12 1
wo

tan q =
]

= vtrain
n
ra
[v

24 2
Ans. –vwoman vwoman u
39.6 m
45°
4. Suppose u is the speed of the boat relative to water, then
35.2 m
velocity of the flow (w.r.t. bank) If t is the time taken to reach the bird, then
vx = ( u cos θ + v0 ) , 35.2 = u xt
and perpendicular to flow will be u
or 35.2 = t ...(i)
vy = u sinθ 2
b 1 2
Time to cross the river t = . and 39.6 = u yt − gt
u sin θ 2
In this time the distance travelled by the boat in the direction u 1
of flow = t − gt 2 ...(ii)
C a B 2 2
After solving, we get, u = 41.86 m/s.
b
3. Given, y = bx − cx 2
u dy dx dx
 \ = b − 2cx ...(i)
A
dt dt dt
www.crackjee.xyz
Motion in a Plane 193
\ T = 1.8 s
d2y d 2x  d 2 x dx dx 
and = b − 2 c x 2 + ⋅  The horizontal distance travelled by the ball in this time
dt 2 dt 2  dt dt dt 
x = vx T = 10 × 1.8 = 18 m Ans.
d2y d 2x 2. If u is the velocity of ball before collision with the inclined
At t = 0, x = 0; Also 2
= −a and 0,
= plane, then after impact, its two components of velocity
dt dt 2
along and perpendicular to plane are;
\ – a = −2c vx2 ux = u sin q
a and uy = u cos q
or vx2 =
2c If T1 is the time between first and second impact, then
a a 1
From equation (i), vy = b −0=b 0 = u cos θ T1 − ( g cos θ ) T12
2c 2c 2
2u
v0 = vx2 + v 2y \ T1 = . (constant)
g
a a
= + b2 × ucos
2c 2c u u
a 
= 1 + b2  Ans. n 
2c   usi

4. For the collision x = v0 cos 60° t1 = v0 cos 45° t2 ...(i)
The range along the inclined plane
1 2 1
gt1 = v0 sin 45° t2 − gt22 ...(ii)
and y = v0 sin 60°t − 1
2 2 R1 = u sin θ T1 + ( g sin θ ) T12
2
After solving equations (i) and (ii), we get
2
∆t = t1 − t2 = 11 s Ans. 2u 1  2u 
= u sin θ × + g sin θ  
g 2  g 
In Chapter Exercise -4.3
1. The velocity components at the point of projection are 2u 2 sin θ 2u 2 sin θ
= +
ux = 10 2 cos 45° =10 m/s g g
uy = 10 2 sin 45° =10 m/s 4u 2 sin θ
Time taken to cover horizontal distance 4 m, =
g
4
t = = 0.4 s The velocity after second impact,
10
At the instant of collision with the wall, the velocity com- v = u sin θ + g sin θt
ponents are 2u
= u sin θ + g sin θ ×= 3u sin θ
vx = 10 m/s g
vy = 10 – g × 0.4 = 6.1 m/s 1
6.1 m/s Range R2 = ( 3u sin θ ) T1 + ( g sin θ ) T12
2
10 m/s 2
2u 1  2u 
= 3u sin θ × + g sin θ  
g 2  g 
10 m/s
4.2 m 8 u 2 sin θ
=
g
So R1 : R2 : R3........ = 1 : 2 : 3 ........ Ans.
10 m/s
2m 3. 300 cos30°

4m
The height attained by the ball above point of projection 300 sin30°
1
h = 10 × ( 0.4 ) − g ( 0.4 )2
2 H
P
= 3.2 m 350 sin30°
The total height of the ball from the ground y
H = 2 + 3.2 = 5.2 m 350 cos30°
The time taken by ball to hit the ground x
R
1 2
5.2 = −6.1 T + gT The situation is shown in figure.
2
or 4.9 T2 – 6.1T – 5.2 = 0 R2 + H2 = 302 ...(i)
www.crackjee.xyz
194 Mechanics
If t is the time when bullets meet, then 5. If y is the height of the rocket at any instant, then
x = 350 cos 30° t, ...(ii) y = x tan θ
R – x = 300 cos 30°t, ...(iii) Differentiating above equation w.r.t. time, we get
1 dy dθ

Also y = 350sin 30°t − gt 2 , ...(iv) = x sec2 θ
2 dt dt
 dθ 

and
1
H – y = 300sin 30°t + gt 2 ...(v) or v = ( r cos θ ) sec2 θ  
2  dt 
After solving above equations, we get t = 0.0462 s,
x = 14 m and y = 8.07 m
= ( ) 1 
9 × 103 ×   × ( 0.02 )
 cos 60° 
4. Suppose u is the speed of the stone relative to boat. The = 360 m/s Ans.
components of speed of stone relative to ground
ux = u cos30° + 5 3 m/s
 3u 
=  + 5 3  m/s
 2  r y
uy = u sin 30°
and
u
= m/s 
2 R x
If T is the time of motion of stone, then for vertical displace- 6. The particles will meet at the centre of the hexagon.
ment to be zero, The displacement of any particle from its initial position
1 2 /2
0 = u yT − gT s =
2 sin 30°
2u y u v
\ T = = The effective velocity ve = v cos 60° = .
g g 2
For horizontal motion, we have
30 3 = u xT
30°
 3u  u
=  + 5 3  × v 60° s
 2  g
After solving, we get u = 19.77 m/s Ans. a/2
s 2a
\ Time taken = = Ans.
ve v

Exercise 4.1 Level -1


1.
(a) The average velocity,
  dy d (5t − 8t 2 )
 and vy = = = 5 – 16 t.
r f − ri (−2iˆ + 8 ˆj − 2kˆ) − (5iˆ − 6 ˆj + 2kˆ) dt dt
v av = =
∆t 10
∴ vt =0 = vx2 + v 2y = 122 + 52 = 13 m/s
(−0.7iˆ + 1.4 ˆj + 0.4kˆ) m/s.
=
   5. (d) x = a sin ω t,
( )
v = u + a t = 3i + 4j + (0.4i + 0.3j) ×10
2. ∴ sin ω t = x/a
and y = b cos ω t,
7i + 7j
=
∴ cos ω t = y/b.
or v = 72 + 72 =
7 2 unit x2 y2
∴ + = 1. It represents an ellipse.
3. (b) Particle will strike the point B if velocity of particle a2 b2
with respect to platform is along AB or component of
its relative velocity along AD is zero, i.e. u cos q = v 1
6. (c) x = × 6 × 42 = 48 m
v 2
or q = cos −1  
u 1
and y = × 8 × 42 = 64 m
dx d (12t ) 2
4. (c) vx = = = 12
dt dt
∴ s = x 2 + y 2 = 482 + 642 = 80 m.
www.crackjee.xyz
Motion in a Plane 195
7. (b) y = ax2 1
dy dx 10. (b) u = = 4 km/h
∴ vy = = a × 2x 1/ 4
dt dt
= 2ax vx = 2 a c x v
5km/h
dv y  dx 
Now, ay = = 2 ac   = 2 ac2. u=4 km/h
dt  dt 
8. (b) Their velocity of approach is ∴ v = 52 − 42 = 3 km/h.
v = u 3 cos 30° + u cos 60°.
320
= 2u. 11. (d) u 2 − v 2 = = 80
4
x x
∴ t = = .
v 2u v
9. (b) Time to cross the river = 5 min. u
The displacement perpendicular to flow = 200 m.
\ Velocity of swimmer 2
200 2 5  2
vsy = = m/s  v  − v = 80
5 × 60 3 3
 
∴ v = 60 m/min
12. (a) The man can cross the river in minimum time when
he swim across perpendicular to flow direction.
13. (c) The resultant path of constant velocity and acceleration
will be parabolic.
14. (d) Because of the constriction of the path, the angle of
strike becomes greater than angle of projection.
15. (d) At the highest point of trajectory, the acceleration is equal
to g.

16. (d) θ = ; the other possible angle to get the same range
36
The displacement covered in the direction of flow = is,
300 m in 10 min. π π 5π 13π
∴ The velocity of river flow − θ = − = .
2 2 36 36
300 1
vr = = m/s u 2 sin 2θ u 2 sin 2 θ
10 × 60 2 17. (b) =
The velocity of swimmer in the direction of flow g 2g
  
v sx = v r + v s ∴ tan θ = 4.
1 1 u 2 sin(2 × 15°) u 2
or vsx = + 0 = m/s 18. (b) R1 = = .
2 2 g 2g
His velocity with respect to bank
2 2 u 2 sin(2 × 45°) u2
2 1 and R2 = =
vs = vsx 2 + vsy 2=   +  g g
3 2
∴ R2 = 2R1 = 2 × 1.5 = 3 km.
5
= m/s
6 1
= 3 km/h 19. (c) 1960 = × 9.8 × t 2
 2
The velocity v s makes an angle θ with the bank, then
∴ t = 20 s
vsy 2/3  5
tan θ = = Now AB = ut =  600 ×  × 20
vsx 1/ 2  18 
4 = 3333 m.
=
3 20. (c) The vertical components of the velocities must be equal
so.
or θ = tan–1(4/3) Ans.
v1 sin 30° = v2
www.crackjee.xyz
196 Mechanics

v2 1 27. (c) ∆ P = 2 mu sin θ


or = .
v1 2 2u sin θ
= (mg) ×
21. (a) The boy velocity = horizontal velocity of the ball g
= u cosθ. = weight × time of flight.
22. (a) Let u is the velocity of projection, then
u 2 sin 2 θ
u2 28. (a) H1 =
Rmax = =d 2g
g
u 2 sin 2 (π / 2 − θ) u 2 cos 2 θ
or u = gd and H2 = =
2g 2g
Let h is the height upto which ball rise, then
0 = u2 – 2gh u 2 sin 2 θ u 2 cos 2 θ
H1 H2 = ×
u2 gd 2g 2g
h =
or =
2g 2g (u 2 2sin θ cos θ)2 R2
= =
d 16 g 2 16
=
2 ∴ R = 4 H1H 2 .
23. (a) uy = u sin θ.
29. (b)
1 y
y = uy t – gt2
2 A
1
or 5 = 25 sin θ – × 10 × 22.
2

1 y
∴ sin θ = ,
2
or θ = 30°. 
x
2u x u y 2 × 10 × 20 x B
24. (d) R = = = 40 m. At any instant,
g 10
25. (c) Time of motion, x 2 + y 2 =
 (constant)
x 80 8
t = = = s. dx dy
u x 30 3 or 2 x + 2y 0
=
dt dt
1 2
Thus, 0 = u sin θ t − gt or 2x vB + 2 y v A =
0
2
40 y
⇒ u sin q = or v B = − vA
3 x
∴ uy = 40/3 m/s.
or v B v tan α
=
u 2 sin 45° u 2
26. (c) H = = 30. (b) If u is the horizontal velocity of the block, then
2g 4g
u sin θ = v,
u2 v
R = = = 4 H. ∴ u =
g sin θ

Exercise 4.1 Level -2


1. (b) Given that : After simplifying,
ux = 0; uy = 10 m/s t = 2s
and ax = 8.0 m/s2; ay = 2.0 m/s2 The y coordinate at
Let at time t the x-coordinate is 16 m. We have t = 2 s is
1 1
x = ux + axt2 y = uyt + ay t2
2 2
1 1
or 16 = 0 + × 8 × t 2 = 10 × 2 + × 2 × 22 = 24 m Ans.
2 2
www.crackjee.xyz
Motion in a Plane 197

2. (c) The velocity of wind, = v = vx i + vy j
a
5.
v1
Velocity of boat w.r.t. bank vb = 3 + 1 = 4 m/s
  
Now v w  = v w – vb = (vx – 4) i + vy j
v1t
v2
b
x
v2t

At any time ‘t’


2 2
x 2 =( a − v1t ) + ( v2t )
dx
For minimum value of x, =0
dt Given; vx – 4 = 0 ⇒ vx = 4 m/s.
  
dx
So 2 x = 2 ( a − v1t ) × ( −v1 ) + v22 × 2t

 b
(
And v w  0 = v w – vb = vx i + v y j − 3j )
dt
= v i + v − 3 j ( )
−2av1 + 2t v12 + v22
or 0 = ( ) vy − 3
x y

Given tan 45° = ⇒ vy = vx + 3 = 3 + 3 = 6 m/s


av1 av1 vx

=∴ t =
2 2
v1 + v2 v2 6. (b) x = 3t1 = 5t2  ⇒ t1 = x/3 and t2 = x / 5
1 2
3. (b) Now h1 = 4t1 – gt1
2
1 4 gx 2
= 4 (x/3) – g(x/3)2 = x – .
a 2 3 18
 1 2
and h2 = 4t2 – gt2
2
v
4 x gx 2
The velocity of first particle, v1 = v = 4(x/5) – g (x/5)2 = –
5 10
The velocity of second particle, v2 = at
   Clearly, h2 < h1.
Relative velocity, v12= v1 − v2 7. (b) vy = 6.1 m/s.
2
or v12
2
v 2 + ( at ) − 2v ( at cos α )
= Thus v 2y = u 2y − 2 gh

dv 6.12 = u 2y – 2 × 9.8 × 9.1


For least value of relative velocity, 12 = 0
dt or uy = 14.68 m/s
u 2y 14.682
d ∴ h= = = 11 m.
or   v 2 + a 2t 2 − 2vat cos α  =0 2g 2 × 9.8
dt  
8. (d) y = ax – bx2
or  0 + a2 × 2t – 2vacos α = 0
v cos α or 0 = aR – bR2
or   t = ∴ R = a/b
(0,0) (R,0)
a
4. (c) According to law of sines or Lami’s Theorem 9. (c) If u1, u2 and q1, q2 are the velocities and angles of
projections then x1 = u1cos q1 t and x2 = u2cos q2 t
∴ x2 – x1 = (u2cos q2 – u1 cos q1)t
75
vB v B / A  vB  vA and y1 = u1sin q1 t – ½gt2 and y2 = u2sin q2 –½gt2
∴ y2 – y1 = (u2sin q2 – u1 sin q1)t
60 y − y  u sin θ2 − u1 sin θ1 
45 Now 2 1 =  2 
x2 − x1  u2 cos θ2 − u1 cos θ1 
vA
It represent vertical straight line for u1cos q1 = u2 cos q2
vA vB v 10. (d) If t is the time of height, then,
⇒ = = B/ A
sin 75° sin 60° sin 45° y = 0 = Kt (1– αt)
1
⇒ vB = 717 kmh–1 ∴ t = .
α
www.crackjee.xyz
198 Mechanics
Maximum height |aR|max = g cos 0° = g.
will occur at t/2, and so
17. (d) v0 cos 
 1   1  v0
Ymax = K   1 − α 
 2α   2 α 
K
= . v0 sin 

If t is the time taken by ball to return the boy’s hand,
11. (d) Given h = Ax – Bx2, on comparing with
then
gx 2 1
y = x tan θ – , we get 0 = v0 cos θt − gt 2
2u 2 cos 2 θ 2
A = tan θ = tan 45° = 1, 2v cos θ
or t = 0
g 10 1 g
and B = = =
2 2 2 2 40
2u cos θ 2 × 20 × cos 45° 1 2
v0 sin q t =
Now, at
A 2
∴ = 40.
B 2
1  2v0 cos θ 
12. (c) The time to reach the maximum height without air or v0 sin q = a
2  g 

resistance
u u a
t1 = = , ∴ tan q =
and g 10 g
with air resistance,
4
u u = = 0.4
t2 = = 10
10 + 1 11 18. (a) v0
u u
∆t t1 − t2  − 
Thus, × 100 = × 100 =  10 11  × 100 (H – y)
t1 t1  u 
 10 
x
= 9 % (decreases)
1 y
13. (c) 500 cosθ = 250  ⇒ cosθ =
or θ = 60°. 2
30°
1 2v sin θ
14. (d) Since 0 = (v sinq) t + (– a) t2 ⇒ t =
2 a If t is the time to hit the inclined, then
1 2
Also, h = (v cos q )t + g t x = v0t ...(i)
2
2v 2 g 1 2
  and H – y = gt ,...(ii)
⇒ h = sin q  cos θ + sin θ  2
a  a 
15. Particle will collide when P hits the inclined plane. So time 0 = v0 cos 30° − g sin 30°t …(iii)
of flight of P,
1 y
0 = ut – (g cos 60°) T2 Also= tan 30° ...(iv)
2 x
After solving above equations, we get
2u
or T = 2 gH
g cos 60° v0 =
5
2u
or 4 =
1
⇒ u = g. u 2 sin 2θ R 2b
g× 19. (a) R = and = u cos q ×
2 g 4 g
16. (a) aR = g cos θ.
The value of θ lies between: v0

θ = 0, 
h
aR
at highest point and < 90° at the g
R /4 R /4
point of projection, and so
www.crackjee.xyz
Motion in a Plane 199

u 2 sin 2 θ h (–cosec2θ)
=

Also, H = ; solving above equations
2g dt
 −d θ 
H or v = h cosec2θ  
We get = h.  dt 
4
20. (d) Let the car is at a distance x from A. = (8 × 103) cosec260°× 0.025
= 266.67 m/s
= 960 km/h
22. (c)

Taking reference line through support S, let xA and xB are


the distances of blocks A and B respectively from S. The
The line of sight with car makes an angle q with AB, then total length of the string,  = 2 x A + 3 xB + c,
we have where c is the some part of string which is over pulley and
x = y tan q. .... (i) somewhere else which remain constant.
Here y remain constant. Differentiating above relation w.r.t. time, we get,
Differentiating equation (i) w.r.t. time, we get d d
= (2 x A + 3 xB + c)
dx d dt dt
= [ y tan θ]
dt dt dx dx
or 0 = 2 A + 3 B
dx  dθ  dt dt
or = y sec2 θ  
dt  dt  dx A 3 dxB
or =
 dθ  dt 2 dt
\ v = y sec2 θ  
 dt  3
or vA = vB  .... (i)
dθ v 40 2
or = = 3
dt y sec θ 40sec2 45°
2
Also aA = aB  .... (ii)
2
1   
= rad / s . We have v AB = v A − vB
2 or vB = v A − v AB
21. (b) x = h cot θ
3
x or vB = vB − v AB
2
or vB = 2v AB [Given v AB = v ]
h
= 2v
 The acceleration of block B ;
∆v 2v − 0 2v
aB =
= =
∆t t t
The acceleration of block A
dx d (cot θ) 3  2v  3v
∴ = h = 3 aB = ×   = .
dt dt 2 2  t  t
Exercise 4.2
1. (a, b, d) 100
Absolute velocity of boatman is Time t = = 25 s
   4
v B = v BR + v R
Drift x = (2) (25) = 50 m
= ( −2ˆi + 4ˆj) + ( 4ˆi ) 100 4j^ vB
 2 2
vB = ( 4) + ( 2) =2 5 m/s
= 2ˆi + 4ˆj 
2i^
www.crackjee.xyz
200 Mechanics
2. (b, c, a) dy
Since the wind is blowing toward the east, the plane must and uy = = 3 – 8t
dt
head west of north as shown in figure. The velocity of the

plane relative to the ground v pg will be the sum of the ve-
 ∴ u = u x2 + y 2y = 22 + (3 − 8t )2
locity of the plane relative to the air v pa and the velocity

of the air relative to the ground v ag. = 22 + 32 = 13 m/s.
N 6. (b,d) u cos θ = v cos 30°
vpg
and 0 = u sin θ – g × 3,
W E vag ∴ u sin θ = 30 m/s.
vag
Also 0 = v sin 30° – g × 1,
S b
∴ v = 20 m/s.
(b) 1. The velocity of the plane relative to the ground is
Now u cos θ = 20 cos 30° = 10 3 m/s.
given by equation :
  
v pg = v pa + v ag ∴ u = 302 + (10 3)2 = 20 3 m/s.
2. The sine of the angle q between the velocity of
2(u cos θ) (u sin θ) 2 × 10 3 × 30
the plane and north equals the ratio of vag and R = =
vpa. g 10
vag 90 km / m = 60 3 m.
sin q = = = 0.45
v pa 200 km / h
7. (a,c,d) v = α x
(c) Since vag and vpg are perpendicular, we can use the
 dv 1 α
Pythagorean theorem to find the magnitude of v pg. = α x −1/2 = .
dx 2 2 x
v2pa = v2ag + v2pg
Thus acceleration,
vpg = v2pa − vag
2
dv α α2
a = v = α x× = .
2 dx 2 x 2
= ( 200 km / h ) − (90 km/ h)2
= 179 km/h. dv α2
Also =
3. (a,d) dt 2
For the ground observer, α2
t α 2t
1 2 or v = ∫ dt = .
x = ut and y = gt , 0 2 2
2
Mean velocity,
1 x2
∴ y = g , it represents a parabola. t α 2t
2 u2 ∫0 2
dt
α 2t 2 α 2t
For observer inside train, v = = =
t 4t 4
x = 0, and so ball apears to fall vertically.
4. (a, c, d) The path of case 1 is (see figure) 1 2 1  α2  2
Thus T1 = T2 + T3. s = at =  t
2 2  2 

T2
s
u ∴ t = 2
 α
u
T3 2 s
α2 ×
Now v = α = α s.
4 2
5. (a,c) x = 2t, 8. (a,c)
x At 30° and 60° , R1 = R2
∴ t =
2 Further, H ∝ sin2 θ and T ∝ sin θ
2
 x  x 3 H
and y = 3   − 4   = x – x2. ∴ ∝ sin 2 θ
2
    2 2 R
dx H
ux = =2 and ∝ sin θ
dt T
www.crackjee.xyz
Motion in a Plane 201
sin 60° > sin 30°  
∴ vP . vQ = 0
H1 H 2 H H
∴ > and 1 > 2
R1 R2 T1 T2 or v 2 cos 2 θ + v 2 sin 2 θ − v sin θ gt = 0
9. (b, c) v
v 2 v sin θ gt
or = = or t cosec θ
Initial velocity of particle in vector form can be written as g
y
Substituting this value of t in Eq. (2), we get
  v ˆ
v vQ v cos θ iˆ +  v sin θ −

= j
Q  sin θ 

 v2
 or vQ
= v 2 cos 2 θ + v 2 sin 2 θ + − 2v 2
2
P
x sin θ
 = v cot q
v=
ˆ ˆ
P v cos θ i + v sin θ j ...(1) 10. (c,d)
Velocity of particle at any time t will be: As maximum height attained by each one is same, so uy is
 also same. As
vQ v cos θ iˆ + (v sin θ − gt ) ˆj ...(2)
= 2u y
  T = ,
Given that vP ⊥ vQ g
So T1 = T2 = T3.

Exercise 4.3
1. (b) The initial velocity along vertical direction is same
u2
(uy = 0) for both the bodies, and so Rmax = , for θ = 45°.
g
1 2
h = 0 + gt , 5. (d) At the highest point of the trajectory,
2 vy = 0, and
2h 
or t = . so, Py = 0.
g
For the two pieces, it is
2. (c) If gravitational force is zero, then ay = 0.  
So, x = u cos θ t and y = u sin θ t P1 y + P2 y = 0.
∴ y = x tan θ. It represent straight line. 6. (a) The time of flight depends only on the vertical com-
The resultant path of the body depends on initial velocities ponent of velocity which remains unchanged in collision
and acceleration. with a vertical wall.
3. (a) For maximum height θ = 90°, or body must be pro- 7. (d) Linear momentum during parabolic path changes
jected straight upwards. Then continuously.
8. (d) Statement-1 is false because angles of projection q
0 = u2 – 2gh,
and (90° – q) give same range but time of flight will be
u2 different. Statement-2 is true because in horizontal direction
∴ h = .
2g acceleration is zero.
2u sin θ 9. (a) In statement-2, if speed of both projectiles are same,
T =
4. (d) , it will maximum, when θ = 0°. horizontal ranges will be same. Hence statement-2 is correct
g
explanation of statement-1.
Exercise 4.4
Passage for (Questions 1 & 2) On substituting the value of sin θ in above equation, we get
5
u 2 sin 2 θ u 2 sin 2θ u = gh
1. (a) Given, h = and 2 R = 2
2g g
Passage for (Questions 3 to 5) :
After simplifying, we get 3. (a) HA = HC > HB
tan θ = 2. 5
2 Obviously A just reaches its maximum height and C has
2  crossed its maximum height which is equal to A as u and θ
2. (d) sin θ = 1 are same. But B is unable to reach its maximum height.
5
www.crackjee.xyz
202 Mechanics
4. (c) 7. A→q;B→r;C→p;D→t
P Q dx
(A) Ux = =1
dt
dx
and Uy = = 1 – 2t
dt
O R
∴Ut=0 = u x2 + y 2y = 12 + 12 = 2 m/s.
Time of flight of A is 4 seconds which is same as the time of
2
flight if wall was not there. d x
(B) ax = =0
Time taken by B to reach the inclined roof is 1 sec. dt 2
TOR = 4 d2y
dy = =–2
TQR = 1 dt 2
∴ TOQ = TOR – TQR = 3 sec. (C) For time of flight,
2u sin θ y = 0
5. (c) From above
= T = 4s or 0 = t – t2
g
∴ t = 1s.
∴ u sin θ =20m/s ⇒ vertical component is 20 m/s (D) For maximum height,
for maximum height 1
t = s.
v² = u² + 2as ⇒ 0² = 20² – 2 × 10 × s ⇒ s = 20m 2 2
1 1 1
6. A → q ; B → r ; C → p ; D → q ∴ H = t – t2 = −  = m.
2 2 4

Exercise 4.5
2u cos θ dy
1. (b) t1 = and = kx ...(ii)
vy =
g dt
2u cos(90° – θ) 2u sin θ Dividing equation (ii) by (i), we get
and t2 = = ydy = xdx
g g
y x
2
∴ t1t2 =
4u sin θ cos θ
=
2R
.
Now ∫ y dy = ∫ x dx
2 g 0 0
g
2. (a) v0 cos θ = v0/2
y 2 x2
1 or = +k
∴ cos θ = , 2 2
or θ = 60°. 2 2
or y= x 2 + constant

v vx iˆ + v y ˆj , we get
3. (a) On comparing with, = 4. (b) Maximum area,
A = πR2
dx 2
vx = = ky ...(i)  v2  v4
dt = π   = π .
 g  g2

Das könnte Ihnen auch gefallen